Download as pdf or txt
Download as pdf or txt
You are on page 1of 528

Physics Question Bank and Notes

JEE / NEET
1 Physical World and Measurement
Physics : Scope, Technology and under standing

ˆ In physics we have study mechanial, thermal, electrical, magnetic and optical charcteristics of a
body.
ˆ To understand such a property, physics developed mechanics, thermodynamics,
electormagnetism, optics and electrodynamics, such like branches.
ˆ Range of physics is from zero to infinite.
ˆ Range of length in physics is from 10-14m (radius of nucleus) to 1026m (length of Galaxy.)
Hence ratio of measurement is in order of 1040.
ˆ Range of measurement of time is 10-22 s to 1018 s.
ˆ Range of mass is from 10–30 kg (mass of an electron) to 1055 kg (mass of Galaxy).
ˆ There are four type of fundamental forces in nature : (1) Gravitational force
(2) Electro magnetic force (3) Weak nuclear force (4) Strong nuclear force.
(1) Universe is made of ......
(A) Only radiation (B) Only matter (C) Vacuum (D) Matter and radiation
(2) Full name of AFM is ......
(A) Atomic force mirror (B) Atomic force microscope
(C) Atomic fire microscope (D) Automatic force microscope
(3) Full name of ESR is ......
(A) Electron spin resonance (B) Electron spin range
(C) Electric spin resonance (D) Electric space radar
(4) The range of physics is about ......
(A) Zero to infinite (B) range of nucleus (C) Earth to sun (D) near around the earth
(5) Physics considered vacuum as one ......
(A) Physical Quantity (B) Physical State (C) Physical unit (D) Infinite
(6) ...... is a branch of physics related to charge and magnetic field.
(A) Mechanics (B) Electrodynamics (C) Thermodynamics (D) Optics
(7) Electromagnetic force is ......
(A) only attractive (B) Attractive and repulsive
(C) Only repulsive (D) Short range force
(8) Strong nuclear force acting in the nucleus is between ......
(1) Proton-Proton (2) Proton-Neutron (3) Neutron-Neutron (4) Proton-Electron
(A) 1, 2, 3 (B) 1, 2, 4 (C) 1, 3, 4 (D) 4

1
(9) During the b-emission, nucleus emits ......
(A) neutron and electron (B) neutron and proton (C) neutrino and electron (D) neutrino and proton
(10) As space is isotropic, which law of conservation is obtained ?
(A) Law of conservation of energy (B) Law of conservation of charge
(C) Law of conservation of linear momentum (D) Law of conservation of angular momentum
(11) ...... is responsible for the conservation of linear momentum.
(A) Homogenity of a space (B) Isotropy of a space
(C) Homogenity of time (D) Isotropy of time
(12) If time is homogeneous, which law of conservation is obtained ?
(A) Law of conservation of energy (B) Law of conservation of charge
(C) Law of conservation of linear momentum (D) Law of conservation of angular momentum
(13) Full name of LHC is ......
(A) Large hedron collider (B) Large heater collider
(C) Large heater collision (D) Large hedron cobalt.
(14) If the resultant external ...... acting on the system is zero, total linear momentum of the system
remains constant.
(A) Force (B) Torque (C) Charge (D) Mass
Ans : 1 (D), 2 (B), 3 (A), 4 (A), 5 (B), 6 (B), 7 (B), 8 (A), 9 (C), 10 (D), 11 (A), 12 (A), 13 (A),
14 (A)
Units and Unit systems :
SI unit system :
Fundamental physical Name of Unit Symbol
quantity
Length (l) metre m
mass (m) kilogram kg
time (t) second s
Electric current (I) Ampere A
Thermodynamic Kelvin K
temperature (T)
Lumnious Intensity (I) Candella cd
Quantity of matter (m) Mole mol

Supplementry Units :

No. Supplementry SI Unit Symbol Formula


physical quantity
arc
1. Plane angle (q) Radian rad q=
radius
area
2. Solid angle (W) Steradian Sr W= =
+A
(radius)2 r2

2
(15) Number of fundamental units in SI system are ......
(A) 5 (B) 6 (C) 7 (D) 9
(16) Which is not a unit of energy ?
(A) joule (B) watt sec
(C) newton meter (D) kilogram-meter/sec2
(17) Which one have derived unit ?
(A) Pressure (B) quanitity of matter
(C) mass (D) Thermodynamic temperature
(18) KWh is unit of which physical quantity ?
(A) Power (B) momentum (C) work (D) Electric potential
(19) Unit of modulus of rigidity is ......
(A) Nm (B) Nm–1 (C) Nm–2 (D) Nm2
(20) Qurie is unit of which physical quantity ?
(A) Energy of g - ray (B) radioactivity (C) Half life (D) Intensity of radiation
(21) SI unit of an angular momentum is ......
(A) kg ms–1 (B) kg m2 s–1 (C) kg m–2 s–1 (D) kg m2 s–2
(22) Which one is supplementary unit ?
(A) second (B) Ampere (C) Candella (D) Steradian
(23) Which one is not a true unit of given physical quantity ?
(A) Power : N ms–1 (B) Torque : N m
(C) Pressure : N m–2 (D) Surface tension : N m2
(24) Parsec is unit of ......
(A) Distance (B) velocity (C) time (D) plane angle
(25) Which one is unit of Intensity of an electric field ?
(A) Vm (B) NC (C) Vm–1 (D) As
(26) Which one is not a unit of time ?
(A) second (B) hour (C) year (D) lightyear
(27) Which one is not a physical quanitity ?
(A) Kelvin (B) Candella (C) Volt (D) All
(28) Which physical quantity having same unit in all the unit system ?
(A) Length (B) Time (C) mass (D) Work
(29) dyne g–1 is a unit of which physical quantity ?
(A) Velocity (B) mass (C) Force (D) Acceleration
(30) Which physical quantity from given below is dimensionless ?
(A) Angle (B) Stress (C) density (D) Latent heat
3
(31) Which relation given below is wrong ?
(A) 1J = 107 erg (B) 1 dyne = 105 N
(C) 1 fm = 10–15 m (D) 1 parsec = 3.08 × 1016 m
(32) The average distance between sun and earth is called ......
(A) 1 Parsec (B) 1 lightyear (C) 1 AU °
(D) 1A
(33) SI unit of moment of inertia is ......
(A) kg m (B) kg m–2 (C) kg m2 (D) kg cm2
(34) Which unit is different than other unit ?
(A) Ws (B) KWh (C) Js (D) eV
(35) If the units for mass, length and time becomes double, then unit of angular momentum becomes ......
(A) Doubles (B) Three times (C) Four times (D) Eight times

(36)
1ns
1Ps
= ......
(A) 10–3 (B) 103 (C) 10–9 (D) 10–6
(37) Ns is a unit of which physical quantity ?
(A) velocily (B) Angular momentum (C) Linear momentum (D) work
(38) The volume and area of surface are equal for a given cube. Then the surface area = ...... unit.
(A) 36 (B) 216 (C) 144 (D) 1000
(39) Nm-2 is not a unit of physical quantity given below ?
(A) Pressure (B) Stress (C) Bulk modulus (D) Strain
Ans. : 15 (C), 16 (D), 17 (A), 18 (C), 19 (C), 20 (B), 21 (B), 22 (D), 23 (D), 24 (A), 25 (C), 26 (D),
27 (D), 28 (B), 29 (D), 30 (A), 31(B), 32 (C), 33 (C), 34 (C), 35 (C), 36 (A), 37 (C), 38 (B),
39 (D)
Measurement : Planet
Measurement for a long distance d
A
D
b q P
Planet
D
B D
Earth

Earth

Distance between Earth and planet, D = Measurement of dimension of planet and Star a =
b d
T D

Where, b = Distance between two positions for observation on the Earth.


a = angular diameter of planet.
q = angle in radian
D = Distance between planet and the Earth d = diameter of the planet

4
ˆ Units for very small and very large distances
Multiples Submultiples
Value Prefix Symbol Value Prefix Symbol
1018 Exa E 10–1 deci d
1015 Peta P 10–2 centi c
1012 Tera T 10–3 milli m
109 Giga G 10–6 micro m
10 6
Mega M 10–9
nano n
103 Kilo k 10–12 pico p
102 Hecto h 10–15 femto f
101 Deca da 10–18 atto a
ˆ For very small distance
1 fm = 10–15 m fm = Fermi
1A ° = 10–10 m A° = Angstrom

1 nm = 10–9 m nm = nanometer
ˆ For very large distance
The average distance between the Sun and the Earth is called 1AU
1 Astronomical unit = 1 AU° = 1.496 × 1011 m
The distance corresponding to 1AU length where 1" angle lubtended, is called 1 parsec (1 pc).
1 Parsec = 3.08 × 1016 m
1 lightyear = 1 ly = 9.46 × 1015 m
ˆ Some units of mass :
1 quintal = 100 kg
1 Metric ton = 1000 kg
1 atomic mass unit (amu) = 1.67 × 10–27 kg
ˆ Some units of time :
1 year = 365.25 days = 3.156 × 107 Sec.
1 LM (Lunar Month) = 27.3 days.
Time taken by moon to complete 1 revolution around the Earth is called 1 LM.
ˆ For a given physical quantity nu = Constant
Where n = Quntitative value, u = unit
\ n1u1 = n2u2
u1 = unit of physical quantity in one system.
u2 = unit of physical quantity in other system.

(40) 1° = ...... rad

(A) 180 (B) (C) (D) 360


S 180 360 n
S n

5
(41) If the unit of length and force increases to four times, the unit of energy ......
(A) Increases to 8 times (B) Increases to 16 times
(C) Decreases to 8 times (D) Decreases to 16 times
(42) If the unit of length and time are taken as km and hr, What is the value of g in km h–1.
(A) 980 (B) 9800 (C) 1,27,008 (D) 12,700
(43) The angle between two observed direction for a planet observed from two diametrically opposite
points A and B of the earth is 1.6°. If the diameter of the earth is 1.276 × 104 km, Find the
distance between earth and planet.
(A) 4.57 × 105 km (B) 4.57 × 108 km (C) 3.84 × 108 m (D) 4.08 × 108 m
(44) Diameter of the sun is 1.393 × 109m. Angular diameter of the Sun is ...... . Distance between Sun
and earth is 1.496 × 108 km and 1" = 4.85 × 10–6 rad.
(A) 1920" (B) 1920' (C) 192.0" (D) 1920 rad
(45) If the angle between two observed direction for moon is 54', When it is observed from the two
diametrically opposite points simultaneously. If the radius of the earth is 6.4 × 106 m. Find the
distance between earth and moon.
(A) 8.153 × 108 m (B) 4.076 × 108 m (C) 5.813 × 108 m (D) 3.581 × 108 m

Ans : 40 (A), 41 (B), 42 (C), 43 (A), 44 (A), 45 (A)

Errors in measurement :
Measurement of inaccuracy is called error.
ˆ Estimation of Error :
(1) Absolute error :
Observations for any physical quantity are a1, a2, ......, an

a1  a 2  ......  a n
¦ ai
n
Mean a = =
1
n n i 1

Absolute error in each observation


Da1 = a – a1, Da2 = a – a2..... Dan = a – an
Average (Mean) Absolute error.

¦ 'ai
' a1  ' a 2  ...  ' a n n
Da = = n1
n i 1

\ Measurement of any physical quantity = a ± D a


(2) Relative error.
'a
da =
a

(3) Percentage error


'a
da × 100 % = × 100 %
a
6
Combination of errors :
No. Formula error

1. Addition Z = A + B DZ = DA + DB
2. Subtraction Z = A – B DZ = DA + DB
'Z 'A 'B
3. Multiplication Z = AB = +
Z A B

'Z 'A 'B


4. Division Z = A/B = +
Z A B

'Z 'A
5. Terms with power Z = An =n
Z A

Significant digits :
“The number of digits whose value is accurately known in a measurement plus one additional digit
about which we not certain are called significant figures (digits)”
Rules to decide significant digits
(1) All non - zero digits are significant.
(2) All zeros between two non - zero digits are significant.
(3) When the value is less than one, All zeros to the right of decimal and left of non - zero digit are
never significant.
(4) All zeros on the right of non - zero digit are not significant.
(5) All zeroes after nonzero mumber in, number having decimal point are significant.
ˆ As number of significant digits after decimal points are more, accuracy in measurement
is more.
(46) A body travels a distance (14.0 ± 0.2) m in (4.0 ± 0.3) s, its velocity is ...... ms–1
(A) (3.5 ± 0.51) ms–1 (B) (3.5 ± 0.41) ms–1 (C) (3.5 ± 0.31) ms–1 (D) (3.5 ± 0.21) ms–1
R1R 2 'R p
(47) For parallel connection of Resistance Rp = R  R then = ......
1 2 R p2

' R1 'R 2 ' R1 'R 2 'R1 'R 2 'R1 'R 2


(A) R + (B) – (C) – (D) +
1 R2 R1 R2 R12 R 22 R12 R 22

(48) Two resistances R1 = (3 ± 0.1) W and R2 = (6 ± 0.3) W are connected in series, net resistance
R = ...... W
(A) 9 ± 0.2 (B) 3 ± 0.2 (C) 9 ± 0.4 (D) 9 ± 0.1
(49) The mass, lenght, breadth and thickness for a cube is (39.3 ± 0.1)g, (5.12 ± 0.01)cm
(2.56 ± 0.01) cm, (0.37 ± 0.01) cm respectively then inaccuracy in measurement of density
is ...... g cm–3
(A) 0.29 (B) 0.41 (C) 0.19 (D) 0.035
(50) For a wire, mass = (0.3 ± 0.003) g radius = (0.5 ± 0.005) mm and length = (6 ± 0.06) cm,
percentage error in density is ......
(A) 1 % (B) 2 % (C) 3 % (D) 4 %
7
(51) The periodic time of second pendulum is 2.0 s and mean absolute error in its measurement is
0.01s, then value of periodic time with error is ......
(A) 2.0 ± 0.10 s (B) 2.0 ± 0.05 s (C) 2.0 ± 0.02 s (D) 2.0 ± 0.01 s
(52) V = (100 ± 5) V, I = (10 ± 0.1) A, then percentage error in measurement of Resistance is ......
(A) 5.1 % (B) 4.9 % (C) 6 % (D) 3 %
(53) When current ‘I’ passes through a resistance ‘R’ in time ‘t’, Heat energy produced is given by
H = I2Rt. Percentage error in I, R and t are 2 %, 3 % and 1 % respectively then percentage
error in H = ......
(A) 6 % (B) 5 % (C) 7 % (D) 8 %
(54) A length of cube l = (1.5 ± 0.02) cm its volume V = ...... cm3
(A) 3.375 ± 0.04 (B) 3.375 ± 0.135 (C) 3.375 ± 0.4 (D) 3.375 ± 0.013
1
2 B2
(55) Physical quantity Z =
A
. Percentage error in measurement of A, B, C, D are 2 %, 1 %,
CD3

3 % and % respectively. Find the percentage error in measurement of ‘Z’.


1
3

(A) 8 % (B) 7 % (C) 6 % (D) 5 %


(56) How many significant digits in 0.0250 ?
(A) 1 (B) 2 (C) 3 (D) 4
(57) Significant digits in 5.4 × 103 is ......
(A) 1 (B) 2 (C) 3 (D) 4
(58) 1.875 + 2.41 = ...... (by considering significant digits)
(A) 4.3 (B) 4.28 (C) 4.285 (D) 4.29
(59) Mass and radius of sphere are 5.13 g and 2.10 mm. Find its density by considering significant
digits.
(A) 132 g cm–3 (B) 130 g cm–3 (C) 132.3 g cm–3 (D) 132.30 g cm–3
(60) Multiplication of 15.235, 3.315 and 2 = ...... (by considering significant digits)
(A) 101.0 (B) 101.00 (C) 101 (D) 100
(61) 1.97855 is round off to three digits then obtained number = ......
(A) 1.97 (B) 1.98 (C) 1.90 (D) 2.00
(62) Significant figures in 71.15, 3.008 and 0.1237×105 are ......
(A) 4, 2, 4 (B) 4, 4, 4 (C) 4, 3, 5 (D) 4, 4, 6
(63) Significant digits in 0.0007 are ......
(A) 1 (B) 2 (C) 3 (D) 4
(64) When 1.71 N is Subtracted from 3.75 N the result in significant figures is ......
(A) 2 N (B) 2.0 N (C) 2.04 N (D) 2.000 N

8
(65) Lenght of simple pendulum measured with scale of least count 1mm is 10 cm. Clock having
dispersions is measures 90 s for 100 oscillations. The value of g = ...... ms-2. (take g = 9.8 ms–2)
(A) 9.8 ± 0.11 (B) 9.8 ± 0.21 (C) 9.8 ± 0.31 (D) 9.8 ± 0.41
(66) Thickness of plate measured with l1 = 40.2 ± 0.1 and l2 = 20.1 ± 0.1, maximum uncertainity in
l1 + l2 = .......
(A) 0.1 (B) 0.2 (C) 0.3 (D) 0.4
(67) Thickness of plate measured by micrometer having least count 0.01mm is 1.03 mm. What is the
percentage error in the measurement of thickness of plate ?
(A) 0.7 % (B) 0.97 % (C) 1 % (D) 1.2 %
(68) 9.15 + 3.8 = ...... (by considering significant digits).
(A) 13 (B) 13.0 (C) 13.00 (D) 13.000
(69) Observations for the measurement in length are 2.01 m, 2.03 m, 2.09 m, 2.07 m and 2.01 m.
Mean absolute error in the measurement is ......
(A) 0.028 m (B) 0.030 m (C) 0.152 m (D) 0.048 m

'I
(70) f=– (gravitational potential), then = ...... .
GM
r I

'r 'r 'r


(A) – (B) (C) 2 (D)
r
r r r 'r

(71) If the percentage error in the measurement of volume of a sphere is 3 %, percentage error in the
measurement of surface area is ......
(A) 2 % (B) 1 % (C) 3 % (D) 4 %
(72) Radius of a sphere is 1.51 cm. Area of sphere by considering significant figures is ......
(A) 28.6 cm2 (B) 28.63 cm2 (C) 28.638 cm2 (D) 28.6381 cm2
Ans : 46 (C), 47 (D), 48 (C), 49 (A), 50 (D), 51 (D), 52 (C), 53 (D), 54 (B), 55 (B), 56 (C),
57 (B), 58 (B), 59 (A), 60 (A), 61 (B), 62 (B), 63 (A), 64 (C), 65 (C), 66 (B), 67 (B),
68 (B), 69 (B), 70 (B), 71 (A), 72 (A)

Dimension and Dimensional Formula

ˆ When any physical quantity is represented in terms of M, L, T, ......, the equation is known as
dimensional formula and power of M, L, T.... is known as dimension.

ˆ If the dimensional formula for a physical quantity is Ma Lb Tc, their values in two different unit
system are n1 and n2 then,

ª M1 º ª L1 º ª T1 º
a b c
n2 = n1 « » « » « »
¬ M2 ¼ ¬ L 2 ¼ ¬ T2 ¼

9
Physical quantity : Formule, Units and dimensional formula.
No. Physical Quantity Formula Unit Dimensional
Formula
1. Speed Distance / time ms–1 M° L1 T–1
2. Acceleration Change in volocity/time ms–2 M° L1 T–2
3. Force Mass × accleration N = kg ms–2 M1 L1 T–2
4. Density Mass/volume kg m–3 M1 L–3 T°
5. Pressure Force/area Nm–2 = Pa M1 L–1 T–2
6. Work Force × displacement Nm = J M1 L2 T–2
7. Energy - J M1 L2 T–2
8. Power Work / time Watt M1 L2 T–3
9. Impulse of force Force × Change in time Ns M1 L1 T–1
10. momentum mass × velocity kg ms–1 M1 L1 T–1
11. Torque Force × position vector Nm M1 L2 T–2
12. Temperature (T) – Kelvin M° L° T° q–1
13. Heat (Q) - J M1 L2 T–2
Heat
14. Specific heat J kg–1 K–1 M° L2 T–2q–1
Q
m'T mass × Temp. diffrence
15. Latent heat Heat J kg–1 M° L2 T–2
mass
16. Gas constant (R) – J mol–1 K–1 M1 L2 T–2q–1

gas constant
17. Boltzmann constant (kB) J K–1 M1 L2 T–2q–1
R
=
NA Avagrado’s No.

18. Plank’s constant (h) Energy / frequency Js M1 L2 T–1


19. Charge (q) Electric current × time As = C M° L° T1A1
20. Surface Charge Charge
density (s) area Cm–2 M° L–2 T1A1
21. Electric current density (J) Current per unit area Am–2 M° L–2 T°A1
22. Electric potential (V) Work JC–1 M1 L2 T–3A–1
Charge
23. Intensity of electric (E) Force/Charge NC–1 or Vm–1 M1 L1 T–3A–1
field
Potential difference
24. Resistance (R) =W M1 L2 T–3A–2
V
Electric current A

R1
Electric current
25. Conductance W–1 = mho M–1 L–2 T3A2
Potrential difference

26. Resistivity (r) RA Resistance × Area Wm M1 L3 T–3A–2


=
l length

10
No. Physical Quantity Formula Unit Dimensional
Formula

27. Conductivity (s) U = RA W–1m–1 M–1 L–3 T3A2


1 l

q1 q 2
28. Permitivity of e = N–1C2m–2 M–1 L–3 T4A2
o 4 SFr 2
vacuum (eo)

29. Capacitance (C) Charge CV–1 or F M–1 L–2 T4A2


potential difference

B=
F
30. Intensity of magnetic qv NA–1 m–1 or tesla M1 L0 T–2A–1
field (B)

31. Magnetic flux (f) N B˜ A Vs or weber M1 L2T–2A–1


JG JG

32. Self inductance (L) NI Vs A–1 or henry M1 L2T–2A–2


I

33. Stress Force / area Nm–2 M1 L–1T–2


34. Modulus of elasticity Stress/Strain Nm–2 M1 L–1T–2
35. Moment of Inertia (I) mass × (Perpendicular distance)2 kg m2 M1 L2To
Force or Energy
36. Surface Tension (T) length area Nm–1 = Jm–2 M1 L0T–2

37. Co-efficient of viscosity (h) Nsm–2 M1 L–1T–1


F
6 Srv
Physical quantity having same dimension :
No. Dimensional Physical quantity
Formula
1. M° L° T–1 Frequency, Angular frequency, Angular Speed, Angular velocity velocity gradient,
decay constant
2. M1 L2 T–2 Work, kinetic energy, potential energy Internal energy, Torque, Heat energy
moment of force
3. M1 L–1 T–2 Pressure, Stress, Bulk modulus, Young’s modulus, modulus of rigidity energy density
4. M1 L1 T–1 Linear momentum, Impulse of Force.
5. M° L1 T–2 Acceleration, Acceleration due to gravity, Intensity of gravitational field
6. M L T
1 1 –2
Force, Weight, Thrust
7. M1 L° T–2 Surface Tension, Surface energy (energy per unit area), spring constant.
8. M° L° T° Strain, relative density, plane angle, solid angle, relative permitivity (Dielectric
constant), relative permeability.
9. M° L2 T–2 Latent heat, Gravitational potential
10. M1 L2 T–2q–1 Heat capacity, gas constant, Boltzmann’s Constant, Antropy

11
(73) Dimensional formula of moment of force couple is ......
(A) M1 L1 T–2 (B) M1 L2 T–2 (C) M2 L2 T–2 (D) M1 L–2 T–2
(74) Dimensional formula of energy density is ......
(A) M1 L1 T–1 (B) M1 L–1 T–2 (C) M1 L2 T–2 (D) M1 L–2 T–1
(75) If E, M, L G are Energy, mass, angular momentum and universal constant of gravitation

EL2
respectively then dimension of is ......
M5G 2

(A) Plane angle (B) time (C) mass (D) Length


(76) Which pair (given below) having same dimensional formula ?
(A) Force and work (B) Torque and Power (C) Energy and Torque (D) Power and Energy
(77) If Speed of light (c), Acceleration due to gravity (g) and pressure (P) are taken as fundamental
unit. Then the dimensions of Gravitational constant (G) in c, g, P is ...... .
(A) –1, 2, –1 (B) 1, 2, –1 (C) 2, 2, –1 (D) 0, 2, –1

§ a ·
(78) Dimensional formula of ab in ¨ P  2 ¸ (v–b) = mRT is ...... . Where V = volume, P = pressure,
© V ¹

T = Temperature
(A) M1 L3 T–2 (B) M1 L5 T–2 (C) M1 L–8 T2 (D) M1 L8T–2

(79) Which one is dimensionally correct ?

(A) v = v0 + at2 (B) F = (C) d = v (D) d =


W 2 v 2  v02
d 2 at 2a

v = final velocity, v0 = initial velocity, a = acceleration, W = work, d = displacement

(80) If A, B and C are physical quantities having different dimension, then which one, given below is true ?

(A) (B) AB + C (C) (A + B)C (D) AB


A–B
C C

(81) Which pair given below having different dimension ?

(A) Torque and Work (B) Angular momentum, Plank’s constant

(C) Impulse of force & linear momentum (D) Tension, Surface tension

– bt
(82) Amplitude of damped oscillation A(t) = Ae 2 m . Dimensional formula of b = ......
A = Initial Amplitude, m = mass, t = time.

(A) M1 L° T–1 (B) M1 L1 T1 (C) M1 L1T–1 (D) M1 L1 T°

12
(83) The number of undecayed atoms at time ‘t’ in a element is given by N = N0 e–lt. Where
N0 = Initial undecayed atoms. Find the dimensional formula of l.

(A) M–1 L° T° (B) M° L° T–1 (C) M° L–1T° (D) M1 L° T–1

(84) Dimensional formula of Power is ......

(A) M1 L–2 T2 (B) M1 L2 T–2 (C) M1 L2T–3 (D) M° L2 T–3

(85) Dimensional formula of Impulse of force is ......

(A) M1 L1 T1 (B) M1 L–1 T1 (C) M1 L1T–1 (D) M1 L2 T–1

(86) M° L° T–1 is dimensional formula of ......

(A) R
L
(B) R
L
(C) LR (D) LR
1

(87) Dimensional formula of Intensity of radiation is ......

(A) M1 L–2 T–2 (B) M° L3 T–2 (C) M1 L°T–1 (D) M1 L° T–3

(88) Distance travelled by particle in time ‘t’ is ‘x’, x = [1 – ekt], v0= initial velocity, then
v0
k
dimensional formula of k = ......

(A) M° L–1 T1 (B) M° L1 T° (C) M° L°T–1 (D) M° L° T1

(89) dx
dt
= ae–bt, a and b are constant, x is a displacement of a particle in time ‘t’. Dimension of

is ......
a
b

(A) Distance (B) time (C) mass (D) velocity

(90) Pressure difference for inner and outer side of bubble formed in air is Pi – Po = 4RT . Where

R = Radius of bubble, T = Surface tension, dimension of surface Tension is ......

(A) M1 L1 T–1 (B) M1 L–1 T–1 (C) M1 L°T–2 (D) M1 L° T–1

(91) Young modulus for steel in MKS is 2 × 1011 Pa then in CGS its value is ...... dyne cm–2.

(A) 2 ×1010 (B) 2 ×1012 (C) 2 ×1013 (D) 2 ×106

(92) In a new unit system units of force, distance and time are 100N, 10m and 10s respectively. What
is the unit of mass in new system ?

(A) 103 kg (B) 104 kg (C) 105 kg (D) 106 kg

(93) u1 and u2 are units of some physical quantity, n1 and n2 are their quantitative values then ......

(A) n = u (B) n = u (C) u = n (D) u = u


n1 u1 n1 u2 n1 u1 n1 n2
2 2 2 1 2 2 1 2

13
(94) Force acting on a body is 10N. If the unit of mass and distance become double and unit of time
becomes half then magnitude of force in new unit system will be ...... N.
(A) 1.6 (B) 16 (C) 160 (D) 1600
(95) Energy of a particle is 10J. If the unit of mass becomes four times, unit of acceleration becomes
double, unit of length becomes half, then energy of particle in new system is ......
(A) 4J (B) 40J (C) 400J (D) 4kJ
(96) Unit of power 100erg min–1, unit of time is 1 h, unit of force is 60 dyne then unit of lenght is ...... cm.
(A) 1 (B) 10 (C) 100 (D) 1000
(97) Dimensional formula of force is Ma Lb Tc then 3a + 5b – 2c = ......
(A) 10 (B) 1.2 (C) 4 (D) 12
(98) Dimensional formula of Electric power is Ma Lb TcAd then, 5a + 2b + c – d = ......
(A) 4 (B) 6 (C) 8 (D) 10
(99) Dimensional formula of angular momentum is Ma Lb Tc and dimensional formula of density is
Mx Ly Tz then ax + by – cz = ......
(A) –5 (B) 5 (C) 25 (D) –25
(100) When 10 N force is act on a particle momentum obtained is 1 SI, frequency of oscillation for a
particle is ......
(A) 1 Hz (B) 10 Hz (C) 100 Hz (D) 1 KHz
(101) Momentum (p), Area (A), time (T) are taken as fundamental quantities, dimension of energy is ......

(A) p1 A–1 T1 (B) p2 A1 T1 (C) p2 A 2 T1 (D) p1 A 2 T–1


–1 1

A + Bt 2
(102) A body is moving along the x-axis, equation of velocity is given by v(t) = 1+ Ct
. Dimensional

formula of A, B and C are ...... .


(A) L1 T–1, L1 T–3, T–1 (B) L1 T–1, L1 T–2, T–1
(C) L1 T–2, L1 T–1, T1 (D) L1 T–1, L1 T–2, L1 T–1

(103) Momentum (p) is given by equation p = mv . Dimension of c and p are ...... .


2
1– v
c2

(A) L1 T–2, M1 L1 T–2 (B) M1 L1T–1, L1 T–1


(C) L1 T–1, M1 L1 T–1 (D) M1 L1T–2, L1 T–2

Dt + Et 2
(104) Force is defined as F = 1+ Asin 2 Z t
, then units of A and D are ...... .

(A) unitless, Ns–1 (B) unitless, Ns (C) m, Ns–1 (D) m, Ns


(105) 100 g mass, 10 cm distance, 0.1s time are taken as units in new system. Unit of work = ...... in
new system.
(A) 0.01 J (B) 0.1 J (C) 1 J (D) 10 J

14
(106) Energy E = Gxcyhz. Where G = universal constand of gravitation, c = velocity of light and
h = plank’s constant then value of x, y, z are ...... .

(A) 12 , –1 , 2 (B) –1 , , 1 (C) –1 , , 1 (D) 12 , –1 , 2


–5 3 5 –3
2 2 2 2 2 2 2 2

(107) If energy (E), velocity (V), and time (T) are taken as fundamental physical quantities. Then,
dimensional formula of surface tension is ......
(A) E1 V–2 T–2 (B) E1 V–1 T–2 (C) E1 V–2 T–1 (D) E–2 V–1 T–3
(108) Dimensional formula of permitivity of vacuum is ......
(A) M–1 L–3 T4A2 (B) M–1 L–3 T2A2 (C) M–1 L–3 T4A–2 (D) M–1 L–3 T–2A–2
(109) If the dimension of a physical quantity is La MbTc, then this physical quantity is ......
(A) Acceleration, If a = 1, b = 1, c = –2 (B) Pressure, If a = –1, b = 1, c = –2
(C) Force, If a = –1, b = 0, c = –2 (D) velocity, If a = 1, b = 0, c = 1
(110) Dimensional formula for the ratio of linear momentum and angular momentum is ......
(A) M° L° T° (B) M° L–1 T° (C) M° L1 T° (D) M° L1 T–1
(111) A physical quantity is given by Z = MxLyTz. If percentage error in measurement of M, L and T
are a, b and c respectively then maximum percentage error in the measurement of Z is ......

(A) x + b + c (B) ax + by + cz (C) ax + by – cz (D) x + b – c


a a
y z y z

(112) 76 cm height of Hg = ...... Nm–2.


Density of Mercury (Hg), r = 13.6 g cm–3.
(A) 1.013 × 105 (B) 1.01 × 10–5 (C) 76 × 10–2 (D) 7.6 × 105
Ans : 73 (B), 74 (B), 75 (A), 76 (C), 77 (D), 78 (D), 79 (B), 80 (D), 81 (D), 82 (A), 83 (B),
84 (C), 85 (C), 86 (B), 87 (D), 88 (C), 89 (A), 90 (C), 91 (B), 92 (A), 93 (B), 94 (C),
95 (B), 96 (C), 97 (D), 98 (B), 99 (A), 100 (B), 101 (D), 102 (A), 103 (C), 104 (A),
105 (B), 106 (C), 107 (A), 108 (A), 109 (B), 110 (B), 111 (B), 112 (A)

Questions depents on experimental skills :


Least-count of Vernier calliperse :
Value of one division on main scale (S)
ˆ L.C. =
Total no. of divisions on vernier scale
ˆ Least count (L.C.) = Value of one division on main scale (1 MSD) - value of one division on vernier
scale (1 VSD)
ˆ Suppose 1 MSD = a unit
If nth division of vernier matches with mth division of main scale.

1 VSD = m
n
× a unit


\ Least count (L.C.) = a – n a = 1– n a unit
m m

15
Least count of micrometer screw :
Pitch (p)
ˆ Least count (L.C.) = Total divisions on circular Scale

Where pitch (p)= distance of one division on main scale.


= Distance travelled in complete one rotation of circular scale.
(113) In vernier calliperse measurement of one division on main scale is ‘x’ cm. nth division of vernier
scale matches with (n–1)th division. Then minimum measurement of vernier calliperse is ...... cm.

§ n –1 ·
(A) n–1 (B) ¨ n ¸ x (C) n– 1 (D) n
x nx x
© ¹

(114) Micrometer screw with pitch 0.5 mm and 50 divisions on circular scale is used to measure
diameter of a sphere. Then least count of micrometer screw is ......
(A) 0.1 cm (B) 0.01 cm (C) 0.001 cm (D) 0.05 cm
(115) 30th division of vernier scale matches with 29th division of main scale in spectrometer.
If value of one division on main scale is 0.5° then minimum measurement (L.C.) = ...... .
(A) One minute (B) Half minute (C) 1° (D) 0.5°
(116) Diameter of a wire is measured with micrometer of least count 0.01 mm. Reading of main scale
is 0 mm and reading of circular scale are 48 divisions then diameter of a wire is ...... .
(A) 0.48 cm (B) 0.048 cm (C) 0.24 cm (D) 0.0048 cm
Ans. : 113 (D), 114 (C), 115 (A), 116 (B)
Assertion - Reason type Question :
Instruction : Read assertion and reason carefully, select proper option from given below.
(a) Both assertion and reason are true and reason explains the assertion.
(b) Both assertion and reason are true but reason does not explain the assertion.
(c) Assertion is true but reason is false.
(d) Assertion is false and reason is true.
(117) Assertion : Light year and wavelength both represent distance.
Reason : Both having dimension of time.
(A) a (B) b (C) c (D) d
(118) Assertion : The distance of stars, which are farthar away than 100 light year can not measured
with the method of parallex removal.
Reason : Angle of parallex removal can not be measured accurately.
(A) a (B) b (C) c (D) d
(119) Assertion : Dimension of Surface tension and Surface energy are equal.
Reason : Their SI units are equal.
(A) a (B) b (C) c (D) d

16
(120) Assertion : y = A sin (wt – kx) and (wt – kx) is dimensionless.
Reason : Dimension of k is M°L1T°

(A) a (B) b (C) c (D) d

(121) Assertion : In all measurement, last significant digit is more in accurate.


Reason : d = 0.9 m, d = 0.90 m and d = 0.900 m the d = 0.900 m is more accurate.

(A) a (B) b (C) c (D) d

ˆ Match the columns :

(122) Match the physical quantity in column-1 with SI unit in Column-2.


Column-1 Column-2
(1) Work (a) Jm–1 (A) 1 (c), 2 (d), 3 (b), 4 (a)
(2) Power (b) Ns (B) 1 (b), 2 (c), 3 (a), 4 (d)
(3) momentum (c) kwh (C) 1 (d), 2 (b), 3 (c), 4 (a)
(4) Force (d) Nms–1 (D) 1 (c), 2 (d), 3 (a), 4 (b)

(123) Column-1 Column-2


(1) Stefan’s Constant (a) JK–1mol–1 (A) 1 (d), 2 (b), 3 (c), 4 (a)
(2) Universal gas constant (b) Fm–1 (B) 1 (a), 2 (d), 3 (b), 4 (c)
(3) Electric permitivity (c) Hm–1 (C) 1 (d), 2 (a), 3 (b), 4 (c)
(4) magnetic permeability (d) Wm–2k–4 (D) 1 (a), 2 (d), 3 (c), 4 (b)

(124) Match the measurement in column-1 with significant digits in column-2.


Column-1 Column-2
(1) 33.015 (a) 3 (A) 1 (b), 2 (a), 3 (d), 4 (c)
(2) 0.054 (b) 4 (B) 1 (d), 2 (c), 3 (b), 4 (a)
(3) 0.003530 (c) 2 (C) 1 (d), 2 (a), 3 (c), 4 (a)
(4) 1.75×10–4 (d) 5 (D) 1 (b), 2 (c), 3 (d), 4 (a)

Ans. : 117 (B), 118 (A), 119 (C), 120 (C), 121 (B), 122 (A), 123 (C), 124 (B)

17
3&4 Kinematics
Vector
Equal Parallel Opposite Antiparallel Aparallel
vectors vectors vectors vectors vectors
Magnitude Equal Unequal Equal Unequal Equal or Unequal
Direction Equal Equal Opposite Opposite any
Angle between q = 0° q = 0° q = 180° or q = 180° or q ¹ 0°, q ¹ 180°
two vectors p rad p rad 0 < q < 360°
ˆ When a vector is multiply by any value or scaler, then direction remain as it is but magnitude
becomes scaler times.
Addition or Subtraction of vectors
ˆ Method of triangle
ˆ Method of parallelogram and
ˆ Algebraic or analytical Method
It is resultant vector of addition of vector and vector B , then
  
ˆ R A
o
|R | = A 2  B2  2ABcos T .

and R makes an angle a with vector A , then


 

Bsin T R
tan a =
A  Bcos T
= y  
A+
Rx B  B
 =
ˆ Magnitude of unit vector is one unit, it is expressed as n . R

Unit vector in direction of A ,



ˆ q a
o 
n A = Vector A
=
A
o magnitude of vector
|A |
ˆ Unit vectors in three dimensions (i.e. cartesian co-ordinate system) X-axis ® i , Y-axis ® j ,
Z-axis ® k Y R
Y
Resolution of a vector Ax
P P
Az 
A
Ay Ay Ay
O

A Y
Ax Az M
Ax
X
O Ax Ay Q
X
In two dimension In three dimension

A = Ax i + Ay j and A = Ax i + Ay j + Az k
 

o o
|A| A x 2  A y2 |A| Ax 2  A y2  Az 2

18
ˆ Angular resolution of a vector (In two dimensions) Y
A = A cosq i + A sinq j


ˆ
ˆ A vectors makes an angle q with x-axis in two dimensions, then Q
S
§ Ay ·

A sinq
\ q = tan–1 ¨ ¸
Ay A
tanq =
Ax © Ax ¹ R q
A cosq
ˆ Subtraction of vectors : P
To make subtraction, add an opposite vector of which will
be subtract, in given vector. O X
N A cosq M


A

– 
 B
A =

R = A + (– B)
  
–B

R = A – B
  

If R = A – B then
  
ˆ
o  Bsin T
|R | = A 2  B2 – 2ABcos T and an angle of R with A is a then, tan a = A  Bcos T
 

Multiplication of vectors

Scaler product Vector Product


(dot product) (cross product)

o o o o
ˆ A˜ B = | A | | B | cos q ˆ Au B = | A | | B | sin q n
   

o o
ˆ A˜ B = The product of the magnitude of first Where n is unit vector in the direction
vector with the projection of second vector on obtained by right handed screw rule.
the first vector
o o oo o o o o
ˆ A˜ B = B˜ A ˆ A u B = – Bu A
§ o o· § o o·
u ¨ Au C ¸
o o o
ˆ A u ( B  C) = ¨ ¸ +
o o o o o o o
© ¹
ˆ A ˜ ( B  C) A ˜ B  A ˜ C
© ¹
A B

ˆ If A and B are parallel then, ˆ If A and B are parallel or anti parallel


   

o o § o o · , then o o
A˜ B = | A | | B | ¨A &B¸ Au B = 0
 

©  ¹ 
§ o o·
ˆ If A and B are antiparallel, then If A and B are perpendicular ¨ A A B ¸
 
ˆ
© ¹
o o
A˜ B = – | A | | B | then, o o = | A | | B | n
   
Au B

19
i j k
o o
ˆ If A and B are perpendicular ˆ Au B =
  Ax Ay Az
Bx By Bz
o o
then A ˜ B = 0
o o
ˆ A˜ B = AxBx + AyBy + AzBz ˆ Area of the triangle formed by A and B ,
 

1  
D= |A×B|
2
ª º
A x Bx  A y B y  A z Bz Sin D Sin E Sin J
–1 « »
ˆ q = cos « » = =
 
ˆ C a
«¬ ( A x  A y  A z ) ( Bx  By  Bz 2 ) »
B
¼
2 2 2 2 2
A B C
b g

A
(1) Magnitude of A and B are 10 unit and 20 unit respectively. If the angle between this two vectors is
 

30°, the magnitude of resultant (R) is ...... unit and angle with vector A is ...... ( 3 = 1.7).


(A) 30 unit, 30° (B) 10 unit, 30° (C) 29 unit, 20° 19' (D) 39 unit, 30° 30'

(2) A force act on a substance having mass m is F = 5 i – 4 j + 2 k N. An angle of this force




with X-axis is ......

§ 1 · § 5· § 3 ·
(A) Cos–1 ¨ ¸
©3 5 ¹
(B) Cos–1 ¨¨ 3 ¸¸
© ¹
(C) Cos–1 ¨
© 5¹
¸ (D) Cos–1 3 5
o o
(3) | A × B | is greater by 73 % than A ˜ B for given A and B . An angle between
   

A and B is ......
 

(A) 0° (B) 30° (C) 60° (D) 90°

(4) If the addition vector R of A and B makes an angle a with A and b with B . Then ......
    

(A) Always a < b (B) If A < B, a < b


(C) If A = B, a < b (D) If A > B, a < b
 
(5) An angle between A = 3 i – 4 j – 2 k and B = 8 i + 3 j + 6 k is ......
(A) 90° (B) 60° (C) 180° (D) 0°
 
(6) If the vectors A = 5 i + 7 j – 3 k and B = – 2 i + m j + 6 k are mutually perpendicular,
then m = ......
(A) 1 (B) 4 (C) 3 (D) –4
  JG JG
(7) If A = 2 i – j + 3 k and B = i + 2 j – k then unit vector in the direction of A u B is ......
1
(A) (3 i + j +2 k ) (B) i – j + k
14
1 1
(C) ( i + j + k ) (D) (– i + j + k )
3 3
20

(8) Area of triangle formed by A = 4 i + 3 j – k and B = i + 2 j + k is ...... unit.


(A) 10 3 (B) (C) (D) 5 3


5 3 3
2 2

(9) | A × B |2 + | A × B |2 = ......
   

(A) zero (B) AB (C) AB (D) A2B2

(10) If | A + B | = | A | = | B |, then the angle between A and B is ...... .


     

(A) 0° (B) 30° (C) 90° (D) 120°

(11) The unit vector, which is perpendicular to A and B both is ...... .


 

o o o o o o o o
AB Au B Au B A˜ B
(A) (B) (C) (D)
AB ABsin T ABcos T ABsin T
(12) If 2
3
i + 1 j + m k is unit vector, then m =
3
...... .
(A) (B) – 13 (C) 1 (D)
2 1
3 3
o o
(13) If | A | = 4, | B | = 2.5, A ˜ B = 8 and an angle between A and B is acute angle then
   

o o
| A u B | = ....... .
(A) 6 (B) 10 (C) 3.2 (D) 2

(14) Aproximate area of parallelogram fromed by A = 2 i – 3 j – k and B = 4 i + 3 k is ...... unit.


(A) 5 (B) 11 (C) 18 (D) 15



(15) The component of A = 3 i – 2 j + k in the direction of B = i + j is ...... .


(A) 1 (B) (C) (D)


1
2 2 14
Ans : 1 (C), 2 (B), 3 (C), 4 (D), 5 (A), 6 (B), 7 (D), 8 (B), 9 (D), 10 (D), 11 (B), 12 (A),
13 (A), 14 (C), 15 (C)

Mechanics
ˆ A branch of physics dealing with properties of a moving body, causes of motion, results due to
motion etc. is known as mechanics.
ˆ Mechanics devides in two branches.
ˆ Kinematics : A branch of mechanics dealing with motion without considering its causes is
known as kinematics.
ˆ Dynamics : A branch of mechanics describing motion along with its causes and properties
of moving body is called dynamics.
Concept of a particle and frame of reference :
ˆ A point like object having mass can be considered as a particle.
ˆ If the distance between two object is very large as compared to their dimensions, these objects
can be treated as particle.

21
ˆ All the particles of a solid body performing linear motion cover the same distance in the same
time. Hence, the motion of such a body can be described in terms of the motion of any of its
constituent particles.
ˆ Particle is relative concept.
ˆ Motion is also a relative concept.
ˆ Motion is described according to a place and a situation of an observer.
ˆ A place and a situation from where an observer takes his observation is called frame of reference.
Frame of reference is arbitrary.
ˆ Frame of reference will be inertial or non-inertial.
Position, Path length and Displacement :
ˆ To describe a motion and position of particle, we can choose three mutually perpeducular axis
name them X, Y and Z.
ˆ If all the three co-ordinates of a particle remain unchanged with time, the particle is considered
at rest with respect to time in this frame. If one or more co-ordinates of a particle changes
with time, we say that particle is in motion with respect to this frame.
Path length Displacement
ˆ The total distance travelled by a particle in ˆ The change in position of a particle in some
some time interval is called the path length time interval is called displacement.
or total distance.
ˆ Path length depends on path. ˆ Displacement does not depends on path. Its
depends only on initial and final position.
ˆ Path length is always positive. ˆ Displacement may be positive, negative or zero.
ˆ Path length is scaler quantity. ˆ Displacement is vector quantity.
ˆ In a normal condition, ratio of path length to displacement is one or more than one.
(16) In a joggers park, a man complete two rotation on a circular path of radius 30 m. Then
difference of path length and displacement is ...... m in given motion.
(A) 120 p (B) 60 p (C) zero (D) 30 p
(17) A person travelled a distance of 40 m along a straight line, then he travelled 30 m in right side.
So, the ratio of total distance to displacement is ......
(A) 1 (B) 1.33 (C) 0.75 (D) 1.4
(18) An ant experienced 60 cm displacement on a cicular path of 60 cm radius. Then total distance
travelled by it is ...... cm.

S
(A) (B) 20 p (C) 60 p (D) 120 p
3

Ans. : 16 (A), 17 (D), 18 (B)

Average speed and average velocity :

ˆ With help of average speed, we know how fast the position of object is changing.

ˆ The ratio of total distance travelled (i.e. path length) to the time interval during which the
motion has been taken place is known as average speed. Thus, distance travelled by object in
unit time is known as average speed.

22
Total travelled distance Path length
average speed = <v> = v = =
time interval time interval
ˆ Average velocity is the ratio of displacement to the time interval, in which the displacement
occurs.Thus, the displacement covered by object in unit time is known as average velocity.

displacement
average velocity =
time interval

ˆ Unit of average speed and average velocity is ms–1.


ˆ Average speed is scaler while average velocity is vector physical quantity.

(19) A vehicle travels different distances with different speeds in the same direction. The average
speed of the vehicle is ......
v1  v2  v3  .....
(A) v1+ v2+ v3 + .... (B) t  t  t  ....
1 2 3

d1 d2 d3 d1  d 2  d3  ....
(C) t + t + t + ..... (D)
1 2 3 d1 d 2 d3
   ....
v1 v2 v3

(20) A cyclist travelled third part of total distance with a speed of 12 kmh–1 and remaining distance
with a speed of 18 kmh–1. so, average speed of his is ...... kmh–1.
(A) 15 (B) 12 (C) 18 (D) 15.43
(21) A person wallking in a straight line, covers 13 part of the distance to be travelled with a speed of
v1 and remaining distance with speed v2. So, avrage speed of his is ......
3v1 v2 3v1 v2 2v1 v2 2v1 v2
(A) v  2v (B) 2v  v (C) v  2v (D) 2v  v
1 2 1 2 1 2 1 2
(22) A car covered 120 km straight line disance in 2 hour, then it terned left and travelled 50 km
distance in 1 hour. So average speed and average velocity during this is ...... and ......
respectively.
(A) 40 kmh–1, 50 kmh–1 (B) 56.67 kmh–1, 43.33 kmh–1
(C) 43.33 kmh–1, 56.67 kmh–1 (D) 50 kmh–1, 40 kmh–1
(23) A length of second arm of a clock is 10 cm. This second arm is moves to 9 from 3. Average
speed and average velocity of this second arm is ...... cms–1 and ...... cms–1 respectively.
(A) 3.33, 3.33 (B) 1.05, 0.6666 (C) 1.111, 3.00 (D) 1.666, 3.33

Ans. : 19 (D), 20 (D), 21 (B), 22 (B), 23 (B)


Instantaneous velocity and Instantaneous speed :
ˆ Instantaneous velocity is obtained by time derivative of position.
ˆ Instantaneous velocity is vector physics quantity and its direction is in direction of tangent
of path.

23
ˆ Magnitude of instantaneous velocity is instantaneous speed.
ˆ In a graph of x ® t, instantaneous speed is slope of tangent at given point of path.

(24) A motion of object is expressed by x(t) = 2t2 – 3t + 4 m. Average speed of object for initial 3 s
and instantaneous speed at t = 3 s is ...... and ...... respectively.

(A) 3 ms–1, 9 ms–1 (B) 9 ms–1, 9 ms–1

(C) 4.5 ms–1, 3 ms–1 (D) 4.5 ms–1, 4.5 ms–1


(25) An equation of motion for a particle is given by v = 2t + 3 ms–1. So, distance travelled by particle
in initial four second is ......
(A) 11 m (B) 3 m (C) 19 m (D) 28 m

(26) An object is moving on a circular path with velocity v at a given instant. When it completes half
rotation change in its velocity will be ......

(A) zero (B) – v (C) –2 v (D) 2 v
 

(27) Motion of an object in a straight line is given by x = 5t2 – 6t + 2.2 m. The position of the object
when its velocity is zero is given by ...... m.
(A) 0.6 (B) 0 (C) 0.4 (D) 1.0
Ans. : 24 (A), 25 (D), 26 (C), 27 (C)
ˆ Acceleration :
ˆ The time rate of change of velocity of moving body is called acceleration.
change in velocity
Average acceleration =
time interval
a =<a>=
' v
't
o
o
Instantaneous acceleration a = d v = time derivative of velocity = v

dt

o o
d2 x
= = Second derivative of position with respect to time = x

dt 2
(28) A motion of particle is given by x(t) = 4t2 + 4t –3 m. So, instantaneous acceleration of particle
for 4th second will be ...... ms–2.
(A) zero (B) 8 (C) 4 (D) 16
(29) For a moving particle, the relation between time and position is given by t = 4x2 + 3x. Where t is in
second and x is in meter. So, acceleration for this particle a = ...... . (as a function of velocity).
(A) 12v (B) 8v2 (C) –8v3 (D) –12v2
(30) For moving particle, If x = At2 + Bt + C then equation for velocity ...... and quation for
acceleration ...... .
(A) 2At + B, 2A (B) 2t + B, 2B (C) At + B, 2A (D) At2, Bt

24
(31) The velocity of a particle becomes 9 i + 2 j ms–1 from (4 i – 3 j ) ms–1 in 5 second. Then
average acceleration is ...... ms–2.
(A) i + j (B) 13 i – 5 j (C) 5 i – 5 j (D) i – j
(32) An acceleration of moving particle is a = 4t ms–2. So, velocity at third second will be ...... ms–1.
(A) 12 (B) 18 (C) 36 (D) zero
Ans. : 28 (B), 29 (C), 30 (A), 31 (A), 32 (B)
Kinetic equations for uniformly acceleration motion :
At t = 0 position of particle x0 = 0 then,
at t = t velocity of particle is v = v0 + at.

displacement of particle is x = v0t + 12 at2

§ v  v0 ·
x = ¨ 2 ¸t
© ¹

v 2  v02
x=
2a
In general, at t = 0, position of particle x = x0 then
v = v0 + at

x – x0 = v0t + 12 at2

§ v  v0 ·
x – x0 = ¨ 2 ¸ t
© ¹

v 2  v02
x – x0 =
2a
A distance travelled in nth second by a perticle, which moves with constant acceleration and v0 as
initial velocity.

dnth = v0 + 2 (2n –1)


a

(33) A car which stands at traffic signal, starts its motion with acceleration 5 ms–2. Then velocity of
car after 4 s and distance travelled by car during this 4 s will be ...... and ...... respectively.
(A) 40 ms–1, 20 m (B) 20 ms–1, 20 m (C) 40 ms–1, 40 m (D) 20 ms–1, 40 m
(34) The ratio of the distances travelled in the fifth and the third second by a particle moving start
from rest over a straight path with constant acceleration 8 ms–2 is ......

(A) 5
3
(B) 9
5
(C) 10
3
(D) 5
9
(35) An object starts its constant accelerated motion with 1 ms–1 initial velocity and it covered same
distance in third second as it covered in initial two seconds. Then constant acceleration of object
during this motion is ...... ms–2.
(A) 1 (B) 5 (C) 2 (D) 3
(36) The velocity of an object moving with constant acceleration is 45 ms–1 at the end of 10 s and
becomes 53 ms–1 at the end of 12 second. Then the distance travelles by object in 15 second will
be ...... m.
(A) 65 (B) 82.5 (C) 110 (D) 525

25
(37) The object srarts from stationary, moves with constant acceleration 4 ms–2 for 10 s then it moves with
constant velocity for 50 s and finally it stops with 2 ms–2 deceleration. Total distance coverd by object
will be ......
(A) 1200 m (B) 1300 m (C) 2000 m (D) 2600 m

(38) In a constant acceleration motion, an object losing 23 v0 velocity in time t. Where v0 is initial

velocity. So time required to velocity becomes zero will be ...... .

(A) t (B) 2t (C) 2 (D) 3


3t 2t

(39) Speeds of on object moving with constant acceleration at point A and B are u and v
respectively. Then the speed of object at midpoint of A and B is ...... .

v2  u 2 vu v2  u 2
(A) (B) (C) (D)
v2  u 2
2 2 2 2
(40) An object starts its motion with 2 ms–1 velocity and acceleration given by a = 6t2 – 2t + 3 ms–2.
So, velocity of object at t = 3 s is ...... ms–1.
(A) 20 (B) 56 (C) 48 (D) 51
Ans. : 33 (D), 34 (B), 35 (C), 36 (D), 37 (D), 38 (C), 39 (A), 40 (B)

Kinetic equations for freely falling body :

ˆ For freely falling body v0 = 0 and a = –g.


upward direction ® Positive y-axis ® Positive.

downward direction ® negative y-axis ® negative.

ˆ For freely falling body,


v = – gt –2gy = v2

y = – 12 gt2 – y = 12 vt

(41) Two different substances are allowed to free falling from towers of height h1 and h2. Time taken

by them to reach on ground are t1 and t2 respetively. So t = ......


t1
2

h12
(A) h (B) (C) (D)
h1 h1 h2
2 h2 h2 2 h1

(42) An object is allowed to fall freely from a tower of 80 m height. Distance covered by it in last
second before touching ground is ...... m. (g = 10 ms–2)

(A) 4 (B) 45 (C) 35 (D) 10

(43) A freely falling body passed a building of height 25 m in last second. Then from ...... m height
that body will be allowed to free fall. (g = 10 ms–2)

(A) 125 (B) 100 (C) 85 (D) 45

26
(44) The time taken to reach on ground by a freely falling object from height h1 and h2 are t1 and t2

respectively then t = ...... .


t2
1

h 22
(A) h (B) (C) (D)
h2 h2 h1
1 h1 h12 h2

(45) An object is thrown from a tower of height h in vertically upward direction with speed v. So time
taken by object to touch a ground is ......
ª 2 hg º
(A) 1+ (B) g 1+ 2 (C) g 1– 2 (D) g «1+ 1+ 2 »
2g h v hg v 2 hg v
v2 v v «¬ v ¼»
(46) A stone is dropped from a balloon, starts its motion with 4 ms–2 in vertically upward direction
after 5 s. So, minimum height from ground obtain by this stone is ...... m. (g = 10 ms–2)
(A) 55 (B) 70 (C) 90 (D) 105
(47) An object is thrown from a tower of height H in vertically upward direction with velocity v0,
when it touch the ground its velocity becomes 2v0. then H = ...... .
3 v02 3 v02 v 02
(A) 2 g (B) (C) 2 g (D) 2 g
v0
g
(48) An object is allowed to fall freely from a tower of hight 150 m, exactly at the same time another
object is thrown from the bottom of the tower in vertically upward direction with velocity of
30 ms–1. So, two object will be meet after ...... time.
(A) 10 s (B) 45 s (C) 5 s (D) 12 s
(49) A ball is allowed to fall freely from 10 m height from a surface. The ball reaches at 5 m height
after colliding with surface. If time of contact for ball and surface is 0.05 s then average
acceleration during contact is ...... . (g = 10 ms–2)
(A) 241.4 ms–2 (B) 965.2 ms–2 (C) 4828 ms–2 (D) 482.8 ms–2
Ans. : 41 (B), 42 (C), 43 (D), 44 (B), 45 (D), 46 (B), 47 (A), 48 (C), 49 (D)
Relative velocity :
ˆ velocity of P with respect to A =vPA, velocity of P with respect to B = vPB, velocity of B with
respect to A = vBA.
According to this vPA = vPB + vBA (B will be eliminated.)

ˆ Thus, velocity of P with respect to A is obtained by addition of velocity of P with respect to B and
velocity of B with respect to A.
ˆ Above matter can be expressed as below :
vBA = vPA – vPB
= –vAP – (–vBP)
= vBP – vAP (P will be eliminated) vBA = vB – vA

Thus relative velocity vBA of moving body B with respect to moving body A will be obtained by
subtraction of vBP (velocity of moving body B with respect to any stationary object or point P) and
vAP (velocity of moving body A with respect to any stationary object or point P.)
Stationary object means reference object or point will be taken any object or point.

27
(50) A truck passes through a traffic signal with a constant speed 15 ms–1 while at same instant, a car
start its motion with constant acceleration 4 ms–2 in the direction of truck’s motion. So, relative
velocity of car with respect to truck after 6 sec is ...... .
(A) 9 ms–1 (B) –9 ms–1 (C) 39 ms–1 (D) –39 ms–1
(51) Car A is moving on a road with constant speed 54 kmh–1. Now car B and car C are moving with
72 kmh–1 in mutually opposite direction on the same road. At any cretain time (instant), distance
of car B and car C from car A is 1.4 km, driver of car B think to overtake car A. So,
acceleration of car B will be ...... .
(A) 1 ms–2
Car B Car A Car C
(B) 1.5 ms–2
(C) 2 ms–2
(D) –1 ms–2

(52) The rain is falling verticully downward with speed of 12 kmh–1. A man running with 5 kmh–1 on a
straight way. velocity of rain experience by man will be ...... .
(A) 7 kmh–1 (B) 13 kmh–1 (C) 17 kmh–1 (D) 119 kmh
–1

(53) The speed of boat is 5 kmh–1 in still water. It crosses a river of width 1.0 km along the shortest
possible path in 20 min. Then, speed of river will be ...... kmh–1.
(A) 1 (B) 3 (C) 4 (D) 5
(54) A stationary man observes that the rain is falling vertically downward. When he starts running
with a velocity of 10 kmh–1, he observes that the rain is falling at an angle 30° with the vertical
direction. The actual velocity of rain is ...... .

(A) 10 3 kmh–1 (B) kmh–1 (C) 5 3 kmh–1 (D) 15 kmh–1


10
3

Ans : 50 (A), 51 (B), 52 (B), 53 (C), 54 (A)

Relative displacement :
At t = 0, relative positions of particle A and B with respect to reference point (origin) are xA0 and
xB0 and relative velocities of particles A and B with respect to reference point are vA and vB. So,
relative displacement of particle B with respect to particle A at t = t will be,
xB – xA = (xB0 – xA0) + (vB– vA) t
xBA vBA
¿
°
¾
°
½

¿
°
¾
°
½

­ ­

Position of particle B with relative velocity of particle B


respect to particle A at t = 0 with respect to particle A.

(55) At t = 0, car A and car B are at 50 m and 130 m away from origin respectively. Both car starts
their motion in same direction with constant velocities 15 ms–1 and 7 ms–1 respectively. When and
where they overtake each other ?
(A) 10 s, 200 m (B) 15 s, 170 m (C) 8 s, 180 m (D) 12 s, 150 m

28
(56) When particles A and B move uniformly towards each other, they get 16 m closer in 2 second
and when they move uniformly in same direction with original speeds they get 16 m closer in 8
second. So, the speeds of these particles will be ...... and ...... .
(A) 5 ms–1 and 3 ms–1 (B) 8 ms–1 and 0.5 ms–1
(C) 4 ms–1 and 4 ms–1 (D) 32 ms–1 and 2 ms–1
Ans. : 55 (A), 56 (A)
Motion in one, two and three dimensions :
Motion in one motion in two motion in three
dimension dimension dimension
Position x r = xi + y j r = xi + y j + zk
    
 

o o o o
displacement Dx = x – x0 ' r = r – r0 ' r = r – r0
 

= (x – x0) i + (y – y0) j = (x – x0) i + (y – y0) j


+(z – z ) k0
o o
| v | = |' r | | v | = |' r |
o o
average speed <v> = ' t
'x
't 't
o o
Instantaneous v = dt v = dt v = dt
dx  dr  dr

velocity
o
o o
o
average <a> = ' t 'v 'v
 a ! = 't  a ! = 't
'v

acceleration
d2x
o o d2
Instantaneous a = dt = 2 = dv = d 2r a = dt ( v ) = dt 2 ( r )
dv  2  d  
dt a dt
acceleration
dt

 
Kinematic v = v0 + at v = v0 + a t v = v0 + a t
   

equations of x = v0t + 12 at2 d = v0 t + 2 a t d = v t + 12 a t2


 
2
  1 

§o o·
0
§o o·
§ v  v0 · ¨ v  v0 ¸ ¨ v  v0 ¸
uniformly accel- x = ¨ 2 ¸t d = ¨ 2 ¸t = ¨ 2 ¸t
 
© ¹ ¨ ¸ ¨ ¸
d
© ¹ © ¹
ereted motion

2ax = v2 – v02 2 a . d = v2 – v02 2 a . d = v2 – v02


   

ˆ Dircetion of instantaneous velocity is in a direction of tangent drawn at given point of path.

Instantaneous velocity in two dimensions is v = vx i + vy j , then an angle made by instantaneous



ˆ
vy
velocity with x-axis is obtained by equation. tan q = v
x
ˆ Velocity is vector physical quantity, hence it can change in three ways :
(1) Magnitude of velocity will change. (In a direction of velocity)
(2) Direction of velocity will change. (In a perpendicular direction to velocity)
(3) Magnitude and direction both will change. (In direction and perpendicular direction of velocity)
ˆ Acceleration is result of change in velocity, hence it results three different ways.
ˆ If only magnitude of velocity changes then acceleration is parallel to velocity. It known as a||.

29
ˆ If only direction of velocity changes then acceleration is perpendicular to velocity. It known as a^.
ˆ If magnitude and direction of velocity both changes then acceleration will be parallel and
perpendicular to velocity. so,
o o o o
a = a & + aA Here a and a A are mutually perpendicular to each other.

&

o
\|a|= a&2  aA 2

ˆ During uniform circular motion, only direction of velocity changes. The direction of acceleration is
perpendicular to the direction of velocity, it means in the direction of centre of circular path. That’s
why this acceleration is called Radial acceleration or Centripetal acceleration (ar or ac.)

ar = ac = v
2
r

(57) The position vector of a moving particle depends on time according to r = at2 i + (bt – 3) j m,


then magnitude of velocity and magnitude of acceleration at time t will be ...... and ......
respectively.
(A) at2 + bt – 3, at2 (B) 2at + b, 2a (C) 4D 2 t 2 + E2 , 2a (D) 2at + b – 3, 2at
(58) Ships A and B are standing 20 km far from each other on a line joining north south. Ship A
moves with 10 kmh–1 in west direction and ship B moves with 10 kmh–1 in north direction. Then
minimum distance between these two ships will be ...... km obtain after ...... minute.

(A) 10 2 , 60 (B) 20 2 , 60 (C) , 15 (D) , 15


10 20
2 2
(59) A particle starts its motion from the origin with velocity 3 j ms–1 and moves in xy plane with


uniform acceleration 2 i + j ms–2. When the value of x co-ordinate is 25 m, then the value of
y co-ordinate will be ...... m.
(A) 25 (B) 54 (C) 27.5 (D) 55
Ans. : 57 (C), 58 (A), 59 (C)

Projectile motion
When an object is thrown in gravitational field of earth it moves with constant horizontal velocity and
only gravitational acceleration. Such two dimensional motion is called a Projectile motion.
For projectile motion,
Horizontal direction Vertical direction
v0x = v0cos q0 = constant v0y = v0 sinq0
ax = 0 ay = –g
Position at time t, x = (v0cosq0)t Position at time t, y = (v0sin q0)t – 1 gt2
2

velocity at time t, vx = v0 cosq0 velocity at time t, vy = v0 sin q0 – gt

Position of projectile at time t, r = (v0 cosq0)t i + [(v0 sinq0)t – 1 gt2] j



ˆ 2

velocity of projectile at time t, v = v0 cosq0 i + (v0 sinq0 – gt) j




30
Equation of trajectory of a Projectile, y = (tanq0)x – x2
g
ˆ
2(v0 cos T0 )2

v 0 2 sin 2 T 0
ˆ The maximum height (H) reached by the projectile, H =
2g

v0 sin T 0
ˆ Time taken to achieve maximum height by the Projectile, tm = g

2 v0 sin T 0
ˆ Time of flight for Projectile, tF = 2tm = g

2 v0 2 sin T 0 cos T 0 v0 2 sin 2 T 0


ˆ Range of a projectile (Distance travelled in horizontal direction) : R = =
g g

2
Maximum Range Rmax = 0g (For given v0, q Should be 45° for maximum Range.)
v
ˆ

ˆ Relation between range and maximum height for projectile tan q0 = 4RH

ˆ For q and 90° – q, ranges covered by projectiles are equal for given v0.
ˆ The ranges covered by projectile at 45° + a and 45° – a are equal.

(60) Two objects are projected with the same velocity at different angles with the horizontal and if the
range is same for both of them. It t1 and t2 are their time of flights then t1. t2 = ......

(A) 2 Rg (B) 2 Hg (C) (D) 2 g


2R R
g

(61) An object is projected at angle of 30° with the horizontal, with kinetic energy K. Its kinetic
energy at maximum height is ......

(A) 0 (B) 4 (C) (D)


3K K 3K
2 2

(62) For a projectile motion y(t) = 12t – 5t2 and x(t) = 5t. Where x and y are in meter and t in
second. So initial velocity ......
(A) 6 ms–1 (B) 12 ms–1 (C) 5 ms–1 (D) 13 ms–1
(63) Bullets are fired with the same initial velocity v in different direction on a plane surface. These
bullets would fall on the maximum area of ...... on this surface.

Sv 2 S 2v 2 Sv 4 Sv 2
(A) (B) (C) (D)
g g2 g2 g2

(64) In a projectile motion, if the maximum height H is one-forth of the range (R), then angle of
projections with horizontal is ......
(A) zero (B) 30° (C) 45° (D) 60°

Ans. : 60 (C), 61 (B), 62 (D), 63 (C), 64 (C)

31
Graphs :
ˆ Graph is always y ® x.
ˆ Shape of graph can be decided by power of y and x in a equation express in terms of y.
– If both have a power 1-1 then graph is straight line, otherwise different shape for different
power. (generally they are considered as a parabolic).
ˆ For a straight line,
– Co-efficient of x (with sign) gives a slope of graph when equation is expressed in terms of Y.
– To obtain a segment on Y-axis, put x-co-ordinate zero and get the value of y.
– To obtain a segment on X-axis, put y-co-ordinate zero and get the value of x.
ˆ If graph is strait line, then equation of line will be y = mx + c.
Where m = slope of line and c = segment on y axis.
ˆ From a graph, a physical quantity which is obtained by product of x and y-axis will be the area
under the curve and x-axis.
ˆ From a graph, a ratio of y to x is obtained by a slope of curve at a point.

Distance ® time velocity ® time Important-point


graph graph

Uniform distance (d) velocity (v) (1) d = 0 at t = 0, in graph of


motion displacement ® time.
(motion with (2) Slope of graph of displacement ® time
v = constant
d = vt
constant = velocity = constant
velocity) 0
(3) Graph of velocity ® time is parallel
0 time (t) time (t)
to time x-axis, so slope = 0
it means acceleration = 0
Constant distance (d) velocity (v) (1) Graph of displacement ® time is
acceleration Parabolic
motion v = at (2) Slope of velocity ® time graph
(i) initial velocity = acceleration = constant
1 2
d= at
2
and initial 0 (3) Initial velocity and initial position Q is
0 time (t)
time (t)
position is zero zero, So graph starts from origin.

distance (d)
velocity (v) (1) Here initial position is zero, so graph
(ii) Initial of displacement ® time starts
d = v0t + at
1 2
velocity ¹ 0 2 v = v0 + at from origin.
Initial v0 (2) Initial velocity is not zero, so graph
position = 0 time (t) of velocity ® time does not start from origin
0 time (t)
(3) Slope of velocity ® time graph
= acceleration = constant.

32
distance (d) velocity (1) Initial position and Initial velocity are
(v)
(iii) Initial non zero so, graphs do not pass

at
velocity ¹ 0 through origin.

2 a2

+
t
t+ 1

0
v
v0

=
Initial r0 (2) Slope of velocity ® time

v
+v
0
0
r
=
position ¹ 0 0 = acceleration = constant

d
0 time (t) time (t)

constant dece- distance (d) velocity (v)


lerated motion (1) Slope of graph velocity ® time is
(i) Till final v0 constant but negative
0
=
velocity (2) Final velocity becomes zero, So
v

becomes zero 0 t0 motion will be posible till t = t0.


0
time (t)
t = t0 time (t)

distance (d) velocity (v)


(i) It final (1) Slope of graph velocity ® time is
velocity v0
constant but negative.
becomes (2) Final velocity not become zero,
negative So motion will continue after t = t0
0 t0 time (t)
0 t0 time (t)

(65) Figure shows a graph of velocity ® time for any particle. It is clear from graph that ...... on
particle in part B.
velocity (v) (A) Force act in direction of motion.
(B) Force act in opposite direction of motion.
(C) Force will be zero
D
B (D) nothing is clear about force.
A C

0 ® time (t)
(66) A graph of displacement ® time makes 30° angle with time axis. After 2s it makes 60° angle
with time angle. So, average acceleration during this motion is ......
1
(A) 3 (B) 1 (C) 2 3 (D)
3

(67) Figure shows a velocity ® time graph for a upward going lift. So, at ...... m height lift becomes
stationary.

velocity (v) ms–1 (A) 12


(B) 32
(C) 44
(D) 24
4

0 3 8
time (t) s

Ans. : 65 (C), 66 (D), 67 (B)

33
Assertion - Reason type Question :
Instruction : Read assertion and reason carefully, select proper option from given below.
(a) Both assertion and reason are true and reason explains the assertion.
(b) Both assertion and reason are true but reason does not explain the assertion.
(c) Assertion is true but reason is false.
(d) Assertion is false and reason is true.
(68) Assertion : Speed will be changed without changing velocity of object.
Reason : When velocity of object is zero then acceleration of it may not be zero.
(A) a (B) b (C) c (D) d
(69) Assertion : An object becomes stationary for a moment while the direction of motion will
changed.
Reason : At given instant, If velocity of object is zero then acceleration of object is also zero.
(A) a (B) b (C) c (D) d
(70) Assertion : The v ® t graph perpendicular to time axis is not possible in practice.
Reason : Infinite acceleration can not be realized in practice.
(A) a (B) b (C) c (D) d
(71) Assertion : Magnitude of average velocity is equal to average speed if velocity is constant.
Reason : If velocity is constant, then there is no change in the direction of motion.
(A) a (B) b (C) c (D) d
(72) Assertion : Particle A is moving northwards and partide B eastwards with same speed. Then,
velocity of A with respect to B is in southeast direction.
Reason : Relative velocity between them is zero as their speeds are same.
(A) a (B) b (C) c (D) d
(73) Assertion : In the v ® t diagram shown in figure,
average velocity between the interval t = 0 and v

t = t1 is independent of t1. vm

Reason : Average velocity for given interval is 2 .


vm

(A) a (B) b
(C) c (D) d
t
Match the columns : t=0 t = t1

(74) Find the appropriate option from column 2 for column-1, when motion is described by
d = 3 + 8t – 4t2.
column-1 column-2
(a) Initial acceleration (p) –16 unit (A) a ® s, b ® r, c ® q, d ® p
(b) velocity at end of 3 s (q) 3 unit (B) a ® s, b ® p, c ® q, d ® r
(c) distance travelled in 2 s (r) 7 unit (C) a ® q, b ® r, c ® s, d ® p
(d) displacement at 1 s (s) –8 unit (D) a ® q, b ® r, c ® p, d ® s

Ans. : 68 (A), 69 (C), 70 (A), 71 (A), 72 (C), 73 (A), 74 (B)


ˆ
34
5 Laws of Motion
ˆ Force is required to starts the motion of static body, to change in motion and to stop the body.
Acoording to external factor force is divided into two parts :

If external factor applies force remaining If external factor applies force without
in contact with a body such force is contact of a body then it is called field force.
called contact force. e.g. Gravitational force
e.g. Frictional force electrical force
magnetic force.

ˆ According to Galileo
(i) Static position and position with uniform motion both are equal, because no force is required
for that.
(ii) Body it self can not change the position of motion. This is called "Property of Inertia".
(iii) Mass of a body is measurement of Inertia.
Three types of Inertia

Inertia of Staticness Inertia due to motion Inertia due to direction

Due to this body maintain Due to this body maintain Due to this body maintain
its steady position its state of motion. its direction of motion.
e.g. e.g. e.g.
(1) A person pushed in back (1) A person pushed in (1) When a car suddenly
ward direction when forward direction when takes a turn passenger
stationary bus suddenly break is applied in pushed outword.
starts. moving bus.
(2) If bullet fires on the (2) A player of long jump (2) When a stone tied
glass of window it runs before taking with a string and
makes a hole but when jump. rotates circularly, if
stone throws on the string breaks stone
glass it breaks whole moves towards the
glass. direction of tangent.
(3) When pillow hit by
stick dust particle goes
down ward
Newton's First law of motion
ÒÒUnless external force acting on a body steady body remains steady and body in motion moving with
constant velocity.ÓÓ

35
So,
(i) First law is actual law of inertia given by Galileo.
(ii) First law gives the definition of force but does not explain about its value.

(iii) Acceleration in a body produced by force. So, S F = 0 then a = 0 and v = constant


  

ˆ First law depends on frame of reference


Two types of frame of references

Inertial frame of reference Non-inertial frame of reference

ˆ It follows Newton's First law. ˆ It does not follow Newton's first law so it
So, F = 0 then a = 0 is not essential that F = 0 then a = 0
   

ˆ In such a frame of reference, for a solution


of problem regarding to motion, Acceleration
of body is taken in opposition direction of
motion, which is equal to acceleration of
frame of reference, called pseudo acceleration
and corresponding force is called pseudo force.

(1) In a bus, going from Ahmedabad to Baroda when suddenly break applies, passenger pushed in
forward direction because...
(A) Some one pushed passenger from backside.
(B) Passenger affraid and starts to run forward.
(C) Due to Inertial upper part of his body is moving with speed of bus and his legs sticks to the
bus.
(D) Due to inertia upper part of his body remains stationary and his legs pushed in backward.
(2) A passenger seating in upper seat of a train moving with velocity of 54 kmh–1 throws a pen in
vertical direction on a passenger seating exactly in lower seat, them this pen...
(A) Falls on head of passenger seating lower.
(B) Falls on front side of passenger seating lower.
(C) Falls on back side of passenger seating lower.
(D) Where pen fall, can not be said.
(3) As Shown in figure a bucket filled 10 litre water hanged with the help of string from point P
(i) If a string suddenly pulled from point R P
(ii) If a string pulled slowly from point R... Q

(A) In first case PQ part of string and in second case SR part of String breaks.
(B) In frist case SR part of String and in second case PQ part of string breaks.
(C) In both the cases PQ part breaks. S
(D) In both the cases SR part breaks. R

36
(4) According to Aristotal concept, to stop the cycle slowly which is moving with constant velocity on
the road...
(A) Force should be applied slowly in the opposite direction of cycle's motion.
(B) It is essential to decrease the force with slow rate in the opposite direction of cycle's motion.
(C) There is no need to apply the force.
(D) The force should be applied perpendicular to the direction of cycle's motion.
(5) Inertia of body is measured with the help of which physical quantity ?
(A) Mass (B) Force (C) Momentum (D) Acceleration
(6) For the graph of speed time, for a substance having mass 22 kg moving in the absence of
external force which one is suitable ?
v
(A) v (B)

t t
v
(C) v (D)

t t
(7) A person sitting on a running horse falls in fornt direction when horse suddenly stops, because.
(A) It is intertia of motion of a person. (B) It is inertia of direction of a person.
(C) It is inertia of motion of a horse. (D) It is inertia of direction of a horse.

Ans. : 1 (C), 2 (A), 3 (B), 4 (C), 5 (A), 6(D), 7 (A)

Newton's second law of motion


ÒÒThe time rate of momentum of a body is directly proportional to resultant external force and it is in
the direction of external force.ÓÓ
Resultant external force µ Time rate of change of momentum.
o
dp
So,

F µ
dt

o
dp
\F =k


dt

37
Unit of F is defined in such a way, so that value of proportionality constant k = 1.


o
.... (1)
p
\F = d


dt

but, p =mv


d
\F = (m v )
 

dt
o
.... (2)
dv
\F =m + v
  dm

dt dt
eqution (2) is more general form. In practise it is taken as special cases


If m is constant If ( v ) is constant,

F = m a ... (3) F = v ... (4)


    dm
dt

e.g. In case of Rocket, For conveyor belt...


Newton's second law
(i) gives measurement of force and defined force only quantitively.

(ii) In eqn - (3) F = 0 then a = 0. (i.e. v = constant) which matches with Newton's first law.
  

(iii) Acceleration of a particle at any moment a at any point is decided by force F acting on that
 

point, at that moment.


(iv) Force depends on time - rate of change of momentum not on momentum.
(v) When more than one force are acting on a body, ‘F’ Shows resultant external force and ‘a’
shows acceleration of centre of mass.
(vi) For a graph of p ® t, slope at any point gives value of force at that time.
Slope = tan q = Force acting on a particle at time t p
ˆ SI unit of force = newton (N)
CGS unit is dyne
ˆ gravitational unit of force : kg wt (Kilogram weight) or kgf (Kilogram force)
1 kgf = 9.8 N
t
q
Cases of variable mass : t

ˆ Rocket's motion :
ˆ In case of Rocket, mass of the system does not remain constant, it varies.
ˆ In a rocket, during the combustion of fuel, gas is ejects from nozzel, which applies force on a
rocket according to Newton's third law.

38
ˆ Thrust acting on a rocket at any moment,
F = – v dm – mg where v = velocity of a gas with respect to rocket.
dt

dm = Rate of change of mass of rocket due to combustion of fuel.


dt

by neglecting the effect of acceleration due to gravity, F = – v dm


dt

Speed of rocket at time t is Where m0 is a mass of rocket at t = 0


§m ·
vR = v ln ¨ 0 ¸ – gt m is a mass of rocket at time t
© m ¹

by neglecting the effect of acceleration due to gravity,


§m ·
vR = v ln ¨ 0 ¸
© m ¹

ˆ Burnt out Speed (vb) Ñ When all the fuel of rocket combusted (burnt), the final speed attains by
rocket, is called burnt out Speed (vb).
m0
vb = v ln ma Where ma = mass of frame of rocket without Fuel.

(8) A block of mass 1500 g is moving with speed of 30 ms–1. 12 N Force is acting in the direction of
motion and 5 N force is acting in the perpendicular direction for 3 s. The speed of a block after
3 s is ...... ms–1.
(A) 56 (B) 0.56 (C) 0.056 (D) 560
(9) A substance of mass 5 kg is moving in Y direction, Force F = kt2 is acting in the direction of
motion. Where k = 15 s–2. The distance travelled by substance in first two sec is ...... m and
speed after two sec is ...... ms–1.
(A) 3, 6 (B) 6, 3 (C) 8, 4 p (D) 4, 8
(10) The graph of momentum à time is as shown in figure, for
a substance. The ratio of force acting on it at t = 3 s and 20
during the first three sec is ......
(A) 1 : 1 (B) 3 : 2
(C) 2 : 3 (D) 1 : 2 0 1 2 3 t

(11) A block of mass 60 g tied with the help of string of mass 30 g and length 30 cm as shown in
Figure. If force acting at point ‘P’ is 1800 dyne, then tension produced at a distance of 10 cm
form point P is ...... N.

60 g P

(A) 1600 (B) 16 (C) 0.16 (D) 0.016

39
(12) A body of mass 30 kg is moving with velocity 20 ms–1 in north direction, making an angle of 30°
with east, the force of 150 N is acting on it in south direction. Find the magnitude and direction
of velocily after 5 s.

(A) 45 ms–1, at an angle of tan–1 with east


5
3

(B) 45 ms–1, at an angle of 60° with west toward north


(C) 22.9 ms–1, angle of 41° with South.
(D) 22.9 ms–1, at an angle of 41° with east towords north.
(13)

90°
5 kg

kg

°
45
5
(a) (b)
As shown in Figure (a) block of mass 5 kg is hanged to spring balance on a stand attached to a
two perpendicular wooden slabs. When an angle decreases to half of initial as shown in
figure (b) change in the observation of spring balance is ......
(A) 50 N (B) Zero (C) 25 N (D) 14.65 N
(14) A body of mass 10 g is moving in one dimension, its momentum changes according to time as
relation p = a + bt2 where a = 3 Ns–1 and b = 2 Ns–2. Find the instanteneous force at t = 3 s and
average force in t = 3 s ?
(A) 0, 12 N (B) 6 N, 12 N (C) 12 N, 6 N (D) 0, 6 N
(15) A child releases freely a ball of 150 g from the 20 m high tower, other child standing on the earth
hits it with a bat. Hence this ball, reach again to first child, If the contact between bat and ball is
0.1 s, then the force acting on a bat by ball is ......
(A) Zero (B) 20 N (C) 30 N (D) 60 N
(16) The graph of force acting on a body versus time is given as F
shown in Figure. If mass and initial velocity of a body is 1 kg 30
25
and 30 ms–1 respectively. What is the velocity of a body at t = 4 s. 20
15
(A) 70 (B) 30
10
(C) 40 (D) 100 5

0 1 2 3 4 5 t
Ans. : 8 (A), 9 (D), 10 (B), 11 (D), 12 (C), 13 (D), 14 (C), 15 (D), 16 (D)
Impulsive force :
When a force acting on a substance for very small duration then it is called impulsive force.
ˆ This force changes with time.
ˆ It changes in a small duration, so difficult to measure. In such a case we measure total effect
during the whole period, so it is called impulse or impulse of force.

40
o

F =
 'P
't

\ F . Dt = D p


Where F . Dt = impulse of force which is equal to change of momentum.




Area enclosed by graph of F ® t Shows impulse of force.


Newton’s third law :
“Action and Reaction are always equal and mutually in opposite direction.”
ˆ According to Newton’s third law,
(i) Forces always produced in pair.
(ii) In a pair any one is considered as action and other as reaction.
(iii) Action and reaction are equal and in opposite direction but they are acting on a different
body so their resultant force is not zero.
(iv) Action and reaction produced at the same instant.
(v) When we are discussing motion of some substance, we have to consider the force acting
on it by others, not acting on others by it.

(17) A ball of mass 100 g collides with a vertical wall at an angle of 45° reflects perpendicularly with
its original direction of motion. In this collision ball losses 50 % of its velocity, the change in
momentum of ball is ...... Ns. Initial velocity of ball is 20 ms–1.
(A) 5 (B) Zero (C) 5 (D) 3

(18) A substance is moving with a velocity of 16 ms–1 under the influence of resistive force. After
4 s its velocity becomes 4 ms–1. If the mass of substance is 2 kg the average resistive force
acting on it is ......N.
(A) 12 (B) 8 (C) 6 (D) 4
(19) A Swimmer of mass 60 kg jumps from height of 5 m in swimming pool. When it comes in
contact with the surface of water, its velocity becomes zero in 0.4 s. The average resistive force
is ...... N.
(A) 1000 (B) 1500 (C) 2000 (D) 2500
(20) A passenger in a airport applies force of 50 N at an angle of 60° with the horizontal on a
trollybag of mass 40 kg. If the surface of airport is frictionless find the acceleration of trollybag.
(A) 1.25 ms–2 (B) 25 3 ms–2 (C) 0.625 ms–2 (D) 25 ms–2

(21) Force acting on a body is given by F = (1200 – 4 × 105t) N. After starting the motion to it moves
with constant velocity, how much impulse of force is acting on it ?

(A) Zero (B) 0.9 Ns (C) 1.8 Ns (D) 3.6 Ns

41
(22) A servant of fire-brigade spraying water at the rate of 30 cm3s–1 from the pipe of 1 cm
diameter to control the fire. what is the force of reaction acting on him ?
(A) 30 N (B) 3 N (C) 0.3 N (D) 0.03 N
(23) Sand is thrown at the rate of 4 kgs–1 on a belt moving horizontal in a automatic machine. How
much force is required so that belt is moving with constant velocity of 5 ms–1 ?
(A) 0 (B) 5 N (C) 20 N (D) 100 N
(24) A Rocket arranged vertically having total mass with fuel is 10,000 kg and gas ejected with
–1
velocity of 108 × 102 kmh–1 from it. If the rate of combustion is 50 kgs what is its initial
acceleration ?
(A) 0 (B) 10 ms–2 (C) 15 ms–2 (D) 20 ms–2
(25) An object is falling with constant rate on a upper part of conveyor belt. The force required to
move it with constant speed of 3 ms–1 is equal to the force needed to move a car of mass 50 kg
with an acceleration of 0.3 ms–2. Find the rate of falling an object on conveyor belt ?
(A) 5 kg s–1 (B) 4 kg s–1 (C) 3 kg s–1 (D) 2 kg s–1
(26) A goods train full of coals moving on a railway track, due to opening of a door of one wagon
coals falling outside with the rate of Dm in time Dt. To maintain the constant velocity of train as
‘v’, how much motion resistive force should be applied ?

(A) Dm 't 'v (B) Dv ''mt 'm


(C) v 't (D) v Dm

(27) Mass of Rocket and its fuel 2000 kg and 18,000 kg respectively. Speed of gas ejected with
respect to surface is 7×103 ms–1 and speed of rocket with respect to surface is 6×103 ms–1. Find
the maximum speed of rocket ?
(A) 10 kms–1 (B) 9.8 kms–1 (C) 2.3 kms–1 (D) Zero
(28) Fuel of mass 14,000 kg is filled in a rocket of mass 21,000 kg. Combustion rate of 300 kg/s and
velocity of gas with respect to rocket is 1200 ms–1, then find the resultant force acting on a
rocket.
(A) 24 ×104 N (B) 12 ×104 N (C) 6 ×104 N (D) 1 ×104 N
(29) Water fills at the rate of 0.3 kgs–1 in wagons of goods train moving while raining. The force
acting on goods train by its engine is 30 N, then find the constant speed of goods train ?
(A) 120 ms–1 (B) 100 ms–1 (C) 60 ms–1 (D) 30 ms–1
(30) By pulling hairs one can not lift a whole body. why ?
(A) Due to fatigue.
(B) Feeling pain when one pulled hairs.
(C) due to oil hand slips.
(D) Actually while pulling the hair, applied force is internal force, whose resultant is zero.
(31) Newton’s third law shows conservation of ......
(A) Force (B) Energy (C) mass (D) momentum
(32) A person of mass 50 kg is standing on a spring balance, now he suddenly jumps from the spring
balance, then observation of spring balance ......
(A) Increases (B) Decreases
(C) First increases then decreases to zero (D) becomes zero

42
(33) A bird of mass 400 g is kept on a spring-balance in a cage. Observation of spring balance is
25 N when bird is stationary, if it is flying in upwards with an acceleration of 2.5 ms–2, the
instanteneous observation of spring balance is ...... .
(A) 24 N (B) 25 N (C) 26 N (D) 27 N
(34) What is the angle between the force of action on a bench by physics textbook kept on it and
force of reaction on a textbook by bench ?
(A) 0° (B) 90° (C) 180° (D) 360°
(35) Equal force of 8 N is acting on both the ends of a massless spring as shown in the figure, the
force of tension acting at any point on the spring is ......
8N 8N
(A) 4 N (B) 8 N (C) 12 N (D) 16 N
(36) When bullet fires from a gun, gun moves in backward direction. It supports Newlon’s which law
of motion ?
(A) First (B) Second
(C) Third (D) It is not related to Newton’s laws.
(37) A carpenter is fitting a nail of mass 20 mg and length 6 cm on a wall with the help of 2 kg
hammer. While collides with nail speed of hammer is 8 ms–1, nail enters half a way in the wall in
three equal strokes of hammer. Find the impulse of force on a nail in each stroke ?
(A) 16 Ns (B) 16 × 10–6 Ns (C) 16 × 10–3 Ns (D) 160 Ns
Ans. : 17 (A), 18 (C), 19 (B), 20 (C), 21 (C), 22 (D), 23 (C), 24 (C), 25 (A), 26 (C), 27 (C),
28 (D), 29 (B), 30 (D), 31 (D), 32 (C), 33 (C), 34 (C), 35 (B), 36 (C), 37 (A)
Momentum :

Product of mass (m) and velocity ( v ) is called momentum ( p ). p = m v


   

ˆ Momentum gives more information than velocity.


ˆ SI unit : kgms–1 or Ns
ˆ Dimensional formula : M1L1T–1
 
Relation of momentum with other physical quantity : kinetic energy K = = 12 p ¹ v
p2
ˆ
2m

De-broglie wave length l = p , Where h = plank’s constant = 6.625 × 10–34 Js


h
ˆ
Law of Conservation of momentum :
“Total momentum of isolated system remains constant.”

If the resultant external force acting on a system F = 0 then,





(i) Total momentum of system p = Constant
(ii) Momentum of the system can individually change, but their vector addition (total momentum)
remains constant.
(iii) Resultant acceleration of the system a = 0


(iv) velocity of the system v = constant




(v) path of motion of a body does not change.


ˆ Law of conservation of momentum is fundamental and universal.

43
(38) A player of circus keeping a disc of mass 6 kg in horizontally static condition in air by firing
bullets of mass 30 g from gun. If he is firing 40 bullets per sec, the velocity of bullet when it
reach to the disc is ...... ms–1.
(A) 0.18 (B) 50 (C) 1.8 (D) 5
(39) If the velocity of a body increases by 100 %, what is the percentage change in momentum ?
(A) 100 % (B) 200 % (C) 300 % (D) 400 %
(40) A steady substance of mass 9 kg divides into three fragments of equal masses. When it explodes,
velocities of two fragments are –3 i cms–1 and 4 j cms–1 respectively. If time interval of
explosion is 3×10–2 s, the velocity of the third fragment is ......
(A) 400 i + 300 j (B) 300 i + 400 j (C) 0.04 i + 0.03 j (D) 0.03 i + 0.04 j
(41) For a substance having constant momentum, probably which physical quantity remains constant ?
(A) Force (B) Velocity (C) Acceleration (D) All the above
(42) A person having rifle is standing on a stationary raft in a lake. Mass of the system (person + rifle
+ raft) is 100 kg. A person is 3 m away from the bank, can fire a bullet of mass 100 g with
velocity 10 ms–1 from his rifle. If he having 100 bullet, what should he do to reach on a bank ?
(There is no friction acting between raft and water and person should not allow to steer or to take
help of any external force).
(A) bullets should be fired in the opposite direction of a bank.
d(m)
(B) bullets should be fired in the direction of a bank.
(C) bullets should be fired in the upward direction.
(D) A person can not reach to a bank, though he fired all the bullets. 4
(43) A substance of mass 12 kg is moving with constant 3
acceleration.The graph of distance versus time for it is as 2
shown in figure, then its momentum is ...... Ns. 1
(A) 48 (B) 24 0 t(s)
1 2 3 4
(C) 16 (D) 6
(44) A person is standing on a stationary raft of mass 60 kg in a lake, the mass of a person is 80 kg. If
person is moving in opposite direction of a bank with a velocity of 7 ms–1 with respect to raft, After
2 sec, find the distance of a person from a bank ? Initially a person is 20 m away from bank,.
(A) 14 m (B) 17 m (C) 23 m (D) 26 m
(45) A bomb suddenly explodes into three fragments from static condition. The ratio of masses are
1:2:3 and velocities of first two fragments are 9 i ms–1 and 6 j ms–1 respectively, the velocity of
third fragment is ...... ms–1 ?
(A) 4 (B) 5 (C) 8 (D) 12
Ans : 38 (B), 39 (A), 40 (D), 41 (B), 42 (A), 43 (C), 44 (D), 45 (B)
Equillibrium of concurrent forces
ˆ The line of action of concurrent forces passes through a single point.
ˆ When they are in equillibrium,

SF =0


\ S Fx = 0 ; S Fy = 0 ; S Fz = 0

44
o o o o
ˆ When two Forces are in equillibrium, F 1 + F 2 = 0 Þ F 1 = – F 2
o o o
ˆ When three forces are in equillibrium, F 1 + F 2 + F 3 = 0
o o o
\ F1 + F 2 = – F 3
When three vectors of forces are arranged head to tail as shown in figure
and formed regular triangle, then the relation is obtained as below :
F1 F2 F3
sin D = sin E
= sin J

o o o
(46) Three forces F 1 , F 2 and F 3 are in equillibrium. Which figure represent this situation ?
o
(A) (B)
o F3
o F1
F1
o
F3

o
o
F2 F2

o
o
(C) (D)
F1 F1
o
F3
o o
F3 F2

o
F2

(47) Resultant of two forces 3F and 2F is R. If first force doubles, resultant force doubles, then find
the angle between these two forces.
(A) 180° (B) 120° (C) 90° (D) 60°

(48) A substance of mass 5 3 kg is hanged with the help of 3m long string. If horizontal force of
50 N is applied to the mid point of string, then what is angle made by upper part of the string
with vertical direction in equillibrium position ?
(A) 30° (B) 45° (C) 60° (D) 90°
(49) Wooden block is kept on the slope of an angle q and given acceleration ‘a’. If block does not
slide on the slope then what is the value of ‘a’ ? (The length and height of slope are 4m and 1m
respectively.) (g = 10 ms–2)

(A) Zero (B) 4 ms–2 (C) 20


ms–2 (D) 5 ms–2
3 4

Ans. : 46 (A), 47 (B), 48 (A), 49 (C)

45
ˆ Friction
When two bodies are in contact with each other, horizontal component to the surface of contact force
(R) is called frictional force or friction (f).

\ R= f 2  N 2 Where N = Normal force


Friction
¯
Is
No there any Yes
relative motion
between body
and surface ? Body
Sliding Rolling
is sliding or
Rolling ?

Kinetic friction
Static friction or sliding friction Rolling friction
¯ ¯ ¯
Static friction characteristics : Kinetic friction characteristics : Rolling friction characteristics :
(1) It is self-ad-justing force. (1) It opposes the relative (1) It is less than static and
As external force increases, motion of contact surface. kinetic friction.
it increases. (2) It does not depend on the
(2) It opposes the impending speed of a body.
motion (3) fk < fs (max)
¯ ¯ ¯
Value : Value : Value :
0 £ f £ msN fk = mkN fr = P r N
mk < ms mr = co-efficient of
¯ ¯ mk = co-efficient Rolling friction
Fext > (fs)max Fext £ (fs)max of kinetic friction
(fs)max = msN \ f = –Fext
R f
ms = co-efficient of static friction
N
ˆ Angle of friction (q) Ñ a
The angle between contact force and Normal force
is called Angle of friction (a).
ˆ Work done against ffrictional force :
(1) Work done for a motion on horizontal plane :
W = m mg×d
q
(2) Work done to apply motion to a body in upward direction
on a slope of an angle q is : W = mg (sin q + m cos q ).d
(3) A substance sliding downward from the slope of an angle q, work done in this case :
W = mg (sin q – m cos q ).d

46
(50) An explosive substance is kept between two blocks of 8 kg and 12 kg. When it suddenly
explodes, substance of mass 8 kg travels a distance of 9 m and become stationary.What is the
distance travellad by substance of mass 12 kg. Frictional force acting on both bodies are equal.
(A) 9 m (B) 6 m (C) 5 m (D) 4 m
(51) As shown in figure, force F is acting on a block of weight ‘W’. If the co-efficient of friction
between block and surface is ms, find the minimum value of F, so that block comes in to motion.
G
F

Ps W Ps W cos T Ps W cos T Ps W
(A) sin T  P cos T (B) sin T  cos T (C) cos T  P sin T (D) cos T  P sin T
s s s

(52) As shown in figure a box is kept on a table, connecting with block B.


Send is falling in the box at the rate of 200 gs–1 and box is moving
with constant velocity of 2 ms–1. Co-efficient of friction between box
and surface of table is 0.2 and mass of block B is 10 kg. After how A
much time box become stationary ? Mass of box is 5 kg.
(A) 100 s (B) 200 s B
(C) 225 s (D) 450 s
(53) Three blocks are arranged as shown in figure, block A and C are
tied with wall. Static friction between A and B is 0.25 and between A
F
B and C is 0.4. Find the minimum value of required force so that
block B performs horizontal motion. B 60°

(A) 37 N (B) 18.5 N [mA = 2 kg, mB = 1 kg

(C) 34 N (D) 10 N g = 10 ms–2] C

(54) A block of mass 8 kg is kept on a horizontal surface. Static friction between block and surface is
0.25. When external force acting on a block are 5 N and 25 N, static friction are f1 and f2
respectively then f1 × f2 = ...... N2
(A) 4 (B) 0.25 (C) 100 (D) 20
(55) A player to play gymnast’s pole moving with constant speed on a pole. The mass of player is
60 kg and co-efficient of friction is 0.2 between his palm and pole.What would be the horizontal
force acting on pole by him ? (g = 10 ms–2)
(A) 600 N (B) 1800 N (C) 2400 N (D) 3000 N
(56) Force of 13.2 N is acting on a stationary block of mass 6 kg kept on a horizontal surface, block
travels a distance of 2 km and attains velocity of 64 ms–1. Co-efficient of kinetic friction between
block and surface is ......
(A) 0.4 (B) 0.5 (C) 0.1176 (D) 0.7

47
(57) A car of mass 2000 kg is moving with constant speed 20 ms–1. When break applies it becomes
stationary. If the frictional force between tyre of car and surface of the road is 8000 N, how
much distance is travelled by a car ?
(A) 50 m (B) 100 m (C) 150 m (D) 200 m
(58) Escalator-belt is moving with constant speed of 2 ms–1. A passenger keeps his bag on it,
co-efficient of friction between bag and belt of escalator is 0.5. Find the distance travelled by bag
with respect to belt before it becomes steady on the belt. (g = 10 ms–2)
(A) Zero (B) 1.2 m (C) 0.6 m (D) 0.4 m
(59) The time taken by block to sliding downward from the smooth surface of an angle 45° is ‘n’
times the time taken by block kept on same type of rough surface. Then what is the co-efficient
of frication between block and surface ?

§ ·2 § ·2
1 1

(A) 1 – 12 (B) 1 (C) ¨1  12 ¸ (D) ¨¨ 1 2 ¸


¸
n 1  n2 © n ¹ © 1 n ¹

(60) The ratio of required force for a block kept on a inclined track of inclination q, for the upward
motion and for the downward motion is 2. If the co-efficient of friction is 0.3, find the value of
an angle q ?
(A) tan–1 (2) (B) tan–1 (0.9) (C) tan–1 (0.3) (D) tan–1 (6)
(61) The force required to placed a substance of mass 102 kg in steady position on a plane making an
angle of 30° with the horizontal is 750 N. If the co-efficient of static friction and kinetic friction of
the surface are 0.4 and 0.3 respectively. Find the magnitude of frictional force ?
(A) 750 N (B) 500 N (C) 250 N (D) 0
(62) The length of friction less surface is 3 m and it is making an angle of 30° with the horizontal. A
body is moving on this inclined track from stationary position and reached at the bottom, after
that it is moving on a frictionless surface in horizontal direction. Co-efficient of friction of surface
is 0.25, then how much distance is travelled by body before it come to rest ?
(A) 8 m (B) 6 m (C) 4 m (D) 2 m
Ans. : 50 (D), 51 (D), 52 (C), 53 (C), 54 (C), 55 (D), 56 (C), 57 (A), 58 (D), 59 (A), 60 (B),
61 (C), 62 (B)
Dynamics of Uniform Circular motion :
Maximum safe speed on a path having inclination ‘q’, radius ‘r’, and co-efficient of friction ms is :

§ P  tan T ·
vmax = rg ¨ s ¸
© 1  P s tan T ¹

Case - I : If path is horizontal, q = 0 vmax = rg P s

Case - II : In the absence of friction (when plane is taking turn in the air, by neglecting air
resistance)

\ Optimum speed v0 = rg tan T (\ ms = 0)

Case - III : When it is required to stop the vehicle on inclined track then, tan q £ ms

48
ˆ To obtain the maximum safe speed on horizontal road, vehicle should be inclined at an angle ‘q’
with vertical.
§ v2 ·
q = tan–1 ¨ rg ¸
© ¹
ˆ For the coin kept on a horizontal disc and should not be thrown outside to it :
(remains on the verge of the disc) C
Ps g
r£ Where, r = Distance of the coin from the center.
Z2
w = angular speed of the disc.
For circular motion in vertical Direction :
Velocity of body at any point on the path of motion is, D B

v0 2 – 2 gl (1– cos T)
q
v= v0 2 – 2 gh =
l
Where, h = height of the object from horizontal
l = length of the string
q = angle made by string with the downward. A
Velocity of body and Tension produced in the string T at different points on the path of motion.

Point Velocity Tension (T) minimum velocity Tmax Kinetic energy


required to start
the motion

A v0 + mg 6 mg mgl
mv02 5
5gl 2
l

v02 – 2 gl 2
1
B – 2 mg 3 mg mgl
mv0 2 3
3gl 2
l

v02 – 4 gl 2
1
C – 5mg 0 mgl
mv02 1
gl 2
l


1
D – 2mg 3 mg mgl
mv02 3
v02 – 2 gl 2 3gl 2
l
ˆ Relation between angular velocity and linear velocity : v = rw
1 rotation = 2p radian
2S S rad
1 rotation/minute = =
60 30 s
(63) A cyclist is moving with speed of 10 ms–1 on a circular path, centripetal force is acting on it is
20 N. If he doubles the speed, the required centripetal force is ...... N.
(A) 20 N (B) 40 N (C) 60 N (D) 80 N
(64) Two substances of mass m1 and m2 are moving on a circular path of equal radii. If these
substances complete 5 rotation and 10 rotation in 2 s respectively, the ratio of centripetal
force =
F1
......
F2

(A) m (B) 2 m (C) 4 m (D) 8m


m1 m1 m1 m1
2 2 2 2

49
(65) Radius of horizontal circular path is 3 m. A cyclist completes 30 rounds in 6 minutes on this path,
What would be the centripetal acceleration ? (p2 = 10.)

(A) 6 ms–2 (B) 6 ms–2 (C) 5 ms–2 (D) 9 ms–2


5

(66) Centre of horizonted circular track is origin. At a point on a circumference whose co-ordinate is
x = –4 m and velocity of a particle at that point is 10 j ms–1. Find the acceleration of a particle at
a point, whose y - cordinate is – 4 m.

(A) 25 i ms–2 (B) 10 i ms–2 (C) 25 j ms–2 (D) 10 j ms–2

(67) A particle is rotating on a circular path with velocity 0.2 ms–1 in a cone shaped cap kept on the
surface. Height of cap is 10 cm and its edged portion is in upper side. What is the height of
particle from the surface ?
(A) 0 cm (B) 9.6 cm (C) 4 cm (D) 10 cm
(68) A body of mass ‘m’ is moving with speed ‘v’ on a circular tath of ‘r’ radius then ......
(A) Magnitude of force changes, but acceleration remains constant. P
(B) Magnitude of force is constant, but acceleration changes.
(C) Magnitude of force and acceleration both changes.
1m
(D) Magnitude of force and acceleration both remains constant.
(69) A particle of mass 10g is moving from point P to point Q on a
semi - circle path as shown in Figure .
Find the centripetal force on it at point ‘Q’ Q

(A) 10 N (B) 0.4 N (C) 6 N (D) 0.2 N


(70) A substance of mass ‘m’ is moving on a circular path of radius ‘r’ with momentum ‘p’, then
centripetal force acting on it is ...... .

pr p2 p2m
(A) pv (B) (C) (D)
m mr r
O A B C
(71) ¬r® ¬r® ¬r®
As shown in Figure, mass of 5 kg, 10 kg and 15 kg are tied to the points A, B and C
respectively. If it is moved circularly from point ‘0’, what is the ratio of centripetal force acting
on a body at A, B and C ?
(A) 1 : 1 : 1 (B) 1 : 2 : 3 (C) 1 : 4 : 9 (D) 1 : 5 : 8
(72) A substance ‘A’ freely falls from 20 m high tower, at the same instant another substance ‘B’ is
moving on a circular path of radius 7 m. When B completes 10 rotations, ‘A’ falls on the surface,
what is centripatal acceleration of a substance ‘B’ ?
(A) 5 × 103 ms–2 (B) 7 × 103 ms–2 (C) 9 × 103 ms–2 (D) 11 × 103 ms–2

50
(73) A motor cyclist is moving with a speed of 36 kmh–1 on a horizontal road, takes a turn on curved
path of 20 m radius, the speed decreased by him, so that he does not thrown out from the road
is ...... ? ( m = 0.18)
(A) 6 ms–1 (B) 4 ms–1 (C) 10 ms–1 (D) 8 ms–1
(74) A cyclist takes a turn with speed 6 ms–1 on a horizontal, frictionless circular path of radius ‘9 m’.
For safety of himself he should alligned the cycle at angle of ...... with vertical ?

(A) tan–1 (2) (B) tan–1 2 5 (C) tan–1 5 2 (D) tan–1 (6)

(75) A body of mass 100 g is tied at the end of 4 m long string and revolves with 5 revolution /
minute. What is the Tension produced in the string ? (p2 = 10)
25S
(A) 9 N (B) 19 N (C) 4
N (D) Zero

(76) A car of mass 1200 kg is moving on a circular path of radius 30 m. A car attains maximum safe
speed v1 when co-efficient of friction is 0.5. If this car moves on a inclined track with an angle
of inclination 45° having same radius and same co-efficient of friction, its maximum safe speed is

v2, then v = ...... .


v1
2

(A) 1 : 6 (B) 6 :1 (C) 5 :1 (D) 1 : 5


(77) A car of mass 500 kg is moving on a inclined curved surface of an angle 30° and co-efficient of
friction 0.4 with maximum safe speed 6.93 ms–1. What would be the optimum speed v0 of this
car ?
(A) 2.66 ms–1 (B) 6.93 ms–1 (C) 1.77 ms–1 (D) 8.3 ms–1
(78) A car of 2000 kg mass is moving on a circular path of radius 270 m and inclination 16.7° is
taking turn. What is maximum safe speed of car, if the co - efficient of friction between tyre of
car and surface of the road is 0.5. (tan 16.7° = 0.3).
(A) 30 ms–1 (B) 40 ms–1 (C) 50 ms–1 (D) 60 ms–1
(79) A cyclist completes one revolution in 3.14 sec, on a circular path of circumference 62.8 m. The
cycle should be alligned at an angle of ...... with upward direction, by cyclist.
(A) tan–1 (1) (B) tan–1 (2) (C) tan–1 (3) (D) tan–1 (4)
(80) A particle performing uniform circular motion in a circular path of radius ‘r’ in XY plane, whose
centre is at origin. Co-ordinates of particle at time ‘t’ is p (r, q), where q = Angle with X-axis


G
then acceleration a of a particle is given by ......

(A) v i  v j (B) – v sin T i  v cos T j


2 2 2 2
r r r r

(C) – v cos T i  v sin T j (D) – v cos T i – v sin T j


2 2 2 2
r r r r

51
(81) A student tied a stone of mass 200 g at the end of thread and rotates it circularly in vertical
plane. The ratio of minimum velocity of this stone at uppermost point and lowermost point of this
circular path is ...... .
(A) 1 : 5 (B) 5 : 1 (C) 1 : 3 (D) 3 :1

(82) A bucket filled with water and tied with thread revolves on a part of redius ‘4 m’. If water doesn’t fall
down from the uppermost point of the path. What would be periodic time of revolution of a bucket ?
(A) 2 s (B) 4 s (C) 6 s (D) 8 s
(83) An object of mass 3 kg is tied with 2 m long thread and hanged in a plane. An object is given a
velocity in horizontal direction such that thread makes an angle of 60° with the upward direction.
How much tension is produced in the thread at this position ?
(A) 60 N (B) 80 N (C) 100 N (D) 120 N
A B
(84) D 10 N force is acting on a sphere starting from point A to point B
as shown in figure. It moves from B to C and then after
20 m
moving on a circular path of radius ‘r’. Finally it stops at point D,
the radius ‘r’ of circular loop is ...... m. (mass of sphere is 2 kg).
C
(A) 10 (B) 20 (C) 5 (D) 18
(85) P As shown in Figure, a particle (P) sticks on a sphere of
radius 24 m. Now, this sphere rolls in horizontal direction, at
p' how much height a particle ‘P’ becomes free from the
h' 20°
O surface of a sphere ?
(A) 30 m (B) 40 m
(C) 20 m (D) 10 m
(86) A stone is tied at the end of 2 m long thread and given a motion with uniform velocity in vertical
upward plane, the ratio of minimum and maximum tension produced at that time is 25Ñ3. What is
the velocity of the stone ?
(A) 2 3 ms (B) 2 5 ms (C) 4 3 ms (D) 4 5 ms
–1 –1 –1 –1

(87) A particle of mass ‘m’ falls from point ‘A’ in a spherical m


A
surface of radius ‘R’ as shown in figure. Which graph
represents the relation of ratio of centripetal force to normal R
force acting on a particle with q at any point. (q ¹ 0 or p)
(A) X (B) X

q q

X X

(C) (D)

q q

52
mN
N
(88) A parabolic glass is arranged as shown in Figure. Here x2 = 20y
q

and co-efficient of static friction of a glass is 0.5. An insect of


mass ‘m’ can sticks steady in a glass upto how much height from
the surface of a table ?
4 cm
(A) 5.25 cm (B) 2.5 cm
(C) 1.25 cm (D) 0.625 cm

(89) A rope of mass 4 kg and length 10 m is pulled by a force of 50 N. Find the tension produced in
the rope at a distance 3 m from the point where force is acting ?
(A) 50 N (B) 35 N (C) 15 N (D) 0
(90) Three blocks are hanged with the help of string having negligible mass T1 T1
from a pulley which is massless and frictionless, as shown in the figure
4 kg
Find the tension T1 and T2 produced in the string and T = ......
T1
4 kg
T2
2

(A) 4 (B) 8 2 kg
(C) 3 (D) 32
(91) Three blocks are hanged on a pulley (friction less) with the help of massless
string as shown in the figure. Pulley it self is hanged from rigid support with the
help of string with negligible weight. Find the tension produced in the string and
acceleration of the blocks ? (g = 10 ms–2) 7 kg
(A) 1.25 ms–2, 12.5 N
(B) 3.75 ms–2, 14.6 N 4 kg
(C) 4.25 ms–2, 125 N 5 kg
(D) 1.25 ms , 78.75 N
–2

(92) What is the acceleration of blocks and tension produced in the string in a
system as shown in the figure.
(A) 2 ms–2, 64 N
(B) 2 ms–2, 96 N 8 kg
(C) 0.5 ms , 64 N
–2

12 kg
(D) 0.5 ms–2, 96 N
(93) Two wooden blocks are hanged from pulley with the help of massless
string as shown in the figure. If they releases the block of mass 4 kg
travels a distance of 3 m, in the same time how much distance is
travelled by block of mass 1 kg.
(A) 3 m (B) 6 m 1 kg
(C) 9 m (D) 12 m
4 kg
53
(94) What should be the value of ‘‘q’ so that system remains in
equillibrium, given in the figure.
q (A) 60° (B) 45°
(C) 30° (D) 0°

m 2m m

(95) As shown in the figure two labours are pulling a block of


mass ‘m’ in upward direction by applying same force. If the
velocities of the string in downward direction, which is in the
hand of labours are equal v, what is the velocity of block ?
(A) v cos q (B) 2 v cos q
(C) v sec q (D) 2 v sec q m

(96) 4m ¬Pulley Two block of mass m and 4m are attached with the string and
passes through a pulley as shown in the figure. A block of mass
q ‘m’ hanging from length ‘l’ performs oscillations at angle q. What
should the minimum co-efficient of friction between block and
surface so that block of mass 4m does not slide ?
m
2 – cos T 1– cos T 3 – 2 cos T
(A) (B) (C) 2cos T (D)
2
3 2 2 4

(97) A mA = mB = 6 kg in the arrangement given in the figure and string


is massless, co-efficient of friction between B and surface is
B 0.5. What shold be the maximum force applied on a block A, So
F
that it does not slide on a block ‘B’ ? (g = 10 ms–2)
T7
(A) 36.72 N (B) 60 N
(C) 96 N (D) 103.78 N T5 T6

T3
T4
(98) If T7 = 60 N and T3 = 2T1, in the given figure. Find the value of T1. m3
(A) 10 N m4

(B) 120 N
(C) 40 N
(D) 160 N T1
T2
m1
T2
m2

54
(99) What is the ratio of acceleration in block A and B, in a A
5 kg
system as shown in the figure ?
(A) 1 : 1 (B) 5 : 2
(C) 2 : 5 (D) 5 : 8

8 kg B

(100) B
All the pulley and string are massless, surface is
8 kg frictionless arranged in the figure. What is the tension
produced in the string ?
(A) Zero (B) 12 N
A
12 kg (C) 24 N (D) 48 N
(101) All the pulley and string are massless, surface is
friction less and block A sliding downward from
the slope during equillibrium, what is the tension
A
m
produced in the string ?
(A) 23 mg sin q (B) 2 mg sin q
3
B
m
q (C) 12 mg sin q (D) 2 mg sin q

(102) Three blocks of mass 5 kg, 10 kg and 15 kg are tied with weightless string and kept on a
frictionless plane. If a body of mass 15 kg is pulled by force of 90 N then T = ......
T2
1

T2 T1 15 kg
10 kg 90 N
5 kg

(A) 3 (B) 13 (C) 2 (D) 12

(103) Four blocks are in contact with each other as shown in the figure. Relative velocity between m3
and m4 is zero. If the contact force between m1 and m2 is F1 and between m2 and m3 is F2 then
F1 = ...... N, and F2 = ...... N.

m4 = 2 kg

m3 = 3 kg m2 = 2 kg
m1 = 1 kg
80 N

(A) 50, 30 (B) 30, 50 (C) 70, 50 (D) 50, 70

55
(104) As shown in the figure, three blocks of mass m1 = 12 kg, m2 = 24 kg and m3 = 8 kg are in
contact with each other. Some force is applied to a point P, hence system moves with
acceleration 2 ms–2 at that time the contact force at R is FR. When some force is applied to

Q system attains velocity of 12 ms–1 in 3 s, and contact force at S is Fs. Then F = ......
FR
s
m2

m1
m3
P R S Q

(A) 2 : 3 (B) 4 : 9 (C) 3 : 2 (D) 9 : 4


(105) As shown in the figure pulley is frictionless and string is massless.
What is the acceleration of the system ? m
(A) Zero (B) 8.66 ms –2

m
(C) 5 ms–2 (D) 3.66 ms–2
30° 60°

(106) Two bodies of masses m1 and m2 are attached


with an identical pulley to a spring balance as
shown in the figure. What is the reading of
spring balance ?

(A) m  m g (B) m  m g
2m m
1 2 mm
1 2
1 2 1 2

m2 m1
(C) 12 (m1+ m2) g (D) (m1 + m2) g

F
(107) Two blocks are tied with the help of string and given a T1
acceleration of 3 ms–2 in upward direction as shown in figure. If the

tension produced in the strings are T1 and T2 then T = ....... 6 kg


T1
2

(A) 3 : 2 (B) 2 : 3
T2
(C) 2 : 1 (D) 1 : 2
12 kg

56
(108) A sphere of mass 2 kg is kept in a cube as shown y
in the figure. A cube is given a motion with velocity

v = (5t i + 2t j ) ms . So that sphere remains stationary


–1
  

with respect to cube. Find the magnitude of resultant force


acting on a cube by a Sphere ?

(A) 29 (B) 89 N
x
(C) 29 N (D) 26 N
(109) A An inclination making an angle of 30° with the
horizontal as shown in the figure. A slot of length
5 m is made in a plane as shown in the figure. A
frictionless cylinder is released in a slot to
perform motion. How much time is taken by it to
°
30

30° travel a distance AO ?


O x
(A) 1 s (B) 2 s (C) 3 s (D) 4 s
Ans. : 63 (D), 64 (C), 65 (A), 66 (C), 67 (B), 68 (B), 69 (B), 70 (C), 71 (C), 72 (B), 73 (B),
74 (C), 75 (B), 76 (A), 77 (C), 78 (B), 79 (D), 80 (D), 81 (A), 82 (B), 83 (A), 84 (D),
85 (B), 86 (D), 87 (C), 88 (A), 89 (B), 90 (C), 91 (D), 92 (B), 93 (A), 94 (B), 95 (C),
96 (D), 97 (B), 98 (A), 99 (C), 100 (D), 101 (C), 102 (B), 103 (C), 104 (B), 105 (D),
106 (D), 107 (A), 108 (D), 109 (B)
Experimental work :
Object : To study the relation between maximum static friction fs(max) and Normal reaction force (N),
and find out the Co-efficient of maximum static friction (ms) between block and horizontal
Surface.
Explaination : Suppose one wants to obtain co-efficient of maximum static friction for surface ‘A’ kept
on a table in a arrangement as shown in the figure.
ˆ Suppose weight of pan = P0
ˆ As a weight in a pan increases slowly, block starts motion on a surface. At that time,
Force of the effort = weight of pan (P0) + weight in a pan (P')
\ P = P0 + P' ....... (1) Weight
and Normal reactional force, block ® ® pulley
N = weight of block (W0) + weight in a block (W)
\ N = W0 + W
and co-efficient of friction of the surface ®pan
Force of the effort (P)
m = Normal reactional force (N) = slope of the graph of P ® N

(110) In a experiment of static friction, the graph of force of the effort (P) versus Normal reactional
force (N) making an angle of 30° with the axis of normal reactionaly force. What would be the
co-efficient of friction of the surface ?
(A) 0.26 (B) 0.58 (C) 0.42 (D) 0.37

57
(111) The graph of force of the effort (P) versus normal reactionary force (N) for a experiment of
static friction done by a student for two surfaces A and B is as shown in the figure, then we can
say that,
(A) The work done against frictional force on a body of equal P
mass moves to a equal distance is more for surface ‘A’ than
B
surface ‘B’.
(B) The work done against frictional force on a body of equal
mass moves to a equal distance is less for surface ‘A’ than A
surface ‘B’.
(C) The work done against frictional force on a body of equal
mass moves to a equal distance is equal for both the
surfaces. N
(D) We can not predict about work from the graph.
(112) The readings (observations) taken by a student for two different surface in a Experiment of
static friction is given as below. Weight of pan and block are equal.
Obs. Weight kept for a displacent of body
No. in a wooden weight in pan for minimum weight (gm. wt)
block (W') gm. wt surface ‘A’ for surface ‘B’
1. 150 10 12
2. 200 20 22
3. 250 30 32
4. 300 40 42
5. 350 50 52
(A) mA > mB (B) mA < mB (C) mA = mB (D) mA ³ mB
(113) The experiment done in a laboratatory for the study of co-efficient of static friction. Which graph
of force of the effort (P) versus normal reactionary force (N) is suitable ?
(A) P (B) P

N N

(C) P (D) P

N N

Ans. : 110 (B), 111 (B), 112 (B), 113 (C)


Assertion - Reason type Question :
Instruction : Read assertion and reason carefully, select proper option from given below.
(a) Both assertion and reason are true and reason explains the assertion.
(b) Both assertion and reason are true but reason does not explain the assertion.
(c) Assertion is true but reason is false.
(d) Assertion is false and reason is true.

58
(114) Assertion : For a particle performing uniform circular motion, linear momentum constantly changes.
Reason : For a particle performing uniform circular motion, magnitude of velocity remains
constant but direction of velocity constantly changes.
(A) a (B) b (C) c (D) d
(115) Assertion : The slope of graph of momentum versus velocity shows the mass of a body.
Reason : Kinetic energy of a body, K = 2 m
P2

(A) a (B) b (C) c (D) d


(116) Assertion : The frame attached with the earth is non-inertial frame of reference.
Reason : The frame of reference moving with acceleration is an example of non - inertial frame
of reference.
(A) a (B) b (C) c (D) d
(117) Assertion : A body kept in a lift, which is moving with constant speed in downward direction.
It’s weight observes less than the original weight.
Reason : According to Galilyo, static position and position of constant speed for a body are equal.
(A) a (B) b (C) c (D) d
(118) Assertion : A player in cricket pulls his hand in backside while catching, because reaction
decreases on his hand.
Reason : While taking catch, as player pulls his hand in backside, the time of contact increases.
(A) a (B) b (C) c (D) d
(119) Assertion : Frictional force is acting in the direction of motion by surface on both the wheels of
a cycle, when it is moving.
Reason : When two surfaces are in contact with each other, frictional force produced.
(A) a (B) b (C) c (D) d
(120) Assertion : “Ball - bearing” is used between two moving parts of machine.
Reason : Frictional force is decreased by “ball - bearing.”
(A) a (B) b (C) c (D) d
(121) Assertion : A cyclist bent his cycle in inner side while moving along the curved path.
Reason : By bending cycle mass of cyclist decreases.
(A) a (B) b (C) c (D) d
(122) Assertion : Action and reaction are equal and in opposite direction according to Newton’s third law.
Reason : Action ³ Reaction
(A) a (B) b (C) c (D) d
(123) Assertion : Effective mass of a freely falling body is zero.
Reason : Acceleration produced on a freely falling body by the earth is equal to ‘g’.
(A) a (B) b (C) c (D) d
(124) Assertion : To attract iron nails by magnet, it should be in contact with magnet.
Reason : A substance is moving under the effect of field force. When force is applied, it is not
essential that this force is in contact with external factor.
(A) a (B) b (C) c (D) d
Ans. : 114 (A), 115 (B), 116 (A), 117 (D), 118 (A), 119 (D), 120 (A), 121 (C), 122 (C), 123 (C),
124 (D)

59
S F = 100 N
Match the columns :
(125) The arrangement of block and pulley is as shown in the
figure. Consider pulley and block massless and ignore
frictional forces.
R
Column - 1 Column - 2
(i) Tension near P (P) 2.5 ms–2
(ii) Tension near R (Q) 50 N
(iii) Acceleration of a block (R) 25 N
of mass 2 kg
P
(iv) Acceleration of a block (S) 3.75 ms–2
1 kg Q
of mass 4 kg
3 kg
(A) i ® P ii ® Q iii ® S iv ® P
(B) i ® S ii ® R iii ® Q iv ® R 2 kg
(C) i ® R ii ® Q iii ® P iv ® S
(D) i ® Q ii ® P iii ® R iv ® Q
(126) Match the columns, in the reference of arrangement as shown in the figure. 4 kg
Surface is friction less, and string is tension less.
Column - 1 Column - 2 N
18
(i) Acceleration of block having mass 1 kg (P) 4 SI
g
1k
(ii) Resultant force on a block having mass 2 kg (Q) 25 SI 2k
g
g
(iii) Normal force on a block having mass 3 kg (R) 2 SI N 3k
60 °
30
(iv) Normal reaction force between the (S) 15 3 SI
block having mass 2 kg and 1 kg
(A) i®R ii ® P iii ® S iv ® Q
(B) i®P ii ® R iii ® Q iv ® S
(C) i®R ii ® Q iii ® S iv ® P
(D) i®Q ii ® P iii ® Q iv ® R
Ans. : 125 (C), 126 (A)
Comprehension Type Questions :
A person of mass 60 kg is standing on a spring balance in a lift. Lift is connected with cable
rotar, to control the speed of lift there’s an arrangement of break and accelerator in it. In this
conditions, select the proper answer in a questions given below. (g = 10 ms–2)
(127) What is the observation of spring balance, when lift is moving with acceleration of 5 ms–2 in
upward direction ?
(A) 300 N (B) 200 N (C) 100 N (D) Zero
(128) What is the acceleration experienced by a person when lift is moving in downward direction with
an acceleration 5 ms–2.
(A) 5 ms–2 (B) 10 ms–2 (C) 15 ms–2 (D) 20 ms–2
(129) What is the observation of spring balance when cable of lift breaks ?
(A) Zero (B) 5 ms–2 (C) 10 ms–2 (D) 15 ms–2
Ans. : 127 (A), 128 (C), 129 (A)
ˆ
60
6 Work, Energy and Power
Work :
ˆ Work = Force × displacement in the direction of force
or
Work = effective component of displacement × force in direction displacement

Work done by Work done by


constant force variable force
Bo o
W = F × d = Fd cos q W= ³ F ˜ dr
 

0 £ q < 90° q = 90° 90°< q £ 180°


W=+ W=0 W=–
o o
ˆ Area enclosed by graph of Foce ( F ) ® displacement ( r ) = work.

ˆ Work :
MKS – Unit = joule
CGS – Unit = erg
Dimensional formula = M1L2T–2
1 joule = 107 erg

(1) A person pull trolly bag with 24 N force and displacement 10 m. If work done in this case is
120 J. Find the angle of trolly bag with vertical direction.

(A) 0° (B) 30° (C) 45° (D) 60°

(2) Two students kept their bag on a table at a height of 1 m. Mass of their bag are equal and time
to kept on a table for student - 1 is three times than student - 2. The ratio of work done by them
is ......

(A) 1:3 (B) 3:1 (C) 9:1 (D) 1:1

(3) A car of mass 2000 kg is moving with speed 108 kmh–1, when brake applies suddenly it stops
after travelling a distance of 15 m. Find the work done on a wheel of a car.

(A) 900 KJ (B) – 900 KJ (C) 300 KJ (D) – 300 KJ

(4) In a cricket ground, player A throws a ball of 150 g in horizontal direction up to 15 m and player
B throws a ball of 300 g in vertical direction up to same distance. Find the ratio of work done on
a ball by gravitational forces in both the cases ?
(A) 1:2 (B) 2:1 (C) 1.1 (D) 0

61
(5) As shown in figure, two blocks of mass m1 and m2 m1
are given motion under the effect of gravitational m2
field. If the ratio of mass are m1 : m2 = 2:3. Find the
ratio of work done.
h
d
(A) 1 : 1 (B) 2 : 3
(C) 3 : 2 (D) 2 : 1
(6) Ramesh tied 1 kg stone at the end of 3 m long string and gives circular motion in a horizontal
plane. How much work is done on a stone by string and gravitational force ?
(A) 4 N, 0 N (B) 0 N, 4 N (C) 4 N, 4 N (D) 0 N, 0 N

(7) Two forces are acting on a 10 kg block as


shown in figure, so block attains velocity of
14.66 ms–1 after travelling some distance. If
block performs motion only in horizontal direction
100 N
30° and kinetic friction of surface is 0.2. Find the
work done on a block by resultant force during
70 N
this motion.

(A) 96.6 J (B) 48.3 J (C) 73.3 J (D) 35 J


(8) Acceleration of 6 kg mass changes according to distance as a(x) = (9x2 + 6x – 3) ms–2. What is
the work done when body travels a distance of 3 m ?
(A) 33 J (B) 99 J (C) 198 J (D) 594 J
o o
(9) Force acting on a 2 kg mass in x-y plane changes according to F = {(3x2 + 2x) i + (6y2 + 2) j } N.


Find the work done on this body when it displaces from pt. P (0, 2) to Q (2, 0).
(A) –12 J (B) –8 J (C) +8 J (D) 12 J
(10) –2 0 2 3 4
D 0 A B C

Particle lying at origin 0 performs motion from 0 to A under the influence of force F = kx2 and

work done in this case is W1. When it travels from B to D work done is W2 then W = ...... .
W1
2

(A) 3 : 2 (B) 3 : 1 (C) 19 : 8 (D) 8 : 19

(11) Displacement of a body having mass 2 kg under variable force changes according to
§ 3 2 ·
S = ¨ 3 + 2 + 5 ¸ m. What is the work done on a body by this force in first two second ?
t t
© ¹

(A) 12 J (B) 18 J (C) 24 J (D) 36 J

(12) A chain of 3m length is kept on a table in such a way so that its 1 m part hanging downward
from the edge of table. The whole mass of chain is 9 kg. How much work is done to bring
hanging part completely on a table ?

(A) 270 J (B) 100 J (C) 30 J (D) 15 J

62
(13) A particle performs motion in two dimension under the influence of force F = (3x2 i + 4 j ) N.


What would be the work done to move a particle from point (2, 3) m to (3, 0) m ?
(A) 0 (B) +7 J

F
(C) 12 J (D) +19 J

È Ø È Ø
(14)  The graph of Force É  ® distance ÉÊ d ÙÚ
Ê F ÙÚ 
for a particle moving along X- axis is as shown d
in figure. Find the work done, when particle
travels first 12 m distance.
(A) 26 J (B) 24 J (C) 40 J (D) 8 J
Ans : 1 (B), 2 (D), 3 (B), 4 (D), 5 (B), 6 (D), 7 (C), 8 (D), 9 (B), 10 (D), 11 (D), 12 (D), 13 (B), 14 (B)

Energy
The ability to do work

Kinetic energy Potential energy


The ability to do work due to its motion The ability to do work due to position or
is called kinetic energy configuration is called potential energy

Formula

K = 2 mv2 = = 2 pv Potential energy due Potential energy due to


1 p2 1
2m
W = Dk Þ Work – energy – theorem to position configuration

Gravitational potential Elastic potential energy


energy : U = mgh in the spring U = 2 kx2
1

h = height from the x = compression of the


reference surface of spring
Total energy the earth k = force constant of
ˆ (mechanical energy) : E = U + K the spring
ˆ Conservation of mechanical energy :
For isolated system under the effect of conservative force, E = U + K = constant
\ DE = DU + DK = 0
ˆ Different form of energy : (1) Mechanical energy (2) Internal energy (3) Heat energy
(4) Electrical energy (5) Chemical energy (6) Nuclear energy.
ˆ Energy - mass equivalence :
According to Einstein's theory of relativity, E = Dmc2
Dm = change in mass, E = Energy, c = velocity of light = 3 × 108 ms–1
E = energy equivalent to Dm

63
ˆ MKS unit of energy = joule
ˆ CGS unit of energy = erg
ˆ Dimensional formula of energy = M1L2T–2
ˆ Traditional unit of energy = calorie, 1 calorie, = 4.186 joule
ˆ Unit of energy in terms of power = 1 kWh = 3.6 × 106 J = 1 Unit
(15) A ball is thrown with velocity ‘v’ in vertical direction. At the same time a block having same
mass is projected at an angle of 30°. Find the ratio of their potential energy at the points on
maximum height on their paths of motion.
(A) 1 : 1 (B) 2 : 1 (C) 4 : 1 (D) 8 : 1
(16) Two objects with mass 16 kg and 4 kg is moving with acceleration of 2 ms–2 and 8 ms–2
respectively from the steady state. After a respective time interval t1 and t2 their kinetic energy

equals to K J. Find the ratio .


t1
t2

(A) 2 : 1 (B) 1 : 2 (C) 1 : 4 (D) 4 : 1


(17) As shown in figure a sphere of simple pendulum
having mass m1 releases from position A, becomes steady 60° 1m
after a collision with a sphere having mass
m2 = 2m1 at mid position of its path of motion. Hence a sphere
of m2 mass attains kinetic energy of 4 J, then m2 = ...... A
m1
(A) 2 kg (B) 3.2 kg
(C) 6 .3 kg (D) 8.2 kg m2

(18) As shown in figure a sphere of mass 1 kg given a


A motion from steady position at point A, which rolls
down to point B. Energy loss of 0.5 J per 12 m

7m B length on the slope. Find the kinetic energy at


point ‘B’.
4m
(A) 40 J (B) 19 J
30° 30°
(C) 8 J (D) 0 J
C
(19) A stationary bomb of mass 20 kg suddenly explodes in two fragments in proportion of 1:4. If
kinetic energy of small fragment is 360 J, find the kinetic energy of larger fragment.
(A) 360 J (B) 180 J (C) 90 J (D) 0 J
(20) A body of mass 800 g freely falls from height of 50 m. When this body comes on the surface of
the earth, potential energy completely converts in to kinetic energy. Find the loss of potential
energy of a body.
(A) 0 J (B) 200 J (C) 300 J (D) 400 J

64
(21) An object is moving on a straight path, when its speed increases by 2 ms–1, its kinetic energy
doubles. What is the original speed of an object ?
(A) 2 ms–1 (B) 2 ± 8 ms–1 (C) 8 ms–1 (D) 2 ± 2 ms–1
(22) 15 J energy is paid by a person to pull an object of mass 3 kg from depth ‘h’ in empty well. 40 %
energy is wasted due to friction. When this object reach to the edge of well suddenly rope breaks and
object goes to bottom of well. if its velocity at bottom is 3 ms–1, find the depth of well.
(A) 6 m (B) 2 m (C) 1 m (D) 0.45 m
(23) Two balls A and B freely falls from the same height. Ratio of their mass are 1:4, when potential
energy of A is three times than potential energy of B. Find the ratio of travelled distance
by them.
(A) 1:12 (B) 12:1 (C) 1:6 (D) 6:1
(24) An object of mass 2 kg freely falls from a height of 60 m collides with the surface of the earth
and reflect upto height 40 m. Loss of energy during this collision is how much part of its original
energy ?
(A) one third (B) Half (C) one fourth (D) one sixth

(25) In diatomic molecule, potential energy between two atoms is given by U(x) = . How
a  b
x12 x6

much energy is required to separate these two atom from stable position ?
b2
(A) 2 a (B) b (C) b (D) b
2 2 2
6a 4a 12 a
(26) A rope of length ‘L’ is tied with balloon. When a person tries to climb in a ballon with the help of
rope, it descends by distance ‘d’. If mass of ballon is M, then what is the ratio of change of
potential energy of a person and ballon ?

(A) d m
L M
(B)
L d
d Mm (C) L  d m
d
M (D) L – d m M
(27) The force required to pull the spring upto 6 cm is 120 N. What is the required work to pull a
spring another 6 cm ?
(A) 5 J (B) 8.4 J (C) 10.8 J (D) 12.4 J
(28) The ratio of stored potential energy of two spring pulled by same force having force constant
600 Nm–1 and 1200 Nm–1 is ‘a’ and the ratio of stored potential energy when they pulled to same
length is ‘b’ then a . b = ...... .
(A) 1 (B) 2 (C) 3 (D) 4
(29) A sphere of mass 0.1 kg is moving with velocity of 10 ms–1 collides with a spring which is at a
distance 2 m from the initial point, which compresses the spring and becomes steady. Find the
compression of the spring. [Force constant of spring is 6 Nm–1 and co-efficient of friction
between sphere and surface is 0.2]
(A) 1 m (B) 2 m (C) 3 m (D) 4 m
(30) When a spring pulled by 2 mm, 36 J energy stored in it. Find the work done to pull a spring
further 2 mm length.
(A) 36 J (B) 72 J (C) 108 J (D) 144 J
65
(31) A spring is arranged vertical, whose end is connected with horizontal surface. A block of mass
‘m’ falls on a spring from height ‘h’ so, spring compresses to distance ‘d’. If the force constant
of spring is ‘k’, what is the total work done ?

(A) mg (h + d) + 12 kd2 (B) mg (h + d) – 12 kd2

(C) mg (h – d) – 12 kd2 (D) mg (h – d) + 12 kd2


(32) A block of mass 2 kg is moving with velocity of 4 ms–1 collides with a spring and compresses it.
Find the compression of the spring if friction force is 16 N and force constant of the spring is
10 kNm–1.
(A) 8.5 cm (B) 2.5 cm (C) 5.5 cm (D) 11.0 cm
(33) The graph of potential energy ® distance is as shown in figure. Which graph of
Force ® distance is suitable ?
(A) F(x) (B) F(x)
U (x)
P P
x x

(C) F(x) (D) F(x) P

0 P x P
x x

(34) An elastic spring is arranged along the x-axis as shown in figure. The graph of
F ® x is as shown in figure. find the work done to bring end of spring from x = A to x = C.

F(N)
50
40
30
20
10
x(m)
0 1 2 3 4 5

x = –1 x = 0 1 2 3 4 5
A C

(A) 40 J (B) 80 J (C) 120 J (D) 160 J

66
(35) A spring is in its normal condition as shown in
figure. What should be the minimum mass of A
so that B is on the verge to loose a contact with
surface ?
A
(A) M
2
(B) M
(C) 2 M (D) 3 M
B
M

Ans. : 15 (C), 16 (A), 17 (B), 18 (B), 19 (C), 20 (D), 21 (B), 22 (D), 23 (B), 24 (A), 25 (C), 26 (B), 27
(C), 28 (A), 29 (A), 30 (D), 31 (B), 32 (C), 33 (C), 34 (B), 35 (A)
Power :
Time - rate to do work is called power.
'W
<P> = 't (average power)

Instantaneous power P = 't = F . v


 
'W

ˆ Different units of power :


(i) Js–1 (ii) watt (W) (iii) horse power (hp), 1 hp @ 746 W
Dimensional of power = M1L2T–3
(36) The force ‘F’ is acting on a car having mass ‘m’ and moving on a horizontal road in such a way
so that its velocity changes from v2 to v1 when it travelled a distance ‘d’. If produced power by
engine of car (P) is constant then v2 = ......


1 1 1 1
(A) Pd  v 2 2 (B) Pd  v 2 3 (C) 3Pd  v 3 2 (D) 3Pd  v 3 3
2m 1 2m 1 m 1
m 1

(37) Water is falling on turbine of A. C. generator from 80 m height with the rate of 20 kgs–1. 20 %
energy from the total energy is converted into electrical energy then what is the electrical
energy ?
(A) 3.2 kW (B) 2.4 kW (C) 4.2 kW (D) 6.8 kW
(38) Water tank of size 3m × 3m × 1m is kept on a terrace of 10 m high building. How much time is
taken to fill the water in this tank with the help of motor of power 10 kW and efficiency 40 % ?
(A) 6.23 min (B) 3.75 min (C) 4.24 min (D) 8.52 min
(39) What is the power of a body of mass 3 kg projected at an angle of 30° with the horizontal, with
velocity of 40 ms–1 ?
(A) 75 W (B) 200 W (C) 300 W (D) 400 W
(40) When a rail of mass 12 × 10 kg pulled by engine, its velocity increases from 3 ms–1 to 5 ms–1 in
6

two minutes What is the power of engine ?


(A) 200 kW (B) 400 kW (C) 600 kW (D) 800 kW
(41) Vehicle of mass 2000 kg and 3000 kg taken time 8 s and 6 s respectively to travel on a slope.
The rario of their power of engine is ......
(A) 1 : 2 (B) 2 : 3 (C) 3 : 2 (D) 2 : 1

67
(42) Force acting on a body of mass ‘m’ is (2 i – 3 j + 4 k ) N and displacement in t s is
3t2 i + 4t j + 6t3 k . Power of body at t = 2 s, is ......
(A) 324 W (B) 300 W (C) Zero (D) 200 W
(43) A soldier fires bullets of mass 20 g with speed 360 kmh at the rate of 180 bullet/min. What is
–1

power of machine-gun ?
(A) 600 W (B) 400 W (C) 200 W (D) 100 W
(44) Heart of an animal pushes 1 cc blood in 1 sec with pressure of 23,000 Nm–2. Find the essential
power.
(A) 0 (B) 0.32 W (C) 0.023 W (D) 0.042 W
(45) An electric motor of 0.5 hp moving with speed of 600 rpm. If the efficiency of motor is 70 %,
what is the work done by motor during one rotation ?
(A) 3.46 J (B) 5.12 J (C) 4.19 J (D) 2.6 J
Ans. : 36 (D), 37 (A), 38 (B), 39 (C), 40 (D), 41 (A), 42 (B), 43 (A), 44 (C), 45 (D)
Collision :
ˆ During the collision, if momentum, total energy and kinetic energy is conserved then it is called
elastic collision.
ˆ During the collision, If momentum and total energy is conserved but kinetic energy is not
conserved then it is called inelastic collision.
Collision in one dimension
ˆ An object of mass m1 is moving with velocity v1 along + x axis and collides with an object of
mass m2 moving with velocity v2 in the same direction.
After collision velocities of mass m1 and m2 are v'1 and v'2 respectively
(1  e) m2 (1  e) m1
v'1 = m  m v1 + m  m v2 and v'2 = m  m v2 + m  m v1
m1 – em2 m2 – em1
1 2 1 2 1 2 1 2

Where e = restitution co-efficient = v – v


v '2 – v '1
1 2

For, complete elastic collision, e = 1


Complete inelastic collision, e = 0
and For other cases, 0 < e < 1
ˆ For complete elastic collision (e = 1)

v'1 = m  m v1 + m  m v2 and v'2 = m  m v2 + m  m v1


m1 – m2 2 m2 m2 – m1 2 m1
\
1 2 1 2 1 2 1 2

ˆ Two objects with mass m1 and m2 moving with velocity v1 and v2 respectively suffers perfect
elastic collision. If their velocities after collision are v'1 and v'2 respectively.

v'1 = m  m v1 + m  m v2 v'2 = m  m v2 + m  m v1
m1 – m2 2 m2 m2 – m1 2 m1
1 2 1 2 1 2 1 2

m1 >> m2 v'1 = v1 v'2 = – v2 + 2v1


m1 << m2 v'1 = –v1 + 2v2 v'2 = v2
m1 = m2 v'1 = v2 v'2 = v1

68
ˆ Inelastic collision :
A body of mass m1 is moving with velocity v1 collides with other body of mass m2 moving with
velocity v2. If the collision is total inelastic then combined velocity after collision,

v=
m1v1  m2 v2
m1  m2

ˆ A body of mass ‘m’ moving with velocity ‘v’ collides inelastically with stationary body of mass ‘m’ then
after collision,

velocity of first body, v'1 = 2 (1 – e)


v
-

velocity of second body, v'2 = 2 (1 + e)


v
-
§ m1 m2 ·
- Decrease of energy during collision, DK = 12 ¨ m  m ¸ (1 – e2) (v1 – v2)2
© 1 2 ¹

- Body falls from height ‘h’ collides on the surface, height achieved by body after n - collision is,
hn = e2n h
§ 1  e2 ·
- Distance travelled by body before it becomes steady, d = h ¨¨ ¸
¸
© 1– e ¹
2

§ 1 e ·
time taken by body to become stationary, t = ¨ 1– e ¸
2h
© ¹
- g

(46) Three stationary spheres of mass 3 kg, 6 kg and 3 kg are kept on a horizontal frictionless
surface. At t = 0 time sphere A moving with velocity 9 ms–1 collides elastically with sphere B.
Then after perfect inelastic collision occurs between B and C. What is the velocity of sphere C
after collision ?

B
A C

(A) 9 ms–1 (B) 4 ms–1 (C) 1 ms–1 (D) Zero


(47) Two spheres of mass m1 and m2 (m2 = 2m1) moving in opposite direction collides with each other.
They become stationary after collision. If velocity of sphere having mass m1 is 12 ms–1, then
what is the velocity of a sphere having mass m2 ?
(A) 12 ms–1 (B) 9 ms–1 (C) 6 ms–1 (D) 3 ms–1
(48) A bomb projected at an angle of 60° with velocity of 200 ms–1 explodes in to three equal fragments,
at maximum height. First fragment is moving with a speed of 100 ms–1 in vertical upword direction,
second fragment moving with the same speed in downward direction. If the total mass of bomb is
3 kg, find the kinetic energy of the third fragment.
(A) 300 J (B) 15 kJ (C) 30 kJ (D) 45 kJ
(49) Nucleus of 92U238 is moving with a speed ‘v’, emits a - particle which is moving with speed ‘3v’
in the same direction. What is the velocity of remaining nucleus ?

(A) 238 (B) 117 v (C) 117 v (D) Zero


4v 113 119

69
(50) A sphere of mass 8 kg moving in east direction with velocity 8 ms–1 collides with strong and a big
wall. Calculate the velocity and kinetic energy of a sphere after collision. Consider collision as a
perfect elastic.
(A) 8 ms–1, 256 J (B) 4 ms–1, 128 J (C) 0, 256 J (D) 0, 256 J
(51) Two block P and Q having same mass ‘m’ is moving with velocities 3 ms–1 and –5 ms–1
respectively on a frictionless horizontal surface collides elastically. What is the ratio of their
momentum after collision ?
(A) 3 : 5 (B) 5 : 3 (C) 9 : 25 (D) 25 : 9
(52) Ramesh throws a ball from 16.8 m high tower in downward direction, after collision with surface
it reflects back upto 4.2 m height. Find the percentage decrement in a velocity of ball.
(A) 100 % (B) 75 % (C) 50 % (D) 25 %
(53) A bomb explodes in two fragments of equal mass after release from a helicopter which is steady
at a certain height, one of the fragment attains horizontal velocity of 15 ms–1. After how much
time the vectors connected two fragment with initial point becomes perpendicular ? (g = 10 ms–2)
(A) 15 s (B) 9 s (C) 6 s (D) 3 s
(54) A ball freely falls from 30 m height. If elastic co-efficient during collision is e, then upto how
much hight ball would be reflect after second collision ?
(A) 15 e m (B) 60 e m (C) 30 e m (D) 30 e4 m
(55) An object with mass ‘m’ thrown in upward direction with a velocity 200 ms–1 After 4 sec object
divides into two fragments having ratio of mass 1:3. If smaller fragment is moving with a
velocity 400 ms–1 in upward direction. Find the velocity of larger fragment.
(A) 200 ms–1 (B) 100 ms–1 (C) 80 ms–1 (D) 0
(56) A ball thrown in downward direction from a height 15 m, collides with the surface and losses
50 % energy, reflects back to height of 10 m, then what is its initial speed ?
(A) 5 ms–1 (B) 10 ms–1 (C) 80 ms–1 (D) 00 ms–1
(57) An object falls from height ‘h’ on the horizontal surface, suffers frequent collision and frequent
reflection. If co-efficient of restitution is ‘e’, then find the travelled distance by object before it
becomes steady.
§ 1  e2 · § 1– e2 · § 1– e2 · § 1  e2 ·
(A) h ¨¨ ¸
2¸ (B) h ¨¨ ¸
2¸ (C) 2 ¨¨ ¸
2¸ (D) 2 ¨¨ ¸

h h
© 1– e ¹ © 1 e ¹ © 1 e ¹ © 1– e ¹
(58) A body of mass 4 kg, moving with velocity 12 ms–1 collides with a body of mass 6 kg and stick
to it. Find the decrease in its kinetic energy.
(A) Zero (B) 288 J (C) 172.8 J (D) 144 J
Ans. : 46 (B), 47 (C), 48 (D), 49 (B), 50 (A), 51 (B), 52 (C), 53 (D), 54 (D), 55 (C), 56 (D), 57 (A), 58 (C)
Assertion - Reason type Question :
Instruction : Read assertion and reason carefully, select proper option from given below.
(a) Both assertion and reason are true and reason explains the assertion.
(b) Both assertion and reason are true but reason does not explain the assertion.
(c) Assertion is true but reason is false.
(d) Assertion is false and reason is true.
70
(59) Assertion : For a body only mass or only energy should not be conserved but mass - energy
combinely conserved.
Reason : According to Einstein’s equation E = Dmc2
(A) a (B) b (C) c (D) d
(60) Assertion : When momentum of lighter and heavy body are same, their kinetic energies are
always equal.
Reason : Kinetic energy does not depend on mass of body.
(A) a (B) b (C) c (D) d
(61) Assertion : Roads on the mountain are curved instead of Straight.
Reason : Slope on the mountains are more so possibility of sliding vehicle is more.
(A) a (B) b (C) c (D) d
(62) Assertion : If a spring is extended or compressed for equal length, equal potential energy is
stored in both the cases.
Reason : Potential energy of the spring is directly proportional to force constant.
(A) a (B) b (C) c (D) d
(63) Assertion : Power of machine-gun P = nK where n = no. of bullets fired per second and
K = Kinetic energy of bullet.
work done by machine-gun
Reason : Power of machine-gun P = time
(A) a (B) b (C) c (D) d
(64) Assertion : Work done by damping force is zero, for a body moving under the effect of damping
force.
Reason : Work depends on an angle between force and displacement.
(A) a (B) b (C) c (D) d
(65) Assertion : If velocity of a sphere moving in horizontal direction becomes double, its kinetic
energy becomes four times.
Reason : Kinetic energy is directly proportional to square of velocity.
(A) a (B) b (C) c (D) d
(66) Assertion : The graph of potential energy stored in the spring ® restoring force is straight line.

Reason : Potential energy stored in the spring is given by U = kx2, where x = compression or
1
2
expansion of a spring.
(A) a (B) b (C) c (D) d
(67) Assertion : During the elastic collision of two body total momentum and total kinetic energy
conserved.
Reason : During the collision, if two body sticks with each other, called elastic collision.
(A) a (B) b (C) c (D) d
(68) Assertion : The work done to come upto bottom for a body having mass ‘m’ on the sloped
friction less surface making an angle q and the work done for the same body to move
downward from the same height are always equal.
Reason : In both the cases gravitational forces are equal.
(A) a (B) b (C) c (D) d
Ans. : 59 (A), 60 (D), 61 (A), 62 (B), 63 (A), 64 (D), 65 (A), 66 (D), 67 (C), 68 (B)

71
Match the columns :
(69) Match the column - 1 with column - 2 :
Column-1 Column-2
(i) Perfect elastic collision (P) losses some kinetic energy
(ii) Perfect inelastic collision (Q) 0 < e < 1
(iii) Partial elastic collision (R) e = 1
(iv) Partial inelastic collision (S) e = 0
(A) i ® Q ii ® R, Q iii ® P, S iv ® Q, S
(B) i ® R ii ® P, S iii ® R, Q iv ® P, R
(C) i ® P ii ® R, S iii ® P, Q iv ® S, Q
(D) i ® R ii ® S iii ® P, Q iv ® P, Q
(70) Match the columms :
Column-1 Column-2
(i) Kinetic energy of a body never be a negative (P) Kinetic energy increases by 100 %
(ii) For uniform circular motion (Q) Kinetic energy increases
(iii) When a bomb explodes from tank (R) Kinetic energy remains constant
(iv) Momentum of a body increases by 50 % (S) Statement is true.
(A) i®P ii ® R iii ® S iv ® Q
(B) i®S ii ® R iii ® Q iv ® P
(C) i®S ii ® R iii ® P iv ® Q
(D) i®R ii ® Q iii ® P iv ® S
Ans. : 69 (D), 70 (B)
Comprehension Type Questions :
The relation between potential energy, kinetic energy and total energy is given as below :

U = mgh, K = 12 mv2, E = U + K and D E = D U + D K = 0

(71) When a body of mass 2 kg freely falls from a tower, what is its kinetic energy at t = 5 s ?
(A) 5000 J (B) 2500 J (C) 2000 J (D) Zero
(72) What is the momentum of a body at that time in above question ?
(A) 100 NS (B) 2500 NS (C) 300 NS (D) Zero
(73) What is the potential energy at that time in above question if total energy is 5000 J ?
(A) 100 J (B) 2500 J (C) 5000 J (D) Zero
(74) What is the change of total energy of a body at a given time ?
(A) 1000 J (B) 2500 J (C) 5000 J (D) Zero

Ans. : 71 (B), 72 (A), 73 (B), 74 (D)

72
7 Rotational Motion
Centre of mass:
“A point at which all the mass of the system can be considered as concentrated.”
ˆ Centre of mass is defined for study of extended objects as a particle.
ˆ The centre of mass should be either inside or outside the body.
ˆ It shows the average position of the mass of the component of the object.
ˆ In symmetrical bodies the centre of mass coincides with the geometrical centre, while the centre of
mass of irregular shape bodies is towards the heavy mass distribution.
ˆ For two particles system

M r cm = m1 r1 + m2 r2
  

where M = m1 + m2
ˆ In component form,
M xcm = m1x1 + m2x2 ; M ycm = m1y1 + m2y2
ˆ If centre of mass is at origin then
o o
m1 r1 + m2 r2 = 0 
  m1
\ r2 r
m2 1

Here the sign of r1 and r2 are opposite. It shows that the mass m1 and m2 are on both the sides of
 
ˆ
the centre of mass.
ˆ m1 > m2 Þ r1 < r2. It means that the centre of mass towards the heavy mass.
ˆ If two particles having same mass then,

o
o o
r1 + r2
r cm =
2
x1  x 2 y1  y 2
\ x cm ; ycm
2 2
ˆ For n- particles system,
o
¦ mi ri
n
o
M r cm =
i 1

¦ mi xi ; ¦ mi yi ; Mzcm = ¦
n n n
Mxcm = Mycm = mi zi
i 1 i 1 i 1

ˆ For a rigid body,

³ r dm ;Mxcm = ³ x dm ; Mycm = ³ y dm ; Mzcm = ³ z dm


o o
M r cm =

73
(1) Three particles each of mass 3 kg are placed at three corners of an equilateral triangle as shown
in figure. The centre of mass with respect to particle 1 is ...... m.
y (A) ( 0.5, 1.33 ) m
3
(B) ( 1.33, 0.5 ) m
1.5 m (C) ( 12, 4.5 ) m
(D) ( 4.5, 12 ) m
x
1 2m 2

(2) Two particles of mass 50 g and 100 g have positions 3 i + 4 j + 5 k cm and – 6 i – 2 k + 4 j


cm with respect to origin. The distance of the centre of mass from the origin is ...... .

(A) 15 cm (B) 10 cm (C) 5 cm (D) 15 cm


y
(3) Find the centre of mass with respect to
origin of a E shape having 2 cm 10
thickness and uniform density distribution. cm

(A) ( 2.6, 2.4 ) cm

(B) ( 2.4, 1.6 ) cm 5

(C) ( 2.4, 5 ) cm

(D) ( 1.4, 2.6 ) cm

O 2 4 6 cm x
(4) Find the centre of mass with respect to a particle of 1g mass of four particles parallelogram
shaped system. ( The length of each side is a ).

2g 3g (A) ( 3a , 0.95a)
4

(B) ( 2 , 4 )
a 3a
a
a
(C) ( 4 , 2 )
3a a
60°
1g 4g
(D) (0.95a, 3a
)
4

(5) A ‘T’shaped object with dimensions shown in the figure,is lying


on a smooth floor. At which point from the surface a force is l
applied so that the object has only the translation motion ?
(Thickness of rod is negligible).
2l
(A) l (B) 23 l

(C) 43 l (D) any where.

74
(6) Y Four bricks each of length L and mass m are
arranged as shown, from the wall. The distance
of the centre of mass of the system from the
L wall is...... .
2
(A) 7 L (B) 8 L
8 7

(C) 12 L (D) 16 L
L 11 15
X
2

L L
L y
4 2
(7) A circular plate of uniform thickness has a diameter of 60
cm. A circular portion of radius 20 cm is removed from O' x
O
one edge of the plate. Then the centre of mass of the
remaining portion with respect to origin is ...... cm
(A) ( 0, –4 ) (B) ( 8, 0) (C) ( 0, –8 ) (D) ( –8, 0)
(8) The particles of 10 g, 20 g, 30 g and 40 g are placed at 2, 6, 8 and 11 hour sysmbols respectively of a
weightless dial of clock having radius 8 cm. Find the co - ordinates of centre of mass of this system.
y (A) ( 1.84, –0.136 ) cm
m4
m1 (B) ( 1.36, –0.184 ) cm
60° 30°
x (C) ( –1.36, –0.184 ) cm
m3 (D) ( –1.49, –0.184 ) cm
m2
(9) Mass density of a rod of length L changes with respect to distance x from one end is according to
equation l = b x, where b = constant. Then the distance of centre of mass from x = 0 is ...... .
(A) L2 (B) L3 (C) 23L (D) 2
3L

(10) The centre of mass of a half portion of a thin ring of mass 2M and
radius R having uniform mass density with respect to its centre is ...... . y

(A) 2SR (B) R


2S
R
S
(C) R (D) R
S x
(11) The centre of mass of cone with a uniform mass density with radius R and height h with respect to its
top is ...... .
(A) 43 h (B) 23 h (C) 4 h (D) 13 h
3

(12) Two spheres of mass M and 4M having radius R and 3R respectively. The distance between
their centres is 10 R. If they are move due to gravitational force then before collision the
distance travelled by the bigger sphere is ...... .
(A) 2 R (B) 8 R (C) 4.8 R y (D) 1.2 R
(13) Locate the centre of mass of a right angle
triangle of uniform mass density.
§x y· §x y· y
(A) ¨ 2 , 2 ¸ (B) ¨ 3 , 3 ¸
© ¹ © ¹
§x y· §x y· x
(C) ¨ 2 , 3 ¸ (D) ¨ 3 , 2 ¸ x
© ¹ © ¹ O
75
(14) The length of a rod is 2 m. Its linear mass density change by equation l = 3 + x, then the centre
of mass of the rod with respect to x = 0 is ...... .

(A) 12 m (B) 12 m (C) 12 m (D) 1 m


13 15
13
(15) The centre of mass of a half portion of a disc of mass m and radius R having uniform mass
density with respect to its centre is ...... .

(A) 4 (B) 4SR (C) 45SR (D) 43SR


3R

Ans. : 1 (B), 2 (A), 3 (C), 4 (D), 5 (C), 6 (B), 7 (C), 8 (D), 9 (C), 10 (A), 11 (c), 12 (D), 13 (B), 14
(A), 15 (D)
Motion of centre of mass :
o
¦ mi vi
n
M v cm =


i 1
o
¦ mi ai
n o
= = F net = Resultant external force.

M a cm
Linear momentum of the system :
i 1

o 
P = M v cm
§o · o
d ¨ v cm ¸
¦
o JG o n
F ext = dt = M dt = M a cm = mi ai
© ¹
d P
i 1

¦ Fi
n o

o
o i 1
= F ext =
¦ mi
n
a cm
M
i 1

ˆ Centre of mass behave like a partical of mass M and move due to external force applied on it.
o
o
Conservation of linear momentum : Ifõ F ext = 0 then dt = 0
d p
ˆ
o o
\ M a cm = 0
JG
\ P = constant \ a cm = 0
o o
M d ( v cm ) = 0 \ v cm = constant.
dt

ˆ The particles of system having individual change in momentum but the total momentum of system
remains constant.

(16) Two particle of mass 2 kg and 4 kg move on a linear path in opposite direction with velocity
2 ms–1 and 3 ms–1, then the velocity of centre of mass of system is ...... .

(A) 3 ms–1 in the direction of first object (B) 43 ms–1 in the direction of first object
8

(C) 43 ms–1 in the direction of second object (D) 3 ms–1 in the direction of second object
8

76
(17) The distances of two object of masses m1 and m2 from the centre of mass of the system are r1
and r2. Due to their gravitational forces the acceleration produced in the object m2 is ...... .
m1 G m1 G m1 G m1 G
(A) ( r  r )2 (B) r 2 (C) ( r – r )2 (D) ( r  r )2
1 2 2 1 2 1 2

(18) A gun of mass M put on a friction less horizontal surface. A bullet of mass m is fired from it, the
distance travel by the gun when the distance travelled by the bullet is x ...... .

§ mM · § · § m· § M–m·
(A) ¨ m ¸ x (B) ¨ m  M ¸ x (C) ¨ M ¸ x (D) ¨ m ¸ x
m
© ¹ © ¹ © ¹ © ¹

(19) A stone of mass 90 kg tied with a mass less thread of length 10 m and on the other hand a person
of mass 60 kg pull this stone. If the surface is frictionless, at which distance the stone and person
can meet to eachother ?
(A) At 4m distance from the person (B) At 4m distance from the stone
(C) At 5m distance from the person (D) Can not meet
(20) A freely falling object A suddenly breaks into two fragments. The mass of first fragment is 4 M
3

and the other is M4


and they are freely fall. After breaking the centre of mass of object A is ...... .
(A) shifted towards heavy fragment.
(B) shifted towards lighter fragment.
(C) move on original path.
(D) shifted in which direction depands on at what height the fragment are made .
(21) When a object thrown upward it explodes in to two part at a point P of certain height . The mass
of bigger piece is four times more than the smaller piece .When the position of small piece is on
the right side of point P is 12 cm, the position of big piece from point P is ...... .
(A) 3 cm on right side (B) 3 cm on left side (C) 4 cm right side (D) 4 cm on left side
(22) The four forces are applied on a object of 2.6 kg as shown in figure. Find out the magnitude of
accelaration of the centre of mass of object . Y
(A) 2 ms–2 (B) 3 ms–2 6N
4N

7N 30°
(C) 4 ms –2
(D) 3.5 ms –2
X
°

O
30

(23) The particles of mass m, 2m, 3m.... nm are placed on a line and 30°
their position from x = 0 is x, 2x, 3x, .... nx, then find out the
2N
position of centre of mass from x = 0.
(2 n  1) (2 n  1) (2 n  1)
(A) (2n + 1)x (B) x (C) x (D) x
2 3 4

(24) What is the displacement of big block (incline ) when small block move and
reach at the bottom of incline ? The mass of big block is 10 times more than the
small block. All the surfaces are friction less.
(A) 0.1 m (B) 0.2 m
(C) 0.01 m (D) 0.02 m 1.1 m

77
(25) A object of mass m moves on X-axis with velocity 5 ms–1 collides with a stationary object of
mass double then it. Due to this the big object explodes in to two pieces of same mass and the
small object is comes in to rest . If one piece move on Y-axis with velocity 3 ms–1 than the
velocity of second piece is ...... .

(A) 24 ms
–1
(B) 4 ms–1 (C) 34 ms
–1
(D) 2 ms–1

(26) A simple pendulum made of a wooden block of mass 500 g .A bullet of


mass 50 g fired from downward side of the block it comes into rest
after entering in to the block. If the displacement of block is 1.1 m M
vertically then the speed of bullet ...... . (g = 10 ms–2 ) v
m

(A) 50 2 ms–1 (B) 51 2 ms–1 (C) 51.6 ms–1 (D) 50 ms–1

(27) Two square plates of length d and a circular plate of


diameter d place as shown in figure . If the mass distribution
of all three plates are same than the position of the centre of
mass of the system with respect to centre of first plate is
...... .

(A) < d (B) > d (C) = d (D) = 1.5 d


(28) Two spheres of mass 5 kg and 10 kg having same radius placed on a horizontal surface. A
sphere of 5 kg mass moves away up to 8 cm from the second sphere. Then how much
displacement of heavy sphere is required so that the centre of mass of the system is shifted 1 cm
towards the centre of lighter sphere.
(A) 2.5 cm towards lighter sphere (B) 5.5 cm towards lighter sphere
(C) 5.5 cm away from lighter sphere (D) 2.5 cm away from lighter sphere
(29) At the two ends A and B of a stationary boat of mass 50 kg and length 5 m , two person of
mass 40 kg and 60 kg are sit. If they meet at the centre of boat for discussion then the
displacement of boad ...... . ( neglect the firiction of water )
(A) 0.5 m, towards A (B) 0.3 m, towards A (C) 0.3 m, towards B (D) 0
(30) A symmetric rectangular box of mass M and length L is placed stationary m
on a frictionless surface. on the bottom of this rectangular a slot is given at
the centre. As shown is figure a sphere of mass m is fix at a rolling comes
in to the rest in the slot then what is the displacement of box ? L
M m 2 (M  m)
(A) M  m (B) L (C) L (D) 2(M  m)
mL mL
m m
(31) A person with a bag is fall down from a tower of height 100 m. The mass of person and bag is
50 kg and 20 kg respectively. If the person wants to landing in to a lake situated at 0.5 m
distance from the bottom of tower, for which velocity the bag can thrown in opposite to the lake
when he come at 20 m downward from the top of tower. (g = 10 ms–2)
(A) 0.5 ms–1 (B) 0.25 ms–1 (C) 1 ms–1 (D) 0.1 ms–1
Ans. : 16 (C), 17 (D), 18 (C), 19 (B), 20 (C), 21 (B), 22 (B), 23 (C), 24 (A), 25 (C), 26 (C),
27 (C), 28 (D), 29 (B), 30 (D), 31 (A)

78
Rotational Motion
Angular displacement :
linear displacement
Q Angular displacement =
radius

q = r
d
q P
O r
  
\ linear displacement d = T × r
ˆ Angular displacement is a vector quantity and its direction can be find out by using right hand
screw rule.
ˆ It is axial vector, so its direction is along the rotational axis.
ˆ Its unit is radian and revolution.
ˆ 1 revolution = 2p rad = 360°
ˆ In case of the fixed rotational axis the angular displacement is different.
Angular Velocity :
o
ˆ If the angular displacement is 'T in time interval Dt then,
angular displacement
average angular velocity =
time
o o
'T T T
\  Z ! = 't = 2 1
't

o o
dT
ˆ Instantaneous angular velocity Z = dt
ˆ Unit : rad s–1
ˆ direction can be find out according to right hand rule.
o
ˆ Z is also axial vector so its direction is along the rotational axis.
o o o
ˆ Relation with linear velocity : v = Z × r
ˆ The magnitude of angular velocity is called angular speed.

w = 2 S = 2pf
T
Angular acceleration :
o
ˆ The change in angular velocity is D Z in time interval Dt then

o o
Average angular acceleration  D ! = 'Z
't
o o
o
dZ d2 T
ˆ Instantaneous angular acceleration D = =
dt dt 2
ˆ Unit : rads–2

ˆ Direction : In the direction of change in angular velocity.

79
 Relation with linear acceleration :
o o o o o
a = D × r + Z × v
o o
ˆ The direction of D × r is found to be along the tangent to the circular path. Hence it is called
the tangential component aT of the linear acceleration. It is responsible for to change only the
direction of motion.
o o o o
ˆ The direction of Z × v is found to be the radial direction towards the centre. Hence Z × v is
called the radial component ar. It is responsible for to change only the magnitude of motion.
o o o
ar = Z × v G
aT
ar = wv (q = 90°)
v2 o
ar = r

q
a
o
= w2r
o
ar
o o
ˆ a = aT + ar O
o
|a| = aT 2  ar 2

= D 2 r 2  Z2v 2

= r D 2  Z4

o
o
ˆ If a makes an angle q with at then,
Z2
tan q = a = D
at
r
ˆ If a = constant then
§ Z  Z0 · Z  Z0
(1) q = ¨ ¸t (2) a = (3) w = w0 + at
© 2 ¹ t

(4) q = w0t + 1 at2 (5) 2aq = w2 – w02 (6) qnth = w0 + 1 a (2n –1)
2 2
(32) The radius of front and back wheels of a bicycle is r1 and r2 respectively, where r1 = 2r2. If the
velocity of the points of both the wheels in contact with ground is v1 and v2 respectively than ......
(A) v1 = 2v2 (B) v2 = 2v1 (C) v1 > v2 (D) v2 = v1
(33) A particle moves with constant tangential component of acceleration on a circular path of radius
10 m. After 2.5 rotation the velocity of particle is 50 ms–1 tengential acceleration is ...... .
S
(A) 25 ms–2 (B) 25 rad s–2 (C) 2500 p2 ms–2 (D) 2500 p2 rad s–2
(34) The radius of a wheel is 2 m and its one point is in contact with ground. When the wheel
complete half rotation the linear displacement of the particle is ...... .

(A) 1 m (B) 2 4 S 2 (C) 2 2 S2 (D) 4 4 S 2


(35) Initial angular speed of a particle is 2 rad s–1 and constant angular acceleration is 3 rad s–2, then
after 4 s its angular displacement is ...... rad.
(A) 10 (B) 32 (C) 14 (D) 18

80
(36) As shown in figure a disc is rolling without slipping on a horizontal surface. B is
the centre of it and AB = BC. If the velocity of particles A, B and C are vA, vB C
w
B
and vC respectively then ...... . A
(A) vC < vB < vA (B) vC = vA = 2vB
(C) vC > vB > vA (D) vC < vB > vA
(37) A fan initially at rest get the angular speed of 500 rpm in 4 s with constant angular
acceleration.Then the time required to get 250 rpm speed from rest position is ...... .
(A) 3 s (B) 2.5 s (C) 2 s (D) 1.8 s
(38) A wheel intially at rest get 3 rad angular displacement in 1st s with constant angular
acceleration, then the angular displacement in 2st s is ...... .
(A) 12 rad (B) 15 rad (C) 9 rad (D) 6 rad
(39) A wheel acquires angular displacement of 50 rotation in 4 sec, then after 5sec its angular speed
is ...... rad s–1.
(A) 40 p (B) 50 p (C) 30 p (D) 50
(40) A wheel intially at rest moves with constant angular acceleration 3 rad, s–2 after this it moves
with constant angular speed for 8 s and then in 8 s it come into the rest with constant angular
retardation. Then total angular displacement of wheel is ...... rad
(A) 384 (B) 284 (C) 256 (D) 356
(41) A wheel rotating with 5 rad s–2 angular acceleration along its axis. After 2 s of motion the magnitude
of radial and tangential componant of a particle at a distance 2 cm from the axis is ...... and ......
cms–2.
(A) 50, 5 (B) 25, 10 (C) 25, 5 (D) 50, 10
Ans. : 32 (D), 33 (A), 34 (B), 35 (B), 36 (C), 37 (C), 38 (C), 39 (B), 40 (A), 41 (D)
Moment of Inertia
‘A characteristic of a object which oppose the change of motion of object either at rest or move with
constant circular motion’
ˆ For a particle I = mr2
where r = perpendicular distance of particle from the axis

¦ mi ri 2
n
ˆ For a system of particle (ununiform distribution) I =
i 1

ˆ For a system of particle (uniform distribution) I = ³ r 2 dm

ˆ SI unit is kg m2 and dimensional formula is M1L2T0


ˆ It depends on the selection of axis and the distribution of mass about it.
ˆ Moment of intertia is a tensor physical quantity.
ˆ Inertia of object is constant while moment of inertia is different.
Radius of Gyration :
‘The perpendicular distance from the axis of a particle at which the total mass of the object is
concentric’
‘The rms perpendicular distances of particles from the axis.’

81
r12  r22  ..... rn2
ˆ For the system of particles k =
n

ˆ Moment of inertia I = Mk2


ˆ Radius of gyration does not depends upon the mass of object but depend upon the volume of it.
Theorem of parallel axis :
I = Ic + Md2
I = Moment of inertia of the body about any axis
Ic = Moment of inertia of the body through its center about any axis.
M = Total mass of the substance.
d = Perpendicular distance between the two axis.
Theorem of perpendicular axis :
If we take X and Y axis in the plane of a planar body then,
Iz = Ix + Iy
Special note :
ˆ (1) For a symmetric two-dimensional object moment of inertia of any axis passing from its
centre of mass and parallel to its plane is same. So, in this case it is not necessary that
two axis are perpendicular to each other.
(2) The intersection point of two perpendicular axis is not always the center of mass of the
object, it may be the point outside the object.
ˆ The moment of inertia with respect to the axis perpendicular to the joining line of two particles is
I = m1r12 + m2r22
ˆ According to the definition of centre of mass, m1r1 = m2r2 and r1+ r2 = r then,

r1 = and r2 = m1r m1 c.m. m2


m1  m2
m2 r
m1  m2 r1 r2
r

§ mm ·
\ I = ¨ 1 2 ¸ r2
© m1  m2 ¹

\ I = mr2 and m = m  m
m1m2
1 2

= Reduced mass
m < m1 and m < m2
ˆ Torque :
‘Torque is the moment of force with respect to a given reference point’
ˆ Torque acting on the particle is
o o o
W = r× F
t = r F sin q
o o
where q = angle between r and F

82
o
r = position vector
r sin q = perpendicular distance of line of action of force = inertia = lever arm
o
ˆ according to the right hand screw rule the direction of torques ( W ) is perpendicular to the plane
o o
formed by r and F
ˆ SI unit : Nm or J
ˆ CGS unit : dyne cm
For a system of particles, R
o l1 l2
o o o
W = W1 + W2 + .... + W n
ˆ For equilibrium of rod F1 F2

o o
W1 + W2 = 0
\ F1l1 – F2l2 = 0
F1l1 = F2l2
o o o o
ˆ F = m a resembles with the equation W = I D
Couple :
Two forces of equal magnitude and opposite directions which are not colinear form a couple.
Moment of couple = magnitude of any one of the two force × perpendicular distance between
the two forces.
ˆ In a torque applied on a object one force is external while the other force are external.
In couple both the force are external.
Angular Momentum :
o o o
ˆ Angular momentum of a particle is l = r × P
l = r psin q
o o
where q = angle between r and p
p = mv = linear momentum of particle
ˆ Angular momentum is axis vector. Its direction can be find out by using right hand screw rule.
ˆ Unit : J s or erg s
ˆ Angular momentum of a particle = Linear momentum × Perpendicular distance of the vector of
linear momentum = moment of linear momentum
o o
ˆ For circular motion r ^ p so L = rp = mvr = mwr2. = Iw

Also, d L = I dt = I a = t
dZ
dt
ˆ If large amount of torque applied for small time interval then ‘Angular impulse of torque is

³ dt = W av Dt = D L = change in angular momentum.


o o
t2
o o o
J = ³ W av dt = W av
t1

Law of conservation of angular momentum :


o o
ˆ W = dL
dt

83
If resultant torque is zero
o o
d L = 0 Þ L = constant
dt
o o o
\ L 1 + L 2 + ..... + L n = constant
L = Iw = constant
\ w µ 1 from this it is clear that
I
when a planet come near to the sun its moment of inertia is decrease. So angular speed is
increase (I µ r2)
ˆ If the radius of planet suddenly increase by x time then its periodic time
T µ I Þ T µ R2
So,T2 = x2T1
ˆ The work due to torque W = t q
T2
for variable torque W = ³ WdT
T1

Rotational kinetic energy KR = 12 Iw2 = 12 Lw = L


2
ˆ 2I

ˆ Power P = tw = I µ w = Iw d Z
dt

o o
P = W . Z (In vector form)
(42) The object of mass 3kg, 5kg, 6kg, and 2 kg are placed at the vertices A, A B

B, C and D of a square length 1 m. Then moment of inertia with respect to


axis perpendicular to its plane and passing through it centre is ...... .
O
(A) 6 kg m2 (B) 8 kg m2
(C) 4 kg m2 (D) 16 kg m2 D C

(43) In above example, find the moment of inertia with respect to AD axis of square.
(A) 11 kg m2 (B) 13 kg m2 (C) 16 kg m2 (D) 8 kg m2
(44) In above example, the moment of inertia with respect to axis BD is ...... .
(A) 16 kg m2 (B) 8 kg m2 (C) 4.5 kg m2 (D) 9 kg m2
(45) The mass of a disc is 10 kg and radius is 0.2 m. It moves with 200 rpm angular speed along the
axis passing through its centre and perpendicular to its plane. To let it in stationary position in 15
sec the tangential force required on the circumfrance is ...... .
(A) 0.5 p N (B) 0.4 p N (C) 0.2 p N (D) 0.44 p N
(46) A disc of mass 5 kg and radius 0.4 m rotates along its axis with 30 rpm. To incerase its angular
momentum 20 % in 5 s required torque is ...... Nm.
(A) 2.6 p (B) 0.16 p (C) 1.6 p (D) 0.016 p

84
(47) When 1500 J energy given to a object its angular speed increase by 1000 rpm to 2500 rpm then
the moment of inertia of a object is ...... kg m2
(A) 0.052 (B) 0.52 (C) 52 (D) 0.026
(48) Two disc rotating about an axis passing through its centre and perpendicular to its plane. The
mass of bigger disc is 2 kg and radius is 0.2 m and angular speed 50 rad s–1 while for smaller
disc the mass, radius and angular speed is 4 kg, 0.15 m and 250 rads–1 respectively. The axis of
both the disc is coinside when the smaller disc get in connect to bigger disc, then the angular
speed of a system of two disc is ...... rads–1.
(A) 200 (B) 140 (C) 153 (D) 105
(49) A particle start to move with constant velocity from the point (10, 10) cm and move parallel to
y-axis in negative direction, then its angular momentum with respect to orgin is ...... .
(A) zero (B) constant
(C) increase (D) intially increase and then decrease
(50) The inner and outer radius of a hollow cylinder is a and b respectively. The moment of inertia
of it with respect to its axis is ...... (density of material is r, length of cylinder is L)
§ a4  b4 · § b2 – a 2 · § b4 – a 4 ·
(A) 2pLr (a + b )
2 2
(B) 2pLr ¨¨ ¸
¸ (C) 2pLr ¨¨ 2 ¸
¸ (D) 2pLr ¨¨ 4 ¸
¸
©
4
¹ © ¹ © ¹

(51) If the sixth portion of a disc of mass M and radius R of uniform density distribution cut then its
moment of inertia with respect to the axis passing through the centre of original disc and
perpendiculr to the plane is ...... .
(A) 12 MR2 (B) 4 MR2 (C) 3 MR2 (D) 1 MR2
1 1
6
(52) A circle made of a wire of length l and uniform linear density l, then its moment of inertia with
respect to axis parallel to its plane and passing tangentially is ...... .
8 S 3Ol 2 3Ol 3 Ol 3
2
(A) 3 Ol 3 (B) 8S2 (C) (D)
8S 2 8S 2
(53) The mass of a semicircular ring of radius R is M. The moment of inertia with respect to axis
passing through the centre of original of ring and perpendicular to its plane is ...... .

(B) MR
MR 2 2 MR 2
(A) 2 (C) 4 (D) non of the above

(54) A square made of four rods of length l and mass m. The moment of inertia with respect to the
axis passing through its centre and perpendicular to the plane is ...... .
(A) 4 ml2 (B) 3 ml2 (C) 1 ml2 (D) 3 ml2
1 2
3 6
(55) The moment of inertia of a square plate of length l and mass m with respect to axis passing through any
one corner and perpendicular to its plane is ...... .
(A) 4 ml2 (B) 3 ml2 (C) 1 ml2 (D) 2 ml2
1
3 6 3
(56) If the moment of inertia of a triangle plate of uniform density with A

respect to sides AB, BC and CA is I1, I2 and I3 respectively then, 4 cm


(A) I2 < I1 (B) I1 + I2 = I3
(C) I2 > I1 (D) I3 is maximum
B 3 cm C

85
(57) Y From a circular disc of radius R and mass 4M, a small disc
of radius 4 is removed from the disc. The moment of inertia
R R
4
of the remaining disc about an axis perpendicular to the plane
O X of the disc and passing through O is ...... .
R
(A) 2.43 MR2 (B) 1.23 MR2
(C) 1.4 MR2 (D) 1.43 MR2
(58) The moment of inertia of thin uniform rod of
length l and mass m about an axis pp'
shown in figure is ...... . P'

ml 2 sin 2 T ml 2 sin 2 T
(A) (B) O q
8 12 A B

(C) ml cos T (D) ml cos T


2 2 2 2
P
8 12
(59) A thin wire of uniform density is having length 4l and mass m is bent to form a rectangle ABCD.
Here side AB is four times the side BC. Then moment of inertia about an axis passing through
AD is ...... .
(A) 0.3 ml2 (B) 0.5 ml2 (C) 0.4 ml2 (D) 0.2 ml2
(60) A solid metallic sphere is rotating about an axis passing through its diameter. Suddenly its volume
is increased by 6 %, then the change in its angular speed will be ...... .
(A) –2 % (B) +2 % (C) – 4 % (D) +4 %
(61) The moment of inertia of a disc of radius 1.5m is 150 kg m . The disc can rotate frictionless
2.

about vertical axis. A man of mass 60 kg is standing on its edge. Now man walks with speed of
2 ms–1 on its edge, then angular speed of a disc will be...... .

(A) 1 rad s–1 (B) S rad s–1 (C) S2 rad s–1 (D) 1.2 rad s–1
1.2

(62) A hollow sphere is placed on a rough horizontal surface. On appling force F on it in horizontal
direction it rolls down without slipping on the surface. Then its angular acceleration is ...... .
F
(A) 5 MR (B) 5 MR
6 6 F
R

(C) MR (D) 5 MR
3F 3 F

P
(63) A uniform rod of mass m and length l is suspended by means of two right
inextensible strings as shown in figure. If one string is cut out then the tension in
the other string is ......
l (A) 4
mg
(B) 2 mg (C) mg (D) 2
mg

(64) If a ball of mass m and radius r is hitted at the height h from its centre, G
h J
§ hv0 ·
velocity gained by it is v0.. Then angular speed obtained by it is ...... ¨ ¸ r
© r2 ¹

(A) 5 (B) 2 (C) 4 (D) 45


2 5 5

86
(65) A weightless rod AB of length 2l is placed vertically on a frictionless horizontal surface. Two
sphere of mass m are attached to two ends of rod AB. impulse of force J is applied parallel to the
surface at the point A and also perpendicular to rod AB, then velocity of end A is ...... .
(A) m (B) 2m (C) 0 (D) m
J J 2J

(66) As shown in the figure find the moment of inertia about x x' axis
of a system of two rods of mass m each and length l each placed x'
perpendicular to each other.
ml 2
(A)
ml 2
(B) O
6 2

ml 2 x
(C) ml (D)
2
12 6 2
(67) Four dises of radius R are cut from a thin square plate of side 4R and mass M. The moment of
inertia of the remaing portion about an axis passing through its centre and perpendicular to its
plane is ...... .
5S
(A) 8 MR2 (B) 3 MR2
8

8 5S MR
(C) 3  8 2
3 8
(D) 8  5S MR2
4R

(68) Two sphere A and B connected to two ends of a 1 m long thin rod and applied difference inpulse of
force at the same time. The instantaneous velocities gained by them are shown in the figure. Then
the angular speed of sphere B with respect to A will be ...... .

(A) 8 3 rad s–1


8ms –1
8
(B) 3 rad s–1 v

60° 30°

(C) 4 3 rad s–1


4
(D) 3 rad s
–1 A 1m B

(69) A turn table of radius r is rotating about vertical axis passing through its centre. A bullet of mass
m is fired in an opposite direction of turn table’s motion from the gun attatched at its edge in the
direction tangent to the edge of table. If the speed of bullet is v, then increase in angular speed of
turn table is ...... . (I0 = moment of inertia of a system of turn table + gun )

(A) (B) (C) I (D) v


mvr 2 mvr mvr
I0  mr 2 I0  mr 2 0 2r
B

(70) A rod of uniform density is having mass M and length L. It is placed vertical on a v0
C

smooth horizontal surface. A particle of mass m moving with velocity v0 collides at m x

point C of the rod and becomes stationary. What should be the height of this point from
the centre of the rod so that the end A of the rod remain stationary ? A

(A) L3 (B) (C) L6 (D) L4


2L
O 5
(71) A right triangle plate ABC of mass M is free to rotate in the vertical plane
A l B about a fixed horizontal axis through A. It is support by a string such that
l the side AB is horizontal. The reaction at the support A is ...... .

C
Mg 2M g
(A) (B) (C) Mg (D) 2 Mg
3
3 3

87
(72) A disc of radius r is formed from a solid sphere of mass M and radius R. If the moment of
inertia of the disc about an axis passing through its edge and perpendicular to its plane is equal to
the moment of inertia of the sphere about its diameter, then r = ......
7
(A) R (B) R (C) R4 (D) 15 R
2 4
15 15
(73) The moment of inertia of a wheel about an axis passing through its centre and perpendicular to its
plane is 2.5 kgm2. It is rotating about this axis with angular speed of 90 rpm. Then the torque
required to stop rotating it in 0.5 min is ...... Nm
S S S
(A) 2 (B) 4 (C) p (D) 3
y p'
(74) The moment of inertia about an axis passing through X axis of a A B
square plate of uniform density is I. Then moment of inertia about q
X
an axis PP' making an angle q with Y-axis will be ...... . x

(A) 2I (B) I cos2 q


(C) I (D) I sin2 q D
p
C

(75) Two spheres of mass 2 kg and 3 kg are attatched to two ends of a rod of negligible mass. Length
of rod is 2m.The rod is rotating with uniform angular speed about an axis perpendicular to its
length. What should be the position of axis of rotation so that minimum work is required to be done
to rotate it?
(A) 1.2 m away from 2 kg (B) 1.2 m away from 3 kg
(C) 0.8 m away from 2 kg (D) 0.6 m away from 3 kg 1
2
(76) A square plate is shown in the figure. The moment of inertia about an
axis passing through plane of plate from 1, 2, 3 and 4 are I1, I2, I3 and
I4 respectively. which of the following is not the moment of inertia about 0
3
an axis passing through its centre and perpendicular to its plane ?
(A) I1 + I2 (B) I1 + I3
(C) I2 + I4 (D) All of the above. 4

(77) A ring of mass M has radius R. It is rotating with uniform angular speed w about its axis. If
two particles each of mass m are placed at diametrically opposite points, then its angular speed
2
will be ...... .
M m M m
(A) (B) (C) (D)
M 2M
w w w w
M M m 2 M m M
(78) A ring of mass M and radius R rolls down on a horizontal surface
w with angular speed w. The angular momentum of ring with respect to
A point A is .......

(A) MR2w (B) 12 MR2w (C) 2MR2w (D) 2 MR2w


3

(79) A disc of uniform density has mass M and radius R. It is placed in a XY


plane in such a way that its centre coincides with origin. If its moment of y y
inertia about an axis passing through point (2R, O) is equal to that about
an axis passing through the point (y, O) then, y = ...... .
2R
R
(A) R (B) R
17 15
4 2 O x

(C) R (D) R
17 17
2 2

88
(80) l A cube shaped block of mass m and side l is moving with
v velocity v on a frictionless horizontal surface. It collides with
P buldged part at point P and then after it rolls. Then its angular
speed after the collision will be ...... .

(A) 4 l (B) 2 l (C) 4 v (D) 4 l


3 v 3 v 3 l 5 v

Ans. : 42 (B), 43 (A), 44 (C), 45 (D), 46 (B), 47 (A), 48 (C), 49 (B), 50 (D), 51 (A), 52 (C), 53 (B),
54 (A), 55 (D), 56 (C), 57 (D), 58 (B), 59 (A), 60 (C), 61 (D), 62 (B), 63 (A), 64 (B), 65 (A),
66 (C), 67 (D), 68 (B), 69 (A), 70 (C), 71 (B), 72 (D), 73 (B), 74 (C), 75 (A), 76 (D), 77 (B),
78 (C), 79 (D), 80 (A)
ˆ If the body is slipping
The friction between the body and the surface is zero

Translatory kinetic energy, KT = 12 mv2

ˆ If the body rotates about fixed axis of rotation, then rotatory kinetic energy KR = 12 Iw2

v2 § k2 ·
= 12 (mk2) 2 = 12 mv2 ¨ 2 ¸
R ©R ¹

§ k2 ·
\ KR = ¨ ¸ (KT)
© R2 ¹
ˆ If the body performes rotational motion and its axis of rotation performes linear motion, the body
is said to be rolling body. Friction between the body and surface, F ¹ 0
ª 2 º
Net kinetic energy KNet = KT + KR = 12 mv2 + 12 Iw2 = 12 mv2 «¬1  R 2 »¼
k

ˆ If the object rolls across a surface in such a way that there is no relative motion of object and
surface at the point of contact, the motion is called rolling without slipping, even in the presence
of friction.
ˆ The works done against friction = energy lost = 0

Net kinetic energy, KNet = 12 mv2 + 12 Iw2

= 12 m R2w2 + 12 Iw2

= 12 [I + mR2]w2

KNet = 12 Ipw2
where I = moment of inertia of rolling body about its centre.
I P = moment of inertia of rolling body about point of contact P.
ˆ In case of rolling without slipping, all points of a rigid body have same angular speed but
different linear speed.
v v 2v
v=0 v v
v
v v v 2v
v=0 v
pure translation pure rotation pure rolling

89
ˆ Rolling without slipping on an inclined plane.
Potential energy at height h = Total kinetic energy at the bottom at an inclined plane.
ª k2 º B
mgh = 12 mv2 «1  2 »
¬ R ¼

ª º2 h
1

« »
ˆ Linear velocity at the bottom of an inclined plane. v = « 2 gh »

d
« k2 »
«1 2
q
»
¬ R ¼
A C

g sin T
ˆ constant acceleration in motion, a =
1 k
2

R2
ˆ Decrease in acceleration due to friction,
a' = g sin q – a
g sin T
=
1 R
2

k2
ˆ Frictional force = F = ma'
ˆ Time taken to reach the bottom of an inclined plane,

ª § ·º 2
1

t = 1 « 2h ¨1  k 2 ¸»
2

sin T ¬ g © R ¹¼

k2
ˆ Here the factor is a measure of moment of inertia of a body and its value is constant for
R2
given shape of the body and it does not depend on the mass and radius of a body.
ˆ The values of velocity, linear acceleration and time of descent ( for a given inclined plane ) is same for
bodies of small or large dimensions, for lighter or havier bodies of equal shape.
k2
ˆ The substance, which has more moment of inertia the ratio , will be more, so magnitude of
R2
velocity and acceleration will be decreased while value of time will be increased.
ˆ For a solid and hollow body of same shape, Isolid < Ihollow. Hence the velocity and acceleration of
a solid body are larger and time of descent is lesser. It means solid body will reach the bottom
first with greater velocity.
ˆ If given body rolls down on an inclined plane of different angle of inclination.
v µ q°, aµq t µ q–1
ˆ The necessary condition for a body rolling without slipping is

§ k2 ·
¨ ¸ tan q
m³ ¨ k2 + R 2 ¸
© ¹

ˆ When a body of mass m is tied to one end of a string which is wound around the solid
body[cylinder, pulley, disc], when the mass is released,it falls vertically downwards, then
90
ˆ Downward acceleration

a =
g
§ 2 ·
1 M¨ k ¸
m © R2 ¹

M = mass of a rigid body (rotating body)


k = Radius of gyration of a rotating body h
ˆ Tension in string T

ª º m
T = mg « »
I
¬« I  m R ¼»
2

I = moment of inertia of a rotating body. mg


ˆ Velocity of a body after travelling a distance h in downward direction,

v =
2 gh
§ 2 ·
1 M¨ k ¸
m © R2 ¹

ˆ Time taken to travel a distance h,


2h ª M § k 2 ·º
t = g «1  m ¨ 2 ¸ »
¬ © R ¹¼

2mgh 2 gh
Angular speed of a rotating body, w = I  mR 2 = R  M (k 2 )
ˆ 2
m
ˆ when a body allows to oscillate about an axis passing from any point except centre of the body
of uniform shape, it is called the compound pendulum.
l2  k 2
Its time period is given by, T = 2 p where L =
L
g l
l = distance between the point of suspension to centre of mass of a body,
k = radius of gyration of a body.
(81) A solid sphere, a hollow sphere and a ring are released from an inclined plane (frictionless) of
height h, which object will reach the bottom first ?
(A) ring (B) solid sphere
(C) hollow sphere. (D) All of them reach the bottom at the same time.
(82) A hollow sphere of mass M and radius R rolls on a horizontal plane without slipping with
the velocity v. Now, it the surface rolls up vertically, the maximum height the body would attain
will be ...... .
2 2
v2 2
(A) 5 (B) 6 (C) 10 (D) 7
6 v 5 v 7 10 v
g g g g

(83) A small object of uniform density rolls up a curved surface with an initial velocity v. It reaches up
3v 2
to a maximum height of 4 g with respect to the initial position. The object is ...... .

(A) Ring (B) hollow sphere (C) Disc (D) hollow cylinder

91
(84) A solid cylinder rolls down an frictionless inclined plane. If angle of inclination with horizontal is q
and the co-efficient of friction between the body and surface is m, then
(A) Frictional force equal to mmg cos q
(B) Frictional force will become resistive force.
(C) Frictional force opposes the linear motion and helps to relational motion.
(D) Frictional force increases as q decreases.
(85) A small sphere of radius r is kept in a hemispherical bowl of radius R. It A
is released from point A. The angular speed of the sphere when it R
reaches at the bottom will be ......
B

§ ·2 § ·2 § ·2 § g ·2
1 1 1 1
(A) ¨ 5 (B) ¨ 7 (C) ¨ 2 (D) ¨ 10
 r ¸¹  r ¸¹  r ¸¹  r ¸¹
g g g
© 7 R © 10 R © 5 R © 7 R

(86) A hollow sphere of radius r rolls down without slipping on inclined plane of height h. A loop of
radius R is formed at the end of an inclined plane. What should be the minimum height of inclined
plane to be kept so the hollow sphere can complete the rotation in a loop ?
(A) 2.7 R
(B) 1.7 R
h
(C) 3 R2
R (D) 2.3 R
(87) As shown in the figure two inclined planes are A C
same in all aspects. In which the AB part is
rough and the BC part is smooth. The kinetic
energy of the body at the bottom of inclined B B
h
h
plane is
(A) more in (i) (B) more in (ii)
C
q q
A
(C) equal in both (i) (ii) (D) information is incomplate (i) (ii)
(88) A solid sphere and a solid cylinder have equal mass. They rolled down on inclined planes of height
h1 and h2 respectively. To have equal velocities for the two objects at the bottom of the inclined
planes the ratio of heights h1 : h2 should be = ......

(A) 14 (B) 5 (C) 14 (D) 7


15 7 5

A
15
(89) A solid sphere rolls down without slipping on an inclined plane of
height 2h. The surface AB is rough and the suface BC is smooth.
The ratio of linear kinetic energy to rotational kinetic energy of
2h B
sphere at a point C is ...... .
(A) 4 (B) 6

(C) 16 (D) 14 q
C
(90) A horizontal turn table is rotating with constant angular speed w. A light coin is placed at distance
r from the centre of the turn table. Then for which of the following condition, the coin will keep
rotating with the turn table ? (co-efficient of static friction is m)
Z2 Pg Pg
(A) r = mgw2 (B) r < Pg (C) r ³ (D) r £
Z 2
Z2
Ans. : 81 (D), 82 (B), 83 (C), 84 (C), 85 (D), 86 (A), 87 (A), 88 (C), 89 (B), 90 (D)

92
Assertion - Reason type Question :
Instruction : Read assertion and reason carefully, select proper option from given below.
(a) Both assertion and reason are true and reason explains the assertion.
(b) Both assertion and reason are true but reason does not explain the assertion.
(c) Assertion is true but reason is false.
(d) Assertion is false and reason is true.
(91) Assertion : A particle is moving on a straight line with a uniform velocity, its angular momentum
is always zero.
o o o
Reason : The angular momentum of a particle is given by the formula l = r × p
(A) a (B) b (C) c (D) d
(92) Assertion : A hollow cylinder of metal and a solid cylinder of wood having equal mass and
dimensions rolled down simultaneously without slipping on an inclined plane. Then the hollow cylinder
will reach the bottom first.
Reason : According to law of conservation of kinetic energy, the kinetic energy of both the
cylinders is unequal at the bottom of the inclined plane.
(A) a (B) b (C) c (D) d
(93) Assertion : Inertia and moment of inertia are same quantities.
Reason : Inertia represents the capacity of a body to oppose its state of motion or rest.
(A) a (B) b (C) c (D) d
(94) Assertion : A ladder is more apt to slip, when you are high up on it, then when you just begin to climb.
Reason : At the high up on a ladder, the torque is large and on climbing up the torque is small.
(A) a (B) b (C) c (D) d
(95) Assertion : When there is no torque is acting on a body with respect to its centre of mass, its
speed remains constant.
Reason : The total linear momentum of an isolated system remains constant.
(A) a (B) b (C) c (D) d
(96) Assretion : If a wheel moves down on frictionless inclined, then it can slips only, cannot rolling.
Reason : For rolling the work against the frictional force is zero.
(A) a (B) b (C) c (D) d
(97) Assertion : A hollow cylinder is more stronger than a solid cylinder used as shaft in motor.
Reason : The value of torque is more for hollow cylinder as compared to solid cylinder for a
given value of angular displacement.
(A) a (B) b (C) c (D) d
(98) Assertion : The linear speed for all the particles of a rolling body is same.
Reason : The rotational motion does not affect the linear momentum of a rigid body.
(A) a (B) b (C) c (D) d
(99) Assertion : If the earth contracts to half than its initial value (keeping mass constant) the length
of the day decrease.
Reason : As the radius of the earth changes, the moment of inertia also changes.
(A) a (B) b (C) c (D) d
(100) Assertion : The value of radius of gyration is constant for a given body.
Reason : Radius of gyration means the rms distance of particles from axis of rotation.
(A) a (B) b (C) c (D) d

93
(101) A circular platform is free to rotate in horizontal plane is rotating with angular velocity w0. When a
child standing on a edge of turn table moves along a chord of the platform and reaches the other end
of a turn table.The angular velocity of the platform w will vary with time t as ......
w w w
w

(A) w0 (B) w0 (C) (D) w


w0
0

t O t O t t
(102) The graph between log L and log P is. (L is angular momentum, P = linear momentum)
log L log L
(A) (B)

log P log P

log L
log L

(C) (D)

log P log P
Ans. : 91 (D), 92 (D), 93 (D), 94 (A), 95 (D), 96 (B), 97 (A), 98 (D), 99 (A), 100 (D), 101 (B), 102 (D)
Match the columns :
(103) A object of mass 0.5 kg is projected in the direction making an angle of 45° with horizontal with
initial velocity of 10 2 ms–1. When it moves in the horizontal direction only, then ...... .
column-1 column-2 (A) i ® R ii ® P iii ® Q
with respect to point of
projection (P) 25 SI (B) i ® S ii ® P iii ® R
(i) Torque acting
on the body (Q) 50 SI (C) i ® Q ii ® P iii ® R
(ii) Angular momentum
of the body (R) 0.4 SI (D) i ® P ii ® Q iii ® S
(iii) Angular velocity of the body (S) None of these

(104) A disc rolls down without slipping on a horizontal surface.


The velocity of its centre of mass is v. If the velocity of O
A q
point A on its edge is vA and angle between OP and OA

is q as shown in the figure, then P

94
column-1 column-2
(i) q = 60° (P) vA = 2 v (A) i®P ii ® Q iii ® S iv ® R
(ii) q = 90° (Q) vA = v (B) i®S ii ® P iii ® Q iv ® R
(iii) q = 120° (R) vA = 2v (C) i®R ii ® Q iii ® P iv ® S
(iv) q = 180° (S) vA = 3 v (D) i®Q ii ® P iii ® S iv ® R
(105) If the resultant force acting on a rigid body is zero, then
column-1 column-2
(i) Linear velocity of centre of mass (P) P = 0
(ii) Angular velocity of the body (Q) Q = constant
(iii) The angular velocity
about an axis passing
from its centre of mass. (R) R = variable

(iv) The angular velocity


about an axis passing
from any point other
than centre of mass.

(A) i ® Q ii ® R iii ® R iv ® R (B) i ® R ii ® Q iii ® P iv ® R


(C) i ® Q ii ® R iii ® P iv ® R (D) i ® R ii ® R iii ® R iv ® R

Ans. : 103 (C), 104 (D), 105 (A)

95
8 Gravitation
Newton’s Universal law of Gravitation :
“Every particle in the universe attracts every other particle with a force which is directly proportional
to the product of their mass and inversely proportional to the square of the distance between them
and the direction of this force is along the line joining them.”
ˆ The magnitude of the gravitational force acting between two particles of mass m1 and m2 lying at
distance r from each other is,

F= (obeys inverse square law)


G m1m2
r2
Where G = universal constant of gravitation
its value is 6.67 × 10–11 Nm2 kg–2 and the dimensional formula is M–1L3T–2.
ˆ Definition of ‘G’ : The gravitational force of attraction between two objects of unit mass each and
placed unit distance apart is called the universal gravitational constant.
Gravitational force in vector form :
Gravitational force acting on the particle of mass m1 by the particle of mass m2 is,
o
=
G m1m2 
F 12 r 12
r2
o
Where r 12 is the unit vector in the direction of F 12 .


Similarly, the gravitational force acting on the particle of mass m2 by the particle of mass m1 is,
o o – G m1m2 
F 21 = or F 21 =
G m1 m2 r r 12
r2
21
2
r G
where 21 is the unit vector in the direction of F21 .
r

Here F 12 = – F 21 and | F 12 | = | F 21 |
o o o o

ˆ Important features of Gravitational force :


ˆ It is acting between any two bodies by virtue of their mass.
ˆ It is always attractive in nature.
ˆ The gravitational force between two objects is independent of intervening medium.
ˆ The gravitational forces are mutually interactive forces.
ˆ The gravitational force is a central force.
ˆ The gravitational force is a conservative force. The work done on the object by it does not depend
on the path taken but only depends on initial and final position. or The work done by it on closed
path is zero.
ˆ The gravitational force between two bodies is independent of the presence of other bodies.
(Two body force)
ˆ The gravitational force by a hollow spherical shell of uniform density on a particle out side the
shell is equal to the force which can be obtained by considering the entire mass of the shell as
concentrated on its centre.
ˆ The force on a particle at any point inside a hollow spherical shell of uniform density is zero.

96
(1) Calculate the value of gravitational force acting between two spheres each of mass 2 kg, when
their centres are 20 cm apart. (G = 6.67 × 10–11 Nm2kg–2)
(A) 6.67 × 10–9 N (B) 6.67 × 10–11 N (C) 6.67 × 10–7 N (D) 6.67 × 10–5 N
(2) Three uniform spheres, each having mass m and radius r, are kept in such a way that each
touches the other two. The magnitude of the gravitational force on any sphere due to the other
two is ......
G m2 2 G m2 3 G m2
(A) G m (B) (C) (D)
2

r2 4r 2 4r 2 4r 2
(3) Three masses, each equal to m, are placed at the three corners of a square of side l. The
magnitude of gravitational force on unit mass at the fourth corner will be ...... .

(A) G m (B) (C) G m 1  2 2 (D)


3Gm 3G m

3l 2 l 2 2
l l2
(4) Two identical solid brass spheres of radius R are placed in contact with each other. The
gravitational force between them is proportional to ......
(A) R2 (B) R–4 (C) R3 (D) R4
(5) Two point masses A and B having mass in the ratio 4 Ñ 5 are separated by a distance of 1 m.
When another point mass C of mass M is placed in between A and B, the gravitational force
between A and C is 15 times the gravitational force between B and C. Then the distance of C
from A is ......
(A) 2 m (B) 2 m (C) 1 m (D) 25 m
3 7 3
(6) The magnitude of gravitational force acting between two particles of mass m1 and m2 separated by a
distance r is F. What would be the change in the distance between them so that the gravitational
force acting between them will become 2F ?
(A) decreased by 29.3 % (B) increased by 29.3 %
(C) decreased by 50 % (D) decreased by 25 %
(7) The gravitational force due to earth on a body of mass m at a height h from the Earth’s surface

is 13 times the force on it at sea level (at surface of the earth). Then h = ......
R
Where R = radius of the earth
(A) 0.414 (B) 0.732 (C) 0.500 (D) 0.314
(8) The distance of the centres of earth and moon is r. The mass of earth is 81 times the mass of
the moon. At what distance on the line joining their centres from the center of the earth, the
gravitational force on any object will be zero ?
(A) 0.9 r (B) 0.7 r (C) 0.5 r (D) 0.25 r
(9) Three particles each of mass m are placed at the three vertices of an equilateral triangle of side l.
What is the resultant gravitational force due to this system of particles acting on another particle of
mass (M) placed at the mid-point of any side ?

(A) 3GM m (B) 4GM m (C) GM m (D) 4GM m


4l 2 3l 2 4l 2 l2
(10) A mass M is split into two parts, m and M – m. If the gravitational force acting between the two

parts is maximum for a given distance, then the ratio m = ...... .


M
(A) 1 (B) 1 (C) 3 (D) 1
2 4 4 5

97
(11) The gravitational force acting between two spheres of mass m and M situated at a distance r in
air is F. Now these spheres are kept in the liquid of specific gravity 5 at a distance r, then the
gravitational force will be ......

(A) 5F (B) F (C) F (D) F


5 25
(12) The gravitational force by earth on a body of mass 1 kg at earth’s surface is 10 N. Then the
gravitational force on a satellite of mass 100 kg revolving around the earth in a orbit at average

distence 3 R from the centre of the earth will be ......


2
(R = radius of earth)
(A) 4.44 × 102 N (B) 6.66 × 102 N (C) 500 N (D) 3.33 × 102 N
(13) The centripetal force acting on a satellite orbiting around the earth is F and the gravitational force
acting on a satellite due to earth is also F. The resultant force acting on satellite will be ......

(A) F (B) 2F (C) Zero (D) 2F


(14) Two particles of equal mass m go round a circle of radius R under the action of their mutual gravita-
tional attraction. The speed of each particle is ......

(A) (B) (C) 1 G m (D) 1


Gm 4G m 1
2R R 2 R 2R Gm
(15) The gravitational force is a ...... force.
(A) conservative (B) non conservative (C) electrostatic (D) repulsive
Ans. : 1 (A), 2 (D), 3 (C), 4 (D), 5 (A), 6 (A), 7 (B), 8 (A), 9 (B), 10 (A), 11 (C), 12 (A), 13 (A),
14 (C), 15 (A)
Gravitational Acceleration
‘The acceleration produced in the body due to the gravitational force of the earth is called the
gravitational acceleration or the acceleration due to gravity (g).
The gravitational acceleration at a distance r (r > Re) from the centre of the earth is

g = G Me ; where_ Me = mass of the earth and Re = Radius of the earth.


r2
At the surface of the earth, r = Re

\g=
G Me
R e2
ˆ The value of the g does not depend on the mass, shape and size of the body but depends on the mass
of the earth and height or depth from the surface of the earth.
ˆ The value of 'g' at the surface of the earth is 9.8 ms–2.
ˆ 'g' is a vector quantity and its direction is always towards the centre of the earth.
Variations in Acceleration Due to Gravity
(1) Due to shape of the earth :
The earth is not completely spherical but is slightly bulged out at the equator and flattened at
the poles. The radius of the earth at equator is nearly 21 km more than the radius at the poles.
\ gpole > gequator ( more by 0.018 ms–2 approximately.)

98
(2) Variation in 'g' with altitude :
The gravitional acceleration at a height h from the surface of the earth is,

g(h) = OR
G Me
R e  h 2

g(h) = (for any height) OR


g
§ h ·
¨1  R ¸
2

© e ¹

g(h) = g §¨1 – 2h ·¸ (for h < < Re)


©R e ¹
for h < < Re,

The absolute decrease, D g = g – g(h) = R


2hg
e

'g
The fractional decrease, g = = 2h
g – g ( h)
g Re

The percentage decrease, 'g × 100 % = 2 h × 100 %


g Re
\ The loss in the weight of a body of mass m at a height h from the surface of the

earth =
2 mgh
Re
(3) Variation in 'g' with depth :
The gravitational acceleration at a distance r (r < Re) from the centre of the earth is

g' = 4 pGrr where r = uniform density of the earth.


At the surface of the earth, r = Re
3

\ g = 4 pGRer
3
ˆ The gravitational acceleration at depth d from the surface of the earth
(at distance r < Re from the centre of the earth)

§ ·
g' = g ¨1– d ¸
©R e ¹
At the centre of the earth, d = Re

\ g' = g §¨1– e ·¸ = 0
R
R © e ¹
Thus, the value of the gravitational acceleration at the centre of the earth is zero.

§ g ·
The absolute decrease = D g = g – g' = d ¨ ¸
© Re ¹
'g g–g'
The fractional decrease = g = = R
d
g e

'g
The percentage decrease = × 100 % = R × 100 %
d
g e
ˆ The rate of decrease of ‘g’ outside the surface of the earth (for h < < Re) is double to that of inside
the surface of the earth.

99
ˆ The graph of g ® r :
g
­

At surface of the earth


g=
G Me ­
R e2 Above the surface of the earth
gµr
­
gµ 1
r2

O r = Re
­ r®
Inside the surface of earth (distnce from centre of the earth)
(4) Variation in effective Gravitational Acceleration 'g' with latitude due to earth’s Rotation :
Equtorial Plane : The plane passing through the center of the earth and perpendicular to its
axis of rotation is called equatorial plane.
Latitude : The angle made by the line joining a given place on the Earth’s surface to the centre
of the Earth with the equatorial line is called the latitude (l) of that place.
At the equator l = 0° and at the poles l = 90°
The effective gravitational acceleration at the place having latitude l is.
g' = g – Re w2 cos2 l
R e Z2 cos 2 O
= g (1 – ), Where w = rate of rotation about its own axis.
g

(i) At the equator l = 0° Þ cos l = 1


R e Z2
g' = g (1 – g
) = g – Rew2

= minimum value of effective gravitational acceleration.


(ii) At the poles l = 90° Þ cos l = 0
\ g' = g
= maximum value of effective gravitational acceleration.
ˆ When a body of mass m is moved from the equator to the poles, its weight increases by an amount,
m (gp– ge) = mw2Re
where gp = gravitational acceleration at poles, ge = gravitational acceleration at equator.
ˆ If earth stops rotating about its own axis then at the equator the value of g increases by w2Re and
consequently the weight of the body of mass m lying there increases by mw2Re.
ˆ Average density of the earth in terms of 'g' and 'G'.
Accepting the earth as a solid sphere of uniform density,
§ g R e2 ·
3¨ ¸
¨ G ¸
r= = = © ¹
(\ Me = )
Me 3M e g R e2
4 SR 3 4 S R e3 4 S R e3 G
e
3
3g
= 4SR G
e

100
(16) A body weighs 81 kgf on the surface of the earth. How much will it weigh on the surface of Mass

whose mass and radius are 19 times and 12 times respectively that of the earth ?
(A) 40 kgf (B) 36 kgf (C) 24 kgf (D) 162 kgf
(17) If the earth were a sphere made completely of lead, then what would be the value of gravitational
acceleration on its surface ? (Radius of the Earth = 6.4 × 106 m, G = 6.67 × 10–11 SI,
Relative density of lead = 11.3)
(A) 22.21 ms–2 (B) 34.49 ms–2 (C) 28.72 ms–2 (D) 14.67 ms–2
(18) The mass of two planets are in the ratio 1: 2. Their diameters are in the ratio 1: 3. The acceleration
due to gravity on the surface of the planets are in the ratio ......
(A) 2 : 1 (B) 3 : 2 (C) 2 : 3 (D) 9 : 2
(19) If the radius of the earth is made three times that of present value. Then for what should be the
approximate change in the value of density of earth, so that the value of gravitatonal acceleration on
the surface of the earth remains constant.
(A) decreased by 67 % (B) increased by 67 %
(C) decreased by 33 % (D) increased by 33 %
(20) A man can jump to a height of 2 m on a planet A. What is the height he may be able to jump on
another planet whose density and radius are respectively, one - quarter and one third that of planet A.
(A) 18 m (B) 24 m (C) 36 m (D) 15 m
(21) The weight of a body on the surface of the earth is 54 N. What would be its weight at height

from the surface of the earth ? Where Re = radius of the earth.


Re
2
(A) 72 N (B) 36 N (C) 18 N (D) 24 N
(22) At what height from the surface of the earth, the value of gravitational acceleration will be half that
on the surface of the earth ? Radius of the earth R = 6400 km
(A) 2650 km (B) 3366 km (C) 1325 km (D) 414 km
(23) A body hanging from a massless spring stretches it by 1 cm at the earth surface. How much will the
same body stretch the spring at a place 1600 km above the earth’s surface ?
(Radius of earth R = 6400 km)
(A) 0.32 cm (B) 0.64 cm (C) 0.16 cm (D) 0.86 cm

(24) At what distance from the centre of the earth the weight of body becomes 16
1
times its weight that
on the surface of the earth ? Radius of the earth is R.
(A) 3R (B) 4R (C) 5R (D) 8R
(25) At what height above the earth’s surface the value of gravitational acceleration be same as that the
gravitational acceleration at a depth of 100 km from the surface of the earth ?
(A) 50 km (B) 100 km (C) 200 km (D) 25 km
(26) How much below the surface of the earth does the acceleration due to gravity become 10 % of its
value at the earth’s surface ? (Radias of the earth R = 6400 km)
(A) 6336 km (B) 5400 km (C) 5760 km (D) 5980 km

101
(27) The ratio of weights of a body of mass m at a height of 30 km above earth’s surface to a depth of
30 km from the surface of the earth is ......
(A) 0.946 (B) 0.962 (C) 0.984 (D) 0.995
(28) Suppose the earth is a uniform sphere of radius R. If the acceleration due to gravity at a place having
latitude 45° and at equator are g' and g'' respectively. Then g' – g'' = ...... (gravitational acceleration
at the poles = g)

(A) 3R Z (B) R Z (C) R Z (D) 2 R Z


2 2 2 2
2 2 3 3
(29) The angular velocity of the earth with which it has to rotate so that acceleration due to gravity on
60° latitude becomes zero is ...... (Radius of earth R = 6400 km, at the poles g = 10 ms–2)
(A) 2.5 × 10–3 rad s–1 (B) 1.25 × 10–3 rad s–1 (C) 2.5 × 10–2 rad s–1 (D) 1.25 × 10–2 rad s–1
(30) The angular velocity of the earth with which it has to rolate so that the weight of a body becomes 13
times the weight at body at equator ...... (Redius of earth R = 6.4 × 106 m, at the poles g = 9.8 ms–2)
(A) 7.8 × 10–4 rad s–1 (B) 6.7 × 10–4 rad s–1 (C) 8.7 × 10–4 rad s–1 (D) 10 × 10–4 rad s–1
(31) The weight of a body on the surface of earth is W. Then the weight of a body at half way mark from
the surface of the earth to centre of the earth is ...... (Consider the density of the earth to be uniform)
(A) 2 (B) W (C) 4 (D) 8
W W W

(32) The density of a planet is twice that of the earth and radius is times that of the earth. Then the
3
2
value of gravitational acceleration on the surface of the planet is how many times the value at sur-
face of the earth ?
(A) 43 (B) 3 (C) 6 (D) 4
3

(33) The mass of a body on a surface of the earth ia M. Then the mass of the same body at the surface
of the moon is ...... .
(A) M
6
(B) M (C) zero (D) infinite
(34) The rate of change of gravitational acceleration (g) at a depth x from the surface of the earth is ......
–8G SU
(A) – 43 Gpr (B) – 23 Gpr (C) (D) – Gpr
3
(35) The value of acceleration due to gravity at a height 1600 km above the earth’s surface is ......
(Value of g at surface of the earth = 9.8 ms–2, radius of earth R = 6400 km)
(A) 8.73 ms–2 (B) 7.59 ms–2 (C) 6.27 ms–2 (D) 9.12 ms–2
(36) If the earth stops rotating about its own axis, then the change in the value of gravitational accelera-
tion at a place having latitude of 45° is ...... (radius of the earth R = 6.4 ×106 m)
(A) 2.74 cms–2 (B) 1.68 cms–2 (C) 1.12 cms–2 (D) 3.34 cms–2

(37) The radius and mass of the earth are R and M respectively. Then the ratio
g
G
= ......
(Where g = gravitational acceleration, G = universal constant)

(A) MR2 (B) (C) M (D) R


M 2
R2 R M
(38) Assume earth to be complete sphere of radius R. If values of gravitational accelerations at a place
having latitude of 30° and at equator are g30 and g respectively. Then, g – g30 = ......

(A) w2R (B) 4 w2R (C) Z R (D) Z R


3 2 2
2 4

102
(39) Assume that the acceleration due to gravity on the surface of the moon is 0.2 times the acceleration
due to gravity on the surface of the earth. If Re is the maximum range of a projectile on the earth’s
surface. What is the maximum range on the surface of the moon ...... (Assume initial velocity of
projection to be constant)
(A) 0.2 Re (B) 0.5 Re (C) 2 Re (D) 5 Re
Ans. : 16 (B), 17 (A), 18 (D), 19 (A), 20 (B), 21 (D), 22 (A), 23 (B), 24 (B), 25 (A), 26 (C), 27 (D),
28 (B), 29 (A), 30 (D), 31 (A), 32 (B), 33 (B), 34 (A), 35 (C), 36 (B), 37 (B), 38 (B), 39 (D)
Mass and Weight
The quantity of matter in the body is called mass. it is the fundamental intrisic property of the body.
Mases are of two types :
(i) Inertial mass : The ratio of the external force applied on a body to the acceleration produced
in it due to the external force is called the inertial mass (mi).
Applied external force
mi = = Fa
Acceleration produced Froce
(Q according to Newton’s second law of motion, mass = ).
acceleration
ˆ The inertial mass of a body is a measure of its inertia.
ˆ It is the measure of ability of the body to oppose the production of acceleration in its
motion by an external force.
(ii) Gravitational mass : The ratio of the gravitational pull of the earth on a body to the
acceleration produced in it due to gravitational force is called the gravitational mass (mg).

\ mg = g
F

From the experiments mi = mg = m


Weight of the body : The gravitational force exerted by earth on a body is called weight of a body.

W=
GM e m
R e2
(Where Me = mass of the earth, Re = radius of the earth, G = universal gravitational constant)
Q W = mg
Its unit is N and directed towards the centre of the earth.
Gravitational Intensity (Gravitational field) :
ˆ Defination : The gravitational force exerted by the given body on a body of unit mass (test
mass) at a given point is called the intensity of gravitational field (I) at that point.
o
o
\ I = m where m = test mass
F

ˆ It is a vector quantity and its direction is towards the centre of gravity of a body whose
gravitational field is considered.
ˆ Its unit is N kg–1 and dimensions are M0L1T–2.

The gravitational intensity due to body of mass on at a distance r is, I = GM ÞIµ 2


1
ˆ 2 r
r
ˆ If the gravitational intensiry is known at a given point the gravitotional force acting on any body
kept at that point in the field can be determined.
ˆ The value of gravitational field intensity at any point is equal to the value of gravitational
acceleration at that point.
ˆ gravitational intensity and gravitational acceleration are different quantities. Their units are
different but equivalent. (\ N kg–1 and ms–2).
ˆ I ® r graph for the earth is gravitational field would be the same as g ® r graph.

103
ˆ As shown in the figure suppose at point P the gravitational intensities are equal and opposite due
to two bodies A and B, Thus at point P resultant gravitational intensity is zero.

A r B

I1 I2
m1 m2
P

x r–x

§ · § ·
here x = ¨¨ m  m ¸¸ r and r – x = ¨¨ m  m ¸¸ r
m1 m2

© 1 2 ¹ © 1 2 ¹

Gravitational Field Intensity for Bodies of Different Shape


Body Position Gravitational Figure
intensity I ® r (graph)

Uniform solid (i) Outside the surface r > R I=


GM
R
r2
I
sphere (ii) On the surface r = R I=
GM
R2 GM
R2
r
(iii) Inside the surface r < R I= O r=R
GM r
R3
R
Uniform spherical (i) Outside the surface r > R I=
GM
r2
I

shell (ii) On the surface r = R I=


GM
R2
(iii) Inside the surface r < R I=0 O r=R r

Thin uniform (i) At a point on its axis I= GM r


(a  r 2 ) 2
3
2 a

circular ring (ii) At the centre of the ring I=0 P l


O
r

ª º
2 GM r « 1 »
Uniform Disc (i) At a point on its axis I= 
a2 « r »
1
¬ r  a2
2
¼
a
or P qI
r
I= (1 – cosq)
2 GM
a2

(ii) At the centre of the disc I=0

104
Gravitational Potential
‘The negative of the work done by the gravitational force in bringing a body of unit mass, from
infinite distance to the given point in the gravitational field is called the gravitational potential (f) at
that point.’

f =  m =  ³ m =  ³ I .d r
o o
r r o o o oF
(Q I )
W F .d r
f f m

=  ³ Gm dr
r

2
f r
Gravitational Potential, f = –Gm
r
If r = ¥ then f = 0 = fmax
ˆ It is a scalar quantity. Its SI unit is Jkg–1 and dimensional formula is M0L2T–2.

o o –d I
ˆ f =  ³ I .d r Û I = dr
ˆ Gravitational Potential Difference :
‘It is defined as the work done to move a unit mass from one point to the other in the gravitational
field.

fA fB
M A B
rA
rB
WA o B § ·
Df = fB – fA = = –GM ¨¨ r  r ¸¸
1 1
m © B A ¹

ˆ Potential due to large numbers of particles is given by scalar addition m1 m2


r2
of all the potentials. In the figure at point P, total gravitational
potential, is given by, r1
P
r4 r3 m3
f =
– G m1
–
G m2
–
G m3 ......... m4 rn
mn
r1 r2 r3

Gravitational Potential for bodies of Different shape

Body Position Gravitational Figure


intensity I ® r (graph)

Uniform solid (i) Outside the surface r > R f=


– GM
r R
sphere (ii) On the surface r = R fsurface = R
– GM

»¼
V
– GM ª

r=R
(iii) Inside the surface r < R f = 2 R «3  R O r
r
¬

(iv) At the centre of the fcenter = = 2 f Surface


–3GM 3 –3GM
2R 2R
sphere (r = 0)

105
Uniform (i) Outside the surface r > R f=
– GM R
r

spherical shell (ii) On the surface r = R f=


 GM V
R r=R r
O
 GM
(iii) Inside the surface r < R f=
 GM
R R

 GM
Thin uniform (i) At a point on its axis f=
a2  r 2 a
P
circular ring (ii) At the centre of the ring f= a
 GM
r

Gravitational Potential Energy


‘The negative of the work done by the gravitational force in bringing a given body (of mass m) in the
gravitational field of the Earth from infinite distance at the given point is called the gravitational
potential energy (U) of that body at that point.’
The gravitational potential energy of a body of mass m at a distance r from the centre of the earth
(r ³ Re) is,

U= = fm (Q f = e )
– GM e m – GM

It is a scalar quantity. Its unit is J and dimensional formula is M1L2T–2.


r r
ˆ
ˆ Gravitational potential energy is always negative because gravitational forces are attractive in
nature.
ˆ As the distance increases, gravitational potential energy increases. (becomes less negative)
ˆ At infinite distance Gravitational potential energy U = 0 = Umax
ˆ Here the potential energy U is of the system consisting of the Earth and the body.
ˆ From the centre of body of mass M is the body of mass m is moved from a point at a distance
r1 to a point at distance r2 (r1 > r2) then change in potential energy,
§1 1·
DU = U2 – U1 = GMm ¨ r  r ¸
© 1 2 ¹
ˆ As r1 is greater than r2, DU will be negative. It means that if a body is brought closer to earth
it’s potential energy decreases.
ˆ Gravitational potential energy at the centre of the earth,
§ –3 GM e ·
Ucentre = mfcentre = m ¨ 2 R ¸ = 2 R
–3 GMe m
© e ¹ e
ˆ If the body of mass m is taken at a height h from the surface of the earth, then change in
potential energy,
m gh
DU = U2 – U1 = 1  h
Re
§ m g Re ·
(i) If h = nRe ; DU = ¨ ( n  1) ¸ n
© ¹

(ii) If h << Re ; DU = mgh (Q h ® 0)


Re
(iii) If h = Re ; DU = 12 mgRe

106
ˆ In case of discrete distribution of mass total potential energy
ª G m1 m 2 G m 2 m3 º
U = SUi = – «   ....»
¬ r12 r23 ¼

Total pairs are formed for a system of n particles.


n ( n –1)
ˆ 2
(40) If gravitational force acting on a body of mass 50 g at piont is 2 N, then what would the magnitude of
intensity of the gravitational field at that point ?
(A) 40 N kg–1 (B) 0.4 N kg–1 (C) 2 N kg–1 (D) 100 N kg–1
(41) The distance at which the magnitute of gravitational field intensity due to thin uniform ring of mass
M and radius R from the centre of the ring on its axis will be zero.

(A) (B) (C) R2 (D) 23R


R R
2 3
(42) The magnitude of gravitational intensity at a point is 20 N kg–1. What would be the magnitude of the
gravitational force on a body of 10 kg mass at this point ?
(A) 100 N (B) 200 N (C) 50 N (D) 400 N
(43) Two objects of equal mass m are placed at a distance d from each other on a horizontal surface. The
value of gravitational potential at a mid point on line joining their centres is ...... (G = universal
gravitational constant)

(A) (B) (C) (D) Zero


–Gm –2 G m –4 G m
d d d
(44) Three particles each of mass m are kept at the three vertices of an equilateral triangle of side b. The
gravitational potential at the centroid of this equilateral triangle is ......

(A) (B) (C) (D)


–3 3 G m –2 3 G m –3 2 G m –9 G m
b b b b
(45) Two bodies of mass m and 9m are placed at a distance r. The gravitational potential at a point on the
line joining them where the gravitational field is zero, will be ...... .

(A) (B) (C) (D)


–6 G m –9 G m –4 G m –16 G m
r r r r
(46) Four particles each of mass m are kept at the four vertices of a square with side l. The gravitational
potential at the centre of the square is ......

(A) (B) (C) (D)


–2 3 G m –4 2 G m –3 2 G m –5 2 G m
l l l l

R
(47) A body of mass m is taken from earth surface to the height h = . The increase in its potential
5
energy will be ......
(accleration due to gravity on the surface of the earth = g, radius of the earth = R).

(A) 7 mgh (B) 6 mgh (C) 4 mgh (D) 23 mgh


6 5 3

(48) Three particles each of mass 2m are kept at the three vertices of an equilateral triangle of side l. The
gravitational potential energy of this system is ......
–3 G m 2 –6 G m 2 –12 G m 2
(A) (B) (C) (D) – G m
2

l l l 2l

107
(49) A body of mass m starts falling from a distance 3R above the Earth’s surface. Its kinetic energy
when it has fallen to distance R above the Earth’s surface is ......
Radius of Earth = R, mass of Earth = M, Universal Gravitational constant = G

(A) (B) (C) (D)


GM m GM m GM m GM m
2R 4R 3R 6R
(50) A body of mass 1 kg is placed at a distance of 4 m from the centre and on the axis of a uniform
ring of mass 5 kg and radius 3 m. Calculate the work required to be done to increases the
distance of the body from 4 m to 3 3 m. (Gravitational constant = G)

(A) 6 (B) 3 (C) 2 (D) 5


G 2G 3G G

Ans. : 40 (A), 41 (A), 42 (B), 43 (C), 44 (A), 45 (D), 46 (B), 47 (B), 48 (C), 49 (B), 50 (A)
Escape velocity
The minimum velocity with which a body must be projected from the surface of earth so that it
escapes from the gravitational field of the earth is known as escape velocity (ve).

Escape velocity ve = 2 GM e
= 2g R e (Q GMe = gR2e )
Re

= 2 ( 4 SU G R e ) R e = Re 8 SGU
3 3
ˆ The value of escape velocity does not depend upon the mass of the projected body but it
depends on the mass and radius of the planet (Here Earth) from which it is being escaped.
(projected).
ˆ On the surface of the earth, ve = 2 g Re

substituting the values of g and Re, ve = 11.2 kms–1


ˆ If the escape velocity required for the body lying on the surface of moon, to make free from the
moon’s gravitational field is ve', then

ve' = 2 GM m
Where Mm = mass of the moon and Rm = radius of the moon.
Rm
substituting all these values, in this case, ve' = 2.3 kms–1

Which is nearly 1 times the escape velocity at the earth’s surface.


6
ˆ A planet will have atmosphere if the speed of molecule in its atmosphere at the temperature pre-
vailing there is less than the escape speed.

ˆ speed of molecules of gas = vrms = 3RT


M
\ That’s why the earth has atmosphere as at earth vrms < ve
while moon has no atomosphere as at moon rms > ve
Escape Energy
‘The minimum energy to be supplied to the body to make it free from Earth’s gravitational field
(from binding with the earth) is called the escape energy of that body. It is often called the binding
energy of the body.’

\ The escape energy of the body of mass m lying on the surface of the Earth =
GM e m
Re

108
(51) The escape velocity for a body projected vertically upwards from the surface of the earth is 11.2 kms–1.
If the body is projected an angle of 45° with the vertical, the escape velocity will be ...... kms–1

(A) 11.2 (B) 11.2 × §¨ 1 ·¸


© 2¹
§ 3·
(C) 11.2 × ¨ 2 ¸
© ¹
(D) 11.2 × 3 1
(52) The escape velocity of a body on the surface of the earth is ve, then the escape velocity on a planet
whose radius is three times and mass is three times that of the earth, is ......
(A) 3 ve (B) 9 ve (C) ve (D) 27 ve
(53) The escape velocity of a body on the surface of the earth is ve. The radius of the earth is 6400 km.
The value of radius of the earth (by contracting the earth), for which the escape velocity would
become 10 times the escape velocity of its present value ...... (Assume mass of the earth to be
constant.)
(A) 6.4 km (B) 64 km (C) 640 km (D) 4800 km
(54) A satellite with kinetic energy K is revolving round the earth in a circular orbit. How much more kinetic
energy should be given to it so that it may just escape from the gravitational field of the earth ?

(A) 2 K (B) 2K (C) K (D)


K
2

(55) A satellite is orbiting close to the surface of the earth. How much additional velocity (appronimately)
should be given to it so that it may just escape into outer space (radius of earth = 6400 km,
gravitational acceleration g = 9.8 ms–2)
(A) 11.2 km s–1 (B) 3.2 km s–1 (C) 8 km s–1 (D) 20.2 km s–1
(56) The escape velocity of a body on the surface of the earth is ve. If mass of the earth is made twice
and radius is made halved, then the escape velocity of a body would become ......

(A) 2 ve (B) 2 ve (C) 43 ve (D) 3 ve


3

(57) The escape velocity on the surface of the earth is v1. The escape velocity on the surface of a planet

whose radius and density are 4 times and 9 times respectively than that of earth is v2. Then v = ......
v1
2

(A) 16 (B) 12 (C) 43 (D) 4


1 3

Ans. : 51 (A), 52 (C), 53 (B), 54 (C), 55 (B), 56 (A), 57 (B)


Kepler's Laws
ˆ First Law (Law of orbits)
‘‘All the planets move in the elliptical orbits with the sun situated at one of the foci.’’
elliptical orbit a = Semi major axis
P
e e b = Semi minor axis
Perigee Apogee
s
sun b a Aphelion distance = Largest distance of planet from the sun.
rmin rmax Perihelion distance = Shortest distance of planet from the sun.
ˆ When planet is far away from the sun it moves slower in the orbit. Thus its kinetic energy is minimum

109
and potential energy is maximum.
ˆ When planet is near to the sun it moves faster in the orbit. Thus its kinetic energy is maximum and po-
tential energy is minimum.
rmax = a + ea = a (1 + e)
rmin = a – ea = a (1 – e)
Where e is the dimensionless number having value between 0 to 1 called eccentricity of the ellipse.
If e = 0, the ellipse is a circle. For earth e = 0.017.
Second Law (Law of Areas) Ñ
‘‘The line joining the sun and the planet sweeps equal areas in equal interval of time it means the

dt
areal velocity d A remains constant..’’
ˆ The areal velocity being constant is the geometrical representation of the law of conservation of
angular momentum.
P3 P2 In equal interval of time,
A2 A1 area of SP1P2 = area of SP3P4
S P1
P4 \ A1 = A2
Third Law (Law of Periods) :
‘‘The square of the time - period (T) of the revolution of a planet is proportional to the cube of the
semi major axis (a) of its elliptical orbit.’’
\ T2 µ a3 Þ T µ a3/2
Satellites
A body revolving around a planet is called its satellite.
ˆ The orbital motion of the salellite depends on the gravitational force by the planet and the initial
conditions.
ˆ The path of these satellites are elliptical with the centre of Earth at a focus. However, the
difference in semi major axis and semi minor axis is so small that they can be treated as nearly
circular.
Orbital velocity
The minimum velocity required to put a satellite into its orbit is known as orbital velocity. The orbital
velocity of a satellite at a distance r (r > Re) from the centre of the earth is,

v0 = =
GM e GM e
r Re  h

Very close to the surface of the earth, v0 = =


GMe
g Re
Re
ˆ The value of orbital velocity does not depend on the mass of the satellite, but depends on the
mass of the planet (here earth) about which it revolves and the radius of the orbit.
ˆ The orbital speed of a satellite when it revolves very close to the surface of the earth,

v0 = 9.8 u 6.4 u 106 = 7.92 km s–1


ˆ The work done by the satellite in a complete orbit (i.e. one complete revolution) is zero.
ˆ If the orbital velocity of a satellite orbiting near the surface of the earth is increased by 41.4 %
( 2 times) then it will escape from the gravitational field of the earth.

110
Different orbital shapes cossesponding to different velocities of a satellite.
(1) If v < vo (vo is the velocity require to maintain satellite in the orbit)
(i) The path would not be circular, rather it will be spiral. The satellite finally falls on the Earth.
(ii) Kinetic energy is less than potential energy Þ Total energy is negative.
(2) If v = v0
(i) The satellite revolves in a circular orbit.
(ii) e = 0 (e ® eccentricity).
(iii) Kinetic energy is less than potential energy Þ Total energy is negative.
(3) If v0 < v < ve (Where ve = Escape Velocity)
(i) The satellite revolves in a elliptical orbit.
(ii) e < 1
(iii) Kinetic energy is less than potential energy Þ Total energy is negative.
(4) If v = ve
(i) The satellite will move along a parabolic path and escape out of the gravitational field of
earth.
(ii) e = 1
(iii) The kinetic energy is equal to the potential energy. Þ Total energy becomes zero.
(5) If v > ve
(i) The satellite will move along a hyperbolic path and escape out of the gravitational field of
earth.
(ii) e > 1
(iii) The kinentic energy is greater than the potential energy Þ Total energy becomes positive.
Time Period of Satellife (T) :

2 Sr 2Sr
T= = 2p GM e Þ T µ r (Q GMe is constant)
r3 2 3
v0
= GMe
r
ˆ The periodic time of satellite is independent of mass of a satellite but depends on the mass
of the planet (here earth) about which it revolves and the radius of the orbit.
Height of satellite from the surface of the Earth
1
§ g R 2 T2 ·3
h = ¨¨ e 2 ¸ –R
¸ e
© 4S ¹

Energy of Satellite :
(i) Kinetic Energy (K) :

K = 12 mv02 = ... (1)


GM e m
2r

L2
Angular momentum of satellite is L = mv0r, then kinetic energy is K =
2 m r2

111
(ii) Potential Energy (U) :
The potential energy of a satellite at a distance r from the centre of the earth is,

U = ...... (2) (Note : From formula f = , U = mf.)


– GM e m – GMe
r r

– L2
= (in terms of angular momentum)
m r2
(iii) Total energy (E) :
E = Potential energy + Kinetic energy

...... (3)
– GM e m
= 2r

– L2
= (in terms of angular momentum)
2 m r2

ˆ Total energy of a satellite is negative.


From equations (1), (2) and (3)
E
K
= –1 Þ K = – E and
Energy K
U
E
= 2 Þ U = 2E
ˆ For any value of r, values of U and E are negative and O
E r®
value of K is positive and K = – E. U
ˆ As r ® ¥ all three energy curves approach a value of zero.
Binding Energy of Satellite :

Total energy of satellite is E = . Negative sign indicates that this satellite is in the bound
– GMe m
2r
state by an attractive force of central body. Thus energy must be supplied to remove it from the orbit
to infinity. The energy required to remove the satellite from the orbit (from the gravitational field of
the planet here the earth) to infinity is called Binding Energy of the system. i.e.

\ Binding Energy (B.E.) = –E =


GMe m
2r

If the orbit of a satellite is elliptical

then, Total Energy E = – = constant.


GM e m
2a

Where a = semi major axis


ˆ When the satellite is closest to the central body (about which it revolves) (at perigee), then kinetic
energy of satellite is maximum. i.e. the potential energy is minimum. (from formula U = E – K)
and when the satellite is farthest from the central body (at apogee), then kinetic energy of
satellite is minimum and potential energy is maximum.
ˆ When the satellite is transferred from one circular orbit of radius r1 to other circular orbit of
radius r2 (r2 > r1) then the variation in different quantities can be shown by the following table.
112
Quantity Variation Relation with r

(1) Orbital Velocity Decreases v0 µ


1
r
3

(2) Time Period Increases T µ r2

(3) Linear momentum Decreases Pµ


1
r

(4) Angular momentum Increases Lµ r

(5) Kinetic Energy Decreases K µ 1r

(6) Potential Energy Increases U µ – 1r

(7) Total Energy Increases E µ – 1r

(8) Binding Energy Decreases B.E. µ 1r


Geo-Stationary Satellite (Geo-Synchronous satellite) (parking satellite)
ˆ The Earth’s satellite having orbital periodic time of 24 hours (equal to the periodic time of
rotation of the Earth about it’s own axis), is called the geo-stationary satellite.
ˆ Geo-stationary satellite revolve around the Earth in the equitorial plane in east west direction.
Height of Geo-stationary satellite from the surface of the Earth :
1
4S r
2 3 § GM e T 2 ·3
T2 = GM Þ r = ¨¨ ¸
¸
© 4S ¹
2
e

Now substituting all values in above equation, we get


\ r = 42,260 km
\ h = r – Re = 42,260 – 6400 = 35860 km
Such an orbit of satellite is known as parking orbit.

The orbital speed of satellite using equation is 3.08 kms–1


GM e
r
Polar Satellite :
ˆ These satellites revolve around the Earth in north south direction at height nearly of 800 km from
the surface of the Earth.
ˆ The time period of these satellites is almost 100 min.
Maximum height attained by a projectile :
Suppose body of mass m is thrown with velocity v in vertically upward direction from the
surface of the earth and it attains maximum height of H. At maximum height its velocity is zero.
According to law of conservation of mechanical energy,
Total energy at the Earth’s surface = Total energy at a height from Earth’s surface.
§ GM e m · § GM e m ·
\ 1
mv2
+ ¨ R ¸ = 0 + ¨ R  H ¸
2 © e ¹ © e ¹
Where Re = Radius of the Earth, Me = Mass of the Earth.

113
ª º
\ v2 = 2GMe « R  R  H »
1 1
¬ e e ¼

§ ·
= 2GMe ¨ (R ) (R  H) ¸
H
© e e ¹

§ ·
= 2gRe2 ¨ (R ) R (1  H/ R ) ¸ (Q GMe = gRe2)
H
© e e e ¹

§ ·2
1

¨ ¸
v2 = v= ¨ ¸
2g H 2g H
¨§ ·¸
Þ
1 H ¨ ¨1  ¸¸
H
© © Re ¹¹
Re

Now, v2 = R  H
2g H Re
e

\ (v2) (Re + H) = 2gHRe Þ v2Re + v2H = 2gHRe


\ v2Re = (2gRe – v2)H
v2 R e
ÞH=
2 g R e – v2
Relative angular velocity of satellite
If satellite revolve around the Earth in the equitorial plane in same sense of rotations as that of earth
about its own axis (from west to east), Then the relative angular velocity of satellite for an observer lying
at the Earth surface is,
wrelative = ws – wE
Where ws = angular velocity of satellite
wE = angular velocity of Earth
2S
Now using equation w = ,
T
2S 2S
T = Z 2S = Z – Z = 2S 2S = E s
T T
relative s E – TE – Ts
Ts TE
(58) The time period of a satellite orbiting close to the surface of the earth is 50 min. The time period of a
satellite orbiting at height three times the radius of the earth from the surface of the earth is ......
(A) 100 min (B) 400 min (C) 50 × 8 min (D) 150 min
(59) The rate of rotation of a planet is 8 times the rate of rotation of earth around the sun. Then ratio of
their radii of orbits of rotation is ......

(A) 24 (B) 4 (C) 16 (D) 12


1 1 1

(60) The mercury (A planet) is revolving around the sun as shown in


elliptical path. The potential energy of the mercury will be minimum R Q
at the point ......
(A) P (B) Q S P
Sun
(C) R (D) S

114
(61) The figure shows elliptical orbit of a planet of mass m about the sun m
S. The shaded area SCD is thrice the shaded area SAB. If t1 is the A D
time for the planet to move from A to B and t2 is time to move from S
B C
t1
C to D then t = ......
2

(A) 1 (B) 2 (C) 3 (D) 4


(62) The period of revolution of planet A is 27 times that of B. The distance of A from the sun is how
many times greater than that of B from the sun ......
(A) 9 (B) 8 (C) 4 (D) 6
(63) Two satellites are revolving into a circular orbit of radii r and 1.01 r respectively around the earth.
Their orbital time periods are T1 and T2 respectively. The period of second satellite is larger than
that of the first one by approximately ......
(A) 0.5 % (B) 1.0 % (C) 1.5 % (D) 3.0 %
(64) The time period of a satellite revolving at a height equal to the radius of the earth from the surface of
the earth is ...... (radius of the earth R, acceleration due to gravity = g)

(A) 2p
2R
(B) 4 2 SR (C) 2p
R
(D) 8p
R
g g g g
(65) A geo-stationary satellite is orbiting the earth at a height 6R above the surface of earth. R being the
radius of the earth. The time period of another satellite at a height of 2.5R from the surface of
the earth will be ......
(A) 6 2 hr (B) 6 hr (C) 10 hr (D) 2 6 hr
(66) Two satellites A and B are revolving in the circular orbit of equal radii around the earth. The mass of
A is 100 times the mass of B. Their time periods are in the ratio of ......
(A) 1 : 100 (B) 100 : 1 (C) 1 : 1 (D) 10 : 1
(67) The rotation period of a satellite of mass m revoling close to the surface of the earth is ......
(A) 72 min (B) 62.2 min (C) 84.6 min (D) 104 min
(68) The time period of revolution of a satellite orbiting close to the surface of the earth in terms of
density of the earth (r) is ......
3S 4S S 2S
(A) GU (B) GU (C) 3
GU (D) GU

(69) Two satellites A and B go round the earth in circular orbits having radii 4R and R. If the orbital
velocity of satellite A is 3v, then the orbital velocity of satellite B is ......

(A) 6 v (B) 12 v (C) 43 v (D) 2 v


3

(70) The orbital velocily of a satellite revolving around the earth in a circular orbit close to the surface of
the earth is v0. The orbital velocity of another satellite revolving at a height one half of the radius of
the earth, from the surface of the earth, is ......

(A) v (B) v (C) 23 v0 (D) 2 v0


3 2 3
2 0 3 0
(71) The orbital time period of satellite revolving around the earth in the orbit of radius r is T. If the same
satellite is revolving in the orbit of radius 2r, the new periodic time is ......
(A) 2T (B) 1.5 T (C) 2.8 T (D) 0.5 T

115
(72) If orbital period of a satellite is T, then its kinetic energy is proportional to ......
2 2
(A) T1 (B) (C) T 3 (D) T
1 3
T3
(73) A satellite of mass m and having kinetic energy K is orbiting around the earth in circular orbit of
radius r. The angular momentum of the satellite is ......

(A) (B) (C) (D)


K K
mr 2
2 m r2 2 K mr 2 2 K mr

Ans. : 58 (B), 59 (B), 60 (D), 61 (C), 62 (B), 63 (C), 64 (B), 65 (A), 66 (C), 67 (C), 68 (A), 69
(A), 70 (B), 71 (C), 72 (D), 73 (C)
Assertion - Reason type Question :
Instruction : Read assertion and reason carefully, select proper option from given below.
(a) Both assertion and reason are true and reason explains the assertion.
(b) Both assertion and reason are true but reason does not explain the assertion.
(c) Assertion is true but reason is false.
(d) Assertion is false and reason is true.
(74) Assertion : The earth suddenly stops rotating about its axis, then the value of acceleration due
to gravity will become same at all the places.
Reason : The value of acceleration due to gravity is independent of rotation of the earth.
(A) a (B) b (C) c (D) d
(75) Assertion : The escape velocities for two objects projected in the direction making an angle of
30° and 60° with the surface of the earth, from the surface of the earth are v1 = 2ve
and v2 = 2ve respectively.
3
Reason : The value of escape velocity does not depend on the angle of projection.
(A) a (B) b (C) c (D) d
(76) Assertion : For the planets orbiting around the sun, angular speed, linear speed, kinetic energy
changes with time but the angular momentum remains constant.
Reason : No torque is acting on the rotating planet. So its angular momentum is constant.
(A) a (B) b (C) c (D) d
(77) Assertion : The weight of a body on the surface of the earth is more at mid night time that of
noon time.
Reason : The gravitational forces exerted on the body by the earth and by the sun are in
opposite direction to each other at noon time.
(A) a (B) b (C) c (D) d
(78) Assertion : The orbital time period of a satellite revolving close to the sarface of the earth is
smaller than that the satellite revolving far away from the surface of the earth.
Reason : The square of the orbital time period is directly proportinal to the cube of the orbital
radius.
(A) a (B) b (C) c (D) d
(79) Assertion : The orbital speed of a satellite is greater than its escape speed.
Reason : Orbit of a satellite is within the gravitational field of earth, whereas escaping is beyond
the gravitational field of earth.
(A) a (B) b (C) c (D) d

116
(80) Assertion : Different planets have different values of escape velocity.
Reason : The value of escape velocity is not a universal constant.
(A) a (B) b (C) c (D) d
(81) Assertion : The gravitational froce exerted on a body by the moon is less than that by
the earth.
Reason : The value of gravitational force depends on the factor M2 for a given mass m and it is
r
very small for the moon. Where r = distance from the centre.
(A) a (B) b (C) c (D) d
(82) Assertion : Gravitational force between two particles is negligibly small compared to the electrical
force.
Reason : The electrical force is experienced by the charged particles only.
(A) a (B) b (C) c (D) d
(83) Assertion : Body becomes weightless at the Earth’s centre.
Reason : The gravitational acceleration increases when distance decreases from surface of Earth.
(A) a (B) b (C) c (D) d
And. : 74 (C), 75 (D), 76 (A), 77 (D), 78 (A), 79 (D), 80 (A), 81 (A), 82 (B), 83 (C)
Graph based questions :
(84) A shell of mass M and radius R has a point mass m placed at a distance r from its centre. The
gravitational potential energy U (r) versus r (distance from centre) will be ......
R
(A) o r (B) o r

U(r) U(r)

o r o r

(C) (D)

U(r) U(r)

(85) The correct graph representing the variation of total energy (E), Kinetic energy (K) and potential
energy (U) of a satellite with its distance from the centre of earth is ......
(A) (B)
E E
Energy
Energy

U K
o r o r
K U
Energy

Energy

(C) K (D) K
o r o r
E
U U

117
(86) The diagram showing the variation of gravitational potential of earth with distance from the
centre of earth is ......
(A) V (B) V

R r R r
O O

V V

R r R r
(C) O (D) O

Ans. : 84 (C), 85 (C), 86 (C)


Comprehension Type Questions :
Paragraph-1
G
The gravitational field in a region is given by I = 5 i + 12 j N kg–1. Answer the following
questions.
(87) Find the magnitude of the gravitational force acting on a body of mass 2 kg placed at the
origin ......
(A) 26 N (B) 30 N (C) 20 N (D) 35 N
(88) Find the potential at points (12 m, 0) and (0, 5 m), if the potential at the origin is taken to
be zero.
(A) –30 J kg–1, –30 J kg–1 (B) –40 J kg–1, –30 J kg–1
(C) –60 J kg , –60 J kg
–1 –1
(D) –40 J kg–1, –50 J kg–1
(89) Find the change in gravitational potential energy if a body of mass 2 kg is taken from the origin
to the point (12 m, 5 m) ......
(A) –225 J (B) –240 J (C) – 245 J (D) –480 J
(90) Find the change in potential energy if the body is taken from (12 m, 0) to (0, 5m)
(A) –10 J (B) – 50 J (C) 0 (D) – 60 J
Paragraph - 2
Assume that orbits of Earth and the Mars around the sun to be circular. An artificial satellite is
launched from the earth which can revolve around the sun in such a way that its apogee is lying on
the axis of rotation of Mars and the perigee is lying on the axis of rotation of earth. The orbital time
periods for earth and the Mars are Te and Tm respectively around the sun and symbols for different
parameters are as under :
Me = mass of earth, Mm = mass of Mars, M = mass of artificial satellite, Le = angular momentum of
earth around sun, Lm = angular momeatum of Mars around sun, Re = semi major axis of arbit of
Earth, Rm = Semi major axis of orbit of Mars, Ee = total energy of the earth, Em = total energy of
Mars.
(91) The orbital time period of a satellite around the sun is ...... (Neglect the effect of gravitational
field by earth and by Mars.)

ª 2 2 º2
3

Te  Tm « Te 3  Tm 3 »
(A) (B) (C) T  T (D) « »
2 Te Tm
« »
Te Tm 2
¬ ¼
2 e m

118
(92) Total energy of the satellite is ......

2 M § R e Ee · 2 M § R e Ee ·
(A) M ¨ R  R ¸ (B) M ¨ R  R ¸
e © e m ¹ m © e m ¹

§ ·
2 Ee M § R e  R m · 2 Ee M ¨ R e  R m ¸
(C) M ¨ R ¸ (D) M ¨ ¸
m © ¹ © Re  Rm ¹
m 2 2
e

(93) Areal velocity of a satellite around the sun is ......


(A) Less than that of the areal velocity of earth.
(B) Greater than that of the areal velocity of Mars.
(C) Same as that of the areal velocity of earth.
(D) Greater than that of the areal velocity of earth.
Ans : 87 (A), 88 (C), 89 (B), 90 (B), 91 (D), 92 (A), 93 (D)

Match the columns :


(94) A satellite is projected vertically near the surface of a planet with speed v. The value of
acceleration of a freely falling body near this planet is found to be 4.9 ms–2. Radius of the planet
is 3200 km. For various values of v, the path of satellite can be predicted. Match the velocity of

satellite with its respective path 2 1.4


Column-1 Column-2
(a) v = 4 km s–1 (p) Elliptical (A) a ® q b®p c®s d®r
(b) v = 5 km s –1
(q) Circular (B) a ® p b®q c®r d®s
(c) v = 5.6 km s–1 (r) Hyperbolic (C) a ® s b®r c®p d®q
(d) v = 6.6 km s–1 (s) Parabolic (D) a ® r b®s c®q d®p
(95) Match Column 1 and Column 2
Column-1 Column-2
(a) Elliptical orbit of a planet (p) Conservation of kinetic energy
(b) Circular orbit of a satellite (q) Conservation of angular momentum
(c) Escape velocity (r) Independent of mass of a satellite

(d) Orbital velocity (s) GM


R
(t) Constant areal velocity

(A) a ® q, t b ® p, q, t c ® r d ® r, s (B) a ® p, r b ® q, r c®t d®p


(C) a ® s b®r c®s d®t (D) a ® p b®q c®r d®s

Ans. : 94 (A), 95 (A)

119
9 & 10 Properties of solid and liquid
Elasticity : The inherent property of a body due to which, body trines to restore the normal shape or
to oppose the change in shape is known as elasticity.
Perfect elastic body : It a body can completely regains its original stape after removal of the
deforming force, it is called a perfect elastic body.
In practice it is impossible to have a perfect elastic body.
The object which can be considerd as the nearest to perfect elastic body is quartz.
Non elastic body : (plastic body) : If a body remains in the deformed state and does not even
partially regain its original shape after removal of deforming force, it is called a perfect non elastic
body. e.g. Wax
Rigid Body : If the relative positions of the particles of the body remain invarient even resultant
force acts on it, the body is called rigid.
Stress : The restoring force arising per unit cross sectional area of a deformed body is called stress.
F Force (F)
Stress s = A
Area (A)
unit : Nm–2
Dimenssional formula : M1L–1T–2
Types of Stress :
(1) Longitudinal Stress (sl) Ñ The stress due to which the length of the body changes is called
longitudinal stress.
Types of longitudinal stress :
ˆ Tensile Stress : The stress which
causes increase in the length of the
body is called tensile stress.
ˆ Compressive stress : If due to the
application of external forces length of
the rod decreases, the resulting stress
is called compressive stress.
(2) Volume Stress or Hydraulic Stress (sV) : The stress produces due to the forces which are
perpendicular to the entire surface of the body is called volume stress. Application of such forces
cause change in the volume of the body.
(3) Shearing Stress or Tengential Stress (ss) : If the force acting on a body is tangential to a
surface of the body it causes shearing strain in the body is called shearing stress.
Note : In the normal position of the body the intermolecular distance r = r0. When external force
acts on it,
ˆ If the body is compressed (r < r0) ® intermolecular forces are repulsive
ˆ If the body expand (r > r0) ® intermolecular forces are attractive.
The Difference between pressure and stress :
Pressure Stress
ˆ Vector ˆ Tensor
ˆ The whole body is acted upon by forces, ˆ The forces should not be perpendicular
acting perpendicularly every where on the body. to the surface.
ˆ It is same on all surface. ˆ It can be different on diffrent surfaces.
It is also possible that there is stress on
one surface and there is no stress on
the other surface.

120
Thermal Stress : When both the ends of a rod is fixed in the rigid support and its temperature
is reduced, the stress induced in the rod is called thermal stress.
Thermal Stress s = Y a DT
Y = Young's modulus a = linear co-efficient of expansion DT = decrease in the temperature.
Strain : (e)
ˆ When an external force is applied on a body its length, volume or shape change is called
strain.
ˆ It is ratio of change in body when deforming force is applied to the original body.
ˆ Strain is unitless and dimensionless physical quantity.
Types of strain
(1) Longitudinal Strain (el) : The ratio of change in length of a body (Dl) when deforming
force is applied to the original length (l) is called longitudinal Strain.
'l
el =
l
ˆ Tensile Strain ® increase in length
ˆ Compressive Strain ® decrease in length
(2) Volume Strain (eV) : It is ratio of when a body is acted upon by the forces everywhere
on its surface in direction perpendicular to the sarface, the volume of the body change to
original volume.
9
ev =
V
(3) Shearing Strain (es) : A force tangential to a cross-section of a body produce the change
in shape, it is called shearing Strain.

es = h
x

Types Stress Strain

Force perpendicular to cross-section Change in length


Longitudinal sl = el =
cross- sectional area original length
'l
sl = el =
F
A l

Volume eV = Change in volume


Force perpendicular at every point of surface
sV = original volume
area of surface

'V
sV = = =P (Where P = Pressure) eV=
F PA
A A V
Tengential force
Shearing ss = es = = tan q
x
area h
x

=
Fs
A
h
q

Unit : Nm–2 Unitless

121
(1) The length of a string is l1 when the tension force of 3 N is applied on the string. The length becomes
l2 when force becomes 4N. What would be the length to the string if the force is made 7 N.
(A) 4l2 – 5l1 (B) 7l2 – l1 (C) 4l2 – 3l1 (D) 3l2 – 4l1
(2) One horizontal rod of length 1 m is rotating about an axis passing through its edge and
perpendicular to its plane. With what revolution per second it should be rotated so that it breaks ?
(breaking stress = 3×109 Nm–2, density ot the material of the rod = 6000 kgm–3.)
(A) 1000 rps (B) 318.2 rps (C) 159 rps (D) 259 rps
(3) A rod of length 2m, mass 1 kg and. Cross-sectional area 10–4 m2 is hanged vertically. 1 kg mass is
suspended at its lower end calculate the stress at the midpoint of the rod. (g = 10 ms–2)
(A) 20 × 104 Nm–2 (B) 105 Nm–2 (C) Zero (D) 15×104 Nm–2
(4) When a mass more than 27 kg is suspended from a wire, it breaks. Another wire is having radius
equal to one third of the first wire, which is made up of the same material. Calculate the maximum
mass which can be loaded using this wire.
(A) 9 kg (B) 3 kg (C) 27 kg (D) 81 kg
(5) Length of a metallic rod of mass m and cross-sectional area A is L. If mass M is suspended at

the lower end of this rod suspended vertically. Stress at the cross-section situated at 4 distance
L

from its upper end is ......

4 A
§ 3m · g
(C) ¨ M  ¸
g
(A) (B) M  (D) (M + m)
Mg g
© 4 ¹A
m
A A
(6) A rod of length 100 cm and negligible weight is hanged using steel
wire and brass wire in such a way that it remains horizontal as 2m Brass
shown in the figure. Asteel = 0.2 cm2 and Abrass = 0.4 cm2. Both Steel
wire
steel and brass wires are of equal length. At what distance on the T1 Wire T2

rod a mass (W) must be suspended so that the tension produced in


steel wire is same as that in brass wire.

(A) m from steel wire (B) m from the brass wire


2 4
3 3

(C) 1 m from the steel wire (D) m from the brass wire
1
4

Ans. : 1 (C), 2 (C), 3 (D), 4 (B), 5 (C), 6 (A)

Hooke's Law and Elastic Moduli


“For small deformations the stress and strain are directly proportional to each other.”
sl µ el Þ sl = Yel
Vl
\Y= H =
FL
l A' L

VV
Bulk Modulus : B = H
V

Comperssibility K =
1
B

122
Modulus of rigidity (Shear Modulus)
Vs
h= H = =
F/ A Fh
s x/h Ax

(7) The density of sea-water on its surface is r. Find the density of water where the pressure is
a Pa. Where Pa = atomspheric pressure and a is a constant. The Bulk modulus of the water is B.

UB UB UB UB
(A) B D P (B) B – D P (C) B  ( D – 1) P (D) B – ( D – 1) P
a a a a

(8) A solid sphere of radius R and made up from the meterial having bulk modulus B is placed in a
cylindrical container having cross-sectional area A and filled with some liquid. A piston of
cross-section A is kept floating on the surface of the liquid. Calculate the relative change in the
radius of the sphere when mass M is kept on the piston.

(A) (B) (C) (D) AB


Mg Mg Mg 3M g
3AB 2 AB AB

(9) A wire of length 0.5 m, redius 0.1 m rotates about an axis passing through its edge and
perpendicular to its plane with an angular speed of 400 rad s–1. Calculate the increase in the length
of the spring. The density of the material of the wire is 104 kgm–3. Young’s modulus
Y = 2 × 1011 Nm–2.

(A) mm (B) mm (C) mm (D) 1 mm


1 1 1
6 3 2

(10) Find the tension force produced in the b


wire when a force F is applied as shown a
F
in the figure. (Y = 2 × 1011 Nm–2)

Y
(A) (B)
F F
§ ab·
L
S b a

2 2 2
¸ Y
© 2 ¹

(C) (D) S ab Y
F F
§ a b ·
S¨ ¸Y
2 2

¨ ¸
© ¹
2

(11) A solid sphere of radius (r) made up from the material having bulk modulus (B), placed in a
cylindrical container filled with liquid. A piston having cross-sectional area (a) is placed in a
container. When a mass (m) is placed on the piston than find fractional increases in
'r
radius r ...... .

(A) B m g (B) 3 m g (C) (D)


a a mg mg
3 Ba Ba
(12) A twist of 0.1 unit per cm is produced in a wire of radius 3 cm. A hollow cylinder of having
internal radius 4 cm and outer radius is 5 cm is under the effect of same couple of force. Find
twist in hollow cylinder per cm.
(A) 0.1 unit (B) 0.455 unit (C) 0.91 unit (D) 1.82 unit

123
(13) l1 l2 l3 The Young’s modulus of three rods having cross-section area
and equal volume are Y 1, Y 2 and Y 3 respectively. Their
Y 1, a1 Y 2, a 2 Y 3, a 3 coefficent of linear expansion are a1, a2 and a3 respectively. A
compound rod made up of these 3 rods is fixed between two
walls as shown in the figure. It has been observed that the
length of the central rod (l2) remains the same even if the

temperature of the system increases. Calculate original l .


l1
3

l1 = length of the first rod l3 = length of the third rod.


§ Y2 D 2  Y1 D1 · Y3 § Y3 D 3  Y2 D 2 · Y1
(A) ¨ Y D  Y D ¸ Y (B) ¨ Y D  Y D ¸ Y
© 3 3 2 2 ¹ 1 © 2 2 1 1 ¹ 3

Y1 D1 Y3 D 3
(C) Y D (D) Y D
3 3 1 1

(14) A wooden board with uniform thickness moves on smooth surface under the influenes of constant
horizontal force (F0) its young modudus is Y. If area of cross section is A, its compressive strain in
direction of force is ...... . (Total length of wooden board = L)

(A) (B) (C) (D)


F0 2 F0 F0 3 F0
AY AY 2 AY 2 AY

(15) Young modulus of Steel, Aluminium and Tungsten wire having same length
and same area of cross-section are Y1 = 2×1011 Pa, Y2 = 0.7 × 1011 Pa 1 2 3
Y3 = 3.6 × 1011 Pa. They are suspended vertically us shown in figure
Effective Young modulus of this arrangement is ...... Pa.
LOAD
(A) 6.3 × 10 11
(B) 2.1 × 10 11
(C) 0.8 × 10 22
(D) 7.099

(16) An average distance between two molecules of an unknown metal is 3.2 × 10–10 m. The constant
of intermolecular force between them is 6 Nm–1. The Young’s modulus for this metal is ...... Nm–2.

(A) 2.33 × 105 (B) 18.75 × 1010 (C) 0.1875 × 1010 (D) 1.875 × 1010

(17) Two rods of length L1 = 10 cm and L2 = 20 cm are fixed between


two walls as shown in figure. Their Young modulus are Y1 and Y2.
wall L1
Their coefficient of linear expansion are a 1 and a 2 . Wall
Where a1 : a2 = 3 : 4. Both rods are not bend even after heating. rod L2

Ratio of young modulus to obtain same value of thermal stress is


Y1 : Y2 = ...... . rod

(A) 1:1 (B) 3:4 (C) 4:3 (D) 4:9

(18) A ring of radius R2 is fixed on a wooden disc of radius R, in such a way that their centres remain

the same. The area of the cross-section of the ring is 100 cm2 and the Young’s modulus of the
meterial of the ring is 2 × 1011 Pa. Calculate the force required for the expansion of the ring.

(A) 4 × 109 N (B) 2 × 106 N (C) 2 × 1013 N (D) 1013 N

124
(19) A graph of stress ® strain for two different material A and
B is shown in the figure. If their Young modulus are A

Stress
YA
YA and YB. Find Y = ...... [tan 36° = 0.75, tan 18° = 0.3]
B B

36°
(A) (B)
2 2
18°
Strain
5 1

(C) 2 (D) 2
1 5

(20) The graph of mass ® elongation for four wires of diffrent length D
C
but same material is as shown in the figure. Which one of the
mass
followings represent the thickest wire ? B

(A) OD (B) OG A

(C) OB (D) OA
O elongation
Ans. : 7 (D), 8 (A), 9 (B), 10 (D), 11 (C), 12 (B), 13 (B), 14 (C), 15 (B), 16 (D), 17 (C), 18 (B),
19 (D), 20 (A)
Poisson’s ratio :
ˆ The ratio of lateral strain to longitudinal strain is known as poisson’s ratio.
' D/D
m= 'l / l
D = Diameter of cross-section

ˆ m < 0.5

For solid < m<


1 1
ˆ 4 3
ˆ For rubber m is very close to 0.5
Elastic potential energy :

U=
A Y ( ' L2 ) A = area of cross-section, Y = Young’s modulus
L = original length, DL = change in length
2L

Energy per unit volume = ×Y × 'LL


1 2
ˆ 2

= × Stress × Strain
U 1
\
V 2
Note :

ˆ for two wire If Y1 = Y2 and F1 = F2

' L2 L2 r12
\ DL µ ' L1 = L1 × r 2
L
r2
Þ
2

ˆ for two wire If Y1 = Y2 and L1 = L2

125
' L2 F2 r12
\ DL µ Þ 'L =
F
r2 1 F1 r22

ˆ Y, B and h decreases with the increase of temperature.


ˆ The restoring torque produce in a wire having twist q is,

SK r 4 T
t= l = length of the wire.
2l

(21) The potential energy of the molecule of air, U = M6 – 12 where M and N are constants. The
N
r r
potential energy in the equilibrium position ...... .

(A) 0 (B) N (C) M (D) MN


2 2 2

4M 4N 4

(22) The poisson’s ratio of an object is 0.1. The longitudinal strain of a rod made from this object is
10–3 . The percentage change it its volume is ...... .
(A) 0.008 % (B) 0.08 % (C) 0.8 % (D) 8 %
(23) The poisson’s ratio of an object is 0.5. The tensile strain is due to this force of 2 ×10–3. The
percentage change in the volume ...... .
(A) 2 % (B) 2.5 % (C) 5 % (D) 0 %
(24) The ratio of diameters of two wires of same length and same meterial is 2 : 3. Both are given
same tension then the ratio of their potential energy per unit volume is ...... .
(A) 2 : 3 (B) 81 : 16 (C) 9 : 4 (D) 16 : 81
(25) What would be the potential energy per unit volume of a wire having tensile strain 20 Nm–2 ?
Y = 2 × 10" Pa
(A) 0.5 × 10–11 Jm–3 (B) 109 Jm–3 (C) 10–9 Jm–3 (D) 2 × 10–9 Jm–3
Ans. : 21 (C), 22 (B), 23 (D), 24 (B), 25 (C)
Fluid pressure

fluid pressure P =
F
ˆ A

1 Pa = 1 Nm–2

1 atm = 1.013 ×105 Pa

1 bar = 105 Pa

1 torr = 133.28 Pa = 1 mm-Hg

1 atm = 76 cm of Hg = 760 mm-Hg

Thrust on the Liquid :

The total force acting on the surface of the liquid by the liquid is called thrust of the liquid.

126
ˆ Pressure due to fluid column : Pa
P – Pa = hrg (gauge pressure)
Total Pressure P = Pa + hrg P h
density
of the r
liquid

Pascal’s law : Pressure in an incomperssible fluid in equilibrium W = mg


F1
is the same everywhere, if the effect of gravity is neglected. m
Principle of Hydraulic press :

P=
F1
= 2
F a A
F2
a A

but 1 = A
F W
a
Where A >> a then F1 << W and F2 = W
Archimedes Principle :
When a body is partially or fully immersed in a liquid the buoyant force acting on it, is equal to
the weight of the liquid displaced by it and it acts in the upward direction at the centre of mass of the
displaced liquid.
[Fb = Vfrfg].
Law of floatation :
Weight of body W = weight of the liquid displaced by the part of body immersed.
Mg = mg (M = Mass of flowting body Vsrs,
Vsrsg = Vf rf g m = Mass of the desplaced liquid = Vfrf)
Us Vf
Uf = Vs
ˆ This relation is also true for accelerated fuel
ˆ Weight force W, Fb - buoyant force.
W > Fb, the body sinks in the liquid
W < Fb, the body floats on the Liquid surface.
W = Fb, the body can remain in equilibrium at any depth in liquid.
Note :
Body is sink in accelerated fluid at that point buoyant force will be upward direction is called
ˆ
buoancy centre.
ˆ For the symmetrical solid body the centre of buoancy is its centre of gravity.
ˆ When the centre of gravity and centre at buoancy are on the same line, the solid would be in the
equilibrium.
ˆ For non-symmetrical body both the centres are not lying on the same line. As a result, the
resultant torque acts on the body and the body will perform rotational motion.
(26) Two objects having different mass are attached at both end of the balance. Where this balance
immersed in the water then it is in balanced position. If mass of one object is 36g and its density
is 9 g cm–3. Find density of second object having mass 72 g.
(A) 3 gcm–3 (B) 3 gcm–3 (C) 1.8 gcm–3 (D) 5 gcm–3
4 2

(27) An object having density 4 kg m–3 in a medium having density 1 kg m–3 is in equilibqium with an
object having density 8 kg m–3 and weight 10 N. Find actual mass of an object. (g = 10 ms–2)
(A) 10 kg (B) 8 kg (C) 4 kg (D) 6 kg
7 3 7

127
(28) A cubical tank is completely filled with water is fixed on a trolly.
If this tank is acclerated with (a). then, P R
a
(i) Pressure at ...... point is maximum
Q S
(ii) Pressure at ...... point is minimum
(A) (i) Q (ii) R (B) (i) S (ii) R (C) (i) Q (ii) S (D) (i) Q (ii) P
(29) liquid
Height of mercury in both arm of manometer (U tube) is
glycerin h
20 cm same. Glycerin having density 1.3 gcm–3 and height of 20 cm
is entered in one arm of a tube. Find the height of a liquid
having density 0.8 gcm–3 entered in other arm of manometer
Hg
so that free end of both liquid in manometer remains same.
(rHg = 13.6 g cm–3)
(A) 10 cm (B) 8 cm (C) 16 cm (D) 19.2 cm
(30) A wooden raft having mass 120 kg having density 600 kgm–3 floats on surface of water. Find the
maximum mass placed on the raft so that it sinks in the water. (g = 10 ms–2)
(A) 80 kg (B) 50 kg (C) 60 kg (D) 30 kg
D
(31) A semisphere bowl having density 3 × 104 kgm–3 is float on d
the surface of liquid. Density of liquid is 1.8 × 103 kgm–3. If
outer diameter of a bowl (D) is 1m. Find internal diameter
r
(d) of bowl.
(A) 0.94 m (B) 0.97 m s
(C) 0.98 m (D) 0.99 m
(32) A sphere of radius (r) is filled with dust of unknown substance as shown in figure and concrete
is filled in remaing part of the sphers of radius R. Specific density of concrete and unknown
substance are 2.5 and 0.5 respectively. When this sphere is placed in water then it is just sink
in water find the ratio of mass of concrete and unknown substance.
unknown substance (density = r2)

R
Water r Concrete (density = r1)

density = s
›

(A) (B) (C) (D) 3


3 5 1
5 3 3
(33) A cubical block floats on surface of liquid such that half of its volume is in the liquid if a

container accelerated in upward direction with accelaration , then ...... part of a block inside
g
4
the water.

(A) 2 (B) 8 (C) 3 (D) 4


1 3 2 3

128
(34) As shown in the figure, a liquid of density 2d filled up to height 2 and a liquid of density d filled
H

up to height . If a cylinder having crosss sectional area and length (L) (Where L < ) is
H A H

placed in the container as shown in figure. Find density of the cylinder (D). (atmospheric
2 5 2

pressure = P0).

(A) d d
5
4 A
H 5

L
2
(B) d
4
5
L
(C) d 4
2d
H
2

(D)
d
5 Frog
(35) A cubical block is partially immersed in water as shown
in the figure. A frog is placed on the surface of block.
l Depth of a block in side the water is l. If a frog jumped
to water than ......
h (A) l decreases and h increase
(B) l increases and h decrease
(C) l and h both increases
(D) l and h both decreases

(36) An oil having density 0.8 gcm–3 is filled in upper part


of a mercury having density 13.6 gcm–3 as shown
Oil
in figure. If a sphere remains in equilibrium such that
its half part in the liquid and half part in the mercury.
Density of material of a sphere is ......
mercury
(A) 3.3 (B) 6.4

(C) 7.2 (D) 12.8

(37) A rod having density (r) is kept in a huge tank filled


Q
with liquid having density (r0) then it remains
equilibrium at an angle q with bottom of container.

R If depth of liquid in a tank is L2 then ......

FB L
S U0 1 U0
(A) sin q = (B) sin q = .
1
2 U 2 U

W U0 U0
P q (C) sin q = (D) sin q =
U U

129
(38) Weight of an object in air is 250 g, in water is 200 g and in liquid is 150 g then ......
(A) density of liquid is one forth of the density of object (B) Object will floats on surface of wator
(C) density of an object is 5 gcm–3 (D) density of liquid is 2 kg m–3
(39) A rectangle block having mass (m) and area of cross-section A is totally immersed in liquid
having density (r). If it is slightly displaced from its equilibrium then it starts oscillation with
periodic time (T). Then ...... .
1
(A) T µ (B) T µ (C) T µ (D) T µ
1 1
A U m
U
Ans. : 26 (C), 27 (D), 28 (A), 29 (D), 30 (A), 31 (C), 32 (B), 33 (A), 34 (A), 35 (D), 36 (C),
37 (A), 38 (C), 39 (A)
Streamlines :
ˆ Streamlines can never intersect each other.
ˆ The tengent drawn at any point represent the direction of velocity of the fluid at that point.
Equation of continuity :
ˆ In fluid mechanics equation of continuity represents law of conservation of mass.
v2
A1v1 = A2v2
\ Av = constant v1 A2

\vµ
1
A

Bernoulli’s equation :
A1
ˆ Bernoulli’s equation for streamline flow which is steady, irrotational, incompressible and
non-viscous.

P1 + rv12 + rgy1 = P2 + rv22 + rgy2


1 1
2 2

P+ rv2 + rgy = constant


1
2

ˆ The first term is known as “Pressure head”


The second term is known as “Velocity head” and
A
The third term is called “Elevation” a
v1 ® v2®
Venturie meter :
r

ˆ It is used to measure the velocity of


A B throat
the fluid.
r = density of the fluid h

r0 = density of the liquid in the manometer

A = Area of the big cross-section

a = Area of throat r0

130
ˆ Velocity at big cross section

2 (U  U 0 ) g h
v1 = a
U (A 2 – a 2 )

ˆ velocity at throat,

2 (U  U0 ) g h
v2 = A
U (A 2 – a 2 )

Torricelli’s law :

ˆ The velocity of the liquid coming out of hole at a depth h from the sarface of the liquid is equal
to the terminal velocity of the freely falling particle from the same height.

2g h where,
v= §A · ( Q A2 << A1) A1 = Area of the free surface of the liquid
2
1– ¨ 2 ¸ » 2g h
© A1 ¹ A2 = Area of the hole

Note :

ˆ A person standing close to the moving train may pulled towards the train because the air which
is in contact with the train also move with large velocity. As a result pressure of the air
decreases. Due to the pressure difference the person may pulled towards the train.

ˆ Blowing off roots by wind storms.

ˆ During a tornado, when a high speed wind blows over a straw, it creates a low pressure. The
pressure below the roof is high. As a result, the roof is lifted up and is then blown off by the
wind.

(40) An incompressible liquid flows in the A


v2 = 2.5 ms–1
horizontal plane, in Y shape joint of a
pipe. What would be the velocity of
the liquid at crossectional area 2.5 A,
as shown in the figure ? A

v1 = 6 ms–1
(A) ms–1 (B) 1.5 ms–1
5
7 2.5 A

(C) ms–1 (D) 2.25 ms–1


7
5 v=

(41) A square hole of side L is sitauted at depth (y) from the top of water tank and a circular hole of
radius R is at depth (4y). When a tank is completely filled with water then the amount of a water
comes out per second in both holes. Radius R = ...... .

(A) L (B) 2 p L (C) 2S (D)


L L
2S

131
(42) Water flows in downward direction in a tabe as shown in figure. Internel A1
diameter at top is 12 × 10–3 m. Speed of water at bottom is 0.6 ms–1.
Find internal diameter of that at a distance 2 × 10–1 m from the top. v1

(g = 10 m s–2)
(A) 5 × 10–3 m (B) 7.5 × 10–3 m A2
v2
(C) 9.6 × 10–3 m (D) 6.4 × 10–3 m
(43) A liquid flows in a tube having length (l) and radius (r) under the plessure difference P, at a rate
of constant volume. (V = volume of liquid). Volume of liquid in a tube having redius 2r is joined
with this tube is ...... (Pressure at series connection both tube is P = constant)
(A) (B) (C) (D)
V V 16 V 17 V
16 17 17 16
(44) Streamline flow of water comes out from the tap, makes a coloum
with continously decrease in cross-sectional area. True explanation of
this statement is ...... .
(A) As the water is coming down its speed is increase so due to
decrease in pressure the atmospheric pressure decreases hence the Water
coloumn of water decreases.
(B) To achive terminal velocity, water comes out reduces its area of coloumn and balance
upward and downward force.
(C) Mass of water at any cross-section is same and water is incompressible. Thus rate of its
volume remains same volume V = Av = constant therefore area dereases due to increares in
speed.
(D) Water beam becomes nerrow due to surface
tension.
(45) A container filled with water is placed on
frictionless horizontal surface. Two holes having
same diameter are in opposite side. If height 10 cm
difference between the holes is 10 cm and area of
cross-section is 0.2 cm2 find the horizontal froce
required to keep container in equilibrium ...... .
r

(g = 1000 cms–2)
(A) 2000 dyne (B) 105 dyne (C) 4000 dyne (D) 5 × 104 dyne
(46) A cylinder filled with liquid perform rotational motion about a vertical rotational axis passing
through its base. Liquid experience upward force near to the wall. If radius of a cylinder is
5 cm and angular speed is 1 rotation/sec. find the height difference of liquid at the centre and
at the wall. (g = 1000 cms–2)
(A) 5 p2 (B) 0.05 p2 (C) 0.5 p (D) 10 p

132
(47) A cylinder filled with liquid performs motion about its axis then liquid experience upward force
near to the wall. If the height difference of liquid near the wall and at the centre is 2.0 cm
then ...... is correct. (r = 0.04 m , w = 2 rps, g = 10 ms–2, p2 = 10)
(A) liquid comes out from the cylinder (B) a liquid does not cames out from the cylinder
(C) liquid just comes out time the cylirder (D) None of these.
(48) Two capillary having same radius and same length are kept on horizontal table. when same
pressure difference is applied at both end then rate of flows of fluid is x. If both tubes are joined
in series and same pressure difference is applied then rate of flow in this combination is ...... .
x
(A) (B) x (C) 2x (D) non of these
2
Ans. : 40 (C), 41 (D), 42 (D), 43 (B), 44 (C), 45 (C), 46 (B), 47 (B), 48 (A)

Viscosity :
Laminar flow : Different layers slide over each other with out getting mixed up in a steady flow,
such flow is known as laminar flow.
Viscous force :

F = h A dx
dv
So,
A = area of contact
\ F µ A dx
dv
= velocity gradient
dv
dx
ˆ hliquid > hgas h = Co-efficient of viscousity
ˆ hliquid ® decrases with increase
in temperature
® hgas increase with increase in temperature.

Stokel’s law : A resistive force on a small smooth, spherical, solid body of radius (r) moving with
velocity (v) through a viscous medium of large dimention having co-efficient of viscousity (h) is given by
F(v) = 6 phrv.
F(v) µ v

ˆ This force is velocity dependent force.

Terminal velocity (vt) : When weight (W) = buoyort fore (Fb) + viscous force (Fv), the resultant force
on the sphere is zero and sphere travels with constant velocity. This velocity is known as terminal
velocity (vt).

2
terminal velocity vt = 2 r g
9 K
(r – r0) r = density of sphere, r0 = dencity of liquid

Poiseiulle’s law : Volume of the liquid passing through the tube in one second is

S P r4
V = 8K l

133
r
ˆ Velocity of a layer situated at distance (x) x

from the axis of tube is v = 4K l (r2 – x2)


P

Where P = Pressure difference l


h = co-efficient of viscousity
Here, NR is dimension less.
Reynold’s Number where r = density
v = velocity If, NR < 2000 Þ Streomline flow
UvD
NR = K h = co-efficient of vescousity 2000 < NR < 3000 Þ flow is unstable
D = diameter of a b tube NR > 3000 Þ flow is turbulent

(49) A square plate of length 0.1 m slides on other plate with speed 0.1 ms–1. If viscous force is
0.002 N and co-efficient of viscosity is 0.01 poise. The thickness of a liquid layer between two
plates is ...... m.

(A) 0.1
v (B) 0.05
(C) 0.005
(D) 0.0005

v0 = 0
(50) A sphere of radius r and density r falls freely
from height 10 cm. Another sphere of same
material falls freely from height h. h cm
10 cm
Radius of other sphere is 2r. If both sphere
maintain their velocity in water then h = ......

(A) 80 cm (B) 40 cm Water

(C) 160 cm (D) Insufficient Information

(51) A small solid sphere acquires terminal velocity in viscous medium match the colour :

A B

(a) Buoyant force acts on sphere (i) Increares

(b) viscous force on sphere (ii) decreases

(c) Resultant force on the sphere (iii) constant

(d) acceleration of a sphere (iv) zero

(A) (a) ® (iii), (b) ® (i), (c) ® (ii), (d) ® (iv) (B) (a) ® (i), (b) ® (ii), (c) ® (iii), (d) ® (iv)

(C) (a) ® (ii), (b) ® (i), (c) ® (iii), (d) ® (iv) (D) (a) ® (iv), (b) ® (ii), (c) ® (iii), (d) ® (i)

134
(52) A small sphere of radius r moving with terminal velocity in liquid having viscous co-efficient hŒ
the ...... is true.
Km g
(A) vr µ (B) vr µ mgrh (C) vr µ r K (D) vr µ
m gr mg
K r
Ans. : 49 (D), 50 (C), 51 (A), 52 (C)

Surface tension, surface energy and capillarity


Cohesive force : The inter molecular attractive force between the molecules of same substance is
called cohesive force.
Example :
(1) It is difficult to seperat two glass plate stic with water
(2) It is difficult to divided a mercury drop in many droplets.
Adhesive force :
ˆ The attractive force between the molecules of different substance is known as adhesive force.
(1) We can write on board
(2) Adhesive force between brick and cement.
Surface tension :
ˆ ‘The force exerted by the molecule lying on one side of an
imaginary line of unit length, on the molecules lying on other
side of the line which is perpendicular to the line and parallel
to the surface is define as the surface tension (T) of a
liquid.’
ˆ Surface of liquid has a tendency to contrac due to surface
tension.
Some interesting phenomencn based on surface tension :
Example :
ˆ Water droplets are spherical
ˆ When shaving brush or painting brush is dipped within the water hairs are well seperated, but
when the brush is taken out of the water, hairs get stick with each other.
ˆ some insects can walk on water sarface.
Surface energy :
ˆ The potential energy stored per unit area in free surface of liquid is known as surface energy
ˆ unit : J m–2 or erg cm–2
Surface tension :

T = 'A
W
ˆ
ˆ Work done to increase the unit surface area is equal to the measure of surface tension.

135
Angle of contact :
Point of contact ˆ The angle between the tangent to the liquid
surface at the point of contact and solid
surface inside the liquid is called angle of
contact.
q
q q = angle of contact

Water drops and bubbles :


(1) A bubble in air.
ˆ Pressure differce = Pi – P0.
\ Pi = pressure in side the bubble
P0 = pressure out side the bubble
ˆ Suppose the bubble is devided into two r
semi spheres as shown in the figure.
ˆ Here there are two free surface (inside and outside)
Force F = T × 2 (2pr) .............................. (1) (bubble has two free surface)
ˆ and force due to excess pressure is
F = (Pi – P0) pr2 ............................ (2)
ˆ compare (1) and (2)

(Pi – P0) = 4rT


(2) bubble inside the liquid
(Pi – P0) = 2rT (It has one free surface)
(3) For water drop :
Pi – P0 = 2rT (It has one free surface)

(4) bubble having charge


ˆ Radius of a bubble increase when charge deposite on its surface.
ˆ Initial pressure inside the bubble

P i = P0 + r
4T
1
0
P

P0 + r
4T V2
–
4T
0
P

P0 + r
1 2 0
r2
2
r
1

136
ˆ final pressure inside the bubble

V
Pi = P 0 + r – 2 
2
(s = surface charge density)
4T
2 0

ˆ If tempereture remains constant according to Boyle’s law


§ 4T · 4 § 4 T V2 · 4
¨ P0  r ¸ pr1 = ¨ P0  r – 2  ¸ pr23
3
© 1 ¹ 3 © 2 0 ¹ 3

ˆ radius (r2) can be calculated from alove equation.


Capillarity
ˆ The phenomenon of rise or fall of a liquid in a capillary held vertically in a liquid is called
capillarity.
ˆ Angle of contact q < 90°, meniscus – concave, water wets the surface, water rise in capillary.

q < 90° q > 90°


q

(fig. A) (fig. B)
ˆ Angle of contact q > 90°, Meniscus – convex, liquid (mercury) doesnot wet the surface liquid
falls in the capillary.
Equation of height :
ˆ Excess Pressure = Pressure due to liquid coloum.

= hrg
2T
R O
R
T = =Surface tension r
(but Cos q = )
r
R R = radius of meniscus q
h
\ h = RU g
2T r = radius of capillary
h = height of the liquid column
2 T cos T
\ h = r Ug r = density of liqnid
q = angle of contact
(1) q < 90° ® h is positive
liquid will rise up.
(2) q > 90° ® h is negative.
liquid will fall down
(3) If q, T, r is constant


1
r

137
(4) For two coaxial tubes having radius r1 and r2 (Inner tube is solid)

h = ( r  r )Ug
2T
2 1

r1

r2

Cross-section of the tube

(53) n water droplets of radious (r) are unite to form a big drop of radious (R) then increase in
tempereture is ...... ( T = Surface tension, specific density of water = 1 unit)

(A)
2T
rJ
(B)
3T
J 1r  R1 (C)
–3 T
rJ
(D)
3T
J 1r  R1
(54) Radins of the arms of U-tabe are r1 and r2. The height difference of r1 r2

a liquid having density (r) filled in the tube is h. If angle of contact


q = o, then surface tension T = ...... . h

Ughr1 r2 Ugh ( r2 – r1 ) h1
(A) 2( r – r ) (B) 2 r1 r2
h2
2 1

2 ( r2 – r1 ) Ugh
(C) Ugh r r (D) 2( r – r )
1 2 2 1

(55) Surface energy of a liquid drop is u. If it divided in to 512 equal droplets, then total surface
energy of all droplets are ......
(A) u (B) 8u (C) 64 u (D) 512 u

(56) Two bubble of soap solution are combine and form a big bubble. If V = change in volume inside
the bubble, S = change in area then which of the following is true. (P0 = atmospheric pressure,
T = Surface tension.)
(A) 3P0V + 4ST = 0
b
a

(B) 4P0V + 3ST = 0

(C) P0V + 4ST = 0


c
(D) 4P0V + ST = 0
(57) Find the work required to be done to double the diameter, of the bubble of soap solution form in
air, T = surface tension = 30 dyne cm–1

(A) 360 p (B) 720 p (C) 90 p (D) 180 p

138
(58) 1000 mercury droplets are unite to form a big drop of radius R. The ratio of total surface energy
of all droplets to the surface energy of big drop is ......
(A) 1 : 10 (B) 10 : 1 (C) 100 : 1 (D) 1 : 100
(59) Two soap bubble of radius 1 cm and 2 cm are combine and form a big bubble. If tempereture
remains constaund during this process. than radius of big drop is ...... .
(A) 2.4 cm (B) 1.5 cm (C) 1.1 cm (D) 0.66 cm
(60) A capillary of radius 0.2 mm held vertical in a container filled with water. Find the pressure
applied on a capillary so that water level in capillary is same as the water surface in cantainer (T
= 0.07 Nm–1, atmospheric pressure P = 105 Nm–2).
(A) 103 (B) 99 × 103 (C) 100 × 103 (D) 101.4 × 103
(61) When an air bubble rises from bottom of a lake to surface of lake, its volume increases by four
times. If 75 cm of mercury coloum producer atmosphere depth of a lake is ...... m. (density of
water is one tenth of that of mercury)
(A) 45 m (B) 7.5 m (C) 22.5 m (D) 12.5 m
(62) When an air bubble rises from bottom of a lake to surface of a lake, its diameter becomers three
times. Tempereture of a bubble remains same. Barometric height at the surface with respect to
relative density of mercury is ......

(A) 26 s H (B) 26 V (C) 9 s H (D) 9 V


H H

Ans. : 53 (B), 54 (A), 55 (B), 56 (A), 57 (D), 58 (B), 59 (C), 60 (D), 61 (C), 62 (B)
Heat transfer :
ˆ Type of heat transfer
(i) Heat conduction (ii) Heat convection (iii) Thermal radiation
Heat (Thermal) conduction :
ˆ ‘The flow of heat energy between the adjacent past of a body due to temperaturs difference be-
tween them is called thermal or heat conduction.
ˆ The constituent particles in solid vibrate about their mean position, depending on their
temperature and not perfom real linear motion.
Non steady state :
ˆ Temperature at every cross section changes with time.
Steady state :
ˆ Thermal steady state temperature at every cross-section remain same. Temperature ture
decrease from hot end to cold end.
Remember, temperature of each parts becomes constant but not equal but it is
gradually decresing from hot end to cold end.
Iso - thermal surface :
ˆ A surface perpendicular to heat conduction maintain at constant temperature is known as
isothermal surface.
ˆ Two isothermal surfaces do not intersect each other.
ˆ Shape of isothermal surface depends upon type of heat conduction and shape of a
conductor.
ˆ Such isothermal surfaces are perpendicular to the heat conduction.

139
Temperature gradient :
ˆ Rate of change of temperature in direction of heat conduction is known as temperature
gradient.
§ ' T ·  dT
Temperature gradient = –''xT = lim ¨ ¸
' x o0 © ' x ¹ dx
ˆ (negative sign indicate that temperature decrease with distance).
unit : C°/m–1 or Km–1
ˆ Heat current
H= = –kA
dQ dT
dt dx

ª T2  T1 º
H = – kA « »
¬ L ¼

ª T1  T2 º
H= = kA « L »
Q
t ¬ ¼
ˆ Amount of heat flows through the conductor in time (t).
ª T1  T2 º
Q = kA « »t
¬ L ¼
k = thermal conductivity, A = area of cross-section, T1 = Temperature at hot end
T2 = Temperature at cold end, L = length of a conductor = Thickness of bottom of a container.
Thermal conductivity (k) :
ˆ Amount of heat flowing per unit time perpendicularly between the planes having unit
temperature gradient between then per unit area is known as thermal conductivity.
MKS unit : cal s–1 m–1 K–1 or Wm–1 K–1. Dimensional formula : M1L1T–3K–1

Thermal resistance R = k A =
L T1 – T2
H
MKS unit : KsJ–1 or K watt–1. Dimensional formula : M–1L–2T3K1
ˆ A compound slab can be obtained by fusing two slabs having different thermal condctirity.
ˆ Following two types of connections are possible.
(1) Series connection : T
Parallel connection
T
1 2
L1 L2

k1 k2
T1 T2 A1
k1
Tx
Q Q
t t
Rs = R1 + R2
A2
k2
1 § L1  L 2 ·
= ¨
A © k1 k2 ¹
¸
T2
T1 L

140
For series connection Rs = R1 + R2

§ L1  L 2 · L1 L2
¨ ¸ = + k A For parallel connection Rp
© sk A ¹ k1 A 2

L1  L 2
\ ks = L R p = R1 + R 2
1 1 1

 2
1 L
k1 k2

k1 A 1 k2 A 2
If : L1 = L2 = L = L1
+ L2

2k1k2
ks = k  k = L (k1A1 + k2A2) (\ L1 = L2 = L)
1 1
1 2 Rp

L
For n slabs \ Rp = k1 A  k2 A 2
1

L1  L 2  ........  L n
ks = kp (A1  A 2 ) = k1 A1  k2 A 2
L L

  .....  n
L1 L2 L
k1 k2 kn

k1 A 1  k2 A 2
\ kp = A1  A 2

If A1 = A2 = A
k1 A  k2 A
kp =
2A

k1  k2
kp =
2
For n - slabs
k1 A1  k2 A 2  ........  kn A n
kp = A1  A 2  ......  A n

Phenomeon of formation of ice in a lake :


ˆ Thickness of ice level increase from x1 to x2 time
where, r = density of water
1 UL
t= (x22 – x12) L = latent heat of water
2 kT
k = thermal conuctivity
\ t µ (x22 – x12) T = negative temp of atmosphere
Remember :

ˆ A compound slab can be divided in series connection and parallel connection.

ˆ If every point on the contact surface are at same temperature then they are connected in series.

ˆ If temperature at every point on the contact surface continously decreases from hot end to
cold end then they are connected in parallel.

141
(63) A cylindrical shell having thermal conductivity (k) is fixed on a cylinder having radius r and
thermal condutivity 2k. Internal and outer radius of shell is r1 and r2 respectively. Temperature at
both ends are T1 and T2 (Where T1 > T2). Find equivailent thermal conductivity ?

k2 2r

T1 r T2
k1

L
(A) 5 (B) 4 (C) 4 (D) 3
4k 5k 3k 4k

(64) Internal and outer radius of a cylindrical shell are 2 cm and 4 cm respectively. Length of a
cylinder is 50 cm. Temperature at internal surface and outer surface are T1 = 0°C and
T2 = 200°C remains constant. Thermal conductivity is 69.3 Wm–1K–1. Calculate the rate of heat
flow perpendicular to outer and inner surface.

dr

r2
r

r1

(A) 2.72 ×104 Js–1 (B) 2.72 ×107 Js–1 (C) 6.28 ×104 Js–1 (D) 6.28 ×107 Js–1
(65) A compound slab is shown in figure. Temperature at top and botton are T1 and T2 respectively.
Find equivalant thermal conduetivity k (Where T1 > T2) Dimensions of each block are show in
figure.

x T1 T1 T1

k2

k1
k4
2x
k3
x
T2 T2 T2

x 2x x

( k1  k4 ) ( k2  k3 )  4 k2 k3 k1  k2 k3  k4
(A) 4( k2  k3 )
(B)
3

4k1k4  (k2  k3 ) (k1  k4 )


(C) (D) non of the above
2k2 k3

142
(66) Calculate the equivalent thormal conductivity of a compound slab shown in figure (Where T1 > T2)

(3)
(1) 2x
T2
2k
k
T1 x
x

T2
k
T1 2k (2) (4)

x x

20 k 40 k 3k
(A) 3k (B) (C) (D)
27 27 2

(67) A spherical thermocol container contains 10 kg ice. Internal and outer radius of a container are
25m and 30m respectively. 335 kJ heat energy is required to melt 1 kg of ice. Thermal
conductivity of thermocol is 0.028 Jm–1K–1s–1. Consider walls of container in thermal steady state.
Calculate the time in which half of the ice melts ?

(A) 90 h (B) 3800 s. (C) 9000 s. (D) 20 h.

(68) The dimensions of the celling of a room are 5 m×5 m×10 cm. Thermel conductivity of concrete is
1.26 W/m°C. At one moment, the temperature outside and inside the room are 44°C and 32°C
respectively. A layer of thermocol of thickness 5 cm and thermal conductivity 0.0275 W m–1°C–1
is laid on the ceiling. A layer of bricks of thickness 7.5 cm and thermal conductivity 0.65 W m–
1
°C–1 laid on the ceiling. Find new rate of heat flow.

(A) 155.8 Js–1 (B) 20.337 Js–1 (C) 0.924 Js–1 (D) 0.0064 Js–1

(69) Five rods of different material but having same dimensions are connected as shown in figure. If
heat current in rod CD is zero find thermal conductivity of rod AD. C
k1 k2
k1 = 370 Wm–1K–1 (Copper)

k2 = 320 Wm–1K–1 (Gold) A k5 B

k4 = 16 Wm–1K–1 (Steel) k3 k4
k3 = ? D

(A) 74.00 (B) 13.83 (C) 18.5 (D) 185

(70) The thickness of ice layer on the surface of lake is 8 cm. Temperature of environment is –12°C
find the time require for the thickness of ice layer becomes 15 cm. Thermal conductivity of ice
0.004 cal K–1cm–1s–1, density of ice 0.92 g cm–3, latent heat of fusion is 80 cal g–1.)
(A) 21.4 h (B) 34.3 h (C) 27.7 h (D) 4.4 h
Ans. : 63 (B), 64 (C), 65 (A), 66 (C), 67 (D), 68 (A), 69 (C), 70 (B)

143
Heat convection
ˆ The transfer of heat, due to the difference in the density of fluid is callled heat convection.
ˆ Here, the constituent particles actually move from one place to the other.
ˆ In heat transfer occurs on the earth, the maximum contribution is of heat convection only.
Natural heat Convection
ˆ Langmuir - Lorentz law

–
dT
= k' T  Ts 4
5
Where, T = temperature of the system
dt
Ts = temperature of surrounding
k' = proportionality constant
Forced heat Convection
ˆ Newton’s Law of cooling
k' = propertionality Constant
– = k' (T–Ts)
dT
dt

Thermal Radiation

ˆ Every substance emitts electromagnetic radiation of difinate frequencies in accordance with its
temperature.

ˆ Thermal radiations are electromagnetic waves only, they travel with the speed of light, in free space.
ˆ The medium is not required for their propagation.
Perfect Black Body :

ˆ The body which absorbs all the radiant energy incident on it is called a perfect black body. e.g. Sun.

ˆ The good absorber of heat is also good reflector of heat.

ˆ When a black body is heated upto certain high temperature it emitts all wave lengths.

Total emissive Power

ˆ The amount of radient energy emitted per unit area per second, at a given temperatue, is called
total emissive power (W)

Its unit is : Wm–2


radient energy absorbed
ˆ absorptivity (a) =
radient energy incident

Total emissive power of the body


ˆ emissivity (e) =
emissive power of the black body at the same temp.

ˆ For perfect black body a = 1 and e = 1

Kirchhoff’s law :

The values of emissivity and absorptivity are equal for every surface.

144
Steafan - Boltzman’s law
ˆ The total emissive power of the body is directly propotional of forth power of its absolute
temperature. Where s = 5.67 ×10–8 Wm–2K–4
W µ T4 s = Steafan Boltzman’s constant
dimenssional formula = M1L0T–3K–4
\W=esT 4

The rate of loss of heat due to radiation Where T = temperature of the system
Ts = temp of the surrounding
= e s A (T4 – Ts4)
dQ
ˆ dt

Wien’s Displacement Law


ˆ With the increase in temperature, the wavelength lm corresponding to maximum value Wl
decreases. Where k = wien’s constant
\ lmT = constant (k) k = 2.89 × 10–3 mK

The D.F. of the constant : M0L1T0K1


(71) The temperature of the body decreases from 90°C to 74°C in 4 min. The temperature becomes
62°C in 8 min. What would be the tempereture of the body at the end of 20 min ?
(A) 36.4 °C (B) 42.4 °C (C) 38.4 °C (D) 40.4 °C
(72) The temperature of a liquid at 100 °C is in contact with the atmosphere having temperature
10 °C. What would be the time require to decrease the temperature of the liquid to 82 °C.
Constant k' = 0.01234567 °C-1/4 min–1.
(A) 18 min (B) 9 min (C) 6 min (D) 12 min
(73) Calculate the change in the wavelength corrosponding to the maximum energy for the perfect
black body whose temperature is increased by 30 %.
(A) 8100 % increase (B) 8100 % decrease (C) 30 % decrease (D) 30 % increase
(74) On decreasing the temperature of a perfect black body, the decrease in the wavelength
corresponding to its maximum energy is 20 %. What would be the percentage change in the
power emitted ?
(A) increases by 316 % (B) decreases by 316 % (C) increases by 416 % (D) decreases by 416 %
(75) What would be the percentage change in the temperature of a perfect black body to decrease its
emissive power by 25 %.
(A) decrease by 30 % (B) decrease by 7 % (C) increase by 7 % (D) increase by 30 %
(76) What would be the percentage change in the emissive power of a perfect black body on
increasing its temperature by 3 times ?
(A) 8100 % (B) 800 % (C) 81 % (D) 8000 %
(77) The temperature of a cup of hot milk decreases 3.65 times faster at temp 360 K than at 320 K
by 1°C. Consider milk as a perfect black body and calculate the temperature of the room.
(A) 310 K (B) 273 K (C) 285 K (D) 300 K
Ans. : 71 (D), 72 (C), 73 (C), 74 (A), 75 (B), 76 (D), 77 (D)

145
Assertion - Reason type Question :
Instruction : Read assertion and reason carefully, select proper option from given below.
(a) Both assertion and reason are true and reason explains the assertion.
(b) Both assertion and reason are true but reason does not explain the assertion.
(c) Assertion is true but reason is false.
(d) Assertion is false and reason is true.
(78) Assertion : The length of a rubber string is L. On appling the tensile force of 5N and 6N the
length becomes a and b respectively. When 9N force is applied the length becomes (a + b – L) m.
Reason : Increase in the length of the string is directly proportional to its original length
(A) a (B) b (C) c (D) d
(79) Assertion : The graph of stress ® strain for two different type of rubbers are as shown in the
figure. Rubber A is more useful as car tyre than ruber B.
Reason : Ruber A releases more energy than B.
Stress

Stress

A B

Strain Strain
(A) a (B) b (C) c (D) d
(80) Assertion : Two wires A and B are of equal material and are also of equal cross-section. The
length of the wire A is double than that of B. The increase in the length of wire A is
double than of B.
Reason : Increasing in the length is directly proportional to the original length.
(A) a (B) b (C) c (D) d
(81) Assertion : Two wires A and B are of equal material and are also of equal length. The diameter
of wire A is double than that of B. Now increase in the length of wire B is 4 times
than that of A.
Reason : Increase in the length of the wire is inversely proportional to its cross-sectional area.

(A) a (B) b (C) c (D) d


(82) Assertion : When a tension force is applied on an object, the restoring force is produced due to
the inter molecular force of attraction.
Reason : The restoring force produced is due to the internal property of the object and not due
to intermolecular force of attraction.
(A) a (B) b (C) c (D) d

146
(83) Assertion : To maintain a piece of paper floating horizontally in air, we must blow air above the paper
and not below.
Reason : In a steady flow of a fluid, for a given mass, the total energy is conserved.
(A) a (B) b (C) c (D) d
(84) Assertion : When a fluid is flowing through a small hole of a vessel than the backforce acts on
the vessel.
Reason : For a given mass of fluid the total energy is fully in the form of kinetic energy.
(A) a (B) b (C) c (D) d
(85) Assertion : The critical velocity of a fluid passing through a tube is inversly proportional to the
radius of the tube.
Reason : The velocity ot the fluid passing through a tube is inversly proportional to the area of
the cross section.
(A) a (B) b (C) c (D) d
(86) Assertion : To keep a light ball rotating about its own axis in air, the blow of air must be as
shown in the figure.

Reason : Due to the viscosity of air there exsist upward thrust.


(A) a (B) b (C) c (D) d
(87) Assertion : The upward lift of an aeroplane when it moves horizontally, is due to the pressure
difference between over and below the wings.
Reason : The velocity of the air over the wings is more than that below the wings.
(A) a (B) b (C) c (D) d
(88) Assertion : No force is acting on an object freely falling with its terminal velocity.
Reason : The weight of the object is balanced by the upward buoyant force.
(A) a (B) b (C) c (D) d
Ans. : 78 (D), 79 (C), 80 (A), 81 (A), 82 (C), 83 (A), 84 (C), 85 (C), 86 (C), 87 (A), 88 (C)
Comprehension Type Questions
Passage-1
L L
Two conducting rods P and Q are of equal
cross-section area (A) and length (L) are kept between two P Q
rigid walls as shown in the figure. Their linear coefficient of
expansion are a1 and a2 and Young’s modulus are
Y1 and Y2 respectively. The temperature of both the rod
increases by T.
(89) The force exerted by any one rod, on the other is ...... .

TA (D1  D 2 )
(A) F = (B) F = TAY1Y2 (a1+a2)
§ 1 ·
¨  1 ¸
© Y1 Y2 ¹

(C) F = TA (Y1+ Y2) a1a2 (D) None of the above


147
(90) The new length of the rod P

ª F º ª F º
(A) L1 = L «1  D1 T  AY » (B) L1 = L «1  D1 T  AY »
¬ 1¼ ¬ 1¼

ª F º ª F º
(C) L1 = L «1  D1 T – AY » (D) L1 = L «1  D1 T – AY »
¬ 1¼ ¬ 1¼

(91) The new length of the rod Q

ª F º ª F º
(A) L2 = L «1  D 2 T  AY » (B) L2 = L «1  D 2 T  AY »
¬ 2¼ ¬ 2¼

ª F º ª F º
(C) L2 = L «1  D 2 T – AY » (D) L2 = L «1 – D 2 T – AY »
¬ 2¼ ¬ 2¼

Passage-2
One end of a steel rod of length 1m and cross-section area
0.01 cm2 is fixed with a rigid support and a sphere of 2 kg is
attached at the other end. Now as shown in the figure the sphere
is given rotation on the circular path of radius 0.2 m with a
constant angular speed w in such a way that the wire makes an
angle q with vertical line. (q = 30°)
(92) Angular speed w = ......
(A) 5 rad s–1 (B) 6.58 rad s–1 (C) 5.37 rad s–1 (D) 9.30 rad s–1
(93) The tension force porduced in the wire is = ......
(A) 23.12 N (B) 40 N (C) 34.6 N (D) 266.5 N
(94) Increase in the length of the wire DL = ......
(A) 4.62 ×1018 m (B) 1.156 ×10–4 m (C) 2 ×10–4 m (D) 1 ×10–4 m
(95) The stress produced in the wire = ......
(A) 20 ×106 N m–2 (B) 16 ×106 N m–2 (C) 24 ×106 N m–2 (D) 4 ×106 N m–2
Passage-3
As shown in the figure mass m and M = 2m are tied
to two ends of a wire of cross - sectional area A passed
over a frictionless pulley. Now the system is made free T
T
from the equilibriam. m

(96) The common acceleration of the blocks is = ...... M

(A) g (B) (C) (D)


g 2g 3g
3 3 2

(97) The stress produced in the wire = ......

(A) (B) (C) (D)


Mg 2 mg 3M g 4mg
A 3A 4A 3A

148
(98) If m = 1 kg, A = 8 ×10–9 m2, Braking Stress = 2 ×109 Nm–2 and g = 10 ms–2. The maximum
value of M for which the wire does not break is ...... .

(A) 4 kg (B) 6 kg (C) 8 kg (D) 20 kg

Passage-4

When a fluid passes through a tube, there exsist relative velocity between fluid layers. As
a result, resistive force is produced at the surface of layers in contact. This force is called

viscous force. According to Newton’s law for Viscous flow, the frictional force F = –h A × .
dv
dx

Where A is area of contact between two layers. h is co-efficient of viscocity and dx is


dv

velocity gradient.

(99) If f is the frictional force required for one solid object to move over another solid object and F
is the frictional force acting between two consecutive layers of the liquid then...

(A) f is indepent of the area of contanct between the surfaces of the solids.

(B) f depends on the relative velocity between the solids.

(C) f depends on the area of the liquid layer.

(D) f is independent to the relative velocity between the liquid layers.

(100) The dimenssional formula for the co-efficient of viscosity.


(A) M1L–1T–1 (B) M1L1T–1 (C) M1L–2T–2 (D) M1L–1T–2

(101) The depth of a river is 5 m. The velocity of the water at the uppermost layer is 2 ms–1. The co-
efficient of viscosity is 10–3 SI unit. Calculate the viscous force acting per unit area of contact ?

(A) 10–4 Nm–2 (B) 2 × 10–4 Nm–2 (C) 4 × 10–4 Nm–2 (D) 5 × 10–4 Nm–2

Passage-5

Figure shows a cylindrical vessel having cross-


sectional area A. Two non viscous liquid which do not
get mixed are filled in this vessel. The density of the
50 cm

liquid-1

liquids are 0.6 g cm–3 and 1.2 g cm–3 respectively. The


height of both of the liquid is 50 cm. A small hole is
50 cm

liquid-2 bored on the Surface of the Vessel, at a height of


25 cm 25 cm from the bottom. The cross-section area of the
x
hole a (<<<A).

(102) The initial speed of the liquid coming out of the hole is ...... .
(A) 88.54 cm s–1 (B) 62.60 cm s–1 (C) 44.27 cm s–1 (D) 31.30 cm s–

149
(103) The initial horizontal range x of the liquid = ...... .

(A) 100 cm (B) 70.71 cm (C) 50 cm (D) 35.35 cm

(104) The height of the hole required to have maximum range x is ...... cm from the bottom.

(A) 66.66 (B) 150 (C) 75 (D) 50

Passage-6

A cylindrical water-tank of cross-sectional area a1 is open at the top. The height of the water
level in the tank is h. A small hole having cross-section area a2 is at the bottom of this tank,
where a1 = 3a2.

(105) The initial speed of the water falling from the tank.

(A) (B) (C) (D)


gh 1 gh
2 gh gh 2 2

(106) The initial speed of the water coming out at the hole ...... .

(A) (B) (C) (D) 2 2 gh


1 gh 3 gh
2
2 gh 2

(107) The time consumed to empty the tank is ...... .

(A) (B) 4 g (C) 6 g (D) g g


2h h 2h 2h
g

Passage-7

Every substance emits electromagnetic rodiation of definate frequency in accordance


with its temperature. This radiation is known as thermal radiation. The energy associated with
this radiations is called radient energy. The thermal radiation propagates in the frce space or air
with the speed of light. The thermal radiation also experiences reflection and refraction same as
those of light and also produce phenomenons like interference, diffraction and polarization.

The body which absorbs all the radient energy incident on it is called perfect black body.

The radient energy emitted per second through the unit area is

W = sT4 Where, T = temperature of the black body, s = Slefan-Boltzman constant

If the body is not perfect black

W = esT4

e = emissivity of the surface.

150
(108) The dimenssional formula for s.
(A) M1L–2T–2K–4 (B) M1L–1T–2K–4 (C) M1L1T–3K–4 (D) M1L0T–3K–4
(109) What is the SI unit of s ?
(A) Js–1K–4 (B) Wm–1K–4 (C) Wm–2K–4 (D) Jm–2K–4
(110) In which part of the electromagnetic wave the thermal radiations are laying ?
(A) Visible light (B) Infrared (C) Ultraviolet (D) microwave
(111) Which appratus is used to detect thermal radiation.
(A) Constant gas thermometer (B) Platinum resistance thermometer
(C) Thermostate (D) Thermopile
(112) An object B of temp T2 is wound on object A having higher temperature T1. (T2 < T1). The rate
of heat loss for object A is ...... .
(A) T14 (B) (T1 –T2)4 (C) T1 –T2 (D) T14 –T24

Ans. : 89 (A), 90 (C), 91 (C), 92 (C), 93 (A), 94 (B), 95 (A), 96 (B), 97 (D), 98 (D), 99 (A) &
(C), 100 (A), 101 (C), 102 (D), 103 (B), 104 (C), 105 (D), 106 (C), 107 (B),
108 (D), 109 (C), 110 (B), 111 (D), 112 (D)

151
12 Thermodynamics
Thermal Equilibrium
When temperature of system A and system B becomes equal, then heat exchanged between them
becomes zero. It is said that thermal equilibrium has been established between system A and system B.
Zeroth law of thermodynamics
When system A and system B are in thermal equilibrium with a third system C then system A and B
are said to be in thermal equilibrium with each other.

TA = TC ½
° ÞT =T
¾ A B
TB = TC °¿

Thermal Expansion
Thermal Expansion
Linear expansion Surface expansion Volume expansion
(1- dimensional) (2-dimensional) (3-dimensional)
- Change in length takes place. - Change in length and breadth - Change in length, breadth
takes place. and height takes place.
( photographic enlargement)

- Dl = alDT

= 'l DA = bADT DV = g V DT
l 'T
a

= 'V
'A
= Coefficient of linear = A 'T
V'T
a b g

expansion
Unit : a = °C–1 or K–1 b = Coefficient of surface g = Coefficient of volume
expansion expansion
b = 2a g = 3a
Unit : °C–1 or K–1 Unit : °C–1 or K–1
Percentage change in density due to volume expansion :

U  U0 J'T
=
U0 1 J ' T
Relation between different scales of temperature :
(1) Celsius and kelvin : TC = Tk – 273

(2) Fahrenheit and celsius : TF = 9


T + 32
5 C

(3) Fahrenheit and kelvin : TF = 9


5
[Tk –273] + 32
Phase diagram :
Graph of P ® T for any substance is called its phase diagram.

152
B Liquid
­ form C

ve
rm
P rve

cur
fo
cu
lid ion

ion
at
So
ris

Fus
po
Va
A
e Triple point Gaseous
rv
cu
n

form
io
at
im
bl
Su

O T®
Curve OA : Sublimation curve. Solid and gaseous form coexists.
Curve AB : Fusion curve. Solid and liquid form coexists.
Curve AC : Vaporisation curve. Liquid and gaseous form coexists.

(1) A gas thermometer is used to measure temperature. When it is dipped in water, triple point
temperature is 273.16 K and pressure is 3×104 Nm–2. When this gas thermometer is dipped in
some other liquid, pressure indicated is 3.5×104 Nm–2 then the new temperature will be ...... .
(A) 54.6 K (B) 45.6 K (C) 54.6 °C (D) 45.6 °C
(2) There are two similar metal strips one of copper and other of brass. Here aB > aC. On
increasing temperature by DT, both strips form an arc of radius R. Then R ...... .

d2 D B  DC ' T
(A) d (aB – aC) DT (B) D  D ' T (C) D  D ' T (D)
d
B C B C d2

(3) On adding steam to 100 g water, temperature of water increases from 24°C to 90°C. How much
steam should be added ?
(A) 25 g (B) 12 g (C) 21 g (D) 100 g
(4) In a temperature scale "A", melting point of water is shown as –160° A and boiling point of
water as –50° A then in its scale, temperature 340 K will be shown as ...... .
(A) –86.3 °A (B) +86.3 °A (C) –86.3 °K (D) –86.3 °C
(5) In a thermometer, if melting point of water is 20 °C and boiling point of water is 150 °C then
50 °C will be shown in this thermometer as ...... .
(A) 85 °C (B) –85 °C (C) 58 °C (D) –58 °C
(6) Mass of ice at –20 °C temperature is 1200 g. To completely melt it, how much steam at 100 °C will
be required ?
Here, specific heat of ice S = 0.5 cal g–1 °C–1
specific heat of water S = 1 cal g–1 °C–1
Latent heat of ice L = 80 cal g–1
Latent heat of steam L = 540 cal g–1
(A) 18.75 kg (B) 18.75 g (C) 1.875 kg (D) 1.875 g

153
(7) A copper sphere of mass 1 kg is heated upto 500 °C and then placed on a big piece of ice at
0 °C then how much ice will melt ?
[specific heat of copper S = 400 Jkg–1 °C–1, latent heat of ice L = 3.5 ×105 Jkg–1]
(A) 0.57 kg (B) 570 gm (C) 5.7 kg (D) 57 kg
(8) On heating a metal sphere to temperature 60 °C, its volume increases by 0.12 % then coefficient
of linear expansion of metal wil be ...... .
(A) 6.66 × 10–6 °C–1 (B) 66.6 × 10–6 °C–1 (C) 5.56 × 10–5 °C–1 (D) 55.6 × 10–6 °C–1
(9) Co-ordinate of triple point of water is ...... .
(A) 4.58 mm-Hg, 273.16 K (B) 4.58 mm-Hg, 0 K
(C) 5.58 m-Hg, 273.16 K (D) 5.58 mm-Hg, 0 K
(10) For values of pressure and temperature at triple point, ...... forms of matter coexists in
equilibrium.
(A) Gas and liquid (B) Solid and gas (C) Solid and liquid (D) All three
(11) Relation between temperature in Fahrenheit (TF) and in Celsius (TC) is ...... .

(A) TF = 9 TC – 32 (B) TF = 5 TC + 32
5 9

(C) TF = 9 TC + 32 (D) TF = 5 TC – 32
5 9

(12) Temperature difference of 10 °C is equal to ...... temperature difference.


(A) 10 °F (B) 20 °F (C) 50 °F (D) 40 °F
(13) Temperature of body of a patient is 40° C. It would be ...... in Fahrenheit scale.
(A) 100 °F (B) 101 °F (C) 102 °F (D) 104 °F
(14) If temperature of a substance changes by 20° C then change in kelvin scale will be ...... .
(A) 293 K (B) 20 K (C) 293 °F (D) –20 °C
(15) Ice at –5 °C temperature is heated slowly till it converts into steam at 100°C. Which of the
following graph shows this entire process ?
(A) (B)
Temperature

Temperature

Heat Heat
(C) (D)
Temperature

Temperature

Heat Heat
Heat on X-axis Temperature on Y-axis

154
(16) A metal sphere of radius R and having specific heat S is rotating with angular speed
f rotation/sec about an axis passing through its centre. Now, on stopping it suddenly, its 50 %
energy is used in increasing its temperature then the equation giving increase in temperature of
sphere will be DT = ...... .

S R2 f 2 S2 R f S2 R 2 f 2
(A) (B) (C) (D)
2 S 2 2 2
5 S2 R 2 f 2 5 S 5 S2 5 S

(17) Heat capacity of aluminium piece of mass 100 g is ...... . (specific heat S = 0.2 cal g–1 °C–1)
(A) 4.4 J °C (B) 44 J °C (C) 4.4 J °C–1 (D) 44 J °C–1
(18) At triple point of water, temperature measured in Celsius scale will be ...... °C.
(A) 0 (B) –273.16 (C) 100 (D) 0.01
(19) At atmospheric pressure, when equilibrium is established between pure water and its vapour,
temperature is taken ...... K.
(A) 100 (B) 273.15 (C) 373.15 (D) 273.16
(20) Value of absolute zero temperature in fahrenheit scale is ...... °F.
(A) 0 (B) –273.15 (C) –459.67 (D) –356.67
(21) At which temperature does value on °C scale and °F scale becomes same ?
(A) 0 (B) 40 (C) –40 (D) 32
(22) At which temperature density of water is maximum ?
(A) 32 °F (B) 39.2 °F (C) 42 °F (D) 4 °F
(23) At which temperature does coefficient of volume expanssion of water becomes zero ?
(A) 0 °C (B) 4 °C (C) 15.5 °C (D) 100 °C
(24) Ratio of heat required to raise temperature of two copper spheres of radii R1 and R2 by 1K is
...... . Here R1 = 2R2

(A) 27 (B) 8 (C) 1 (D) 8


8 27 8 1

(25) A thermodynamic system moves in states (i) from P1, V to 2P1, V and (ii) P1, V1 to P1, 2V1
work done in both cases is ...... .
(A) 0, 0 (B) 0, P1V1 (C) PV1 , 0 (D) PV1 , P1V1
(26) 100 g pure water is heated from 25°C to 50°C temperature. If we neglect expansion of water,
change in internal energy will be ...... .
(specific heat of water = 4184 J kg–1 K–1)
(A) 1046.00 cal (B) 10460 cal (C) 1046.00 J (D) 10460 J

(27) For isothermal process of an ideal gas, P = ...... .


dP

(A) –g V (B) – V (C) – J V (D) –g2 V


dV dV dV dV

155
(28) For adiabetic process of an ideal gar dP = ...... .
P

(A) –g V (B) – V (C) – J V (D) –g2 V


dV dV dV dV

(29) Amount of heat required to raise temperature of a substance by 1° C is called ...... .
(A) Water equivalent (B) Heat capacity (C) Entropy (D) Specific heat
(30) Unit of coefficient of linear expansion is ...... .
(A) °C (B) °C–1 (C) m °C (D) m °C–1
(31) Length of a metal rod is 50 cm. On increasing its temperature by 100 °C, how much increase in
its length takes place ? (for metal, a = 1.1 × 10–5 °C–1)
(A) 5.5 × 10–2 m (B) 5.5 × 10–2 cm (C) 5.5 × 10–3 m (D) 5.5 × 10–3 cm
(32) Radius of a circular disc made of copper is 10 cm and there is a hole of radius 1 cm at its
center. On heating the dics, area of hole ...... .
(A) increases (B) decreases
(C) does not change (D) hole will be destroyed
(33) 5 mole gas at temperature 20 °C is adiabetically compressed at pressure 1 atm such that its
volume becomes tenth part of its original volume then final temperature is ...... .
(A) 736 K (B) 846 K (C) 736 °C (D) 523.5 K
(34) An ideal gas having volume 3 Litre and pressure 20 atm is isothermally expanded to make volume
24 L. Work required is ...... .
(A) 15600 J (B) 12600 J (C) 13750 J (D) 12.600 J
(35) A crystal has coefficient of linear expansion in one direction as "a" and in all perpendicular
direction coefficient of linear expansion is "b". Then coefficient of volume expansion for this
crystal becomes ...... .
(A) 2a + b (B) a + 3b (C) a + 2b (D) 3a + b
(36) For adiabetic process of an ideal gas, relation between pressure and temerature is ...... .
(A) Pg Tg–1 = constant (B) PVg = constant
(C) PV = constant (D) P1–g Tg = constant
(37) Dimensional equation of g in equation PV = constant for adiabetic process is ...... .
g

(A) M0L1T–1 (B) M1L1T0 (C) M1L0T1 (D) M0L0T0

(38) On adiabetically compressing a gas at 1 atm pressure, its volume becomes half of original volume
then new pressure will be ...... m – Hg. [g = 1.4]

(A) (B) 0.76 × (2)1.4 (C) 7.6 × (2)0.4 (D) 0.76 × (2)0.4
0.76
1.4
(2)

(39) Temperature of a substance on kelvin scale is T K and same temperature on fahrenheit scale is
T°F then T = ...... .
(A) 40 (B) 313 (C) 574.25 (D) 301.25

156
(40) Air inside tyre of vehicle has pressure 4 atm and temperature 27 °C. Suddenly tyre bursts, then

new temperature of air becomes ...... . [g = 5 ]


7

–2 –2 2 2
(A) 300 (4) 7 (B) 400(3) 7 (C) 300(4) 7 (D) 400(3) 7

(41) 95 K temperature on kelvin scale is equivalent to ...... on fahrenheit scale.


(A) –288° F (B) –146° F (C) –338° F (D) 178° F
(42) On heating a metal wire, its length increases by 2 % then increase in its area of cross-section is
...... .
(A) 1 % (B) 2 % (C) 3 % (D) 4 %
(43) A glass beaker at 4 °C temperature is completely filled with water and kept in a fridge. Now, its
temperature goes below 4 °C, then ...... .
(A) water will come out.
(B) no change in level of water.
(C) water will go in the beaker.
(D) water will initially go inside and then come out.
(44) A long rod of LA + L B is made by joining rod having length LA and LB of metal A and B
respectively. Coefficient of linear expansion of A and B are aA and aB respectively. When
temperature of rods are increased up to T °C, change in length of every rod is equal, then ratio
LA
L A  L B = ...... . (aC acoefficient of combine linear expansion).

DC DA
(A) (B) (C) aA . aC (D) aA + aB
DA DC

(45) Two thermometers-one having celsius scale and other having fahrenheit scale, are kept in a hot
substance showing 212° F temperature. When fahrenheit thermometer shows temperature
140 °C then celsius thermometer will shows decrease in temperature by ...... .
(A) 40° (B) 30° (C) 60° (D) 80°
(46) Length of a metal wire at 30 °C temperature is 30 cm then its length at 10 °C temperature
is ...... .
(a = 11 × 10–6 °C–1 )
(A) 30 cm (B) 29.99 cm (C) 30.10 cm (D) 29.10 cm
(47) Efficiency of carnot engine at temperature (i) 100 K and 500 K and (ii) T K and 900 K are
same. Then value of T = ...... .
(A) 250 K (B) 280 K (C) 200 K (D) 180° K
(48) On increasing temperature of a metal sphere upto 30 °C, its volume increases by 0 0.30 % then
coefficient of its volume expansion (g) will be ...... .
(A) 0.00003 °C–1 (B) 0.0003 °C–1 (C) 0.0001 °C–1 (D) 0.001 °C–1
(49) In thermal expansion, ratio of coefficient of linear expansion (a), coefficient of surface expansion (b)
and coefficient of volume expansion (g) is ...... .
(A) 3 : 2 : 1 (B) 2 : 3 : 1 (C) 1 : 2 : 3 (D) 1 : 3 : 2

157
(50) Amount of heat required to convert substance of unit mass from solid state to liquid state at
constant temperature is called ...... .
(A) Heat energy (B) Latent heat of fusion (C) Specific heat (D) Internal energy
(51) Depending on phase diagram, match the following :
Column-1 Column-2
(a) Solid and gaseous form of substance P Sublimation curve
coexists
(b) Liquid and gaseous form of substance Q Fusion curve
coexists
(c) Solid and liquid form of substance R Triple point
coexists
(d) All three forms of substance coexists S Vaporisation curve
(A) a ® S ; b ® R ; c ® P ; d ® Q (B) a ® P ; b ® S ; c ® Q ; d ® R
(C) a ® Q ; b ® P ; c ® S ; d ® R (D) a ® R ; b ® Q ; c ® P ; d ® S
Ans. : 1 (D), 2 (C), 3 (B), 4 (A), 5 (A), 6 (B), 7 (A), 8 (A), 9 (A), 10 (D), 11 (B), 12 (C),
13 (D), 14 (B), 15 (A), 16 (D), 17 (D), 18 (D), 19 (C), 20 (C), 21 (C), 22 (B), 23 (B),
24 (D), 25 (B), 26 (D), 27 (B), 28 (A), 29 (B), 30 (B), 31 (B), 32 (A), 33 (A), 34 (B),
35 (C), 36 (D), 37 (D), 38 (B), 39 (C), 40 (A), 41 (A), 42 (D), 43 (A), 44 (A), 45 (A),
46 (B), 47 (D), 48 (C), 49 (C), 50 (B), 51 (B)

1st law of thermodynamics :


DU = DQ – DW
where,
(A) DU = Change in internal energy of system
® depends only on initial and final state of system.
® If temperature of system increases, DU = positive
® If temperature of system decreases, DU = negative
® It is a function depending only on temperature of system
(B) DQ = Change in heat energy of system
® If heat given to system, DQ = positive
® If heat lost by system, DQ = negative
(C) DW = Work done
® If work done by system (its volume increases), DW = positive
® If work done on system (its volume decreases), DW = negative
1st law of thermodynamics for different processes :
(A) Isothermal process :
ˆ Temperature remains constant during entire process.
ˆ DT = 0 Þ DU = 0
\ 0 = DQ – DW
\ DQ = DW

158
ˆ Boyle's law : PV = constant

ˆ Work done W = SPDV = ³ PdV


§ V2 · § P1 ·
W = mRT ln ¨ V ¸ = mRT ln ¨ P ¸
© 1¹ © 2¹

§ V2 · § P1 ·
= 2.303 mRTlog ¨ V ¸ = 2.303 mRT log ¨ P ¸
© 1¹ © 2¹

(B) Adiabetic Process :


ˆ Exchange of heat energy between system and surrounding DQ = 0
ˆ DU = – DW
\ if work done by system, DU = negative
& if work done on system, DU = positive

ˆ Work done W = SPDV = ³ PdV


P1 V1  P2 V2 P R (T1 – T2 )
W= J 1
= J 1

ˆ Relation between P, V and T :


1J
PV = constant, TV = constant, TP J –= constant
g g–1–

(C) Isobaric process :


ˆ Pressure of system remain constant
ˆ DP = 0
(D) Isochoric process :
ˆ Volume of system remains constant
ˆ DV = 0
\ W = P (DV) = P(0) = 0
\ DU = DQ
(E) For isolated system :
ˆ DQ = 0 Þ DU = 0 Þ U = constant
DW = 0
ˆ Heat capacity :
'Q cal J
HC = Unit :
'T
;
°C K
ˆ Depends on type and mass of substance.
ˆ Specific heat :

'Q
C= = [for solid and liquid]
HC
m m' T

159
ˆ depends only on type of substance
cal ; J
Unit :
g °C kg K
ˆ Specific heat of gas at constant volume  (CV) :

§ 'Q ·
CV = ¨ P ' T ¸ v = constant =
fR
© ¹ 2

ˆ Specific heat of gas at constant pressure  (CP) :

§ 'Q · § f ·
CP = ¨ P ' T ¸ P = constant = ¨ 1  2 ¸ R = +R
fR
© ¹ © ¹ 2

ˆ Relation between CP and CV :


CP – CV = R (for ideal gas)
f2
g= C = =1+ f
CP 2
V f

(52) During a thermodynamic process, 1000 J heat is lost on doing 100 J work. Thus, change in its
internal energy will be ...... .
(A) –900 J (B) +900 J (C) +1100 J (D) –1100 J
(53) In a thermodynamics process, on changing pressure of gas, it releases 200 J heat and
100 J work is done on it. If initial internal energy of system is 10 J then final internal energy
will be ...... .
(A) 290 J (B) 90 J (C) –290 J (D) –90 J
(54) 420 J work is done on a system, then change in its internal energy is ...... cal.
(A) 420 (B) +100 (C) –420 (D) –100
(55) For hydrogen gas, CP = 3400 cal kg–1 °C–1 and CV = 2400 cal kg–1 °C–1. Work required to
increase temperature of hydrogen gas from 30° C to 40° C at constant pressure is ...... J if mass
of hydrogen gas is 10 kg.
(A) 100 cal (B) 1000 cal (C) 100000 cal (D) 10 cal
(56) If temperature of 100 m gas at 1 atm pressure is increased from 27° C to 627° C adiabetically,
3

then final pressure will be ...... . (Take g = 1.5)


(A) 27 atm (B) 2.7 atm (C) 270 atm (D) 2700 atm
(57) Heat Q is given to a diatomic (rigid rotator) gas at constant pressure then work done by gas
is ...... .

(A) 3 Q (B) 2 Q (C) 7 Q (D) 2 Q


2 3 2 7

(58) When a system is taken from initial state (i) to find state (f) through path iaf, Q = 500 cal and
W = 100 cal is needed. When system is taken through path ibf, Q = 2000 cal then
W = ...... on path ibf. a
(A) 1400 cal (B) 1900 cal
(C) 1600 cal (D) 1500 cal i f
b

160
(59) For an ideal gas, specific heat at constant pressure is
7
2
R then ratio of specific heats at constant

pressure to that at constant volume is ...... .

(A) (B) (C) (D)


5 7 9 7
7 5 7 9
(60) 5.6 L Helium gas at STP is adiabetically compressed to volume 0.7 L. If initial temperature is
T1 then work done during the process is ...... .

(A) RTl (B) RTl (C) RTl (D) RTl


3 9 8 9
2 2 9 8

(61) During adiabetic process, relation between pressure and volume is P3 µ


1
then ratio of
V4
specific heat is ...... .
(A) 1.80 (B) 1.33 (C) 1.67 (D) 1.42
(62) On expanding 10 mole ideal gas at 100 K constant temperature, its volume increases from
10 L to 20 L. Work done during this process is ....... .
(A) 5763 J (B) 5673 J (C) 57.63 J (D) 567.3 J
(63) Work done during adiabetic compression of 1 kilo mole gas is 146 kJ. During this process, its
temperature increases by 7 °C. This gas will be ....... . (R = 8.3 Jmol–1 k–1)
(A) Monoatomic (B) Diatomic (C) Triatomic (D) Polyatomic
(64) Coefficient of volume expansion of glycerine is 49×10 °C . On increasing its temperature by
–5 –1

20 °C, percentage decreases in its density is ...... .


(A) 10 % (B) 0.98 % (C) 1 % (D) 9.8 %
(65) If g is ratio of specific heats and R is gas constant then molar specific heat at constant pressure
CP = ...... .
JR JR
(A) J  1 (B) J  1 (C) J – 1 (D) J – 1
R R

(66) If g is ratio of specific heats and R is gas constant then molar specific heat at constant volume
Cv = ...... .
J 1 J –1
(A) (B) (C) J  1 (D) J – 1
R R
R R
(67) During an adiabetic process, pressure of a gas is directly proportional to cube of its temperature.
Then for this gas g = ...... .

(A) (B) (C) (D)


3 5 7 5
2 3 5 7

(68) 1 mole ideal gas at temperature T1 K does 6R J work adiabetically. If g = 3 then final
5

temperature of gas is ...... .


(A) (T1 + 4) K (B) (T1 – 4) K (C) (T1 + 8) K (D) (T1 – 8) K
(69) Latent heat of vaporisation for water is 2240 J. If energy required to vaporize 1 g water is
168 J then change in internal energy is ...... .
(A) 2408 J (B) 2240 J (C) 2072 J (D) 1904 J

161
(70) For cyclic process shown in figure, net heat absorbed
P (N m–2)
by system in every cycle is ...... . 4 × 105
(A) 10 p unit
(B) p unit
2 × 105
(C) 100 p unit
(D) p2 unit
V
2 × 102 4 × 102 (L)
(71) In the figure, ideal gas 1 and 2 move from state A to state
­ 1
P B by different path. If change in internal energy for path 1
and 2 are (DUint)1 and (DUint)2 then ....... .
A B
(A) (DUint)1 = (DUint)2
(B) (DUint)1 < (DUint)2
2
(C) (DUint)1 > (DUint)2
V® (D) (DUint)1 = 5 (DUint)2
(72) Which part of graph of P ® V shown in figure represents
­
1 2
Isothermal process, Isochoric process and Isobaric P
process ...... respectively.
(A) 12 ; 34 ; 23
3
(B) 12 ; 14 ; 34
(C) 23 ; 34 ; 12
4
(D) 34 ; 12 ; 23

­
(73) P (4P, V) (4P, 4V) For cyclic process in graph of P ® V shown in figure,
4P
work done = ...... .
3P
(A) 2 PV
2P
(B) 4 PV
P (P, 4V)
(P, V)
(C) 9 PV
(D) 6 PV
V 2V 3V 4V

C
(74) As shown in figure, 1 mole He gas experience cyclic ­
process ABCA. During the process, 1000 J heat is obtained P
from the gas then work done during stage BC is ...... .
(R = 8.3 1 mol–1 K–1) A B
(A) +3490 J (B) 1490 J TA = 300 k TB = 600 k

(C) –3490 J (D) – 1490 J V®

162
(75) Cyclic process of m mole Ar gas is shown in figure.
V(m3)
T2 C Efficiency of thermodynamic process is ...... .
BT1
2
(A) 100 %

(B) 25 %
1 A T2
(C) 75 %
(D) 50 %
5 10
P (N/m2) ®
(76) Which of the following is the graph of b ® P for an ideal gas at constant temperature where b =

compressibility of gas =
 d V/ dP
V
.
(A) ­ (B) ­
b b

P® P®

(C) ­ (D) ­
b b

P® P®

(77) Liquid O2 at 1 atm pressure is heated from 50 K to 300 K at constant pressure. Rate of heating
is constant. Which of the following shows graph of change in temperature with time ?
(A) B (B)
B
Timperature®

Timperature®

A
A

Time Time ®

C
Timperature®

(C) (D) D
B
B
Timperature®

C
A A

Time ® Time ®

163
(78) 1 mole ideal gas moves from state A to state B by two different ways. Firstly, volume is changed
from V to 3V by isothermal expansion and then volume is decreased from 3V to V at constant
pressure. Which of the following is the graph of P ® V showing these two processes ?
(A) (B)
­ B ­ A
P P

A B

V V® 3V V V® 3V
(C) (D)
­ A
­ A P
P

B
B

V V® 3V
V V® 3V
(79) In the figure, a system moves on path 1-2-1. In the P ® V
graph, different paths are shown such that each time thermal P 1 a
equilibrium is set up between system and environment. During b
which closed path is work done maximum positive ? c
d
(A) 1 – b – 2 – f – 1 (B) 1 – c – 2 – e – 1
e
(C) 1 – d – 2 – e – 1 (D) 1 – a – 2 – f – 1 f 2

(80) P For graph of P ® V of a cyclic process, shown in figure,
i=f
change in internal energy of gas DU = ...... and net heat
exchange DQ = ...... .
(A) positive, negative (B) positive, zero
(C) zero, negative (D) zero, positive
V
P (N m–2)×104
(81) During cyclic process shown in figure, net heat
A B
absorbed by system per cycle is ...... . 30
(A) 20 × 106 J
(B) 2 × 105 J
(C) 200 × 107 J 10 C
(D) 20 × 107 J

100 300 V (m3)

Ans. : 52 (A), 53 (D), 54 (B), 55 (C), 56 (A), 57 (C), 58 (C), 59 (B), 60 (D), 61 (B), 62 (A), 63 (B), 64
(B), 65 (C), 66 (D), 67 (A), 68 (B), 69 (C), 70 (B), 71 (A), 72 (C), 73 (C), 74 (C), 75 (D), 76
(A), 77 (D), 78 (D), 79 (B), 80 (C), 81 (A)

164
Efficiency of heat engine :

W Q1 - Q2 Q2
h= Q = Q1 = 1 – Q1
1

h < 1 (always)
where Q1 = Heat absorbed from heat source at high temperature
Q2 = Heat released in sink at low temperature
Coefficient of performance of refrigerator :

Q2 T2
a= = =
Q2
W Q1 - Q2 T1 - T2

a > 1 but never infinity


where Q1 = Heat released by working substance in surrounding at higher temperature (T1)
Q2 = Heat absorbed by working substance from arrangement at lower temperature (T2)
Efficiency of Carnot engine :

T2 Q2
h=1–T = 1– Q
1 1

where T1 = Temperature of heat source


T2 = Temperature of sink

(82) Efficiency of a heat engine is 30 %. During each cycle, difference of heat absorbed and heat
released is 60 J. Then heat absorbed from heat saurce during every cycle, is ....... and that
released in sink is ...... .
(A) 100 J, 63 J (B) 150 J, 65 J (C) 200 J, 63 J (D) 200 J, 140 J
(83) A heat engine absorbs 50 kJ heat from heat source. If its efficiency in 30 % then it releases ......
heat in sink.
(A) 35 kJ (B) 350 kJ (C) 35 J (D) 350 J
(84) If heat engine absorbs 2 kJ heat from heat source and releases 1.5 kJ heat in sink then
efficiency h = ...... .
(A) 5 % (B) 25 % (C) 50 % (D) 2.5 %
(85) A Carnot engine absorbs heat 3×106 cal from heat source at temperature 627 °C and releases
some heat in sink at temperature 27 °C then work done is ...... .
(A) 8.4 × 106 cal (B) 2 × 106 J (C) 8.4 × 106 J (D) 12 × 106 J
(86) Efficiency of a Carnot engine is 40 % and temperature of sink is 400 K. Keeping temperature of
heat source constant, if efficiency is to be made 80 %, temperature of sink should be made ...... .
(A) 300 K (B) 667 K (C) 532 K (D) 133 K
(87) A heat engine works between temperature 227° C and 127° C of Carnot cycle. If it absorbs 6
kJ heat from heat source then it converts ...... heat into work,
(A) 1.2 × 103 J (B) 1.2 × 103 cal (C) 1200 J (D) 1200 cal
165
(88) Efficiency of a heat engine with sink temperature 300 K is 40 %. How much the temperature of
heat source should be increased so as to increase the efficiency by 50 % by keeping sink
temperature constant.
(A) 2500 K (B) 250 K (C) 250 K (D) 200 K

(89) Efficiency of a heat engine is


1
6
. When temperature of sink is reduced by 62 °C, its efficiency

doubles. Temperature of heat source will be ...... .


(A) 37 °C (B) 99 °C (C) 62 °C (D) 52 °C
(90) Efficiency of a Carnot engine is 20 %. It work as heat system for a refrigerator. If 50 J is work
done on the system then how much heat will sink absorb ?
(A) 200 cal (B) 100 cal (C) 200 J (D) 100 J
(91) Coefficient of performance of a refrigerator is a = 5. If it absorbs 120 J heat per cycle from
cold reservoir then how much heat does it release in every cycle to hot reservoir at higher
temperature ?
(A) 96 cal (B) 144 cal (C) 96 J (D) 144 J
Ans. : 82 (D), 83 (A), 84 (B), 85 (C), 86 (D), 87 (A), 88 (A), 89 (B), 90 (C), 91 (D)
Assertion - Reason type Question :
Instruction : Read assertion and reason carefully, select proper option from given below.
(a) Both assertion and reason are true and reason explains the assertion.
(b) Both assertion and reason are true but reason does not explain the assertion.
(c) Assertion is true but reason is false.
(d) Assertion is false and reason is true.
(92) Assertion : Its difficult to find reversible process in practice.
Reason : Most of the processes lost on energy.
(A) a (B) b (C) c (D) d
(93) Assertion : When air comes out of balloon, it feels instantly cool.
Reason : Air experiences adiabetic expansion while coming out.
(A) a (B) b (C) c (D) d
(94) Assertion : Carnot cycle is useful in understanding efficiency of heat engine.
Reason : It shows probability of obtaining maximum possible efficiency at a given temperature.
(A) a (B) b (C) c (D) d
(95) Assertion : On cooling milk kept in a glass in a room, its disorderness (entropy) decreases.
Reason : on cooling a hot substance, it does not dissolved. second law of Themodyhamics.
(A) a (B) b (C) c (D) d
(96) Assertion : Entropy (Disorderness) of an isolated always increases.
Reason : Processes occuring in isolated system are adiabetic.
(A) a (B) b (C) c (D) d

166
(97) Assertion : A Temperature on surface of Sun is 6000 K. Now, by focusing sunrays with help
of huge lens, one can obtain 8000 K temperature.
Reason : This temperature can be obtained according to thermodynamics second law.
(A) a (B) b (C) c (D) d
(98) Assertion : Refrigerator absorb heat from low temperature and releases at high temperature.
Reason : Normally heat can not be flow from high temperature to low temperature.
(A) a (B) b (C) c (D) d
(99) Assertion : An efficiency of Carnot engine will increase when temperature of sink will decrease.

Reason : h = 1 –
T2
T1

(A) a (B) b (C) c (D) d


(100) Assertion : Internal energy of ideal gas depends only on temperature and not on volume.
Reason : Temperature is more important than volume.
(A) a (B) b (C) c (D) d
(101) Assertion : Internal energy and temperature of system will be decrease in adiabatic
compression process.
Reason : An adiabatic process is very slow process.
(A) a (B) b (C) c (D) d
(102) Assertion : When a bottle of cold drink like pepsi is opened, some fogg will produced around it.
Reason : As low temperature, gas get adiabatic expansion and vapour of water cools.
(A) a (B) b (C) c (D) d
Ans. : 92 (A), 93 (A), 94 (A), 95 (A), 96 (A), 97 (D), 98 (C), 99 (A), 100 (A), 101 (D), 102 (A)
Comprehension Type Questions :
P
Paragraph : (N m–2)
A B
A P ®T cyclic process done on 1 mole Ar gas 4 × 105

is shown in figure along path ABCD. D C


2 × 105

O 100 K 200 K 300 K T

(103) Work done to take Ar gas from A to B at constant pressure (4 × 105 N m–2) is ...... .
(A) 16628 J (B) 1662.8 J (C) 166.28 J (D) 16.628 J

(104) Work done to take Ar gas from B to C at constant temperature (300 K) is ...... .
(A) 17.29 J (B) 172.9 J (C) 172900 J (D) 1729 J

167
Paragraph : V (c c)
A process ABCA on 1 mole Ar is shown in figure . 400 c c C
(105) Work done during isochoric process AB is ...... .
(A) 0 J (B) 300 J
100 c c A
(C) 100 J (D) 200 J B

(106) Work done during isothermal process BC is ...... . 100 K 400 K



(A) 46.11 J (B) 461.1 J (C) 3586 J (D) 4611 J
(107) Work done during adiabatic process CA is ...... .
(A) 0 J (B) 1000 J (C) 3200 J (D) 2494 J
Paragraph :
Pressure of gas and volume change while heat of gas remain constant. This process is known as
Adiabatic process. For such process PV = constant. Process is very rapid and walls of a system are
g

thermal insulator, so no exchange of heat takes place between system and its environment. For this
changes, DQ = 0 and according to thermodynamics first law DQ = DU + DW = 0 \ DU = –DW
Answer the following questions according to above paragraph :
(108) Bicycle's tyre burst suddenly. Changes in air pressure and volume will be ....... .
(A) Isothemal (B) Adiabatic (C) Isobaric (D) Isochoric
(109) The temperature of gas, which is suddenly compressed in system, ...... .
(A) Increase (B) Decrease
(C) Constant (D) Depend on environment temperature
(110) When gas in system is suddenly compressed then internal energy of gas will be ....... .
(A) increase (B) decrease
(C) constant (D) no comment
(111) The specific heat of gas during adiabatic process ...... .
(A) 1 (B) –1 (C) 0 (D) infinite (¥)
Match the columns :
(112) A thermodynamic processes are shown in column-1 and in column-2 equation of work done are
given. Match it appopriately.
Column-1 Column-2
(a) Adiabatic process (P) W=0

P R(T1  T2 )
(b) Isothermal process (Q) W= J 1

§ V2 ·
(c) Isochoric process (R) W = 2.303 mRT log ¨ V ¸
© 1¹

(d) Isobaric process (S) W = P DV


(A) a ® P ; b ® R ; c ® Q ; d ® S (B) a ® Q ; b ® R ; c ® S ; d ® R
(C) a ® R ; b ® S ; c ® P ; d ® Q (D) a ® S ; b ® Q ; c ® R ; d ® P

168
(113) Column-1 Column-2
(a) Adiabatic process (P) DU = 0
(b) Isothermal process (Q) DQ ¹ 0; DU ¹ 0, DW ¹ 0
(c) Isochoric process (R) DW = 0
(d) Isobaric process (S) DQ = 0
(A) a ® R ; b ® P ; c ® Q ; d ® S (B) a ® Q ; b ® R ; c ® S ; d ® P
(C) a ® S ; b ® P ; c ® R ; d ® Q (D) a ® P ; b ® S ; c ® Q ; d ® R
(114) Different thermodynamic processes are shown in graph of (P) ® (V)
Column-1 Column-2 E C
G
(a) Graph AB (P) Isochoric process ­
P B
(b) Graph GH (Q) Adiabatic process A H
F
(c) Graph EF (R) Isobaric process D
(d) Graph CD (S) Isothermal process V®

(A) a ® Q ; b ® P ; c ® S ; d ® R (B) a ® P ; b ® Q ; c ® R ; d ® S
(C) a ® S ; b ® R ; c ® P ; d ® Q (D) a ® R ; b ® S ; c ® Q ; d ® P
(115) Match according to concept of heat transfer :
Column A Column B

(a) Heat required to convert a gas from liquid. (P) 2256 kJ

(b) Heat required to convert a liquid from solid. (Q) 333 kJ

(c) Heat required to convert 1g ice to water (R) Heat of fusion

(d) Heat required to convert 1g water to vapour. (S) Heat of vaporization

(A) a ®B b®C c ® D d®A


(B) a ®D b®C c ® B d®A
(C) a ®A b®B c ® C d®D
(D) a ®D b®A c ® B d®C

Ans. : 103 (A), 104 (D), 105 (A), 106 (D), 107 (D), 108 (B), 109 (A), 110 (A), 111 (C), 112 (B),
113 (C), 114 (D), 115 (B)

169
13 Kinetic Theory of Gases
Boyle's Law
"For a gas having sufficiently low density, its pressure is inversly proportional to its volume, at
constant temperature."

Pµ 1 Þ PV = constant ................... (1)


V
Þ P1V1 = P2V2

(A) But volume V = m


U
From equation (1),

§m·
\ P ¨ ¸ = constant
©U¹
If mass m is constant, then

= constant
P1 P2
P =
U
Þ
U1 U2

(B) Number of molecules in unit volume, n = N Þ V = N


V n
from equation (1),

P Nn = constant
If N is constant then, = constant Þ = constant
P P1 P2
n
= n2
n1
Charle's Law
"For a gas having sufficiently low density, at constant pressure, its volume is directly proportional to its
absolute temperature."

VµT = constant
V V1 V2
Þ
T
Þ
T1
= T2

(A) But density r = ÞV m = constant


m m
U
V
=

So, m = constant Þ r T = constant r1 T1 = r2 T2


UT
(B) At constant pressure for a given mass of gas Volume at 0 °C is V0 then Volume at t °C is

Vt = V0 1  t
273.15
.
Gaylussac's Law
"For a gas having sufficiently low density, at constant volume, its pressure is directly proportional to its
absolute temperature."

PµT Þ P = constant
P1 P2
T
Þ
T1
=
T2
(A) At constant volume, for a given mass of gas pressure at 0 °C is P0 then pressure at t °C is

Pt = P0 ª«1  t º»
¬ 273.15 ¼

170
(1) If speed of sound in air is vs and its rms speed in air is vrms then ......

§ J ·2 § J ·2 §J·
1 1 2
(A) vs = vrms (B) vs = vrms ¨3¸ (C) vrms = vs ¨ ¸ (D) vs= vrms ¨ ¸
© ¹ ©3¹ ©2¹
(2) On increasing temperature of a gas filled in a closed container by 1°C, its volume increases by
0.4 %, then intial temperature of gas would be ........
(A) 250 °C (B) 150 °C (C) 250 K (D) 150 K
(3) Volume of a gas at 2 atm pressure and 746.3 K temperature for question 2 is .........
(A) 100 m3 (B) 100 cc (C) 1 m3 (D) 1 cc
(4) ...... = constant in Boyle's law.

(A) (B) V T (C) PV (D)


P P
T V
(5) To double the volume of a given quantity of ideal gas at 27 °C, temperature should be ...... °C
at constant pressure.
(A) 600 (B) 270 (C) 370 (D) 54
(6) On decreasing volume of a gas at constant temperature by 5 %, its pressure ......
(A) decreases by 5.26 % (B) increases by 5.26 % (C) increases by 11 % (D) decreases by 11 %
(7) 2 mole ideal gas is mixed with 3 mole diatomic rigid rotator gas. Molar specific heat at constant
volume will be ......

(A) 1.2 R (B) 2.1 R (C) R (D) R


3 5
2 2
(8) A gas is filled in a closed container at temperature 250 K, on increasing its temperature by 1 K,
percentange change in its pressure is ......
(A) 0.8 % (B) 0.2 % (C) 0.4 % (D) 0.1 %
(9) Two gases having equal mass are in thermal equillibrium. If their pressure are Pa and Pb and
volumes are Va and Vb respectively, then ......

(A) PaVa = PbVb (B) V = (C) Pa ¹ Pb ; Va = Vb (D) Pa = Pb ; Va ¹ Vb


Pa Pb
a Vb
(10) At what temperature will volume of an ideal gas becomes 3 times than that at 0 °C at constant
pressure ?
(A) 819 °C (B) 646 °C (C) 546 °C (D) 182 °C
(11) 1 mol oxygen gas is filled in a container at pressure P and temperature T. In another similar
container, 1 mol Helium gas is filled at temperature 2T then pressure of helium gas will be .......
(A) 8P (B) P/8 (C) P (D) 2P
(12) Volume of a gas at temperature 27 °C is V. If its temperature is increased to 327 °C at constant
pressure, then its volume becomes ......
(A) V/2 (B) V (C) 2V (D) 3V
(13) At what temperature will pressure of 1 g N2 gas be equal to pressure of 1 g O2 gas at 15 °C ?
Molecular mass of O2 and N2 are 32 and 28 respectively.
(A) 13 °C (B) 15 °C (C) 56.4 °C (D) –21 °C

171
(14) Kinetic theory of gas provides support to ......
(A) Boyle's law (B) Charle's law
(C) Boyle's and Charle's law (D) None of the laws.
(15) At constant temperature, on increasing pressure of 1200 ml gas from 70 cm-Hg to 120 cm-Hg,
its volume becomes ......
(A) 400 ml (B) 500 ml (C) 600 ml (D) 700 ml
Ans. : 1 (B), 2 (C), 3 (B), 4 (C), 5 (C), 6 (B), 7 (B), 8 (C), 9 (A), 10 (C), 11 (D), 12 (C), 13 (D),
14 (C), 15 (D)

Avogadro's hypothesis
"For given constant temperature and pressure, number of molecules per unit volume is same for all gases."
Number of molecules in 1 mole gas is NA = 6.023 × 1023
Graham's law for gas expansion
"If two different gases are mixed at constant temperature and pressure, then rate of mixing is
inversely proportional to square root of density of gas."

rµ Þ r = density
1
U

rµ 1 Þ M = Molecular mass of gas


M

U2
= =
r1 M2
U1
\
r2 M1

ˆ If volume V of gas mixes in time t then

r= t
V

= ×
r1 V1 t2
r2 V2 t1
Ideal Gas Equation :
PV = mRT for m mole gas

PV = §¨ R ·
¸T for 1 mole gas
© NA ¹

= kBT kB = N = 1.38 × 10 –23 JK–1 = Boltzmann's constant


R
A
= NkBT for N molecules

Þ PV = MR T for 1 g gas

where, r = specific Gas constant


PV = rT

172
PV = mrT for m g gas.

where r = = Gas constant per unit mass.


R
M

unit of r = Jg–1 k–1

Van-der-waal's correction

(A) Correction in Volume : (V– b) Where b = 1 RTc


8 Pc

(B) Correction in pressure : §¨ P + a ·¸ Where a = 27 R Tc


2 2

© 2
V ¹ 64 Pc
Here Pc = Critical pressure, Tc = Critical temperature and Vc = Critical volume

for 1 mole gas §¨ P + a ·¸ (V – b) = RT


© 2
V ¹

§ 2 ·
for m mole gas ¨ P  P a ¸ (V – mb) = mRT
© 2
V ¹

(16) In equation PV = RT value of constant R at STP, is ......


(A) 2 cal K–1 (B) 10 cal K–1 (C) 0.2 cal K–1 (D) 200 cal K–1
(17) A gas is filled in container at temperature 27 °C. To take out (release) half the mass of gas from
container, up to how much temperature should the container will be heated ?
(A) 54 °C (B) 177 °C (C) 277 °C (D) 327 °C
(18) On increasing the temperature of a gas in closed container by 1°C, its pressure increases by
0.4 %. Then initial temerature of this gas would be ......
(A) 250 K (B) 250 °C (C) 2500 k (D) 2500 °C
(19) In a metallic cylindrical container, pressure of gas at 27 °C temperature is 2 atmosphere. On
making its temperature 54 °C, pressure becomes ...... atmosphere.
(A) 1 (B) 2 (C) 2.18 (D) 1/2
(20) At constant pressure, 1 litre ideal gas is heated from 27 °C to 97 °C then its final volume
becomes ...... litre.
(A) 1.2 (B) 1.9 (C) 19 (D) 2.4
(21) At constant temperature, how much percentage decrease in pressure of gas should be done so as
to increase its volume by 10 % keeping mass constant ?
(A) 8.1 % (B) 9.1 % (C) 10.1 % (D) 11.1 %
(22) A mixture of 8 g oxygen, 14 g Nitrogen and 22 g Carbon Dioxide at 27 °C temperature is filled
in a container of 4 litre. Then pressure of mixture is ...... N m–2.
(A) 5.79 × 105 (B) 6.79 × 105 (C) 7.79 × 103 (D) 7.79 × 105
(23) A mixture of 8 g O2 ; 14 g N2 and 22 g CO2 gases at 27 °C is filled in a container of 10 litre
Then pressure of mixture is ...... (Take R = 0.082 units)
(A) 1.4 atm (B) 2.5 atm (C) 3.075 atm (D) 8.7 atm

173
(24) Two containers of equal volume are filled with same gas at pressure P1 and P2 and temperature
T1 and T2. Now when both containers are joined then their common pressure and temperature
becomes P and T respectively. then ratio P/T ......

(A) (B) (C) (D)


P1T2 + P2 T1 P1T2 + P2 T1 P1T2 + P2 T1 P1T2 + P2 T1
T1 T2 T1 + T2 2 T1 T2 T1 – T2

(25) Main difference between ideal gas and real gas is related to .....
(A) change of state (B) temperature (C) pressure (D) mole
(26) Two different gases have pressure P, volume V and temperature T each. Now, keeping volume and
temperature same, when both gases are mixed, pressure of mixture will be .....
(A) P/2 (B) P (C) 2P (D) 4P
Ans. : 16 (A), 17 (D), 18 (A), 19 (C), 20 (A), 21 (D), 22 (D), 23 (C), 24 (C), 25 (A), 26 (C)

ˆ Pressure of Gas :
PV = NkBT N = Number of molecules

P = N kBT
V

P = nkBT Where  n = = Number of molecule in unit volume.


N
V
ˆ PV = mRT

Mass of gas
PV = M RT m= =
M M
0 M0 Molecular mass of gas

P= T
M R
V M0

U RT
P= M where  r = = Density of gas
M
0 V

ˆ P = 1 r <v2> where <v2> = vx2 + vy2 + vz2


3
<v2> = <vx2> + <vx2> + <vx2>

vrms = <v2> = 3<vx2>


3P
U

Where  v 2 ! = vrms

= 3k B T where m = mass of one molecule.


m

= where M0 = molecular mass


3RT
M0

Internal energy Eint =


f
ˆ mRT
2
Here f = Degrees of freedom

174
Monoatomic gas (He, Ne, Ar,...) f = 3
Diatomic gas (H2, O2, N2,...) f = 5 (Rigid rotator)
CO2 f=7

(27) At a given temperature, pressure of an ideal gas is proportional to ...... (Here r = density of gas)
1
(A) U (B) U (C) r (D) r2
1 2

(28) Average kinetic energy of an ideal gas per mole is ......

(A) 1 kBT (B) 2 RT (C) 3 kBT (D) 2 RT


1 3
2 2
(29) At what temperature rms speed of O2 molecule will be equal to that of H2 molecule at
1000 K ?
(A) 160 K (B) 16 K (C) 1600 K (D) 16000 K
(30) For H2 molecule at 300 K, vrms = 1000 m s then vrms for O2 molecule at 1200 K = .......
–1

(A) 500 m s–1 (B) 50 m s–1 (C) 5000 m s–1 (D) 5 m s–1
(31) Which of the following equation is incorrect ?

(A) vrms = 3P (B) vrms = (C) vrms = 3P


(D) vrms =
U
3RT 3k B T
U M0 m
(32) Cl2 gas is filled in a closed container. If pressure of the gas is doubled and temperature is made
four times then how much times will its density become ?

(A) 2 (B) 2 (C) 4 (D)


1 1
4
(33) vrms speed of dust particles having mass 1.38 × 10 kg at NTP is = .......
–10

(A) 9.49 × 10–9 cms–1 (B) 9.49 × 10–9 ms–1 (C) 9.49 × 10–6 cms–1 (D) 9.49 × 10–6 ms–1
(34) Mean kinetic energy of a gas molecule at 127 °C temperature is 6.21 ×10–21 J. Then its kinetic
energy at temperature 327 °C is ......
(A) 9.315 × 10–21 J (B) 9.315 × 1021 J (C) 9.315 × 10–23 J (D) 9.315×10+23 J
(35) For Oxygen gas filled in a container, mass is 5 g, pressure is P, absolute temperature is T and
volume V. Equation of state for this ideal gas would be ......

(A) PV =
5
32
RT (B) PV = 5RT (C) PV = 2 RT
5
(D) PV = 16 RT 5
(36) Kinetic energy of CO2 molecule at 500 K temperature is E, then kinetic energy of CO molecule
at same temperature is .......

(A) E (B) 32 E (C) 16 E (D) E


32
(37) At a given temperature, ratio of rms speeds of molecules of H2 and He is ......
(A) 1 : 2 (B) 2 :1 (C) 1 : 2 (D) 2 : 1
(38) For 1 mole He gas, Cv = ......

(A) R (B) R (C) R (D) R


3 7 1 5
2 2 2 2

175
(39) If mole number of an ideal gas is m and its degrees of freedom is f, then its internal energy is ......

(A) f m kBT (B) mkBT (C) (D) fmRT


f f 3
mRT
2 2 2

(40) For 1 mole Ar gas at constant pressure, molar specific heat is ......

(A) Cv = R (B) CP = R (C) Cv = R (D) CP = R


5 5 3 3
2 2 2 2

(41) For 1 mole gas,


R
Cv
= 0.672 then molecules of this gas will be ......

(A) diatomic (B) triatomic (C) monoatomic (D) polyatomic


(42) If absolute temperature of a gas is made nine times, rms speed of its molecules become .......
time.

(A) (B) (C) 3 (D) 3


1 1
3
3

(43) Mean free path in gases is of the order of ......


(A) 1A° (B) 10 A° (C) 10+3 A° (D) 10–3 A°
(44) Length of straight path of molecules between two consecutive collisions is called ......
(A) free path (B) janpath (C) mean free path (D) Degrees of freedom

(45) If ratio of vapour density of two gases is 64 then at constant pressure, ratio of their rms speed is ......
1

(A) 1 : 8 (B) 8 : 1 (C) 1 : 8 (D) 8 :1

(46) At normal temperature and pressure, number of molecule of O2 gas per cubic meter is
2.5 × 1025. Then mean free path of O2 molecule is ...... (d = 3.4 A°)
(A) 8.7 × 10–8 cm (B) 8.7 × 10+8 cm (C) 7.8 × 10–8 m (D) 7.8 × 10+8 m
(47) Diameter of molecule of Ar gas is 3.56 × 10–10 m. Mean free path of this molecule at 27 °C
temperature and 1 atm pressure is ......
(A) 7.3 × 10–6 m (B) 7.3 × 10+8 cm (C) 7.3 A° (D) 7.3 × 10–8 m
(48) If mean free path of molecules of H2 gas at temperature T and pressure P is d, then mean free

path of molecules of H2 gas at temperature 4T and pressure 4 will be ......


P

(A) 16 d (B) 16 d (C) 1.6 d (D) 1.6 d


1 1

(49) Degreees of freedom of molecules of CO2 gas is ......


(A) 3 (B) 5 (C) 7 (D) 9

(50) If for a given gas g = 5 , then this gas would be ......


7

(A) Ne (B) Ar (C) He (D) H2

176
Assertion - Reason type Question :
Instruction : Read assertion and reason carefully, select proper option from given below.
(a) Both assertion and reason are true and reason explains the assertion.
(b) Both assertion and reason are true but reason does not explain the assertion.
(c) Assertion is true but reason is false.
(d) Assertion is false and reason is true.
(51) Assertion : When temperature of gas is increased from 27 °C to 927 °C, rms speed of its
molecules becomes four times.

Reason Ñ vrms µ T
(A) a (B) b (C) c (D) d
(52) Assertion : Molecules of monoatomic gas (He) performs both liner motion and vibrational
motion f = 3
Reason Ñ For He gas f = 5,
For linear motion f = 3
For vibrational motion f = 2
(A) a (B) b (C) c (D) d
(53) Assertion : For ideal gas (Ar), PV = constant according to Boyle's law.

Reason Ñ vrms =
3 RT
M

(A) a (B) b (C) c (D) d


(54) Assertion : At constant temperature, rms speed of molecules does not change with change in
volume of gas.
Reason Ñ rms speed of molecules does not depends on volume of gas.
(A) a (B) b (C) c (D) d
(55) Assertion : Energy E of gas at 0 K temperature is zero.
Reason Ñ At 0 K temperature, energy of molecules of gas is zero.
(A) a (B) b (C) c (D) d
(56) Assertion : Mean free path l of gas molecules is inversely proportional to density of gas.

Reason Ñ Mean free path is inversely proportional to pressure of gas.


(A) a (B) b (C) c (D) d
(57) Assertion : Three molecules have speed 1 v, 2 v and 3 v Their vrms speed is 2 v.

Reason Ñ vrms = v 2
(A) a (B) b (C) c (D) d

177
Match the columns :
(58) In column 1 and column 2, gas laws and related quantites are given. Match them appropriatly.

Column-1 (Gas laws) Column-2 (Quantities)


(i) Boyle's law P Pressure = Constant
(ii) Charle's law Q Volume = Constant
(iii) Gaylussac's law R Temperature = Constant
(iv) Equation of state S Quantity of gas = Constant.

(A) (i) ® S (ii) ® Q (iii) ® P (iv) ® R


(B) (i) ® Q (ii) ® R (iii) ® R (iv) ® P
(C) (i) ® R (ii) ® P (iii) ® Q (iv) ® S
(D) (i) ® P (ii) ® S (iii) ® S (iv) ® Q
(59) Match the following appropriately
Column-1 Column-2

(i) vrms of gas P 2 kBT


1

(ii) Energy related to each degrees of freedom Q 5

3P
(iii) Kinetic energy per mole R U

(iv) Degrees of freedom of O2 molecule S 2 RT


3

(A) (i) ® R (ii) ® P (iii) ® S (iv) ® Q


(B) (i) ® P (ii) ® Q (iii) ® R (iv) ® S
(C) (i) ® Q (ii) ® R (iii) ® P (iv) ® S
(D) (i) ® S (ii) ® P (iii) ® Q (iv) ® R

Ans. : 27 (C), 28 (D), 29 (D), 30 (A), 31 (A), 32 (B), 33 (D), 34 (A), 35 (A), 36 (D), 37 (B), 38 (A),
39 (C), 40 (B), 41 (C), 42 (C), 43 (C), 44 (A), 45 (B), 46 (C), 47 (D), 48 (A), 49 (C), 50 (D),
51 (D), 52 (D), 53 (B), 54 (C), 55 (D), 56 (A), 57 (D), 58 (C), 59 (A)

178
14 & 15 Oscillations and Waves
Periodic Motion and Oscillatory Motion
ˆ A repeated motion along a fixed path, about a fixed point at a regular interval of time is called
periodic motion.
e.g. The motion of hands of a clock, the motion of the Earth around the Sun, the motion of the
Earth about its own axis etc.
ˆ If a body moves to and fro, back and forth or up and down about a fixed point, at a regular
interval of time is called Oscillatory motion. e.g. the motion of the pendulum of the clock, The
motion of a loaded spring etc.
ˆ All Oscillatory motions are Periodic but all Periodic motions are not Oscillatory.
Simple Harmonic Motion (SHM)
The periodic motion of a body on a fixed linear path, under the influence of the force acting towards
the fixed point and directly proportional to the distance from the fixed point is called simple harmonic
motion. The body performing SHM is called simple harmonic oscillator (SHO).
The displacement of SHO at time t
y(t) = A sin (wt + f)
A = Amplitude of oscillations (ymax= A)
Also,
y(t) = A sin (wt + f)
y(t) = B cos (wt + f)
y(t) = a sin (wt) + b cos (wt)

Where, A = a 2  b2

a
initial phase f = tan–1 b

q = wt + f is called phase of the oscillator at time t.


is called initial phase. S
SHO starts its motion Initial Phase 2

(f) (radian)
(1) From fixed point (y = 0) 0
towards positive end
0
S
p
(2) From positive end (y = +A) 2
(3) From fixed point (y = 0) p
towards negative end 3S

3S
2
(4) From negative end (y = –A)
2

179
SHO Starts its motion Initial Phase
f (radian)
ˆ From the mid point of fixed point and positive end

positive end (y = A ) and move toward 5S S


2 6 A 6
(1) positive end S rad
x-axis
2
6 A
5S

(2) negative end 7S 2 11S
6 6 6
ˆ From the mid point of fixed point and negative end
negative end (y = – A ) moves towards y-axis
2

(1) negative end 7S


6
11S
(2) fixed Point 6
ˆ SHO starts its motion from :

(1) y = + A and moves towards


2

(a) Positive end thean f = S rad


4

(b) Negative end then f = 3S rad


4

(2) y = – A and moves towards


2

7S
(a) Positive end then f = rad
4

5S
(b) Negative end then f = 4 rad

(3) From y = +
3 A and move towards
2

(a) Positive end then f = S rad


3

(b) Negative end then f = 2 S rad


3

(4) from y = –
3 A and moves towards
2

5S
(a) Positive end then f = 3 rad

4S
(b) Negative end then f = 3 rad

180
ˆ Phase at the end of time t when periodic time is T; q = 2 S t + f
T

When frequency is f; q = 2p f t + f

ˆ When SHO completes n oscillations q = 2p (n) + f

Velocity (v) and Acceleration (a) of SHO

ˆ Velocity of SHO at time t, v(t) = Aw cos (wt + f)

ˆ When displacement is y, v = ± w A 2  y2

ˆ Acceleration of SHO at time t, a(t) = –Aw2 sin (wt + f)

ˆ When displacement is y, a = –w2y

ˆ The graph of y(t) ® t, v(t) ® t and a(t) ® t [when f = o]

+A

O t

–A

Aw

O t
–Aw

w2A
O t

–w2A

t=0 T T 3T T
4 2 4

ˆ At fixed point (y = 0) velocity of SHO, vmax = ± wA

ˆ At end points (y = ±A) acceleration of SHO is maximum, amax = ± w2A

= Þ T = 2p
amax vmax
\ w
vmax amax

Also, v max = A
2

amax

181
ˆ When displacement of SHO is y1, its velocity is y1 and when displacement is y2, velocity is v2 than

ª v12 y2 2 – v2 2 y12 º2 ªy 2 –y2 º2


1 1

Amplitude A = « » Periodic time T = 2p « 22 12 »


«¬ v12  v2 2 »¼ ;
«¬ v1  v2 »¼

y = + A Positive end ˆ v = 0, amax = –w2A, f= S


2

2S vmax
f= y= v= , a= f= 3
3A 3 amax S
3
ˆ
2 2 2

3S vmax amax S
f= 4 y= v= , a= f= 4
A
2
ˆ 2 2

5S S
f= y= v= v , a = max f= 6
A a
6 2
ˆ 3
2 max 2

moving from end point towards the mean position moving from the mean position towards
the positive end
v increases v decrease
a decreases a increases
y = 0 ...... mean ˆ vmax = Aw, a = 0, f = 0....

3S
y = –A ...... negative end ˆ v = 0, amax= +w2A, f=
2

(1) SHO starting from the mid point of its mean position and negative end, moves towards its negative
end, completes one oscillation in 0.5 s. Its initial phase f = ...... rad and its phase at the end of
10 s, q = ...... rad.

(A) f = 3S , q = 83S (B) f = 7 S , q = 247 S


2 2 6 6

(C) f = 5S , q = 245S (D) f = 11S , q = 251S


6 6 6 6
(2) SHM is given by y = 2 sin 10p t + 7 cos 10p t. Where y is in cm and t is in sec. The
amplitude, periodic time and initial phase of the oscillations are ......
(A) A = 5 cm, T = 0.2 s and f = 48°52' (B) A = 3 cm, T = 0.5 s and f = 48°52'
(C) A = 3 cm, T = 0.2 s and f = 48°52' (D) A = 3 cm, T = 0.5 s and f = 52°48'
(3) Periodic time of SHO is T, it starts its oscillation from the mean position. In what time it completes
7 th of its oscillation ?
8
(A) 7 T (B) 10 T (C) 12 T (D) 13 T
9 11 12
8
(4) SHO starts its oscillation from y = and move towards the mean position. Its phase when it
A
2
completes 10 2 oscillation, q = ...... rad.
1

S 43S 45S S
(A) 41 (B) (C) (D) 87
4 4 4 4

182
(5) When the displacement of SHO is 1 cm, its velocity is 2 cms–1 and when displacement is 2 cm,
its velocity is 1 cms–1. Its amplitude A = ...... cm and periodic time T = ...... s.

(A) 5 , 6.28 (B) A = 0, 0 (C) 5 , 3.14 (D) A = 10, 6.28

(6) The displacement of SHO is given by y = 10 sin 2p t 


1

; where y is in cm and t is in s.
12
Calculate its initial displacement, initial velocity and initial acceleration.
(A) y0 = 10 cm, v0 = 10p cms–1, a = –20p cms–2

(B) y0 = 5 cm, v0 = 17.32 p cms–1, a = –10p2 cms–2

(C) y0 = 10 cm, v0 = 10 p cms–1, a = –10p2 cms–2


(D) y0 = 5 cm, v0 = 17.32 p cms–1, a = –20p2 cms–2

(7) Amplitude of SHO is 0.05 m. When its displacement is 0.03 m, its acceleration is –3.0 ms–2.
Calculate its velocity at this time. Also calculate maximum velocity and maximum acceleration of
the oscillations.
(A) v = 0.4 ms–1, vmax = 0.4 ms–1, amax = 5 ms–2

(B) v = 0.5 ms–1, vmax = 0.5 ms–1, amax = 4 ms–2

(C) v = 0.4 ms–1, vmax = 0.5 ms–1, amax = 5 ms–2


(D) v = 0.5 ms–1, vmax = 0.4 ms–1, amax = 4 ms–2

(8) Amplitude and periodic time of SHO are 20 cm and 3 s respectively. It starts its oscillation from
the positive end. Calculate the time taken by it to travel first 10 cm distance towards the mean
position.
(A) 0.25 s (B) 0.50 s (C) 0.75 s (D) 1 s

(9) Amplitude and periodic time of SHO are A and T respectively. Calculate the minimum time to

travel distance from its mean position.


3A
2

(A) (B) (C) (D)


T 3T T T
2 2 6 8

(10) A particle is moving in the X–Y plane. Where x = A cos (w t + f) and y = A sin (w t + f). The
path of the motion of the particle is ...... .
(A) linear (B) circular (C) parabola (D) irregular

(11) Two particles are performing SHM along y-axis, with the equal amplitude A and equal angular
frequency w. The distance between their mean positions is y0, (Where y0 > A). If the maximum
distance between the two particles during their motion is (y0 +A), calculate the phase difference
between them, in radian.

S S S
(A) (B) (C) (D) p
3 4 6

183
(12) The periodic time of SHM of a particle 16 s. At time t = 2 s, it passes from its mean position,
and at time t = 4 s, its velocity is 2 ms . Then amplitude of the oscillations A = ...... m.
–1

(A) (B) (C) (D)


8 16 32
S S S
32 2
S

(13) Amplitude of SHO is 1 m. When it is at a distance of 0.5 m from the mean position, it receives
blow in the direction of its motion. soits, instantaneouly, velocity becomes 3 times to its initial.
Find the new amplitude of its oscillations.

(A) 5m (B) 7 m (C) 11 m (D) 13 m


(14) Velocity of SHO at its mean position is 2 ms–1 and its acceleration at its negative end is 1ms–2.
Then A = ...... m, T = ...... s.
(A) A = 4 m and T = p s (B) A = p m and T = 4 s
(C) A = p m and T = 4p s (D) A = 4 m and T = 4p s
(15) A particle perfoms SHM along the path of length 20 cm. When it is 6 cm away from its mean
position, its velocity is 16 cms–1. Calculate its acceleration when it is 3 cm away from its
positive end.
(A) 7 cms–2 (B) 14 cms–2 (C) 21 cms–2 (D) 28 cms–2
(16) An amplitude and periodic time of SHO are 10 cm and 2p s respectively. Calculate its velocity in
cms–1, when its acceleration is 8 cms–2.
(A) 2 (B) 4 (C) 6 (D) 8

S
(17) The displacement of SHO performing SHM along X-axis is x(t) = 20 sin (15t + 0.5) cm. And
3
the displacement of SHO performing SHM along Y-axis is y(t) = 8 [sin 10pt + 0.75 cos 10pt]
cm. Calculate the ratio of their amplitudes and periodic times.
Ax Tx Ax Tx
(A) A = 2 ; T = 2 (B) A = 1 ; T = 2
y y y y

Ax Tx Ax Tx
(C) A = 2 ; T = 1 (D) A = 1 ; T = 1
y y y y

(18) The displacement of SHO is y(t) = 200 sin 35S t  D cm. It starts its oscillations from a point
100 cm away from its mean position and moves towards its positive end. Calculate its phase
(in rad) at the end of 10 s.
S 17 S 37 S
(A) 0 (B) (C) (D)
6 6 6

2S
(19) The periodic time of SHO is
5
s. Its velocity at the mean Position is 10 5 cms–1. Calculate its
displacement when its velocity is 10 cms–1.

(A) 4 cm (B) 4 5 cm (C) 5 cm (D) 2 5 cm

184
(20) A simple pendulum performs SHM about x = 0, with an amplitude A and periodic time T. It’s
velocity at mean point is 0.02 ms–1. Now its amplitude is made doubled by keeping its length
constant. Calculate the velocity of the pendulum at its mean position.
(A) 0.01 ms–1 (B) 0.02 ms–1 (C) 0.04 ms–1 (D) 0

(21) The graph of y ® t for an SHO is shown in figure. Its acceleration at the end of time t = s
1
y 2
is ...... cms–2.
4 cm
S S2
(A) – (B) –
t
2
y=0
2
2 4 6
(C) – 2 p (D) – 2 p 2

–y
(22) Periodic times of two SHO are T and
3T
4
. They begin their motion simultaneously from their
mean positions. What is the difference between their phases when 1 oscillation of the oscillator
having periodic time T is completed ?
(A) 62° (B) 72° (C) 110° (D) 120°

(23) SHO perfoms SHM on the path of length 24 cm with the frequency of s , Calculate its
3 –1
2S
displacement when the magnitudes of its velocity and acceleration become equal.
(A) 3 cm (B) 6 cm (C) 7 cm (D) 9 cm

(24) Angular frequency of SHO is 2 3 rad s–1. If at time


12
, its displacement is 2 cm, then its
T

velocity v = ...... .
(A) 6 cms–1 (B) 12 cms–1 (C) 18 cms–1 (D) 24 cms–1
(25) What would be the amplitude of SHO whose velocity is a and acceleration is b.
1 1 1 1
ª D 2 y 2  E2 y 2 º 2 ª D 2 y  E2 y 2 º 2 ª D 2 y  Ey 2 º 2 ª D 2 y  E2 y º 2
(A) « » (B) « » (C) « » (D) « »
«¬ E »¼ «¬ E »¼ «¬ E »¼ «¬ E »¼

Ans. : 1 (B), 2 (C), 3 (C), 4 (D), 5 (A), 6 (D), 7 (C), 8 (B), 9 (C), 10 (B), 11 (A), 12 (B), 13 (B),
14 (D), 15 (D), 16 (C), 17 (A), 18 (D), 19 (A), 20 (C), 21 (B), 22 (D), 23 (B), 24 (B), 25 (C)

Force Acting on SHO (Variable force depending on the displacement)


ˆ For SHM along y-axis; Fµy
F = –ky
ˆ For SHM along x-axis, Fµx
F = –kx
Where, k = force constant
k = mw2

= ; T = 2p
k m
w
m k

185
Oscillations of the loaded Spring :
(1) Series Connection (2) Parallel Connection (3) Connection of two
Loaded spring

k1
k1
k1
k2
m
k2

m k2
m

An equivalent force constant In both at the case equivalent force constent


k1 k 2
k = k +k k = k1 + k2
1 2

or k = k + k Periodic time,
1 1 1
1 2

Periodic time T = 2p T = 2p
m (k1 + k 2 ) m
k1 k 2 k1 + k 2

(26) N springs are having equal force constant k. When they are connected in series an equivalent
force constant is ks and when they are connected in Parallel the equivalent force constant
becomes kp. Than,

(A) ks = N and kp = kN (B) ks = and kp = N2k


k k
N2

(C) ks = and kp = kN (D) ks = N and kp = N2k


N 2
k k

(27) 5 springs each of equal mass m and equal force constant (k) are
connected as shown in the figure. Calculate the periodic time of SHM
of the system. k k

(A) T =
m
2p k
k
(B) T = 2 2 p k
m

k k
(C) T = 2p 2 k
m

(D) T = 2p 2 k
3m m

186
(28) As shown in the figure a block of mass m is attached with the springs having force constant
k1 and k2. The periodic time of SHM is T1. When the springs having force constant 4k1 and 4k2
are used, the periodic time is T2. Then,
(A) T2 = 2T (B) T2 = 4T1 k1 k2
m
(C) T2 = 1 (D) T2 = 1
T T
2 4
[Note : If we take the series connection of springs instead of parallel connection of springs then
answer will remain same in this question.]
(29) The length of the spring having force constant k is l. The spring is divided in two parts of length

and . A block of mass m is attaced as shown in the figure. calculate the


3l l
4 4
periodic time of SHM of the System.
3l l
4 4
m

S S
(A) T = 2p (B) T = (C) T = (D) T = 2p 3 k
3m 3m m m
k 2 k 2 k

(30) Four identical springs each of force constant k are


connected as shown in the figure. calculate the frequency
of the simple harmonic oscillations. k k

(A) (B)
1 k 1 4k
2S m 2S m
k k

(C) (D)
1 2k 2 k
2S m S m
m

(31) When a body of mass 1 kg is suspended at the end of the spring, its length is increased by
9.8 cm. Now if the body is given SHM, what would be the periodic time of its oscillation ?
(A) 0.2 p s (B) 2 p s (C) 20 p s (D) 200 p s
(32) A body of mass 100 g is suspended at the end of an elastic spring. Amplitude of its SHM is
A1. Now, when the body is at its mean position, another body of mass 21 g is kept on it. It both of
A1
the objects are jointly perform SHM, the amplitude becomes A2. Than A = ......
2

(A) (B) (C) (D)


10 20 11 15
11 15 10 20
(33) When mass m is suspended by a spring of force constant k and given SHM, its period is T. Now the
spring is cut in two equal parts and arranged in parallel and the same mass m is oscillated by them
then the new periodic time will be ...... .

(A) T (B) 2 (C) 2T (D) 2T


T

187
(34) Four identical springs each of force constant 1000 Nm–1 are
M
connected as shown in the figure. If a body of mass 10 kg
is kept at the top and the system is given SHM, the periodic
time will be ...... .

(A) 6.28 s (B) 3.14 s

(C) 0.628 s (D) 0.314 s

(35) A block P of mass m is kept on a frictionless surface.


Block Q of the same mass m is kept on block P. A k Q
spring of force constant k is connected as shown in ms

wall
the figure. The co-efficient of friction between P and P
Q is ms. Both the blocks are oscillating togather with
equal amplitude A. The maximum static frictional frictionless surface
force between the blocks is ...... .

(A) kA (B) (C) ms kA (D) ms mg


KA
2

(36) A U-tube is partially filled with a liquid of density r. The length of the liquid column in each arm
is the same. Now the free-surface of the liquid in one arm is given a displacement 3.92 mm and
allowed to oscillate. calculate the frequency of its SHM.

(A) (B) (C) (D)


5 10 25 50
S S S S

(37) When the compression of an elastic spring is 1 cm, the restoring force produced in it is 2 N.
When an object of mass 0.5 kg is kept on the spring it is compressed by y and object performs
SHM with the periodic time T, calculate y and T.

S S
(A) y = 0.025m, T = s (B) y = 0.25 m, T = s
10 100

S S
(C) y = 0.025 m, T = s (D) y = 0.25 m, T = s
100 10

(38) As shown in the figure, an object of mass M performs SHM with an amplitude A. The amplitude
of point P is.

k1 A k2 A k1 k2
(A) k + k (B) k + k
p M
1 2 1 2

k1 + k 2 k1 + k 2
(C) k A (D) k2 A
2

Ans. : 26 (A), 27 (B), 28 (C), 29 (B), 30 (A), 31 (A), 32 (C), 33 (B), 34 (D), 35 (B), 36 (C), 37 (A), 38 (B)

188
Energy of SHO

Potential energy of SHO U = ky k = force constant = mw2, y = displacement


1 2
ˆ 2

K. E. of SHO, K = 2 mw2 (A2 – y2)


1
ˆ

The total energy (Mechanical energy) of SHO, E=K+U= m w2 A2 = k A2


1 1
ˆ 2 2

ˆ P. E. U µ y2
K. E. K µ (A2 – y2)
M. E. E µ A2 (independent of y)

ˆ If displacement y increases, P. E. (U) increases, K.E. (K) decreases but, E = constant.


U decreases y = +A K = 0, U = max. = E U increases

K increases y= K=U= K decreases


A E
2 2
E constant E constant

y=0 U = 0, K = max. = E

U increases y= K=U= U decreases


A E
2 2
K decreases y = –A K increases
E constant K = 0, U = max = E E constant
ˆ If the change in the P. E. = DU, The change in the K. E. = DK then DK = –DU
The change in the P. E. and the change in the K. E. is always equal and opposite, so that the
total change is always zero.
[If U increases then K decrease and if U decreases then K increases.]
So net change is always zero.
DK + DU = 0 [law of conservation of M. E.]
So net mechanical energy remain constant.
ˆ The graph of P. E. (U) ® displacement (y) :

Potential energy

U=E E U=E
U= ky2
1
2

y = –A y=0 y = +A
U=0
189
ˆ The graph of K.E. (K) ® displacement (y) :

Kinetic enery
K=E
K= E– ky2
1
2

y = –A y=0 y=A
K=0 K=0

ˆ The graph of M.E. (E) ® displacement (y) :

Mechanical energy
E E E
E does not depend on displacement
E = constant

y = –A y=0 y=A

ˆ Energy of SHO as function of time

Energy ­ (f = 0)
E
E
2

O T T 3T T 5T 3T 7T T t®
8 4 8 2 8 4 8

ˆ The time (T) during which SHO completes 1 oscillation, the values of K. E. and P. E. becomes
twice maximum and twice minimum. Thus, if frequency of SHO is f, the frequency for K. E.
or P. E. becomes 2f.

(39) What would be the values of displacement, velocity and acceleration of SHO when its K. E.
becomes equal to its P. E.

ZA Z2 A ZA
(A) y = ± ;v=± 2 ;a=± (B) y = ± ;v=± ; a = ±Z A
A A 2

2 2 2 2 2

(D) y = ± 2 ; v = ± 2 ; a = ± Z A
ZA Z A 2
ZA
(C) y = ± ;v=± ;a=±
A A 2

2 2 2 2

(40) When the K. E. of SHO is 43 J, its P. E. is 58 J. Now its K. E. increases and becomes 61 J.
Calculate its displacement. [force constant k = 20 Nm–1]

(A) 2m (B) 2m (C) 12 m (D) m


1
2

190
(41) The mass, periodic time and amplitude of SHO are 20 g, 2p s and 10 cm respectively. Calculate
its K. E., P. E. and M. E. for the displacement 6 cm.
(A) K = 6.4 × 10–5 J, U = 3.6 × 10–5 J, E = 10–6 J
(B) K = 4.6 × 10–5 J, U = 5.4 × 10–5 J, E = 10–4 J
(C) K = 3.6 × 10–5 J, U = 6.4 × 10–5 J, E = 10–6 J
(D) K = 6.4 × 10–5 J, U = 3.6 × 10–5 J, E = 10–4 J
(42) The periodic time of SHO of mass 20 kg is 6 s. It starts its oscillations from its mean position. Its
velocity at the end of 1 s is 2 ms–1. Calculate its K. E. and P. E. at this time.
(A) K = 40 J ; U = 120 J (B) K = 40 J ; U = 160 J
(C) K = 80 J ; U = 160 J (D) K = 80 J ; U = 120 J
(43) The force constant of a spring is 400 Nm–1. When a massive body is suspended, the restoring
force produced in the spring is 40 N. Calculate the total mechanical energy of the spring.
(A) 0.2 J (B) 2 J (C) 20 J (D) 200 J
(44) When 100 J energy is given to the SHO of mass 2 kg, it perfoms SHM with the amplitude 1 cm.
Calculate the force constant and angular frequency.
(A) k = 2 × 106 Nm–1, w = 106 rads–2 (B) k = 2 × 106 Nm–1, w = 103 rads–2
(C) k = 2 × 10–6 Nm–1, w = 106 rads–2 (D) k = 2 × 10–6 Nm–1, w = 103 rads–2
(45) Mass of SHO is 0.1 kg. The total length of the path of SHM is 20 cm. The K. E. of the SHO at
its mean position is 8×10–3 J. Calculate angular frequency and force constant.
(A) w = 4 rads–1 , k = 1.6 Nm–1 (B) w = 4 rads–1 , k = 0.4 Nm–1
(C) w = 2 rads–1 , k = 0.4 Nm–1 (D) w = 2 rads–1 , k = 1.6 Nm–1
(46) The mechanical energy of SHO is E. Calculate its kinetic energy at the mid point of its mean
position and end point.

(A) E (B) 0 (C) 4 (D) 4


E 3E

(47) What would be the displacement of SHO when its potential energy becomes 14 times its maximum value ?

(A) 0 (B) (C) (D)


A A 3A
2 2 2

(48) Calculate the ratio of potential energy and total energy of SHO at any instant of time.

ª y º2
1
ªyº
2
y2
(A) (B) « » (C) « » (D)
y
A ¬A¼ ¬A¼ A

(49) What would be the change in the mechanical energy of SHO on decreasing its amplitude by 25 %.
(A) decreases by 56.25 % (B) decreases by 43.75 %
(C) increases by 56.25 % (D) increases by 43.75 %

191
(50) When the displacement of SHO is y1, its potential energy is E1 and when displacement is y2, the
potential energy is E2. For the displacement (y1 + y2) where (y1 + y2) < A) the potential energy is
E. then ...... .

(A) E = E1 + E2 (B) E = E1 + E2

(C) E2 = E12 + E22 (D) E = E + E


2 E1E 2
1 2

(51) One end of an elastic spring (mass less) is fixed at a rigid support. At the other end, a solid
cylinder is attached in such a way that the cylinder can rotate without slipping. The force constant
of a spring is 4 Nm–1. Now cylinder is given 0.5 m displacement and allowed free to move. The
cylinder performs SHM and rotational motion about its own axis. Calculate K. E., rotational K. E.
and periodic time.

(A) K = J, Kr = J, T = 2p (B) K = J, Kr = J, T = 2p
1 1 3m 1 2 3m
3 6 2k 3 3 2k

(C) K = J, Kr = J, T = 2p (D) K = J, Kr = J, T = 2p
1 1 2k 2 1 2k
3 6 3m 3 3 3m

(52) The displacement of SHO is ...... % of its amplitude, when its kinetic energy is 25% of it
potential energy.
(A) 69.88 % (B) 96.44 % (C) 49.88 % (D) 89.44 %
(53) SHO, starting from its mean position, completes 1 oscillation in 12 s. At what time its kinetic
energy becomes 25 % of its total energy (or decreases by 75%)
(A) t = 2 s (B) t = 0.5 s (C) t = 4 s (D) t = 8 s
S
(54) An initial phase of SHO is 3 rad. Total energy is E. Calculate its initial kinetic energy and initial
potential energy.

(A) K0 = 4 , U 0 = E4 (B) K0 = E , U 0 = 0
3E

(C) K0 = 0 , U 0 = E (D) K0 = E4 , U 0 = 4
3E

Ans. : 39 (C), 40 (A), 41 (D), 42 (A), 43 (B), 44 (B), 45 (A), 46 (D), 47 (B), 48 (C), 49 (B), 50 (A),
51 (A), 52 (D), 53 (A), 54 (D)

Simple Pendulum
ˆ Only for small oscillations, the motion of simple pendulum is SHM.
If m = mass of the sphere, l = length of the pendulum, g = gravitational acceleration

Force constant of the simple pendulum k =


mg
l

angular frequency of the simple pendulum w =


g
l

Periodic time of the simple pendulum, T = 2p g


l

192
T2
For constant g (place is same), T = T µ l ; (l < Re)
l2
ˆ l1
1

T2 1
For constant length at different planets, T = Tµ g
g1
ˆ g2
1

ˆ For the pendulum of thin metallic wire, T µ l ; l µ temperature

\ Temperature increases periodic time increases and oscillations becomes slow.

ˆ The value of g is less at mountains or in mines than that at surface of the earth.

\ Periodic time increase at mountains or inside the mines. [oscillations becomes slow]

Simple pendulum in a lift (Elevator)

(1) Elevator is moving with acceleration a :


moving upward geff = g + a ; moving downward, geff = g – a

(2) Elevator is moving with retardation or deceleration (–a)


moving upward, geff = g – a ; moving downward, geff = g + a

Periodic time T = 2p g
l
eff

(3) If elevator is freely falling, a = g


geff = g – g = 0

\ T = ¥ That is Pendulum does not oscillate.

Simple pendulum in a train

When the train is moving with an acceleration or retardation a,

geff = g 2  a2 [a is '+ve' or '–ve' a2 will be positive]

T = 2p g
l
eff

Second Pendulum

Periodic time T = 2 s

length on earth l » 100 cm » 1 m

193
Graphs for Simple Pendulum :

T T
straight line straight line

l l
g

T2
T

Parabola
straight line

l l

T2
T

Hyperbola

straight line

l g
g

(55) The periodic time of a simple pendulum is doubled on increasing its length by 7.5 m. The original
length of the pendulum l = ...... m.
(A) 1.5 m (B) 2m (C) 2.5 m (D) 3.0 m
(56) By keeping the length of a simple pendulum constant, it is taken at a place where gravitational
acceleration reduces by 75%. Then the periodic time of the pendulum
(A) increases by 100 % (B) decreases by 100 % (C) increases by 200 % (D) decreaser by 200 %
(57) The mass of a planet is 4 times that of earth and the diameter of the planet is doubled than that
of the earth. If the periodic time of the pendulum on the earth is Te. What would be the periodic
time of the same simple pendulum on the planet ?

(A) Te (B) 2 Te (C) 2 Te (D)


Te
2
(58) A hollow metallic sphere filled with mercury is taken as sphere of a simple pendulum. If some
part of the mercury flows out of the sphere ......
(A) period and oscillations does not change (B) T decreases, Oscillations becomes slow
(C) T decreases, Oscillations becomes fast (D) T increases, Oscillations becomes slow

194
(59) The period of a simple pendulum in a stationary elevator is T. When an elevator moves up with
an acceleration of , its period is T1. If it moves downward with the same acceleration, its
g
4
period is T2. Then T1 = ...... .
T2
T1 T1 T1 T1
(A) T = (B) T = (C) T = 5 (D) T = 3
2 2 3 5
2 5 2 3 2 2

(60) An electric charge –q is induced on a metallic sphere of a simple pendulum. There is positive
charge on the horizontal surface below this pendulum. What would be the period of the
oscillations of the simple pendulum ?

ª º2
1

« l »
ª º2
1

(A) T = 2p « ml » (B) T = 2p « »
¬ qE ¼ « g  qE »
¬« m ¼»

ª º2
1

« l » ª º2
1

(C) T = 2p « » (D) T = 2p « l »
« g – qE » ¬g¼
¬« m ¼»

(61) Calculate the effective gravitational acceleration at a place, where periodic time of a simple
pendulum of length 0.25 m is 1 s.
(A) p2 (B) 4p2 (C) 2p (D) 0.25 p2
(62) The ratio of frequencies of two simple pendulum kept at the same place is 5 : 4. Then the ratio of
their length is ...... .

(A) 5 (B) (C) 25 (D) 1


4 2 16
5

(63) The length of a simple pendulum is l. When an iron sphere is used as a bob of this pendulum, the
period of its simple harmonic oscillation is T. If a steel sphere of the same volume is used instead
of iron sphere the period becomes 2T. Calculate the length of the pendulum if the density of steel
is n times the density of iron.

(A) 4l (B) (C) 4nl (D) 4n2l


4l
n
(64) A simple pendulum is suspended from the celling of an aeroplane. If the plane starts moving on
the horizontal run-way with a constant acceleration of 12.49 ms–2. Calculate the periodic time of
simple harmonic oscillations of the pendulum. The length of the pendulum is 1m and g = 10 ms–2.

(D) S
S
(A) 2p (B) 2 (C) 2p2
2

2
(65) When a child swing, in sitting and standing inside the swing, the periodic time of the oscillations is
T and T' respectively, then.
' ' ' '
(A) T = T (B) T > T (C) T < T (D) T = T2
195
P
(66) A pendulum of length l m lift at P, when it reaches Q, it
losses 20 % of its total energy due to air resistance. The
velocity at Q = ...... ms–1. (g = 10 ms–2) 1m
(A) 4 ms–1 (B) 6 ms–1
(C) 8 ms–1 (D) 10 ms–1
(67) A simple pendulum with a bob of mass m oscillates along Q
P O R
PQR path. Its motion is simple periodic motion. Calculate
the velocity of the bob when it passes through point Q.
H
(A) v = mgH (B) v = 2 gH

(C) v = 2g H (D) v = 2g H Q
(68) The length of a simple pendulum is 0.9 m. When it passes through its mid point, it velocity is 5 ms–1.
calculate its velocity when it makes an angle of 60° with the vertical line. [g = 10 ms–2]
(A) 4 ms–1 (B) 3 ms–1 (C) 2 ms–1 (D) 0
(69) On the surface of a planet, when the length of the simple pendulum is kept 25 m, its periodic
time is T. At a height of 2000 km from the surface of this planet, when the length is kept 9 m,
the periodic time remains same as T. Then the radius of the planet is ...... .
(A) 2400 km (B) 3000 km (C) 6000 km (D) 8000 km
(70) The periodic time of a simple pendulum is T1. When the point of suspension from the rigid support
T12
is moved by y = kt , its periodic time becomes T2. Then
2
= ...... . [g = 10 ms–2, k = 1]
T22

(A) 7 (B) 5 (C) 7 (D) 6


6 6 5 5

Ans. : 55 (C), 56 (A), 57 (A), 58 (D), 59 (C), 60 (B), 61 (A), 62 (C), 63 (A), 64 (B), 65 (C), 66 (A),
67 (D), 68 (A), 69 (B), 70 (B)
Natural Oscillations Damped Oscillations, Forced Oscillations and Resonance :
ˆ Natural Oscillations : The oscillations performed in absense of any type of resistive forces are
known as Natural oscillations (free oscillations)
frequency of the natural oscillations = f0.
natural angular frequency w0 = 2pf0.
ˆ Damped Oscillations : The oscillations performed in the presence of resistive force, which are of
decreasing amplitude are called damped oscillations.
ˆ Forced Oscillations : The oscillations performed in the Presanse of external periodic force
with constant amplitude are called forced oscillations.
Natural Oscillations Damped Oscillations Forced Oscillations
ˆ Amplitude (A) remains ˆ A exponentially decrease ˆ A remains constant with time
constant with time with time
ˆ The resultant Force F = –ky ˆ F = –ky –bv ˆ F = –ky – bv + F0 sin wt

196
ˆ differential equation
d2 y
dt 2 + m
dt
b
+w02y = 0
dy d2 y
dt 2 + m b dy
dt
+w02y = 0 sin wt
m
F

d2 y
+ w02y = 0
dt 2
ˆ Solution; ˆ y(t) = A'(t) sin (w't + f) ˆ y(t) = A sin (wt + f)
y(t) = A sin (w0t + f)
F0
ˆ A doesnot depend on time t ˆ A'(t) = A e  bt ˆA=
[ m 2 ( Z 02  Z 2 ) 2  b 2 Z 2 ] 2
2m 1


§ Z y0 ·
ˆ Natural angular frequency w' = f = tan–1 ¨ v ¸
2
k  b
m 2m © 0 ¹

w02 = , w0 = b = damping co-efficient of


k k
m m
the medium

ˆ Natural frequency b=
F
v
(for small velocity)

f0 = 21S 2p If velocity is large


k
m

b= (n depends on velocity)
F
vn
N s
unit of b = Nsm–1
m
[b] = M1L0T–1
For Forced oscillations (y)
Displacement (y) ® time (t) Amplitude (A) ® time (t)
(1) constant amplitude (2)
y(t) A constant amplitude
y = +A f=0

y=0
t

y = –A t

For Damped Oscillations


S
Displacement (y) ® time (t) (f = 0) Displacement (y) ® (t) (f = )
2
(3) (4)
A'(t)
y = +A A'(t) y = +A
y(t)
y (t)

y=0
time (t)
time (t)

y = –A y = –A

197
3S
Displacement (y) ® time (t) (f = ) Damping Ossicillation
2
(5) (6) Amplitude (A') ® time (t)
|
A'(t) A
y = +A y(t) A max
amplitude exponentially decreases with time

y =0
time (t)
y = –A
O
time (t)
Resonance

Amplitude of forced oscillation


Z0

1
2
 Z2

As w move toward w0, A increases


ˆ When w = w0, A = maximum. This phenomenon is called Resonance.
For Resonance f = f0
When the frequency of external periodic force becomes equal to the natural frequency of the
oscillations, the amplitude of the oscillations becomes maximum. This phenomenon is called
resonance.
w
0
is called resonant angular frequency
f0 is called resonant frequency

At the time of resonance A =


F0
ˆ bZ
ˆ b ® 0; A ® ¥
Resonance Curves
ˆ b=0Þ A=¥
With the increase of b, maximum b1 = 0
amplitude decreases. A
b1 < b2< b3< b4
ˆ When the external periodic force is
acting on the system and frequency of b2
b3
the external periodic force becomes b4
equal to the natural frequency Z
(or nearly equal), the system oscillates w = w0 Z0
with a very large amplitude and the Z
=1
system may break or collapse. e.g. Z0

(1) When soldiers are marching on a suspended bridge, the frequency of the external periodic
force becomes equal to the natural frequency of the bridge, the bridge might be collapse.
(2) The gusts of wind exerts external periodic force to the trees and structures. If this
frequency becomes equal to the natural frequency, tree oscillate with very large amplitude
and collapses.

198
(3) When a bridge is designed, care is taken so that the external force due to gusts of wind
and natural frequency of the bridge do not becomes equal.
Frequency of the Seismic waves (f) Ñ
The frequency (f) of the seismic waves is very less than the natural frequency (f0) of low
rise structures. (f < f0)
The frequency (f) of the seismic waves is greater than the natural frequency (f0) of
high rise structures. (f > f0)
Therefore, in an earthquake, low and high-rise structures remain less affected while
medium high structure fall down. (f = f0)
Significant Amplitude :
1
e
times the amplitude of the damped oscillator is called its significant amplitude.

(71) In what time the amplitude of damped oscillator becomes 1e times of its maximum value ?

(A) 2 m (B) 2b (C) (D) m


b m 2m 2b
b
(72) The mass of an oscillator is 100 g. It is oscillating in a medium having damping co-efficient
0.805 dyne s cm–1. Calculate the time during which its amplitude decreases by 80 % of its initial
value.
(A) 200 s (B) 400 s (C) 600 s (D) 800 s
(73) The mass of a damped oscillator is m. Its initial amplitude is A0. If at time t, its amplitude
becomes At, then the damping co-efficent of the medium is ...... .

ª º ª º ª § At ·º ª § A0 ·º
« » « » « t ln ¨ ¸» « t ln ¨ ¸»
« 2m » « 2m » « © A0 ¹» « © At ¹»
(A) b = « § At ·» (B) b = « § A0 ·» (C) b = « 2 m » (D) b = « 2 m »
« t ln ¨ ¸» « t ln ¨ ¸» « » « »
¬« © A0 ¹ ¼» ¬« © At ¹ ¼» ¬« ¼» ¬« ¼»

(74) The mass and periodic time of a damped oscillater is 500 g and 2 s respectively. Its amplitude
reduces by 50 % of its initial value, When it completes 50 oscillations. Calculate the damping
co-efficient of the medium.

(A) 0.6930 dyne s cm–1 (B) 6.930 dyne s cm–1 (C) 0.06930 dyne s cm–1 (D) 69.30 dyne s cm–1

(75) The amplitude of damped oscillator becomes 0.8 times of its initial value in 5 s. Then it becomes
N times of its initial value in next 10 s. What would be the value of N ?

(A) 0.813 (B) 0.729 (C) 0.512 (D) 0.343

(76) The periodic time of a simple pendulum in air is T0. If the pendulum oscillate in the medium of a
liquid, the period becomes T. If the density of the liquid is 4 times the density of the material of
the sphere, calculate T.

(A) T = (B) T = 2T0 (C) b = T (D) T


T0 3 2
2 2 0 3 0

199
(77) When the damped oscillator completes 100 oscillations its amplitude becomes 20 % of its initial
value. What will be the amplitude when it completes 200 oscillations ?
(A) 10 % of A0 (B) 4 % of A0
(C) 6 % of A0 (D) 8 % of A0
(78) The mass of a damped oscillator is m. The damping co-efficent of the medium is b. How many
oscillations are performed in 1 s ?

(A) 2S
k  b
(B) 2S mk  2bm
2 2
1 1
m 2m

(C) mk (D)
2 2
 b k  b
1 1
2S 2m 2S m 2m

(79) What would be the time taken by a damped oscillator to acheive its significant amplitude
(average life time) ?

(A) (B) (C) (D)


2m b 0.6930 m 0.6930
b 2m b 2mb

(80) What would be the amplitude of the forced oscillations at the time of resonance ?
bZ bZ
(A) a (B) m a (C) (D)
m a0 a0
0 0 bZ bZ

(81) During earthquake, which one of the following structure have maximum posibility of fall down ?
(A) High rise structure (B) low rise structure
(C) medium high structure (D) All of them having equal probability
(82) In the case of forced oscillations, the resonant wave becomes more sharp when.
(A) the magnitude of resistive force is less (B) the magnitude of external periodic force is less.
(C) the frequency of external periodic force is less. (D) the damping coeffcient of the medium is less.
(83) If w
0
is the natural angular frequency and w is the angular frequency of the external periodic
Z0
force, then at the time of resonance .
Z

Z0 Z0 Z0 Z0
(A) ³1 (B) =0 (C) =1 (D) =¥
Z Z Z Z
(84) A damped oscillator of mass m performs damped oscillation in the medium of damping
co-efficient b. At time t1 and t2, its amplitude is A1 and A2 respectively. Which one of the
following is true ?

b(t1 -t 2 ) b(t 2 -t1 )


(A) A2 = A1 e 2m (B) A2 = A1 e 2m

§ t +t ·
–b ¨ 1 2 ¸
–b(t1 + t 2 ) © 2 ¹
(C) A2 = A1 e 2m (D) A2 = A1 e 2m

200
(85) At a certain time the amplitude of a damped oscillator is 10 % of its initial value. Now the
oscillator is allowed to oscillate in the medium having damping co-efficient twice that of the
previous one. Calculate the percentage decrecse in its amplitude during the same time.
(A) 20 % (B) 5 % (C) 2 % (D) 1 %

Ans. : 71 (C), 72 (B), 73 (B), 74 (B), 75 (C), 76 (D), 77 (B), 78 (D), 79 (A), 80 (C), 81 (C), 82 (D),
83 (C), 84 (A), 85 (D)
Waves
ˆ The motion of the disturbance propagating in the medium (or in free space) is called a wave.
ˆ The wave is neither a 'Physical body' travelling in the medium nor medium Particles are moving
as a single unit.
Mechanical waves : The waves which require elastic medium for their transmission are called
mechanical waves.
Non-mechanical waves : The waves which do not require any elastic medium [even propagate in
the medium] and also propagate in the free space are called non mechanical waves [They are also
known as electro magnetic waves]
Transverse waves : The waves in which the oscillations of the particles are in a direction
perpendicular to the direction of propagation are called transverse waves.
Longitudinal waves : The waves in which the oscillations of the particles are in the direction of
propagation of the wave are called longitudinal waves.
ˆ Sound waves are mechanical and longitudinal.
ˆ waves produce on the string are mechanical and transverse.
ˆ Light waves are non mechanical and transverse.
Intensity of the wave (I) : Energy passing through a unit area, taken in the direction normal to the
propagation, in one second is called intensity of the wave.

Energy (E)/Time (t)


(I) =
Area (A)
Js –1 watt
Its unit is : = = Wm–2
m 2 m2

[I] = M1L0T–3
Note :
Dimensional Formula Quantity
MT : –1
damping coefficient to the medium
MT–2 : Force constant of the spring, surface tension.
MT : –3
Wave intensity, emissive power ot the surface

1
Intensity I µ E (= kA2)
2

\ I µ A2, Intensity µ (Amplitude)2

201
Wave length (l) and Velocity (v) of the wave
l

O p 2p 3p 4p 5p

t=0 l t=T

Wave length (l) : The distance between two particles having phase difference 2p rad is called
wave length (l).

Velocity of the wave (v) : The effect of disturbance travels distance l in time T.

O Z
\ Velocity of the wave v = = lf = ;k= = wave vector
2S
T k O

Note : distance º Phase difference º time

l º 2p º T

Wave equation
y

O x t=t

The displacement of a particle at a distance x from the origin at time t is,


y = A sin (w t – kx) [for positive X-direction]

x
y = A sin 2p (f t – )
O

x
y = A sin 2pf (t – )
v
For the wave propagating in the – X direction take x = –x.
2S x
The phase difference between the two particles having seperation x is, d =
O
ˆ
Note : The time derivative of wave equation represent velocity of the particle at a distance x from
the origin, at time t.
[It does not represent velocity of the wave]

v= Þ v = Aw cos (w t – kx)
dy
dt
ˆ The slope of the wave form at a distance x from the origin at time t

= –Ak cos (w t – kx)


dy
dx

202
S
(86) The minimum distance between the two particles having phase difference 17
2
is ...... A°.
[k = 6.28×108 rad cm–1]
(A) 4.25 (B) 8.5 (C) 17 (D) 3.4
(87) The value of a wave vector is 10 p rad cm . Calculate the phase difference between two
–1

particles having 3.6 cm distance.


(A) 1.8p (B) 3.6p (C) 18p (D) 36p
(88) Wave equation is y = 10 sin (4pt –px) cm, t is in second. Calculate the ratio of velocity of a
particle at 38 cm away from the origin at the end of 10 s and velocity of the wave.
(A) 40 p (B) 10 p (C) 0.1 p (D) 4 p
(89) An amplitude of a progressive harmonic wave is 5 cm. The displacement of a particle at a distance

4 cm away from the origin, at the end of 2 s is cm and the displacement of a particle 16 cm
5
2
away from the origin, at the end of 4 s is 2.5 cm. Calculate the values of w and k.
7S S 5S S S 3S S S
(A) w = 24 , k = 24 (B) w = 24 , k = 24 (C) w = 24 , k = 24 (D) w = 24 , k = 12

(90) The frequency of a wave is


10
S
Hz. If the maximum displacement of particles of the medium is
0.4 cm, the maximum velocity of the particle is ...... cms–1.
(A) 2 (B) 4 (C) 8 (D) 10
(91) The wave equation is y = 10 sin (pt – px) cm. Time t is in second. Calculate the displacement

and acceleration of a particle which is 2 cm away from the origin, at the end of s.
13
6
(A) y = 5 cm, a = –5p2 cms–2 (B) y = 10 cm, a = –10 p2 cms–2
(C) y = 5 cm, a = –10 p2 cms–2 (D) y = 10 cm, a = –5p2 cms–2
(92) The frequency of an electromagnetic wave is 150 MHz. Calculate the value of wave-vector in
rad m–1.
S 3S 3S
(A) p (B) (C) (D)
2 2 4
S
(93) The wave equation for a progressive harmonic wave is y = 0.5 sin (0.05t + 0.02px – ) cm.
6
Where time t is in second. Calculate the minimum distance between the two particles having
S
phase difference rad.
4
(A) 3.125 cm (B) 6.25 cm (C) 12.5 cm (D) 25 cm
(94) The frequency and velocity of a wave are 1 kHz and 330 ms respectively. Calculate the
–1

minimum distance (in metre) between the two particles having phase difference 60°.
(A) 11×10–2 (B) 5.5 × 10–2 (C) 6.6 × 10–2 (D) 3.3 × 10–2
S
(95) For particle-1, wave equation is y1 = 10 sin (50pt + 3 ). For particle-2, wave equation is
y2 = 10 cos pt. Calculate the phase difference of the displacement [or velocity or acceleration] of
the particle-2 with respect to particle-1.
S S S S
(A) (B) (C) – (D) –
6 3 6 3

203
(96) The wave equation for a wave propagetting in X-direction is y = 0.008 cos (a x –bt) m. The
wavelength and periodic time are 0.08 m and 0.5 s respectively. Calculate values of a and b.

(A) a = 25p, b = 4p (B) a = 4 p, b = 25p (C) a = 50 p, b = 2p (D) a = 2p, b = 50 p

x
(97) The wave equation is y = y0 sin 2p (f t – ). The maximum velocity of a particle is 4 times the
O
velocity of the wave. Calculate the wave length of the wave.
S y0 S y0 S y0
(A) (B) (C) (D) 4py0
4 8 2

(98)
The wave equation is y = 0.01 sin 2p 0.05  0.02 m [Where y and x are in metre, t is in
t x

second] Calculate the values of maximum velocity and maximum acceleration.


(A) vmax = 0.4p, amax = 4p2 ms–2 (B) vmax = 16 p, amax = 4p2 ms–2

(C) vmax = 16 pms–1 , amax = 16p2 ms–2 (D) vmax = 0.4 p ms–1 , amax = 16p2 ms–2
S
(99) The displacement of some particle of the medium is given by y = 10–6 sin (100 t + 20x +
4
)m.
(where x is in metre, t is in second). Calculate the wave-speed.
(A) 5 ms–1 (B) 0.5 ms–1 (C) 5p ms–1 (D) 5p2 ms–1
x
(100) The wave equation for one dimensional progressive harmonic wave is y = 10 sin 20p (t – ) m.
160
(where x is in metre, t is in second). Calculate the slope of the wave at a distanace of 320 m, at
the end of 2 s.
S 5S 5S 3S
(A) –10 (B) – (C) – (D) –
5 4 8 8

Ans. : 86 (A), 87 (D), 88 (B), 89 (B), 90 (C), 91 (A), 92 (A), 93 (C), 94 (B), 95 (C), 96 (A), 97 (C),
98 (D), 99 (A), 100 (B)

Speed of waves in a medium :


ˆ Speed of transverse wave on stretched string

v= Where T = Tension force


T
P
m = linear mass density of the string

ˆ Speed of longitudinal wave (sound wave) in a medium :


E = Elastic constant of the medium
v= U
E
B = Bulck's modulus
Y = Young's modulus
v=
B
U P = Pressure
r = density of the medium
v=
Y
U

JP Cp
v= g = C
U v

204
J RT
v= rV = mass
UV
V = volume of gas
v=
J RT m = Molar mass of gas.
M

Note :
ˆ The phenomenon of the propagation of sound in the gas is adiabatic
ˆ At a constant temperature, velocity of sound in the gas is independent of the Pressure.
ˆ Velocity (speed) of sound increases with increasing humidity
ˆ The speed of sound is comparatively much greater in solid than that in the liquid and in the gas.
[except Vulcanized rubber]
ˆ At STP, the speed of sound in air v = 332 ms–1.
ˆ In practice, the range of wave length of audible sound is from l = 1.7 cm to l = 17 m.
ˆ Waves having l < 1.7 cm are not audible. They are known as Ultrasonic waves.
ˆ Waves having l > 17 m are also not audible. They are know Infrasonic waves.
ˆ An object moving with a velocity greater than the velocity of sound is called supersonic.
When such supersonic body (e.g. an aeroplane) travels in air, it produces energetic
disturbance. Such disturbance moves in backward direction and diverge in the form of a cone.
Such waves [disturbances] are called Shock waves. When Shock waves collides with structure,
a very huge sound is produced.

The speed of supersonic is measured in Mach Number

Velocity of source
Mach Number =
Velocity of sound

ˆ Speed of the sound is directly proportional to its absolute temperature.

v1
v2 =
T1
T2

ˆ For two different gases having equal pressure (P) and equal value of g, the ratio of velocity of
sound.

v1 U2
v2 = U1
, r1 and r2 are the density of the gas.

ˆ For sound, the temperature coefficient of expansion (a) is given by


205
vT  v0
a= , where, vT = velocity of sound at T °C, v0 = velocity of sound at 0 °C. The unit
T
of a is °C–1.
ˆ For sound waves, the time interval between two successive condensation and rarefaction is equal

to .
T
2
ˆ When a person hear echo sound from the reflector at a distance d, then velocity of sound.

v= d
2d
t reflector
Person
\ Time interval of Echo. t = v
2d

§ I ·
ˆ Sound intensity level is given by L = 10 log ¨ I ¸ Where I0 = minimum intensity (refrence
© 0¹
intensity), I = intensity of the sound
Sound intensity level is meausured in decibels (dB).

(101) A sound wave of frequency 400 Hz is propagating with the speed of 332 ms–1. What would be
the minimum time to form rarefaction at a place where maximum condensatin occurs.

(A) s (B) s (C) s (D) s


1 1 1 1
800 200 332 664
(102) The speed of sound in H2 is 1225 ms–1. By taking volume ratio 1:2 of H2 and O2, a mixture
H2O2 is prepared. What would be the speed of sound (in ms–1) in the mixture ? [density of O2 is
16 times that of H2]
(A) 2450 (B) 1000 (C) 500 (D) 250
(103) At what temperature the speed of sound would be double than its value at NTP ?
(A) 600 K (B) 1200 K (C) 150 K (D) 75 K
(104) A person standing at the mid point of the two parallel walls claps his hand, hears its echo after
1 s. Calculate the distance between the two walls (in metre) if the speed of the sound in air is
332 ms–1.
(A) 332 (B) 116 (C) 664 (D) 58
(105) A stone is dropped in a well from the height of 20 m from the water surface. The sound of
collision of the stone with water is heard after 2.06 s (after dropping). Then the velocity of sound
in ms–1 is ...... . [take g = 10 ms–2]
(A) 333 (B) 300 (C) 350 (D) 260
(106) The minimum intensity of an audible sound is 10 Wm . Calculate the sound intensity level (in
2 –2

decibel) when the intensity of the sound becomes 107 Wm–2.


(A) 5 (B) 3 (C) 30 (D) 50
(107) A sound of intensity level 50 dB is how many times powerful than the sound of intensity level
20 dB ?
(A) 30 (B) 300 (C) 900 (D) 1000
(108) What would be the minimum distance (in metre) of reflector from the person (source) for
listening the echo of sound ? Velocity of sound is 330 ms–1.
(A) 16.5 (B) 33 (C) 66 (D) 99

206
(109) An intensity of a sound wave decreases by 10 % while passing through a slab. Such two slabs
are kept together and sound wave is allowed to pass through it. Calculate the percentange
decrease in the intensity of the sound wave.
(A) 20 % (B) 19 % (C) 21 % (D) 10 %
(110) Young's modulus of a matter is 13.2×10 Nm . The density of this matter is 3.3×102 kg m–3.
10 –2

Calculate the velocity (in ms–1) of longitudinal wave in this matter.


(A) 500 (B) 1000 (C) 2000 (D) 2500
(111) Velocity of sound in a gas at STP is 273 ms . Calculate the temperature coefficient of velocity
–1

in ms–1 K–1.
(A) 0.5 (B) 0.25 (C) 0.8 (D) 0.75
(112) A wire PQR is prepared by connecting two wires PQ and QR of equal radius. The length and
mass of wire PQ are 2 m and 0.025 kg respectively. Those for wire QR are 1 m and 0.05 kg
respectively. The tension produced in wire PQR is 80 N. Calculate the time taken by the sound
(in second) to travel from P to R.
(A) 0.025 (B) 0.05 (C) 0.25 (D) 0.5
Ans. : 101 (A), 102 (C), 103 (B), 104 (A), 105 (A), 106 (D), 107 (D), 108 (B), 109 (B),
110 (C), 111 (A), 112 (B)
Principle of Superposition
‘When two or more waves superpose at same particle of the medium, the resultant displace-
ment of a particle at the superposition is equal to the vector sum of the individual displacement
produced by each wave.
Reflection of Waves
ˆ Reflection from the rigid support :
ˆ When a wave is reflected from the rigid support, its phase incident wave
is increased by p.
\ “crest” becomes “trough”
and “trough” becomes “crest”
reflected wave
If yi = A sin (w t – kx) its reflected wave,
yr = –A sin (w t + kx)
[Note : The negative sign indicates that the phase increases by p. The sign inside the bracket
changes indicates that direction of propogation is reversed]
ˆ From the free end
ˆ The reflection at free end (open boundary) takes place incident wave
without any change in the phase.
“crest” remains “crest”
“trough” remains “trough” reflected wave
If yi = A sin (w t – kx)
yr = A sin (w t + kx)
Stationary Waves
The resultant wave obtained due to the superposition of two waves having equal amplitude (A),
equal wave length (l) [or equal frequency f ] and travelling in mutually opposite directions, is called
Stationary wave.
ˆ equation for the stationary wave y = –2A sin kx cos w t
ˆ energy does not propagate in this type of wave.

207
Nodes
The positions in a stationary wave where the amplitude always remains zero are called
the “Nodes”.
O 3O
The nodes are located at a distance x = 2 , l, 2 , 2l, ......, 2 from the end x = 0
nO
ˆ
O
ˆ The distance between two successive node is
2
O
ˆ The first nodel point is at a distance of 2 (from x = 0 end).
Antinodes
The positions in a stationary wave where the amplitude always remains maximum are called
“Antinodes”.
O 3O 5O O
ˆ Antinodes are located at a distance x = , , ......, (2n-1) from the end x = 0.
4 4 4 4
O
ˆ The distance between two successive antinode or two successive node is
2
O
ˆ The distance between successive node and antinode is
4
O
ˆ The first antinodel point is at a distance of (from x = 0 end).
4
rigid support

A A A

N N N N

O
O O
2
O
4 2

Normal Modes of Vibration


Fundamental frequency (First harmonic)
n=1 A

f1 = l1 = 2L
v
2L
Second harmonic (First overtone)

f2 = 2 L l2 = L
2v
A N A
n=2
f2 = 2f1
Third harmonic (Second overtone)

f3 = l3 = L
3v 2
N
2L 3
A A N A
n=3 f3 = 3f1

nth harmonic (n–1th overtone)

fn = ln =
nv 2L

n=n
2L n
fn = nf1

208
Note :

ˆ For oscillations having nth harmonic;

number of closed loops = n

number of Antinodes = n

number of Nodes = (n–1)

ˆ The difference between two successive harmonic is equal to fundamental frequency.

fn – fn–1 = f1 or fn+1 – fn = f1

Stationary Waves in Pipes

ˆ For closed pipe n = 1 fundamental frequency (first harmonic)


frequency wave length n = 2 third harmonic (first overtone)
n = 3 fifth harmonic (second overtone)
fn = = (2 n  1)
v (2 n –1) 4L
n = n (2n –1)th harmonic ((n –1) overtone)
n
l
4L

Note : For closed piple all harmonics are not possible only f1, 3f1, 5f1, 7f1 ...... possible.

ˆ Open pipe

frequency wave length n = 1 fundamental (first harmonic)


n = 2 Second harmonic (first overtone)
fn = =
nv 2L
n
n = 3 third harmonic (second overtone)
l
2L n

fopen pipe = 2 fclosed pipe n = n, nth harmonic ((n –1)th overtone)

(113) The possible maximum wave length of the stationary wave produced on the string of length
100 cm is ...... cm.
(A) 25 (B) 50 (C) 100 (D) 200
(114) In a case of stationary wave, the distance between successive node and antinode is 0.01 m. If
the speed of the components of the wave is 320 ms–1, calculate the frequency of the stationary
wave.
(A) 4 kHz (B) 8 kHz (C) 800 kHz (D) 0
(115) The fundamental frequency for an open-pipe is 512 Hz. If it is closed at one end the fundamental
frequency becomes ...... Hz.
(A) 256 (B) 512 (C) 1024 (D) 0
(116) The air column in a closed pipe experiences first resonance with a tuning fork of frequency
160 Hz. The length of the air column in the closed pipe is ...... cm. (v = 320 ms–1)
(A) 25 (B) 50 (C) 2.5 (D) 5
(117) A closed organ pipe and an open organ pipe are tuned to the same fundamental frequency. What
is the raito of lengths.
(A) 2 : 3 (B) 3 : 4 (C) 1 : 2 (D) 3 : 2

209
(118) What would be the minimum length (in cm) of an open-pipe to have resonance with the tuning
fork of 160 Hz ? (v = 320 ms–1)

(A) 10 (B) 25 (C) 50 (D) 100

(119) The frequencies of two consecutive overtone are 285 Hz and 325 Hz respectively. What would
be the fundamental frequency ?
(A) 20 Hz (B) 40 Hz (C) 80 Hz (D) 305 Hz

(120) The length of a closed pipe is 130 cm. The frequency of stationary waves form is equal to the
frequency of third overtone. Calculate wavelenth (in cm) of the wave.
(A) 40 (B) 80 (C) 130 (D) 260

(121) A closed pipe and an open pipe have their first overtones identical in frequency. Their lengths are
in the ratio ...... .
(A) 1 : 2 (B) 3 : 4 (C) 4 : 5 (D) 5 : 6

(122) A string of length 100 cm is oscillating with 10th harmonic. The number of nodes and antinodes
form on the string are ...... and ...... respectively.
(A) 9 and 10 (B) 10 and 9 (C) 10 and 11 (D) 11 and 10

(123) The length of a closed pipe is 125 cm. The sound wave is produced with a tuning fork of
frequency 320 Hz. Now water is filled gradually in this tube. For which height of water column
will resonance occur ? v = 320 ms–1.
(A) 25 (B) 75 (C) 100 (D) 125

(124) A string is attached with rigid supports separated by 100 cm distance. It is observed to have
resonant frequencies of 295 Hz and 415 Hz. There is no other resonant frequency between these
two. Then, the lowest resonant frequency for this string is ...... .
(A) 120 Hz (B) 60 Hz (C) 220 Hz (D) 250 Hz

2S
(125) The wave equation for a stationary wave produced on a stretched string is y = 10 sin 7
x cos 70p t

Distance between two successive node is ...... cm. [Where x and y are in cm, t is in sec.]

(A) 1.75 cm (B) 3.5 cm (C) 6.5 cm (D) 7.5 cm

(126) The length of the wire of guitar is 100 cm. Its fundamental frequency is 250 Hz. Calculate the
length of the wire required to have 500 Hz frequency.
(A) 50 cm (B) 100 cm (C) 200 cm (D) 250 cm

S
(127) The stationary wave produced on a string is given by y = 10 sin 4
x cos 40pt [Where x and y
are in cm, t is in s] The positions of antinodes from the rigid support are ...... .
(A) 2 cm, 6 cm, 10 cm, 14 cm... (B) 4 cm, 8 cm, 16 cm, 20 cm...
(C) 2 cm, 4 cm, 6 cm, 8 cm.... (D) 4 cm, 6 cm, 8 cm, 10 cm...

210
(128) A block is attached at the free end of the sonometer wire. The fundament frequency for the
vibrations of the wire is 500 Hz. When the block is immersed in the water the fundamental
frequency becomes 300 Hz. Now, instead of water the block is immersed in the liquid. the
fundamental frequency becomes 100 Hz. Calculate the specific density of the liquid.
(A) 1 (B) 1.5 (C) 2 (D) 2.5
(129) A string of length 35 cm is vibrating with the frequency of 3 kHz. The velocity of the wave is
350 ms–1. Find the fundamental frequency and number of closed loops formed on the string.
(A) f1 = 1000 Hz, n = 6 (B) f1 = 500 Hz, n = 5
(C) f1 = 500 Hz, n = 6 (D) f1 = 300 Hz, n = 10
(130) The wave equation for a progresive harmonic wave propagating in the negative X-direction is
y = 20 sin (4pt + 3px). The wave reflects from the rigid support. The equation for the reflected
wave is ...... .
(A) y = 20 sin (4pt – 3px) (B) y = –20 sin (4pt + 3px)
S S
(C) y = 20 sin (4pt – 3px + ) (D) y = 20 sin (4pt + 3px + )
2 2
2S
(131) The wave equation for a progressive harmonic wave is y = 10 sin (4pt – 5
x). It reflects from
the rigid support. If the intensity of the reflected wave is 0.81 times the intensity of the incident
wave, what would be the wave equation for the reflected wave ?
2S 2S
(A) yr = –8.1 sin (4pt + x) (B) yr = –0.81 sin (2pt + x)
5 5
2S 2S
(C) yr = –9 sin (4pt – 5 x) (D) yr = –9 sin (4pt + 5 x)

Ans. : 113 (D), 114 (B), 115 (A), 116 (B), 117 (C), 118 (D), 119 (B), 120 (A), 121 (B), 122 (A),
123 (C), 124 (A), 125 (B), 126 (A), 127 (A), 128 (B), 129 (C), 130 (C), 131 (D)
Beats
The Phenomenon of the loudness of sound becoming maximum and minimum periodically due to
super-position of two sound waves of equal amplitude (A) and slightly different frequencies
(f1 – f2 < 7) is called ‘beats’.
The number of beats in unit time = f1 – f2

The periodic time of sound intensity becomes maximum or minimum T =


1
f1  f 2

ˆ By filing one of the prongs of a tuning fork, its frequency will increase a little.
ˆ By putting some wax on one of the prongs of a tuning fork, its frequency will decrease a little.
Doppler Effect
Whenever there is a relative motion between a source of a sound and a listener, with respect to
medium in which the waves are propagatting, the frequency of the sound experienced by the listener
is different from the frequency emitted by the source. This phenomenon is called
Doppler effect.
If, fs = Original frequency of the sound emitted by the source, fL = frequency of the sound
experienced by the listener, vs = velocity of the source of sound, vL = velocity of the listener
v = velocity of the sound.
§ v  vL ·
The general formula for the frequency experienced by the listener is, fL = ¨ v  v ¸ fs
© s ¹

211
Spaceial cases :
(1) The listener is moving towards the stationary source
§ v  vL ·
vs = 0, vL = positive; fL = ¨ v ¸ fs
© ¹

(2) The listener is moving away from the stationary source.


§ v – vL ·
vs = 0, vL = negative; fL = ¨ v ¸ fs
© ¹

(3) The source is moving towards the stationary listener.


§ ·
vL = 0, vs = negative; fL = ¨ v  v ¸ fs
v
© s ¹

(4) The source is moving away from the stationary listener.


§ ·
vL = 0, vs = positive; fL = ¨ v  v ¸ fs
v
© s ¹

(5) Both (source and listener) are moving towards each other (approaching each other)
§ v  vL ·
vs = negative, vL = positive; fL = ¨ v – v ¸ fs
© s ¹

(6) Both are moving away from each other


§ v – vL ·
vs = positive, vL = negative; fL = ¨ v  v ¸ fs
© s ¹

(7) The listener is moving away from the source and the source is moving towards the listener, with
relative velocity.

§v–v ·
vs = negative, vL = negative; fL = ¨ v  vL ¸ fs
© s ¹

(8) The listener is moving towards the source and the source is moving away from the listener with
relative velocity.

§ v  vL ·
vs = positive, vL = positive, fL = ¨ v  v ¸ fs
© s ¹

(132) Frequencies of two tuning forks are 320 Hz and 480 Hz respectively. They produced sound waves
in air having difference in the wave length 48 m. Calculate velocity of sound in air (in ms–1)
17

(A) 280 (B) 300 (C) 340 (D) 360


(133) Two waves having wavelengths 50 cm and 50.5 cm produced 6 beats in 1s. Calculate the
velocity of the waves.
(A) 303 ms–1 (B) 404 ms–1 (C) 505 ms–1 (D) 606 ms–1
212
(134) The wave lengths of two pitches of the sound are 90
175
m and
90
173
m respectively. Both of them

produces 4 beats in 1s with the third pitch. Calculate the fixed frequency of this third pitch.
(A) 174 Hz (B) 348 Hz (C) 522 Hz (D) 696 Hz
(135) Frequencies of three sound sources having equal intensity are 312 Hz, 316 Hz and 320 Hz
respectively. Calculate the number of beats produced by any two successive source in 1 s.
(A) 2 (B) 4 (C) 6 (D) 0
(136) A tunimg fork P, produces 4 beats in 1s with a tuning fork Q of frequency 384 Hz. filing one of the
prongs of tuning fork P, it produces 3 beats in 1s. Calculate the original frequency of the tuning fork
P. (in Hz).
(A) 380 (B) 388 (C) 381 (D) 387
(137) A tuning fork M, produces 5 beats in 1s with a tuning fork N of frequency 588 Hz. After loading
one of the prongs of tuning fork M, it produces 3 beats in 1s. Calculate the original frequency of
the fork M. (in Hz).
(A) 583 (B) 593 (C) 585 (D) 591
(138) 51 tuning forks are arranged in the ascending order of their frequencies. Any two consecutive
forks produce 3 beats in 1s. If the frequency of the last fork is 3 times that of the first fork,
calculate the frequency of 26th tuning fork.
(A) 120 Hz (B) 150 Hz (C) 170 Hz (D) 190 Hz
(139) 21 tuning forks are arranged in the ascending order of their frequecies. Any two consecutive forks
produces x beats in 1s. The frequency of 21st fork is 1.4 times that of the first fork. Calculate x if
the frequency of 11th fork is 120 Hz.
(A) 2 (B) 4 (C) 6 (D) 8
(140) Two tuning forks are of frequency 350 Hz and 355 Hz produces beats. After what time (least)
the minimum occurs at a place where maximum is occured ?

(A) s (B) s (C) s (D) s


1 1 1 1
5 10 15 20

(141) A tuning fork produces 2 beats in 1s with the stretched wire of sonometer of length 80 cm and
60 cm. What would be the frequency of the tuning fork ?
(A) 12 Hz (B) 14 Hz (C) 16 Hz (D) 18 Hz
(142) A tuning fork produces 5 beats in 1s with the sonometer wire of length 50 cm. If the length of
the wire is reduced by 2 cm then also the number of beats remains 5. Calculate the frequency of
the tuning fork. (in Hz).
(A) 490 (B) 245 (C) 390 (D) 295
(143) A tuning fork having unknown frequency produces 4 beats in 1s with the fork having 350 Hz
frequency and produces 6 beats in 1s. with the fork having frequency 360 Hz. Find the unknown
frequency.
(A) 354 Hz (B) 346 Hz (C) 366 Hz (D) 358 Hz

213
(144) The frequencies of three sound waves of equal amplitude are (f1–2), f1 and (f1+2) respectively.
If they superpose to produce beats, calculate number of beats produced in 1s.
(A) 1 (B) 2 (C) 3 (D) 4
(145) f1, 1.5f1, 2.25f1, 3.375f1 ...... are the frequencies of the tuning forks in the ascending order. Any
two consecutive forks produces N beats in 1 s. Calculate frequency f1.
(A) N (B) 2N (C) 3N (D) 4N
(146) When two tuning forks are made vibrate they produces 4 beats in 1s. Now a strip is attached
with one of the prongs of fork-2, they produces 6 beats in 1s. Calculate the frequency of fork-2
if the frequency of tuning fork-1 is 200 Hz.
(A) 196 Hz (B) 194 Hz (C) 204 Hz (D) 206 Hz
(147) What would be the difference of angular frequencies in order to hear the beats clearly, in the
case of sound waves ?
(A) > 6p (B) £ 6p (C) > 12 p (D) £ 12p
(148) Using the superposition, for which of the following waves phenomenon of beats can be possible ?
(A) y1 = A1sin w1t and y2 = A2sin w2t (B) y1 = A1sin wt and y2 = A2sin wt
(C) y1 = Asin wt and y2 = Asin wt (D) y1 = A sin w1t and y2 = A sin w2t
(149) Wave equation for two waves propagating the medium and producing beats are; y1 = A sin 2pf1t
and y2 = A sin 2pf2t. What would be the resultant amplitude ?

(A) A' = (B) A' = 2A


A
2

§ f1  f 2 · ª f1  f 2 º
(C) A' = 2A cos 2 p ¨ ¸t (D) A' = 4A2 cos2 2 p « »t
© 2 ¹ ¬ 2 ¼

(150) Two harmonic waves having slightly different frequencies f1 and f2 superpose on each other to
produce beats. The loudness of sound in unit time becomes ......
(A) (f1 – f2) times maximum and (f1 + f2) times minimum.
(B) (f1 – f2) times maximum and (f1 – f2) times minimum.
(C) (f1 + f2) times maximum and (f1 – f2) times minimum.
(D) (f1 + f2) times maximum and (f1 + f2) times minimum.
(151) The ratio of the frequencies of the sound of a car horn heard by a stationary traffic police when
the car is moving towards and away from him is 1.5. If the speed of the sound is 340 ms–1.
Calculate the speed of the car in ms–1.
(A) 68 (B) 78 (C) 48 (D) 58
(152) The frequency of the sound of a car horn experienced by a stationary listener, when car is
moving towards him is 5 % more than its original frequency. If the speed of the sound is
325 ms–1, find the speed of the car in ms–1.
(A) 6 (B) 8 (C) 15 (D) 25

214
(153) Find the difference of apparent frequencies of the sound of a car horn heard by a stationary
listener when the car is moving towards and away from the listener with a speed of 72 kmh–1.
The frequency of the sound emitted by the horn is 1000 Hz, velocity of sound v = 320 ms–1.
(A) 124.8 Hz (B) 142.8 Hz (C) 184.2 Hz (D) 0
(154) Two trains are moving towards a stationary listener with the speed of 72 kmh–1 and 36 kmh–1.
The frequency of the sound of the whistle of both the trains is 200 Hz. Velocity of sound is
320 ms–1. Calculate the number of beats heared by the listener in 1 s.
(A) 4 (B) 5 (C) 7 (D) 8
(155) The whistle of an engine, approaching a hill with the speed of 72 kmh–1 produces sound of
frequency 600 Hz. Find the frequency heared by the driver of the same engine, of the sound of
whistle reflected from the hill. The speed of sound is 320 ms–1.
(A) 680 Hz (B) 700 Hz (C) 780 Hz (D) 860 Hz
(156) The driver of a stationary train at the railway plateform blows the whistle of sound frequency
700 Hz. A person is moving towards the train with the speed of 36 kmh–1. Calculate the
frequency experienced by the person. Velocity of sound v = 350 ms–1.
(A) 640 Hz (B) 720 Hz (C) 780 Hz (D) 820 Hz
(157) A source of sound is moving towards the listener with the speed of 72 kmh–1 and the listener is
moving away from the source with the speed of 36 kmh–1. The source emitts the sound of
frequency 990 Hz with the speed of 350 ms–1. Calculate the frequency experienced by
the listener.
(A) 660 Hz (B) 900 Hz (C) 1020 Hz (D) 1300 Hz
(158) A rickshaw is moving with the speed of 10 ms–1. A loudspeaker on this rickshaw emits the sound
with the speed of 330 ms–1. A car is behind this rickshaw and moving towards the rickshaw with
the speed of 108 kmh–1. Calculate the ratio of the frequency experienced by the driver of the car
to the original frequency of the sound emitted by the loudspeaker.

(A) (B) (C) (D)


36 36 18 18
32 17 34 17

(159) A Radar transmits radio waves of frequency 103 MHz towards an aeroplane. The frequency of
the reflected radio waves observed by Radar is 5 kHz more than the frequency send by it.
Calculate the speed of the aeroplane. Speed of the radio wave is 3×108 ms–1.
(A) 0.5 kms–1 (B) 1 kms–1 (C) 1.5 kms–1 (D) 3 kms–1
(160) A sound of frequency 500 Hz is performing uniform circular motion on the circumference of a
circle of radius 50  cm, with a constant angular speed of 20 rads–1. A person is standing very far
away on the line passing through the centre of this circle and along the plane of the circle. (The
person is stationary). The velocity of the sound is 340 ms–1. Calculate the maximum and minimum
frequency experienced by the person.
(A) 515 Hz and 486 Hz (B) 846 Hz and 515 Hz
(C) 515 Hz and 400 Hz (D) 648 Hz and 515 Hz

215
(161) As shown in the figure, a boy is in between a wall
and a stationary observer. The boy is walking towards
the wall at a speed of 2 ms–1 in a direction at right
angles to the walll. The boy blows a whistle. The
Observer Boy
observer hears 4 beats in 1 s. If the speed of the
sound is 332 ms–1. Calculate the frequency of the (Steady) Wall
whistle.
(A) 150 Hz (B) 200 Hz (C) 330 Hz (D) 440 Hz
(162) A source emitting a sound of frequency f, which is placed at a very large distance from the
listener. The source starts moving towards the listener with a constant acceleration a. Calculate
the frequency experienced by the listener corresponding to the sound emitted just after the source
starts. The speed of the sound is v.

2vf 2 vf 2 2 vf 2 vf 2
(A) (B) (C) (D)
2vf  a 2vf  a 3 vf  a 2 vf  a

(163) A stationary listener experiences the frequency of a sound of the horn of a car moving towards
him with the difference of 10% with the original frequency. Velocity of sound is 330 ms–1.
Compute the velocity of the car in ms–1.
(A) 10 ms–1 (B) 20 ms–1 (C) 30 ms–1 (D) 40 ms–1
(164) A train moving towards a stationary listener with a constant speed of 108 km h–1. The driver of
the train keeps on blowing the whistle continuously. Calculate the ratio of the frequencies heard
by the listener, for the train coming towards him and moving away from him. Velocity of the
sound is 330 ms–1.
(A) 9 : 8 (B) 9 : 5 (C) 6 : 1 (D) 6 : 5
(165) As shown in the figure a train has just completed semicircu-
lar path on a U-shaped railway track. The engine is at one Q
end of the semicircular path while the last coach is at the
other end of the path. The driver blows a whistle of fre-
lùÖë
quency 160 Hz. Compute the apparent frequency heared by R
a passenger in the middle of a train. The velocity of the
sound is 330 ms–1.
P
(A) 160 Hz (B) 200 Hz engine
(C) 80 Hz (D) 320 Hz
Ans. : 132 (C), 133 (A), 134 (D), 135 (B), 136 (A), 137 (B), 138 (B), 139 (A), 140 (B), 141 (B),
142 (B), 143 (A), 144 (B), 145 (B), 146 (A), 147 (D), 148 (D), 149 (C), 150 (B), 151 (A),
152 (C), 153 (A), 154 (C), 155 (A), 156 (B), 157 (C), 158 (D), 159 (C), 160 (A), 161 (C),
162 (A), 163 (C), 164 (D), 165 (A)
Questions based on practicals :
(166) In an experiment of determining the force constant of a spring, dead weight is 100 g. When 100
g mass is suspended, the length of the spring increases by 1 mm and equilibrium is maintained.
Now if the mass is given SHM, Calculate its periodic time. [g = 103 cms–2]
(A) p s (B) 2 p s (C) 0. p s (D) 0. 02 p s

216
(167) In an experiment of simple pendulum, the diameter of the sphere is 1.98 cm. The length of the
pendulum is 50 cm. Calculate the total time taken for 25 oscillations. [g = 980 cms–2]
(A) 38.1 s (B) 35.8 s (C) 53.8 s (D) 13.8 s
(168) In an experiment to determine the force constant of a spring by the method of oscillations, when
the mass 250 g is suspended the periodic time is 0.5 s. What would be the increase in the length
of the spring, when the system is in the equilibrium ? (p2 = 10 and g = 103 cms–2.)
(A) 0.625 cm (B) 6.25 cm (C) 0.625 mm (D) 0.625 m
(169) What would be the slope of l ® T2 graph in an experiment of simple pendulum ?

2S 4S2 g g2
(A) g (B) (C) (D)
g 4S2 4 S2

(170) In an experiment of simple pendulum, what is the necessary angular amplitude so that the motion
of the pendulum can be considerd to be SHM ?
(A) greater than 10° (B) greater than 6° (C) less than 4° (D) in between 6° and 4°
(171) A rubber cork is used as the rigid support in an experiment of simple pendulum. By mistake of
experimenter, the string comes out of the cork from a thin cruck and oscillates from a point
above the lowest end of cork. Then ...... .
(A) T will be more then its actual value. (B) T will be less than its actual value.
(C) T increase, oscillations becomes slow. (D) T increases, oscillations becomes fast.
(172) In an experiment of simple pendulum, the length of the pendulum is taken as 50 cm, 60 cm, 70 cm,
80 cm and 90 cm. The periodic time is measured by taking 20 oscillations for each length. Then with
the increase of length.
(A) periodic time decreases, oscillations become slow
(B) periodic time decreases, oscillations become fast
(C) periodic time increases, oscillations become slow
(D) periodic time increases, oscillations become fast
(173) The length of second pendulum from the graph of l ® T2 is 99.4 cm. The slope of this line
is ...... cms–2.
(A) 980.5 (B) 49.75 (C) 24.85 (D) 100
(174) In an experiment of resonance tube, which one of the following is correct regarding the
frequency of the fork and balancing length ?
(A) length decreases with the increase of frequency
(B) length increases with the increase of frequency
(C) length will not change with the frequency
(D) Initially length increases with increase of frequency and than length decreases
(175) In an experiment of resonance tube, what would be the velocity of the sound wave at 0° C ?
vt vt
(A) v0 = vt (1 + 2 aT) (B) v0 = 1  1 D T (C) v0 = vt + 2 aT2 (D) v0 = 1  1 D T 2
1 1
2 2

217
(176) In an experiment of resonance tube, three readings are taken by using three different frequen-
cies. An average value of measured fl is 82.55 Hz m. Calculate the velocity of the sound at the
given constant temperature.
(A) 3 ×108 ms–1 (B) 165.1 ms–1 (C) 330.2 ms–1 (D) 825.5 ms–1
(177) In an experiment of measuring speed of sound using resonance tube, a student measures length
of tube at first resonance at 12 cm, on winter morning (at comparatively low temperature). When
same experiment is repeated with same tuning fork in summer afternoon (comparatively high
temperature), length for second resonance is found to be x cm. Then which of the followin
goptions is correct ?
(A) 12 > x (B) x > 36 (C) 36 > x > 12 (D) 36 > x > 24
Ans. : 166 (D), 167 (B), 168 (B), 169 (C), 170 (C), 171 (A), 172 (C), 173 (C), 174 (A), 175 (B),
176 (C), 177 (B)

Assertion - Reason type Question :


Instruction : Read assertion and reason carefully, select proper option from given below.
(a) Both assertion and reason are true and reason explains the assertion.
(b) Both assertion and reason are true but reason does not explain the assertion.
(c) Assertion is true but reason is false.
(d) Assertion is false and reason is true.
(178) Assertion : All oscillatory motions are periodic motions but all periodic motions are not oscillatory.
Reason : For small oscillations, motion of the simple pendulum is oscillatory motion.
(A) a (B) b (C) c (D) d
(179) Assertion : The kinetic energy and the mechanical energy of SHO is equal at the end points.
Reason : At the end point, velocity of SHO is zero.
(A) a (B) b (C) c (D) d
(180) Assertion : Acceleration of SHO a = –w y. 2

Reason : Acceleration of SHO is always negative


(A) a (B) b (C) c (D) d
(181) Assertion : At mean position, acceleration of SHO is zero
Reason : At mean position, velocity of SHO is zero
(A) a (B) b (C) c (D) d
(182) Assertion : The mechanical energy of SHO does not depend on its displacement.
Reason : The mechanical energy of SHO E = 1
2
kA2.
(A) a (B) b (C) c (D) d
(183) Assertion : At mean position of SHO, its kinetic energy is equal to its mechanical energy.
Reason : At mean position of SHO, its velocity is zero.
(A) a (B) b (C) c (D) d
(184) Assertion : Oscillations performed by a hard-spring are slow.
Reason : The force constant of hard spring is high.
(A) a (B) b (C) c (D) d

218
(185) Assertion : If the kinetic energy of SHO increases, its potential energy decreases and if its
P. E increases, its K. E. decreases.
Reason : The mechanical energy of SHO remains constant.
(A) a (B) b (C) c (D) d
(186) Assertion : The mechanical energy of SHO does not depend on its maximum displacement
Reason : The maximum displacement of SHO is equal to its amplitude.
(A) a (B) b (C) c (D) d
(187) Assertion : The periodic time of the simple pendulum increases with the increase of mass of
the bob.

Reason : The periodic time of SHO is given by T = 2p k .


m

(A) a (B) b (C) c (D) d


(188) Assertion : On doubling the amplitude of the simple pendulum its period remains the same.
Reason : The period of the simple pendulum is independent of its amplitude.
(A) a (B) b (C) c (D) d
(189) Assertion : The amplitude of the damped oscillations decreases with time.
Reason : There exsi resistive force of air on the oscillator.
(A) a (B) b (C) c (D) d
(190) Assertion : An oscillator can possess more than one natural frequency.
Reason : Natural oscillations are performed in the absanse of all external forces.
(A) a (B) b (C) c (D) d
(191) Assertion : An amplitude of the forced oscillations remains constant.
Reason : No external force acts on the forced oscillator.
(A) a (B) b (C) c (D) d
(192) Assertion : During an earthquake, high-rise structures fall down.
Reason : The frequency of the Seismic waves is very greater than the natural frequency of
high-rise structures.
(A) a (B) b (C) c (D) d
(193) Assertion : Mechanical waves require some elastic medium for their propagation.
Reason : Mechanical waves Propogate due to an elastic property of the medium.
(A) a (B) b (C) c (D) d
(194) Assertion : Electromagnetic waves space require any medium and even propagate in the
free-space.
Reason : Electromagnetic waves do not propagate in the medium.
(A) a (B) b (C) c (D) d

219
(195) Assertion : Longitudinal waves are also called pressure - waves.
Reason : The pressure of the different regions change with time, during the propagation of
longitudinal waves.
(A) a (B) b (C) c (D) d
(196) Assertion : Longitudinal waves can propagate in the solid-medium.
Reason : Solid-medium can posses shearing strain.
(A) a (B) b (C) c (D) d
(197) Assertion : The origin of earthquake (epicentre) can be determined by using seismograph.
Reason : Both transverse and longitudinal waves are produced during an earthquake.
(A) a (B) b (C) c (D) d
(198) Assertion : Wave equation represents the displacement of a particle at a distance x from the origin.
Reason : The time derivative of wave equation represents velocity of the wave.
(A) a (B) b (C) c (D) d
(199) Assertion : When a wave changes its medium, its wave length remains constant.
Reason : The wavelength is a property of the medium.
(A) a (B) b (C) c (D) d
(200) Assertion : When a wave changes its medium, its frequency remains constant but its wave
length does not.
Reason : Frequency is the Property of the source, and wavelength is a Property of the
medium.
(A) a (B) b (C) c (D) d
(201) Assertion : Speed of the transverse wave Propagating on stretched string does not depend on
frequency and amplitude of the wave.
Reason : Elasticity and inertia of medium are necessary for the propagation of the
mechanical waves.
(A) a (B) b (C) c (D) d
(202) Assertion : The phenomenon of propagation of sound in air is adiabatic.
Reason : Isothermal bulk modulus is equal to the Pressure of the air.
(A) a (B) b (C) c (D) d
(203) Assertion : When “crest” of the wave is incident at a rigid support, it becomes “trough” due to
the reflection from the rigid support.
Reason : The Phase of the wave increases by p rad when it reflects from the rigid support.
(A) a (B) b (C) c (D) d
(204) Assertion : Energy does not Propagate in the Stationary wave.
Reason : Stationary wave is not Prograssive.
(A) a (B) b (C) c (D) d
(205) Assertion : For closed-pipe fn represents (n–1)th Overtone.
Reason : All the harmonics are Possible for closed-pipe.
(A) a (B) b (C) c (D) d

220
(206) Assertion : During the Phenomenon of beats, the loudness of sound becomes 2 (f1 – f2) times
maximum.
Reason : The number of beats in unit time is (f1 – f2).
(A) a (B) b (C) c (D) d
(207) Assertion : In Doppler effect, the wavelength of sound waves in the front of the source
decreases while behind the source, its wavelength increases.
Reason : There is relative displacement between the source of sound and wave.
(A) a (B) b (C) c (D) d
Ans. : 178 (B), 179 (D), 180 (C), 181 (C), 182 (a), 183 (C), 184 (B), 185 (A), 186 (B), 187 (D),
188 (A), 189 (A), 190 (B), 191 (C), 192 (D), 193 (A), 194 (C), 195 (A), 196 (A), 197 (B),
198 (C), 199 (D), 200 (A), 201 (B), 202 (C), 203 (A), 204 (B), 205 (C), 206 (D), 207 (A)

Comprehension Type Questions :

(208) A particle perfoms SHM along the path of length 20 cm. Initially it is at the mid point of its mean
position and positive end, and start moving towards the mean position. It completes 2.5
Oscillations in 8 s.
(1) Its amplitude A = ...... cm
(i) 20 (ii) 10 (iii) 5 (iv) 40
(2) Its initial phase f = ...... rad
S 5S 7S 11S
(i) 6 (ii) 6 (iii) 6 (iv) 6

(3) Its phase at the end of 2.5 oscillation q = ...... rad.


35 S S S S
(i) (ii) 25 (iii) 45 (iv) 15
6 6 6 6
(4) Its periodic time T = ...... s.
(i) 1.6 (ii) 0.8 (iii) 3.2 (iv) 4.8
(A) 1 (i) 2 (ii) 3 (iii) 4 (iii) (B) 1 (ii) 2 (ii) 3 (iii) 4 (iii)
(C) 1 (ii) 2 (ii) 3 (i) 4 (iii) (D) 1 (ii) 2 (ii) 3 (iv) 4 (iv)
2S
(209) An amplitude and periodic time of SHO are 10 cm and 3
s respectively :

(1) Its velocity at its mean position v = ...... cms–1


(i) 0 (ii) 10 (iii) 20 (iv) 30
(2) Its acceleration at its mean position a = ...... cms–2
(i) 0 (ii) 30 (iii) 60 (iv) 90
(3) Its velocity at the positive end v = ...... cms–1
(i) 0 (ii) 10 (iii) 20 (iv) 30
(4) Its acceleration at the negative end a = ...... cms–2
(i) 0 (ii) 30 (iii) 60 (iv) 90
(A) 1 (i) 2 (i) 3 (i) 4 (i) (B) 1 (iv) 2 (i) 3 (i) 4 (iv)
(C) 1 (iv) 2 (iv) 3 (iv) 4 (iv) (D) 1 (iv) 2 (i) 3 (iv) 4 (i)

221
(210) The mechanical energy of SHO is twice its kinetic energy.
(1) What would be its displacement ?
(i) y = ± (ii) y = ± (iii) y = ± (iv) y = 0
A A 3A
2 2 2
(2) What would be its velocity ?
vmax
(i) v = 0 (ii) v = vmax (iii) v = (iv) v =
vmax
2 2
(3) What would be its acceleration ?
amax
(i) a = 0 (ii) a = amax (iii) a = (iv) a =
amax
2 2
(4) What would be its potential energy ?
E
(i) U = 0 (ii) U = (iii) U = (iv) U = 2E
E
2 2
(A) 1 (i) 2 (iii) 3 (i) 4 (iii) (B) 1 (ii) 2 (iii) 3 (iii) 4 (iv)
(C) 1 (ii) 2 (iv) 3 (iii) 4 (iii) (D) 1 (ii) 2 (iii) 3 (iii) 4 (ii)
S
(211) The wave equation for a prograssive harmonic wave is y = 10 sin (4pt – x) cm.
5
(Where x and y are in cm, t is in s)
(1) The wave length of the wave l = ...... cm ?
(i) 10 (ii) 5 (iii) 20 (iv) 30
(2) What would be the frequency of the wave ?
(i) 0.5 (ii) 2 (iii) 20 (iv) 50
(3) What would be the wave-vector in rad cm ?–1

2S 3S S 2S
(i) 5 (ii) 5 (iii) 5 (iv) 3
(4) What would be the velocity of the wave in cms–1 ?
(i) 10 (ii) 20 (iii) 15 (iv) 30
(A) 1 (i) 2 (ii), 3 (iii) 4 (iv) (B) 1 (i) 2 (iii) 3 (ii) 4 (ii)
(C) 1 (ii) 2 (ii) 3 (iii) 4 (iv) (D) 1 (i) 2 (ii) 3 (iii) 4 (ii)
S
(212) The wave equation for a progressive harmonic wave is y = 10 sin (2pt – 8
x)
[Where x and y are in cm and t is in s.]
(1) What would be the velocity of the wave in cms–1 ?
(i) 0 (ii) 8 (iii) 16 (iv) 32
(2) What would be the displacement of a particle at a distance 4 cm away from the origin at the
end of 1 s ?
(i) 10 cm (ii) –10 cm (iii) 4 cm (iv) 5 cm
(3) What would be the velocity of a particle at a distance 16 cm away from the origin at the end
of 1 s. [in cms–1]
(i) 0 (ii) 10 p (iii) 20 p (iv) 30 p
(4) What would be the acceleration (in cms ) of the particle in the question 3.
–2

(i) 0 (ii) 20 p (iii) 40 p2 (iv) –40 p2


(A) 1 (iii) 2 (ii) 3 (iii) 4 (i) (B) 1 (iii) 2 (i) 3 (iii) 4 (iv)
(C) 1 (ii) 2 (iii) 3 (iv) 4 (iv) (D) 1 (ii) 2 (ii) 3 (ii) 4 (i)

222
(213) The stationary waves produced in a 20 cm long string fixed at both the ends with rigid support

S
are represented by y = 20 sin 4 x cos 80 S t . (Where x and y are in cm at t is in s)

(1) Wavelength of the wave in cm is ......


(i) 8 cm (ii) 2 cm (iii) 20 cm (iv) 5 cm
(2) Velocity of the wave in cms–1 is ...... .
(i) 20 cms–1 (ii) 80 cms–1 (iii) 160 cms–1 (iv) 320 cms–1
(3) The positions of nodes from x=0 (in cm) are ...... .
(i) 1, 5, 9, 13 (ii) 4, 8, 12, 16 (iii) 2, 6, 10, 14, 18 (iv) 3, 7, 11, 15
(4) The positions of antinodes from x = 0 (in cm) are.
(i) 1, 5, 9, 13 (ii) 4, 8, 12, 16 (iii) 2, 6, 10, 14, 18 (iv) 3, 7, 11, 15
(A) 1 (i) 2 (iv) 3 (ii) 4 (iii) (B) 1 (i) 2 (i) 3 (iii) 4 (ii)
(C) 1 (i) 2 (iv) 3 (iii) 4 (ii) (D) 1 (iii) 2 (iii) 3 (ii) 4 (iii)
Ans. : 208 (C), 209 (B), 210 (D), 211 (D), 212 (A), 213 (A)

Matching Column Type :

(214) Column-1 represents time in terms of periodic time T and Column-2 represents phase at that
time. Correctly match the columns. (f= 0).

Column-1 Column-2

a t= 8 (i) q=p (A) a (iii), b (i), c (iv), d (ii)


T

S
b t= (ii) q=5 (B) a (iii), b (i) c (ii) d (iv)
T
2 4

S
c t=5 8 (iii) q= 4 (C) a (ii), b (iii), c (i), d (iv)
T

3S
d t=3 (iv) q= (D) a (iv), b (i) c (ii) d (iii)
T
4 2

(215) The SHO is given 100 J energy to perform SHM. Values of kinetic energy and potential energy are
given in column 1 and columns 2 respectively. Match them :

Column-1 Column-2
a K=0 (i) U = 40 J (A) a (iv), b (ii), c (i), d (iii)

b K = 50 J (ii) U = 90 J (B) a (ii), b (iii) c (i) d (iv)

c K = 10 J (iii) U = 50 J (C) a (iv), b (iii), c (i), d (ii)

d K = 60 J (iv) U = 100 J (D) a (iv), b (iii) c (ii) d (i)

223
Match the columns :

(216) Match the column-1 (DF) with column-2 (physical quantity) :


Column-1 Column-2

a M1L0T–1 (i) Wave intensity (A) a (iii), b (iv), c (i), d (ii)

b M1L0T–2 (ii) damping force (B) a (ii), b (iv), c (i), d (iii)

c M1L0T–3 (iii) damping coefficients (C) a (iii), b (iv), c (ii), d (i)


d M1L1T–2 (iv) force constant of spring (D) a (iii), b (ii), c (i), d (iv)
(217) For simple pendulum, graph of Y-axis ® X-axis is given in column 1. In column 2, shape of graph
is given. Match them.
Column-1 Column-2
a T2 ® l (i) Straightline (A) a (i), b (ii), c (iii), d (iv)
b T2 ® g (ii) Straightline (B) a (ii), b (iv), c (iii), d (i)
c T® l (iii) Paraboba (C) a (iii), b (i), c (ii), d (iv)

d T® l (iv) Hyperboly (D) a (iv), b (ii), c (iii), d (i)

(218) Match the velocity and acceleration of SHO in column-2 with its displacement in column-1.
Column-1 Column-2

a y=± (i) v= (A) a (ii), b (iv), c (i), d (iii)


A vmax
2 2

b y=± (ii) a= (B) a (iv), b (iii), c (i), d (ii)


2A amax
3 2

vmax
c y= ± (iii) v= (C) a (ii), b (iv), c (iii), d (i)
A
2 2

2 amax
d y= ± (iv) a= (D) a (i), b (iii), c (ii), d (iv)
3A
2 3

(219) Correctly Match the values of kinetic energy of SHO with its displacement in column-1.
Column-1 Column-2

a y= (i) K= 4 (A) a (iv), b (iii), c (i), d (ii)


A 3E
2

b y= (ii) K= (B) a (iv), b (iii), c (ii), d (i)


A E
3 4

c y= (iii) K= (C) a (iv), b (i), c (ii), d (iii)


A 2E
2 3

d y= (iv) K= 2 (D) a (iv), b (i), c (iii), d (ii)


3A E
2

224
(220) Match the column-1 and column-2 for SHO :
Column-1 Column-2
a At mean position (y = 0) (i) K increasese, U decreases.
b negative end (y = –A) (ii) K decreases, U increases.
c moving from m. p. towards positive end (iii) U = E and K = 0
d moving from negative end toward m.p. (iv) K = E and U = 0

(A) a (iv), b (iii), c (i), d (ii) (B) a (iii), b (iv), c (i), d (ii)
(C) a (iv), b (iii), c (ii), d (i) (D) a (iii), b (iv), c (ii), d (i)
(221) Correctly match the characteristic of the wave in column-2 with the given wave in column-1.
Column-1 Column-2
a Sound waves (i) Nonmechanical and transverse
b light waves (ii) mechanical, transverse and longitudinal
c seismic waves (iii) mechanical and transverse
d waves on the string (iv) mechanical and longitudinal

(A) a (iv), b (iii), c (i), d (ii) (B) a (i), b (iv), c (ii), d (iii)
(C) a (i), b (iv), c (iii), d (ii) (D) a (iv), b (i), c (ii), d (iii)
(222) Correctly match the Dimensional formula in column-2 with the physical quantity given in column-1.
Column-1 Column-2
a Wave vector (i) M1L0T–3
b mass density (ii) M1L–1T–2
c Elastic constant (iii) M0L–1T0
d Intensity of wave (iv) M1L–1T0

(A) a (iii), b (iv), c (i), d (ii) (B) a (iii), b (iv), c (ii), d (i)
(C) a (iv), b (iii), c (ii), d (i) (D) a (iii), b (i), c (iv), d (ii)
(223) Correctly match the frequency given in column-2 Corresponding to various harmonic or overtone
for clarinet given in column-1 :
Column-1 Column-2
a Second harmonic (i) f2 = 3 f1
b Seccond overtone (ii) f3 = 5 f1
c Third harmonic (iii) f5 = 7 f1
d Third overtone (iv) Not possible

(A) a (iv), b (ii), c (iii), d (i) (B) a (i), b (ii), c (iii), d (iv)
(C) a (iv), b (i), c (ii), d (iii) (D) a (iv), b (ii), c (i), d (iii)

225
(224) Correctly match the amplitude given in column-1 with column-2.
Column-1 Column-2

§ f1  f 2 ·
a Amplitude of damped oscillation (i) 2A sin 2p ¨ ¸ t
© 2 ¹
b Amplitude of forced oscillation (ii) A

c Amplitude of Stationary waves (iii) A


– bt
e 2m
d resultant amplitude in (iv) 2A sin kx
phenomenon of Beast

(A) a (iii), b (ii), c (i), d (iv) (B) a (iii), b (ii), c (iv), d (i)
(C) a (ii), b (iii), c (i), d (iv) (D) a (ii), b (iii), c (iv), d (i)
(225) The various relative motion between the source of sound and the listener is given in column-1.
The frequency experienced by the listner is given in column-2. Correctly match the columns.

Column-1 Column-2

§ v  vL ·
a The source and the listener are moving (i) fL = ¨ v  v ¸ fs
© s ¹

towards each other.

§ v – vL ·
b Both are moving in the opposite (ii) fL = ¨ v – v ¸ fs
© s ¹

direction.

§ v  vL ·
c Source is moving towads the (iii) fL = ¨ v – v ¸ fs
© s ¹

listener and the listener is moving


away from the source.
§ v – vL ·
d listener is moving towards the source (iv) fL = ¨ v  v ¸ fs
© s ¹

and the source is moving away from


the listener.

(A) a (iii), b (iv), c (ii), d (i) (B) a (iii), b (iv) c (i) d (ii)
(C) a (iv), b (iii), c (ii), d (i) (D) a (iv), b (iii) c (i) d (ii)

Ans. : 214 (B), 215 (D), 216 (A), 217 (B), 218 (C), 219 (A), 220 (C), 221 (D), 222 (B), 223 (D),
224 (B), 225 (A)

226
3 Current Electricity
Electric Current

Electric Charge Q ne nev


l Electric Current  Ÿ I nef
time t t 2S r
In SI system, unit of electric current is Ampere (A).
Dimensional formula of electric current is M o Lo T o A1 or M o Lo T –1Q1 .

1mA 103 A ; 1P A 106 A

Electric Current, I
dQ
, ? dQ I dt
dt

l Electric charge Q ³ dQ ³ I dt
(1) An Electron revolves in circular orbit of radius 5.3. u1011 m in hydrogen atom with constant
speed of so that 1.06 mA current is constituted.

(A) 2u106 ms 1 (B) 1.1 u 106 ms 1 (C) 2.2 u 106 ms 1 (D) 1.5 u 106 ms 1

(2) 9 mA electric current is flowing through a conducting wire then number of electrons passing
through it in 3min is .
(A) 2 u 1018 (B) 1 u 1018 (C) 2 u 1019 (D) 1.01 u 1019

(3) A current through a wire varies with time as I I 0  D t where I 0 100 A and D 8 As 1 . Then
the charge flows across a cross section of the wire in first 20 sec .
(A) 2000 (B) 3600 (C) 1600 (D) 400

h2
(4) An electron in the hydrogen atom is revolving around nucleus in the orbit of radius
16S 2 me 2

4S e2
with speed . Then the equation of electric current will be . Here
h
m = mass of Electron, e = Charge of electron.

4S 2 me5 4S 2 me3 32S 2 me5 32S me3


(A) (B) (C) (D)
h3 h5 h3 h5

(5) The current flowing through wire changes with time as I (3  2t ) then the electric charge
flowing through any cross-section of wire in time t 0 s to t 4s will be C
(A) 20 (B) 24 (C) 28 (D) 14

Ans. : 1 (C), 2 (D), 3 (B), 4 (C), 5 (C)

296
l Drift velocity and Mobility
'I
Electric current density, J
I
'a Cos T A

Drift velocity of electron in conductor, ( v d )


Effective displacement of electron
Time Interval

a.W eE Ÿ a
eE
vd but F ma
m

ª eE º
? vd « m » W , where, W Relaxation time of electron
¬ ¼

,
l Electric current density, J nevd
A
VE nevd

1 ª Ee º ª 1º
E ne « W» «' V U »¼
U ¬m ¼ ¬

? Resistivity, U
m
ne2W

V
l Mobility for material, and conductivity V
vd
ne
E ne

SI unit of mobility is m 2 v 1 s 1 .

(6) An electron covers 4 u 104 m distance in presence of electric field and 104 m distance in
absence of electric field. Then it’s drift velocity is . Electric field is applied for
10 s .

(A) 3 u105 ms1 (B) 4 u103 ms1 (C) 2 u105 ms 1 (D) 3 u104 ms1

(7) 10A electric current is flowing through the copper wire having cross sectional area of 6 mm .
2

The drift velocity of electron flowing through this wire is .


M cu 63.5 kg/kmol , Density of copper 8920 kg m –3

(A) 1.2 u 103 ms 1 (B) 1.2 u 103 ms 1 (C) 1.2 u 104 ms 1 (D) 0.12 ms 1

(8) An electric current density of 5Am2 is obtained when 8 u108 Vm1 electric field is applied to
conducting wire, then resistivity of conductor is .
(A) 1.6 u 108 : m (B) 2 u 108 : m (C) 16 u 105 : m (D) 20 : m

(9) 6 u 1012 electrons are flowing through any cross-section of conductor per unit time with drift
velocity 8 u 1010 ms1 . If cross-sectional area of conductor is 4 cm 2 then electric current flowing
through conductor is A.
(A) 307.2 (B) 30.72 (C) 3.072 (D) 6.015

297
(10) In two conducting wires of same material, same current is flowing. If ratio of radii of wires is 1:4
and drift velocity of electron in thick wire is vd then drift velocity of electron in thin wire is
.

(A) (B) (C) 16 vd (D) 4 v d


vd vd
16 4

(11) The relaxation time for collision of electron with proton in a conducting wire is 18.2 u 1012 s ,
then mobility of conductor is .
(A) 16 Cs kg –1 (B) 1.6 Cs kg –1 (C) 1.6 u 102 Cs kg –1 (D) 3.2 Cs kg –1

(12) In conducting wire having electron density 8 u 1012 , the average time interval between two

successive collision for electron with ions is 4 u 1012 s and mass of ions is 2.56 u 1027 kg , then
the resistivity of that conducting wire is :m .

(A) 31.25 u 1010 (B) 0.31 u 1010 (C) 0.66 u 1010 (D) 0.36 u 1010

(13) Area of the plane shown in figure is 2cm2 . 4A current is flowing through the wire, then the
electric current density at point P of conductor will be.

'a
G
(A) 4 3 u 104 Am 2
4
(B) u 104 Am 2
60° 3
30°
P (C) 4 u 104 Am2 (D) 2 u 104 Am2

(14) Two conductors having same diameter have resistivities U1 and U 2 , and lengths l1 and l2 . Then
resultant resistivity of series combination of these two conductor is .
U1l1  U 2l2 U1l2  U 2 l1 U1l2  U 2 l1 U1l1  U 2l2
(A) l1  l2 (B) l1  l2 (C) l1  l2 (D) l1  l2

(15) The current density along the axis of a cylindrical conductor having radius equal to R is given

by J . Then the current along the conductor is . The distance from the
r3
J0
R4

axis is r.

S J0 R 2 2S J 0 R S J0 R 2 S 2 J0 R
(A) (B) (C) (D)
2 5 5 2
(16) 0.4 mm diameter copper wire is connected with 2 mm diameter iron wire and 4 mA current is
passed through copper wire then current density in iron wire will be .
(A) 1.27 u 103 Am2 (B) 2 u 105 Am2 (C) 1.5 u 106 Am2 (D) 3 u 103 Am2

Ans. :6 (A), 7 (C) , 8 (A), 9 (B), 10 (C), 11 (D) 12 (B), 13 (C), 14 (A), 15 (B), 16 (A)

298
V
Ohm’s law : R ŸV IR.
I
l Electrical Resistivity and conductivity

Resistance R U , where U Resistivity of material of wire


l
A

I Ul
V IR J U l , but V El
A
?E J U

Conductivity, V unit of conductivity is m 1 or siemen m1


1
U

Limitations of ohm’s law


(i) In practice, for some devices, V – I relationship are nonlinear e.g. diode, transistor

V
(ii) Relation of V and I depends on sign of V.
I
(mA)

O
–V V

$
I

(iii) The relation between V o I is not unique. i.e. there is more than one value of V for same
current (I).
I

V1 V2 V3 V

(17) By stretching uniformly, the length of copper wire is increased by n times or it’s radius is made
1
times then it’s resistance will be increased by times.
n
(A) n (B) n2 (C) n 3 (D) n4
299
(18) The electric current is flowing through two parallel wires having same material. If the ratio of

lengths and radii are 86 and 64 then the ratio of currents flowing through wires is .

(A) 3 (B) 13 (C) 89 (D) 2

(19) The material of resistivity U is filled between two concentric spherical shells of radil a and b
(b ! a ) then the resistance of space between these two spherical shells is .

U U ª1 1 º U ª1 1º U §1 1·
  2» ¨  ¸
4S «¬ b a »¼ «
(A) 4S (a  b) (B) (C) (D)
4S ¬ a 2
b ¼ 4S © a b ¹

(20) The dimensions of block is 6 cm u 4 cm u 2 cm . The ratio of maximum and minimum resistances
obtained between mutually opposite sides is .
(A) 9 : 1 (B) 1 : 9 (C) 1 : 6 (D) 1 : 18
(21) The resistance per unit length of wire is 6 : . If the wire is bent to form a circle of radius
12 cm then the resistance between two diametrically opposite points will be .
(A) 0.72 S : (B) 0.36 S : (C) 0.24 S : (D) 1.44 S :

(22) The charge flowing through each resistance changes with time as Q D t  E t 2 , then current
through resistance R will be maximum at time .

D D2 D3
(A) (B) (C) (D)
2E
2E D 2E 2 E

(23) The conducting cylinder of length l have inner and outer radii are r1 and r2 . If resistivity of material
of cylinder is U then the resistance between inner and outer wall at cylinder is .

U l ª r1 º ª r1 º ª r2 º Ul
(A) 2S ln « r » (B) 2S l ln « r » (C) 2S l ln « r » r2  r1
U U
(D)
¬ 2¼ ¬ 2¼ ¬ 1¼ 2S

(24) A wire of resistance 8 : is bent from the center by 180° then it’s ends are joined and is twisted,
So it’s resistance becomes .
(A) 8 : (B) 2 : (C) 4 : (D) 1 :
(25) A copper wire is stretched to make it longer by 0.1 %. The percentage increase in its resistance
is .
(A) 0.1 % (B) 0.4 % (C) 0.2 % (D) 2 %
(26) When P.d. across two terminal of copper wire is increased the current flowing throgh it also
increases. Then for n electric charge flowing through wire per unit volume and drift velocity Vd
of electric charge, which of the fallowing statement is true ?
(A) n constant and v d decreases (B) n constant and v d increases.

(C) n increases and v d decreases (D) n increases andõ v d remains constant

300
(27) Masses of three wires of copper are in the ratio 2 : 3 : 5 and their lengths are in ratio 3 : 5 : 7,
then the ratio of their resistance is .
(A) 20 : 30 : 50 (B) 125 : 15 : 1 (C) 3 : 5 : 7 (D) 135 : 250 : 294

Ans. : 17 (B), 18 (B), 19 (B), 20 (A), 21 (B), 22 (A), 23 (C), 24 (B), 25 (C), 26 (B), 27 (D)

Colour code for carbon Resistors :

First Second Band Third Band Fourth Band


Tenth Digit Ones Digit Multiple Tolerence

Black 0 0 10ü –
Brown 1 1 101 +1%

Red 2 2 102 +2%

Orange 3 3 103 +3%

Yellow 4 4 104 +4%

Green 5 5 105 –

Blue 6 6 106 –

Violet 7 7 107 –

Gray 8 8 108 –

White 9 9 109 –

Gold – – 10-1 +5%

Silver – – 10-2 + 10 %

None – – – + 20 %

To remember colour code


B B R O Y Goes to Bombay Via Gwalior.
l Temperature dependence of resistivity
The empirical formula for resistivity and temperature

UT UT 0 >1  D (T  T 0 ) @ , where D temperature co-efficient of resistivity

UT Resistivity at a temperature q

UT0 Resistivity at proper reference temperature T 0

The above equation can be written in the form of resistance as follows

Rt  R o
R T0 >1  D (T  T 0 ) @ . Temerature co-efficient of Resistivity, D
Ro u t
RT

301
(28) Resistance of copper wire is 5 : at 50 °C temperature and 6 : at 100 °C temperature then
resistance of wire at 0qC temperature will be :.
(A) 1 (B) 2 (C) 3 (D) 4
(29) At temperature, the resistance of copper wire will be four times than it’s
resistance at 27 °C temperature. For copper D 4 u 103 C1 .
(A) 354 °C (B) 758 °C (C) 1023 °C (D) 1516 °C
(30) For a wire resistance at temperature 300 C is 3.1 : and at temperature 100q C is 4.5 : then
temperature co-efficient of resistivity of wire, D .
(A) 0.0012 qC1 (B) 0.0024 qC1 (C) 0.0032 qC1 (D) 0.008 qC1
(31) An electric, toaster have resistive wire of Nicrom. When small current is passed through it at
room temperature ( 27 qC ), it’s resistance is obtained 75.3 : . When toaster is connected with
230 V supply, the current flowing through it is 2.68 A . If from Nicrom, D 1.7 u 104 qC1 then
it’s final temperature will be .
(A) 747 qC (B) 847 qC (C) 897 qC (D) 927 qC
(32) In a platinum resistance thermometer, the resistance of platinum at 0 qC is 5 : and at 100 q C
is 5.23 : . When thermometer is placed in a heatbath, the resistance of platinum is obtained
5.795 : then temperature of heat bath is .
(A) 278 qC (B) 346 qC (C) 372 q C (D) 412 q C
(33) The resistance of tungsten wire of bulb have resistance 18: at 27 q C temperature. Steady
current of 0.25A is flowing through it when bulb is connected with supply of 45 V . For tungsten,
D 4.5 u 103 K 1 then find the temperature of Bulb filament. Suppose Ohm’s law is obeyed.
(A) 2160 K (B) 1800 K (C) 2070 K (D) 2300 K
(34) Two materials have the value of D1 and D 2 as 5 u 104 qC1 and 3.8 u 104 q C1 respectively.
The resistivity of first material U20 2.4 u 108 : m . If new material is made by combining these
two materials such that it’s resistivity does not change with temperature, then what should be the
resistivity U20 of second material ? Take reference temperature 200 C . Assume that the
resistivity at the new material is equal to the sum of resistivity of component materials.
(A) 3.185 u 106 :m (B) 3.158 u 109 :m (C) 3.185 u 108 :m (D) 3.158 u 108 :m
(35) As per colour code of carbon resistor, the resistance for the colour of Indian National flag, from
upper to lower colours in order, is :.
(A) 39 u 105 r 20 % (B) 59 u 105 r 20 % (C) 39 u 105 r 10 % (D) 39 u 105 r 5 %
(36) The value of a carbon resistor is 1760 : to 2640 : , then colour code at that carbon resistor is

(A) Brown, Red, Brown, No colour (B) Red, Red, Black, No colour
(C) Red, Black, Red, No colour (D) Red, Red, Red, No colour

Ans. :28 (D), 29 (B), 30 (D), 31 (B), 32 (B), 33 (D), 34 (D), 35 (A), 36 (D)

302
l Electromotive force (emf) and terminal voltage of electric cells.
The relation between terminal voltege and emf of cell, V  Ir
Where, r internal resistance of electric cell.

³ F < dl
o o
Electric Force, Fe eE , work W n

Non electrical force Fn

When Fn Fe then electric current flowing through the battery becomes zero I o.
which is called Open circuit condition.

V
l Charging current for lead Accumulator, I Rr

l Electrical energy consumed in resistance I 2 Rt


l Electrical energy consumed in D.C.source = VIt
l Energy consumed to charge electric cell = EIt

(37) An emf of battery is . It’s terminal voltage is obtained V on connecting resistance R : with it
then it’s internal resistance is .

5 § 9 ·
(A) r
V
R (B) r ¨ V ¸ R
© ¹
(C) r  9 5 (D) r
V
R

(38) By an electric cell, the electric current is passed through resistance R1 for time t . Now by the
same cell, the electric current is passed through resistance R2 for same time. If in both the
cases Joule heat produced is same then internal resistance of electric cell is .

R1  R2 R1  R2
(A) (B) (C) R1 u R 2 (D) R1 < R 2
2 2
(39) Electromotive force of electric cell is .
(A) Electric force (B) Non electric force (C) Energy (D) Electromagnetic force
(40) When electric cell is in open circuit condition then .
(A) r 0 (B)  (C) V (D) Fn  Fe

(41) Two batteries having same emf of 2 V and same internal resistance of 1 : are connected in
series with external resistance R, then maximum power in R will be .

(A) 3.2 W (B) (C) 5 W (D) 2W


16
W
9

(42) An electric cell have emf 2.2 V. It’s terminal voltage obtained 1.8 V on connecting 5 :
resistance with it then internal resistance of cell will be .

(A) : (B) : (C) : (D) :


10 9 9 5
9 10 5 9

303
(43) Electric current of 0.75 A is obtained on connecting 4 : resistance across electrodes of electric
cell. But when 10 : resistance is connected, terminal voltage obtained is 3.75 V, then electric
current flowing through cell is A at that time.
(A) 0.25 (B) 0.34 (C) 0.50 (D) 1
(44) 0.9 A current is obtained on connecting 2.4 : resistance with electric cell. On connecting 7 :
resistance with the same cell, 0.3 A current flows then it’s internal resistance is :.
(A) 0.4 (B) 0.5 (C) 0.3 (D) 0.2
(45) Two electric cells have same emf and internal resistances r1 and r2 . These two electric cell
are connected in series with external resistance R. The electric potential difference across two
terminal of first electric cell is obtained zero then value of resistance R is .
r1  r2
(A) r1 r2 (B) r1  r2 (C) r1  r2 (D)
2

Ans. : 37 (A), 38 (D), 39 (C), 40 (C), 41 (D), 42 (A), 43 (B), 44 (B), 45 (C)

Kirchoff’s Rules :

(i) Kirchoff’s first rule ® ¦I 0

(ii) Kirchoff’s second rule ® ¦ IR ¦


(a) If our journey through the resistor is in the direction of flow of current which is
arbitrarily choosen, IR should be considered negative and if our journey is in the
opposite direction of electric current then take IR positive.
(b) The emf of the battery should be considered negative while moving from positive
terminal to negative terminal through battery but if Journey is from negative terminal
to positive terminal through battery then emf should be considered positive.
(iii) Equivalant resistance for series combination of resistances, R R1  R 2  ..........................  R n

(iv) Equivalant resistance for parallel combination of resistances

  .......................... 
1 1 1 1
R R1 R 2 Rn

(46) Six resistances each having resistance of 10 : are connected as shown in the figure. The equivalent
Q resistance between points P and R will be .

S T (A) 20 : (B) 80 :
3

(C) 80 : (D) 80 :
P R
11

304
(47) In the given electrical circuit, the value of unknown resistance R should be :,
so that resultant resistance between P and Q is also R.

(A) 3 (B) 39
10:
P Q (C) 69 (D) 10
3:

3: R

(48) When the switch is on in the following circuit, the electric current flowing through it is
.
2: 4:
20V 5V
(A) 6 A (B) 4.5 A
2:
(C) 3 A (D) 0 A

(49) When 7A current is flowing in electric circuit then the electric potential difference between B
B and C will be volt.

4: 2: (A) 16 (B) 10
I1 I1
A C (C) 8 (D) 5
7A 7A
I2 10: 5: I2

(50) In the following circuit, the effective resistance between P and Q is .

A B
(A) 8 : (B) 8
3
:

(C) 24 : (D) 4 :
P O Q

D C
Resistance of each wire is 8 :

(51) A cube is made by connecting 12 wires each of resistance 12 : . The equivalent resistance
between two end points of any diagonal of cube will be .
(A) 6 : (B) 5 : (C) 10 : (D) 12 :

305
(52) Five resistances are connected as shown in figure. On the dotted line shown in figure two
resistances of 2 : are connected. The ratio of the resistance between A and B in the circuit
before and after joining resistances on dotted line is .
C B
(A) 75 (B) 35
2: 2:
2: 2: 2:
(C) 53 (D) 65

A D
(53) The equivalent resistance between points A and B will be in the given circuit.

R R R

R R (A) R (B)
3R
4
A R R R B
R
R (C) (D) 2R
R
R 2
R R
R

1
(54) A circular wire of radius 8 cm have resistance per unit length. Battery of 10 V is connected
S
between points A and B on circular wire then current flowing through the battery is .
Points A and B form right angle at center O.

(A) 3 A (B) 5 A
O B
(C) 3.33 A (D) 10 A

A
10V
I

(55) Nine resistances each with value R are connected as shown in figure. Then equivalent resistance
between A and B will be .

A
(A) (B) R :
7
R:
6

(C) R: (D) R:
3 2
5 9

B C

306
(56) The equivalent resistance between points A and B in the given circuit is .
C D
(A) 50 :
10:
(B) :
60
11
10: 10: 10:
10:
B (C) (D) 60 :
10: 80
:
A
E 7
10:

(57) In a given circuit, resistance of each resistor is r. Then equivalent resistance between A and B is
r r
r
r r (A) 34 r (B) 23 r
A

r r r (C) 8
15
r (D) 8
7
r

(58) Seven resistances each of 10 : are connected with 2 V battery as shown in the figure, then current
flowing through ammeter will be .
E F J

(A) 2 A (B) 1 A
2V 10 : 10: : 10: 10: 10: (C) 0.4 A (D) 0.8 A
:
10
10

H A K
G

(59) Thirteen resistors each of resistance R : are connected in the circuit as shown in the figure.
The effective resistance between A and B is .
C

(A) 2R : (B) :
4R
3
A R B
(C) (D) R :
2R
:
3

D
(60) The effective resistance between points A and B in the given network shown in the figure will be
.
G
r r
F
r r (A) r : (B) 2r :
r
A r O B
r
(C) 4r : (D) :
r
r
C E
2

r r
D
307
(61) The effective resistance between points A and B in given adjacent circuit will be
A
10: 10:
(A) 10 : (B) 5 :
10: 10: 10:
(C) 20 : (D) 14 :
B
10: 10:

(62) The equivalent resistance of given network is .

2: 2: 2: 2: 2: 2: 2: 2:

A B

4: 4: 4: 4: 4: 4: 4:
4:

(A) 8 : (B) (C) (D) 32 :


16 32
: :
3 3

(63) The electric current flowing through the battery, of 10V and internal resistance 0.5 : ,
connected with an infinite network formed by resistances each of value 4 : is A.

10 V ˜ 0.5 :

(A) 0.88 (B) 0.5 (C) .74 (D) 0.2

(64) The equivalent resistance between points A and B in the following network will be

Each resistance is of 2 : .
C

(A) 2 : (B) 1 :
F (C) 3 : (D) 4 :
A
B
D E

(65) Each resistance of network is of 4 : , then the effective resistance between points A and B will
be .

A B

(A) 1 : (B) 2 : (C) 4 : (D) 8 :

308
(66) Two wires with same length have ratio of their cross-sectional area 3:1. If resistance of thick wire
is 10 : then the equivalent resistance of series combination of both wire is .

(A) 40 : (B) (C) : (D) 100 :


40 5
:
3 2
(67) A polygon of n sides is formed from the wire of uniform cross-section and resistance R. Sides of
polygon are in even number. Then the ratio of equivalent resistance between mutually opposite
points and equivalent resistance between two end points of any side will be .
4 n  1 2 n  1 4 n  1 2n 2
(A) (B) (C) (D) n  1
n2 n2 n
(68) Three resistances are in ratio 1 : 2 : 3. The equivalent resistance in parallel combination is 6 :
then equivalant resistance in series combination will be .
(A) 36 : (B) 84 : (C) 66 : (D) 18 :
(69) When two resistances R1 and R2 are connected in series, their equivalent resistance is 50 :
and 12 : when connected in parallel, then the values of these resistances will be
and .
(A) 30 :, 20 : (B) 40 : , 10 : (C) 35 : , 15 : (D) 45 : , 5 :
(70) When R1 and R 2 are connected in series, the equivalent resistance is R S and when connected
R1
parallel the equivalent resistance is R p . If R S < R P 16 and R 4 then R
1
2

and R 2 .
(A) 2 : , 0.5 : (B) 1 : , 0.25 : (C) 8 : , 2 : (D) 4 : , 1 :
(71) In series combination of two resistors, the equivalent resistance is S. When connected parallel,
the equivalent resistance is P. If S = nP then the minimum value of n will be .
(A) 3 (B) 4 (C) 2 (D) 1
(72) The galvanometer shows zero deflection in the circuit shown in figure. If internal resistances of
battery A and B is negligible then value of R is .
Q S
500 : (A) 500 : (B) 1000 :
I
B R A 2V (C) 200 : (D) 100 :
12 V

P T
(73) For the given network currents; I1 , I 2 and I 3 are .
B
I2

(A) (B) A A ,
I3 5 5 15 5 5 12
4: A, A, A A, A
10V 1: I 2  I3 C 2 8 8 3 8 8
A
I1 I2 I3 I1 5V
4:
(C) (D)
5 15 5 5 5 15
A, A, A A, A, A
D
I1  I 2 2 8 3 3 2 3

309
(74) The resistance of galvanometer is 5 : is connected in adjacent circuit, then current flowing through
the galvanometer will be .

10 : 5:

(A) (B)
1 2
A A
5: 17 17

10 : (C) (D)
3 4
5: A A
17 17

10 :

10 V

(75) For the given circuit, obtain current flowing through resistance R1 and electric potential at junction O.

2
V2 10 V
: (A) I1 19 A, Vo 12.375 V
1 V0
30

O
R2 (B) I 2 1.75 A, Vo 12.15 V
V1 15 V R1 15 :
(C) I1 0.175 A, Vo 12.375 V
R3

V3 8V
3
45

(D) I1
:

0.19 A, Vo 1.215 V

Ans. : 46 (C), 47 (C), 48 (B), 49 (B), 50 (B), 51 (C) 52 (C), 53 (C), 54 (C), 55 (D),
56 (C), 57 (C), 58 (D), 59 (C), 60 (D), 61 (D), 62 (C), 63 (A), 64 (A), 65 (B),
66 (A), 67 (A), 68 (C), 69 (A), 70 (C), 71 (B), 72 (D), 73 (A), 74 (B), 75 (C)

(1) The current flowing in circuit for series combination of two cells, I
eq

R  req

Where, eq 1  2 Equivalent emf of series connection of two cells

and, req r1  r2 Equivalant internal resistance of series connection of to cells.

(2) The current flowing in circuit for parallel combination of two cells, I
eq

R  req

r  2 r1
Where, Equivalent emf of parallel combination of two cells.
1 2

r1  r2
eq

Equivalant internal resistance of parallel combination of two cells.


r1 r2
r1  r2
req

310
(3) For mix combination of n cells, the current flowing in the circuit

¦
n

¦r where, m = number of rows formed by series combination of n cells.


i 1
I
R i

(76) Resistance of an ammeter is 0.02 : . When it is connected with a battery, it shows 8 A current.
Now if 3 : resistance is connected in series, the current is decreased by 6 A then emf of
battery is V and internal resistance is .
(A) 0.49 V, 2 : (B) 0.98 V, 8 : (C) 8 V, 0.98 : (D) 2 V, 4.9 :
(77) n electric cells each with emf and internal resistance r are connected in closed cricuit. In this,
cell A is connected in destructive manner. Then except for cell A the electric potential difference
for each cell is .

§ n  1· § n · §n  2·
(A) (B) ¨ n ¸ (C) ¨ n  1 ¸ (D) ¨ ¸
2
n © ¹ © ¹ © n ¹

(78) Two cells of emf 10 V, Internal resistance 2 : and emf 4 V and internal resistance 1 : are
connected with 20 : resistance as shown in figure, then current flowing through 20 :
resistance is .

10 V (A) 0.06 A (B) 0.03 A


20 : 4V
2: 1: (C) 0.1 A (D) 2 A

(79) Two cells each with emf 4 V and internal resistance 2 : are connected in parallel then current
flowing by these two cells from 1 : resistance is A.
(A) 0.5 (B) 1 (C) 2 (D) 4

Ans. :76 (C), 77 (A), 78 (B), 79 (C)

(a) Wheatstone Bridge


R1 R3
(1) Ratio of resistances in balance conditation of wheatstone bridge, R = R
2 4

l1
(2) Experimentally resistance measured with meter bridge, R1 = R 2 100 – l
1

(b) Potentiometer

H
(1) The current flowing through potentiometer circuit, I
R + LU + R

Where, LU = Resistance of potentiometer wire

311
ª HU º l
(2) emf of battery measured with potentiometer, Vl « »
¬ R  LU  r ¼

HU
(3) Potential gradient of potentiometer wire, V
Vl
l R  LU  r .

H l1
(4) Comparison of emf of two cells,
H
1

l2
.
2

H l3  l4
(5) For constructive and destructive combination of cells H
1

2 l3  l4

ªH º ªl 1  l 2 º
«  1» R « »R
(6) Internal resistance of cell by potentiometer, r «V » ¬ l2 ¼
¬ ¼

(80) When 5 : resistance is in one branch and R : resistance is in other branch of wheastone
bridge, the null point is obtained at distance l1 . It another resistance R : is connected in parallel
with R then new null point is obtained at 1.6 l1 . Then value of R will be .
(A) 10 : (B) 15 : (C) 20 : (D) 25 :
(81) When D.C. voltage is applied between P and Q in the following circuit, the current flowing
through 4 : resistance is 1 A, then p.d. between points P and Q is .
4:
O
(A) 0.5 V (B) 3.2 V
P 3: Q (C) 1.5 V (D) 1 V
0.5 :

N 1:

0.5 :

(82) Length of a potentiometer wire is 200 cm. The emf of standard battery is V. It is used to find
emf of battery having 1 : internal resistance. If null point is obtained at 40 cm from the positive
terminal then emf of battery is .

(A) (B) (C) (D)


2 5 4

(83) The current flowing through the battery in the given network will be .
4:

12 :
(A) 1.55 (B) 3
2: 4:
(C) 3.5 (D) 4
I 6 :

7V
312
(84) In the balanced condition of meterbridge if x resistance is connected in left branch and y
resistance is connected in right branch, the null point is obtained at distance 39.5 cm. If value of
resistance y is 12.5 : then value of resistance x will be .
(A) 6.7 : (B) 8.2 : (C) 9.1 : (D) 10.5 :
(85) In the electric circuit of potentiometer, two cell are connected in series in
(i) helping mode (ii) opposing mode then the null points are obtained at distances 6 m and 2 m
respectively. Then the ratio of emf of cells will be .
(A) 1 : 1 (B) 1 : 2 (C) 2 : 1 (D) 3 : 1
(86) As shown in the figure to find internal resistance of 1.5 V battery, the potentiometer of emf 2 V
is used. In open circuit condition of battery, the null point is obtained at distance 76.3 cm. Now,
9.5 : resistance is connected in outer circuit, the null point is obtained at distance 64.8 cm then
internal resistance of cell will be .
2V
(A) 1.5 : (B) 1.6 :
(C) 1.7 : (D) 1.8 :

A B
1.5 V

9.5 :
(87) In experiment of potentiometer for comparison of two cells of emf 1 and 2 , the null point is
obtained at distance 64 cm in series combination (helping mode). If poles of 2 are reversed, the

null point is obtained at distance 32 cm then 1


.
2

(A) 1 : 1 (B) 2 : 1 (C) 3 : 1 (D) 4 : 1


(88) 20 : resistance is in one branch and 60 : resistance is in other branch of meterbridge. If resistances
are interchanged in the branches then the null point will be displaced by distance .
(A) 33.3 cm (B) 66.67 cm (C) 25 cm (D) 50 cm
(89) The specific resistance of potentiometer wire is 1012 : and current flowing through it is 0.5 A.
If area of cross section of wire is 106 m2 then potential gradient will be Vm1 .

(A) 2.5 u 107 (B) 5 u 107 (C) 7.5 u 107 (D) 10 u 107
(90) The potentiometer wire of length 10 m and resistance 40 : is connected with resistance box
and cell of 2 V. If potential gradient is of 0.1 mVcm –1 then the resistance R = from
the resistance box. .
(A) 260 : (B) 760 : (C) 960 : (D) 1060 :

Ans. : 80 (B), 81 (B), 82 (C), 83 (A), 84 (B), 85 (C), 86 (C), 87 (C), 88 (D), 89 (B), 90 (B)

313
Electrical energy and power

Electrical energy consumed, W


V2
VIt I 2 Rt t
R

Joule’s law, The heat energy produced, P I 2R t

Now, J ŸH
W W I 2 Rt
H J J

(91) In the following circuit the heat produced per minute in R 2


3 : resistance will be J.

R1 6 :
(A) 640 (B) 1280
R2 3 :
(C) 960 (D) 320

+ – R3 1 :

12 V
(92) There are 15 bulbs each of 40 W, 5 bulbs each of 100 W, 5 fans each of 80 W and a heater of
1 kW in a house. Electric source has potential of 220 V. Then the fuse in the house should be
with minimum capacity of A.
(A) 12 (B) 14 (C) 8 (D) 10
(93) An electric motor draws 5 A electric current when connected with 200 V D.C. supply. If
efficiency of motor is 60 % then resistance of winding wire of motor will be .
(A) 4 : (B) 8 : (C) 16 : (D) 24 :

(94) n bulbs are made such that each have power P for same supply voltage. If all are connected in
series with same supply voltage then power drawn in each will be .

(A) P (B) nP (C) (D)


P P
n n2

(95) Two filaments of same length are first connected in series and then connected in parallel. For the
same current of source, the ratio of heat produced in both the cases will be .
(A) 1 : 2 (B) 4 : 1 (C) 1 : 4 (D) 2 : 1

(96) Two resistances R1 and R2 are connected with a battery of zero internal resistance. The Joule
heat produced in parallel combination should be 5 times than the Joule heat produced in series
combination. If resistance R1 100 : then R 2 :.

(A) 200 or 30 (B) 249 or 51 (C) 262 or 38 (D) 410 or 65


(97) The rate of melting of ice when 210 V supply is given to the wire of 20 : resistance passed
through ice will be .
(A) 6.56 gs –1 (B) 5.66 gs –1 (C) 1.92 gs –1 (D) 0.85 gs –1

314
(98) A bulb B1 of 100 W-250 V rating and two bulbs B2 and B3 of 60 W, 250 V rating are connected with
supply of 250 V as shown in the figure. If W1 , W2 and W3 are output power of bulb B1 , B2 and B3
respectively then
B1 B2
.
(A) W1 > W2 = W3 (B) W1 > W2 > W3
(C) W1 < W2 = W3 (D) W1 < W2 < W3
B3

250
(99) The water in the electric kettle start boiling after 15 minute. Now if length of heating wire of

electric kettle is made times than initial length, then the same amount of water will start boiling
2
3
after time for same supply voltage.
(A) 8 minute (B) 10 minute (C) 12 minute (D) 15 minute
(100) The series combination of four identical resistors connected with battery consumes 20W power.
If these four resistances are connected parallel to the same battery then power consumed will
be
(A) 80 W (B) 100 W (C) 5 W (D) 320 W

Ans. : 91 (B), 92 (A), 93 (C), 94 (C), 95 (B), 96 (C), 97 (A), 98 (D), 99 (B), 100 (D)

Question related to experiments


(101) Which of the following circuit can be used to prove Ohm’s law ?
(A) (B)

(C) (D)

(102) The dimensions of a conductor having specific resistance U are as shown in figure. Then what
will be effective resistance between A and B ?

B
b
Ub Ua
(A) (B)
ac bc
a
c
U ab Uc
(C) (D)
c ab
A
315
(103) The resistance of a conductor is 2 : at 300 K temperature, then at what temperature it’s
resistance will be 4 : ? For this resistance D 1.25 u 103 qC1 .
(A) 1100 °C (B) 827 °C (C) 1127 qC (D) 800 qC
(104) The equivalant resistance of parallel combination of n resistances is x : . When one resistance is
removed from this combination, the equivalent resistance becomes y : . Then the value of
removed resistance will be .

y  x
xy xy
(A) x  y (B) y  x (C) (D) xy

(105) The V o I graph for a conductor at temperature T1 and T2 is as shown in figure. Then the term
T2 – T1 proportional to .

T2 (A) c os 2T (B) s in 2T
(C) cot 2T (D) tan 2T
Voltage T1

T
T
current
(106) As shown in the figure, eight resistances each of 4 : are connected with battery of 6 V and
negligible internal resistance. Then the current flowing in circuit I = .

6V (A) 0.25 A (B) 0.50 A


I
A B C D (C) 0.75 A (D) 1.0 A

F E
(107) The electric current flowing through the battery in given circuit will be A.

6:
(A) 2 A (B) 3 A

3: 12 : (C) 6 A (D) 9 A

I
+ 4:
9V–

(108) A wire of resistance R is compressed uniformly till its radius becomes n times the initial radius,
then new resistance of wire will be .

(A) (B) (C) (D) nR


R R R
n4 n2 n

316
(109) Three identical resistors connected in series with source of emf consumes 100 W power. If
three resistances are connected in parallel with same source of emf then the power consumed
will be W.

(A) (B) 100 (C) 300 (D) 900


100
3

(110) V o I graphs for series and parallel connection of two identical resistors are as shown in figure.
Which graph is for parallel connection ?
(A) a
b (B) b
V (C) a and b
B a
(D) Not any one from a and b
A

0 Io
(111) In an experiment of a meter bridge, unknown resistance x W and resistance 12 : are connected
with thin uniform wire of meter bridge. After connecting electrical components properly, the null
point is obtained with jocky key at distance 60 cm from point A on AB wire, then value of
unknown resistance will be . (Positive terminal of battery is connected with A)
(A) 18 : (B) 8 : (C) 16 : (D) 4 :
(112) In the circuit of meterbridge, coil P and Q with small resistance are connected in two gaps. At
that time, the null point in obtained with jockey key at 40 cm from the end of P. If 60 :
resistance is connected parallel to Q, the null point is displaced by 20 cm then what will be
resistance of P and Q ?
(A) 50 : and 75 : (B) 60 : and 30 : (C) 20 : and 40 : (D) 10 : and 50 :
(113) Six resistances each of value r are arranged as shown in the figure, then effective resistance
between A and B will be .
D
r (A) (B) 2r
2
r 3
r
r
A C
r
r (C) r (D)
r
r 2
B
(114) For any conductor, V o I graphs at different temperatures are as shown in figure, then .
T3

T2 (A) T1  T2  T3 (B) T1 T2 T3

V T1

T1  T3
(C) T1 ! T2 ! T3 (D) T2
2
0 oI
317
(115) In an experiment of meter bridge, when resistance P is balanced by Q, the null point from one
end of wire is obtained at distance 20 cm. If P < Q and 4P is balanced with Q then the null
point will be at distance .
(A) 50 cm (B) 80 cm (C) 40 cm (D) 70 cm
(116) The resistances of four branches P, Q, R, and S of a wheatstone bridge are 10 : , 30 : , 20 :
and 60 : respectively. The emf and internal resistance of cell are 5 V and 2 : respectively. If
resistance of galvenometer is 60 : then current drawn from the cell will be .
(A) 2 A (B) 0.2 A (C) 0.15 A (D) 0.174 A
(117) In the circuit of wheatstone bridge, different resistances are connected as shown in figure. Current
flowing through galvanometer is zero. If heating effect are neglected then in which following
condition, the current through galvanometer will not be zero ?
(A) emf of cell is doubled
R1 R2
(B) Exchanging battery and galvenometer
(C) Resistances of circuit are doubled

(D) Interchanging resistances R1 and R2


R3 R4

(118) In the circuit of potentiometer, the null point is obtained at distance 35 cm when a cell of 1.25 V
is connected. If the cell is changed, the null point displaces to distance 63 cm then emf of second
cell will be .
(A) 1.75 V (B) 2 V (C) 2.25 V (D) 2.5 V
(119) In an experiment of potentiometer, the null point is obtained at 250 cm for one cell. If parallel to
this cell, a 2 : resistance is connected then the null point is obtained at 125 cm, then the
internal resistance of cell will be .
(A) 2 : (B) 4 : (C) 0.5 : (D) 1 :
(120) A wire of length 10 m is connected with steady voltage source of potentiometer. When primary
cell connected with it is in open circuit condition, the null point is obtained at 7.8 m. When
current is flowing through primary cell and resistance of 10 : connected with it, the null point is
obtained at distance 7 m, then internal resistance of primary cell will be .
(A) 1.24 : (B) 1.36 : (C) 1.14 : (D) 1 :
(121) The resistance and length of potentiometer wire are 10 : and 100 cm in an experiment of
potentiometer. A cell of 2 V emf with negligible internal resistance and resistance R are
connected in series with it. If null point is obtained at length 40 cm for a voltage source of 10 mV
emf then value of resistance R will be .
(A) 900 : (B) 820 : (C) 790 : (D) 670 :
318
(122) A battery of emf 2 V and internal resistance 1 : is connected with the potentiometer of length

200 cm and resistance 4 : and electric current is passed then the potential gradient will

be .
(A) 8 u 10 3 Vcm –1 (B) 4 u 103 Vcm –1 (C) 6 u 10 3 Vcm –1 (D) 2 u 10 3 Vcm –1

(123) A steady voltage source is connected with potentiometer wire of length 4 m. The null point is
obtained at distance 1 m for Leclanche cell. If length of potentiometer wire is increased by 1 m
then for the same Lenchanche cell, the null point will be obtained at .
(A) 1.25 cm (B) 1.4 m (C) 1.75 m (D) 1.2 m

(124) Resistance of potentiometer wire of length 15 m is 30 : . It is connected in series with 5 V

battery and 20 : resistance, then the potential difference between two point seperated by

40 cm on the wire will be .


(A) 0.02 V (B) 0.06 V (C) 0.08 V (D) 0.1 V

Ans. : 101 (A), 102 (A), 103 (B), 104 (B), 105 (C), 106 (D), 107 (B), 108 (A), 109 (D),
110 (A), 111 (B), 112 (A), 113 (D), 114 (A), 115 (A), 116 (D), 117 (D), 118 (C),
119 (A), 120 (C), 121 (C), 122 (A), 123 (A), 124 (C)

Comprehension Type Questions :

Passage I :

In an experiment, the current flowing through a cell and potential difference across it’s terminal
are measured and following observation table is prepared.

S.No. V (volt) I (Ampere)

1. 1.2 0.04
2. 0.8 0.12
3. 0.4 0.20

(125) What is the emf of cell used in experiment ?


(A) 1.5 V (B) 1.4 V (C) 2 V (D) 2.5 V

(126) What will be the maximum current that can be withdrawn from cell ?
(A) 0.25 A (B) 0.28 A (C) 0.3 A (D) 0.35 A
(127) What will be maximum power can be withdrawn from cell ?
(A) 50 mW (B) 98 mW (C) 9.8 mW (D) 49 mW

319
Passage II :

A battery of emf 14 V and internal resistance 1 : is connected with another battery of emf
20 V and internal resistance 2 : in opposing mode and circuit is completed. Find the following
quantities in this condition.

(128) Current flowing in circuit.

(A) 1 A (B) 3 A (C) 2 A (D) 4 A

(129) Electric power in both batteries


(A) 14 W, 20 W (B) 15 W, 10 W (C) 30 W, 45 W (D) 28 W, 40 W
(130) Terminal voltage of both batteries
(A) 12 V, 24 V (B) 15 V, 20 V (C) 10 V, 30 V (D) 8 V, 12 V
(131) Electric power consumed in both batteries.
(A) 8 W, 4 W (B) 4 W, 8 W (C) 5 W, 10 W (D) 6 W, 9 W
(132) Bulbs A and B have ratings 40 W, 110 V and 100 W, 110 V respectively, then find the
resistances of their filament.
(A) 120 :, 250 : (B) 302.5 :, 121 :
(C) 100 :, 110 : (D) 25 :, 60 :
(133) If above two bulbs are connected in series with supply of 220 V then which bulb will be fused ?
(A) Bulb A (B) Bulb B
(C) Both A and B (D) No one will be fused

Ans. : 125 (B), 126 (B), 127 (B), 128 (C), 129 (D), 130 (A), 131 (B), 132 (B), 133 (A)

Assertion - Reason type Question :


Instruction : Read assertion and reason carefully, select proper option from given below.
(a) Both assertion and reason are true and reason explains the assertion.
(b) Both assertion and reason are true but reason does not explain the assertion.
(c) Assertion is true but reason is false.
(d) Assertion is false and reason is true.

(134) Assertion : As temperature is increased, the drift volecity of electrons in metal decreases.
Reason : If temperature is increased, the conductivity of metal decreases.
(A) a (B) b (C) c (D) d
P+Q (R+S)
(135) Assertion : When Wheatstone bridge is in balanced condition R AC P+Q+R+S
Reason : Points B and D are at same electric potential.
(A) a (B) b (C) c (D) d

320
(136) Assertion : Two bulbs of 60 W and 200 W are given. When they are conected in series, bulb of 60 W
glow more bright and when connected in parallel, 200 W bulb glow more bright.
Reason : In series connection, power directly proportional to resistance and in parallel
connection, power inversly proportional to resistance.
(A) a (B) b (C) c (D) d
(137) Assertion : The ratio of resultant resistance of two resistors of same resistance when first
connected in series and then connected in parallel is 4:1.
Reason : In series connection, resistancce increases and in parallel connection, the resistance decreases
(A) a (B) b (C) c (D) d
(138) Assertion : The resistance of fuse wire is more and it’s melting point is high.
Reason : Fuse wire is used for small electric current only.
(A) a (B) b (C) c (D) d
(139) Assertion : Voltmeter measures terminal voltage (V) of battery, not it’s emf ( H )
Reason : When voltmeter is connected with battery, electric current passes through battery.
(A) a (B) b (C) c (D) d
(140) Assertion : For semi conductors, as temperature is increased, their resistivity decreases.
Reason : Semiconductors obey Ohm’s law.
(A) a (B) b (C) c (D) d

Ans. : 134 (B), 135 (B), 136 (A), 137 (B), 138 (D), 139 (A), 140 (C)

Matching type Questions :


(141)
Column-1 Column-2
(a) Resistivity (p) M -1 Lo T -2 A1 (A) a (r), b (p), c (s), d (q)
(b) Mobility (q) 1
M L T 2 -3
(B) a (p), b (q), c (r), d (s)
(c) Electomotive force (r) M1 L3 T -3 A -2 (C) a (q), b (r), c (s), d (p)

(d) Electric Power (s) M1 L2 T -3 A -1 (D) a (s), b (p), c (q), d (r)

(142)
Column-1 Column-2

(a) Kirchoff’s first law (p) ¦ IR ¦ H


H l (A) a (s), b (q), c (r), d (p)
(b) Wheatstone bridge (q) H l
1 1

2 2 (B) a (p), b (r), c (q), d (s)

(c) Kirchoff’s second law (r) ¦I 0 (C) a (q), b (p), c (r), d (s)
(D) a (r), b (s), c (p), d (q)
R1 R3
(d) Potentiometer (s) =
R2 R 4

Ans. : 141 (A), 142 (D)

321
1& 2 Electrostatics

Electric Charge

l Like mass, the electric charge is also fundamental and intrinsic property of matter.

l Electric Charge is scalar quantity. It has two types

(i) Positive Charge (ii) Negative charge


l If electron is removed from the body, it becomes positively charged and its mass is slightly
decreased.

l When the body gain the electron, it becomes negatively charged and mass is slightly increased.

The change in mass of the body = nme m .


Q
l
e e

Where me 9.1u1031 kg.

Units of Electric Charge and Relation Between Them


1 Stat C or 1 esu of charge
SI Unit o Coulomb (C) CGS Unit
1 ab C or 1 emu of charge

Relation : 1 C = 3 × 109 Stat C = 10


1
ab C

l Practical units of charge


(i) amp u hr = 3600 C
(ii) 1 faraday = 96,500 C
l The smallest unit of electric charge o Stat Coulomb
l The biggest unit of electric charge o Faraday

l The dimensional formula of electric charge o T1A1 or Q1

l The minimum magnitude of electric charge on any body is e = 1.60217733 × 10–19 C. It is


known as basic or fundamental charge.

l The number of electrons in 1 C negative charge is n 6.25 u 1018

l The presence of electric charge can be detected by electroscope


Quantization of electric Charge
Magnitude of all charges found in nature are in integral multiple of a fundamental charge.
Q ne Where n integer and e = 1.6 × 10–19 C

227
l Protons and neutrons consists of fundamental particle known as Quarks.

It has two types : (i) Up quark ( u ) o  e (ii) Down quark ( d ) o 


2 1
e
3 3

l Composition of proton : uud

Composition of neutron : udd

l If any body consists n1 number of proton and n2 number of electron then total charge on

it is Q n2  n1 e

Conservation of Electric Charge


l The algebraic sum of electric charges in an electrically isolated system always remains
constant irrespective of any process taking place.
l In every chemical or nuclear reaction, the total charge before and after the reaction remains
constant.

e.g. Pair Production : 2J rays o 1e0  1e0

Nuclear reaction : 92 U
238
o 90 Th
234
 2 He 4

0
n1 o 1P1  1e0

Electrostatic Induction :
Body can be charged by the following methods :
(1) By Friction Ñ
When two bodies are rubbed together, equal and opposite charges are produced on both the bodies.
l In the method of Friction, when an electron is transfer from one body to another body, the
body which loss the electron becomes positively charged and its mass is reduced. While the
body which gain the electron becomes negatively charged and its mass is increased.
l The pair of charged body :

FGW PASER
( ) ()

l F = Fur, G = Glass, W = Wool


l P = Plastic, A = Amber, S = Silk, E = Ebonite,
R = Rubber
(2) By Induction Ñ
If a charged body is brought near a neutral body, opposite charge is induced at the near end and
similar charge at the farther end on neutral body.

228
Charging substance Charged substance Disconnecting
brought near to connected to the Earth charged substance
uncharged substance (Earthing) from the earth

Maximum induced charge Qc r Q ª«¬1  K º»¼ Where K di-electric constant of chargeless


1
l

substance.
(3) By Conduction Ñ
When two identical bodies, one of them is charged and the other is neutral, brought in contact,
the charge is distributed half-half on them. Hence the neutral body is charged.

l If the two sphere of radius R1 and R2 and total charge Q , brought in contact and then
separate the charge on them is :

R1  R2 and 2
QR1 QR2
R1  R2
q1 q

(1) A copper sphere of mass 2.0 g contains about 2 u 1022 atoms. The charge on the nucleus of
each atom is 29e. If the charge on sphere is 2 & then fraction of electrons removed
is .
(A) 5.8 u 1023 (B) 1.25 u 1013 (C) 6.28 u 1023 (D) 2.16 u 1011

(2) A substance of mass 1 g consists of 5 u 1021 molecules. If from 0.01 % molecules of the
substance 1 electron is removed, then total electric charge on the substance is C.
(A) + 0.08 (B) + 0.8 (C) – 0.08 (D) – 0.8
(3) Total charge on 75 kg electrons is C.
(Mass of electron, me = 9 × 10–31 kg)
(A) – 1.25 × 1013 (B) – 6.25 × 1018 (C) – 1.33 × 1013 (D) – 1.6 × 1019
(4) Calculate negative charge in 100 g of water.
(A) 1.33 × 1013 C (B) 5.34 × 106 C (C) 6.25 × 1018 C (D) 2.55 × 108 C
(5) If 1010 electrons are incident on a substance per second, then what time would be taken by it to
get total 1 C electric charge ?
(A) 20 Days (B) 20 Years (C) 2 Hours (D) 2 Days

229
(6) Two chargeless sphere A and B are in contact with each other. As shown in figure, a negatively
charged rod is brought near without contact to sphere. Now, if sphere A and B are slightly
separated and the rod is removed, the charge on sphere A and sphere B is .

sphere – A sphere – B

(A) positive and positive (B) A Positive and B Negative


(C) Negative and Negative (D) A Negative and B Positive
(7) Sphere having radius 2 cm has 40 & charge and other sphere having radius 3 cm has 20 PC
charge. If they are connected with a conducting wire, the charge move from sphere of radius
2 cm to sphere of radius 3 cm is .

(A) 24PC (B) 72PC (C) 16 PC (D) 32 PC


Ans. : 1 (D), 2 (A), 3 (C), 4 (B), 5 (B), 6 (B), 7 (C)
Coulomb's Law :

In 1785 a scientist Charles Augustin Coulomb gavea the law to find out the electric force between
two point charges.

‘The electric force between two stationary point charges is directly proportional to the product of
their charges and inversely proportional to the square of the distance between them.’

FD 1 2 Ÿ
qq qq
F k 1 2
r2 r2

Where k is proportionality constant it is known as Coulomb’s constant. Its value depends upon
two factors :

(i) Unit system (ii) Medium in which the charge is placed.

SI System : k
1 9 u 109 Nm 2 C 2
l
4 S 0

CGS System : k 1

Where 0 permitivity of free space = 8.85 u 1012 C2 N 1 m 2

l Another unit of is 0 : farad / metre

230
l Relative Permittivity (r ) Or Dielectric Constant ( K ) :

Permittivity of medium ()


Dielectric constant of medium K =
Permittivity of vaccum (0 )
r 
0
K

l Coulomb’s Law In Terms of dielectric constant (K) :


When two charges are put in a medium, the electric force between them

1 q1q2
4S  r 2
Fm

 Ÿ  K 0
But 0
K

q1q2
4 S 0 K r 2
Fm 1

Therefore Fm F
K

l For Insulator (dielectric substances) K ! 1 thus, Fm  F

l In vacuum K 1 , For air K 1.0006 1

l For Conductor K f

Vector form of Coulomb’s law :

o §o o ·
¨ r1  r2 ¸
kq1q2
F12
o o
l The electric force on q1 due to q2 is | r1  r2 |3 © ¹

o §o o ·
¨ r2  r1 ¸
kq2 q1
F21
o o
l The electric force on q2 due to q1 is | r2  r1 |3 © ¹

Important points of Coulomb’s law


l The coulombian force acting between two charges is mutually interactive. The force acting
between two charges is equal in magnitude and opposite in direction. The ratio of electric
force between charges q1 and q2 is 1:1.

o o
l Coulomb’s law agrees with Newton’s Third law. F12  F21

l Coulomb’s law is applicable for the distance more than 1015 m (nuctear distance) and it can
be applied for the point charges only.

231
l If q1q2 ! 0 then two charges repel each other and if q1q2  0 then they attract each other.
l Charge Q is divided in charges q1 and q2 and if the force acting between them is maximum

then q1
Q
q2 .
2
l When a material medium of dielectric constant K is placed between the charges, the force

between them becomes K


1
of the force between them in vacuum. Fm F
K
l If a dielectric medium of dielectric constant K and thickness t is partially filled between
the charges q1 and q2 which are at a distance r then the electric force between

them is. F
kq1q2

r t  t K
2

l The coulombian force acting between two charges is not influenced by the presence of a third
charge. Hence, the coulombian force is called a two body force.
l If two point charge q1 and q2 are separated with medium of thickness t1 , t2 ,..... tn and
dielectric constant K1 , K 2 ,..... K n respectively then the electric force between them is.

k q1q2
ª n º
F

«¦
2
K i ti »
¬« i 1 ¼»

In above case, if the distance between two charge is taken t then equivalent dielectric
constant.

ª º
¦ Ki ti »
2
«
n

« »
« »
¦ i »»
i 1
K
«
n

«¬ ¼
t
i 1

l If the force between two charges at distance r1 is F1 and at distance r2 is F2 then

F1r12 F2 r22 .

l If the force in medium of dielectric constant K1 is F1 and in dielectric constant K 2 is


F2 than F1 K1 F2 K 2 .
l If two identical sphere carry charge q1 and q2 and force acting between them is F.
They are brought in contact and then separated then the force acting between them

q1  q2 2
is F’ F.
4 q1q2

l A sphere of mass m, atomic number Z and atomic mass A has electric charge

where Avogadro’s number


(%Ze ) NA m
q NA
A
232
Principle of equilibrium of electric forces :
l To solve some questions of electrostatic, Lami's theorem is very useful. According to this theorem,
o o o
if three forces are in equilibrium, as shown in figure, it means that F1  F2  F3 0 then,
o o
F1 F2

J F1 F2 F3
E D sin D sin E sin J

o
F3
 q1 Q q2  q1 Q q 2
l or
m
 r1 
o m
 r2 
o m
 r1 
o m
 r2 
o

As shown in figure, charges are placed on one line. q1 and q 2 are like charges while Q is unlike

charge then,

r1  r2 2
When the force on q1 is zero then Q
q2
l
r12

r1  r2 2
When the force on q2 is zero then Q
q1
l
r22

q2 r22
l When the force on Q is zero then q1
r12

l If all three charges are like charges then condition for equilibrium of Q is,

 q1 Q q2
q1 r12
m
 r1 
o m
q2  r2 
o
r22

l The magnitude of distance for the same magnitude of force in vacuum and medium

Fvacuum Fmedium

1 q1q2 1 q1q2
4S 0 r 2 4S 0 K r ’2

r
Ÿ r’
K

233
l Null points or neutral points due to charge q1 and q2 at distance r Ñ

l If the neutral point is at the distance d from the charge q1 then,

d r
q2
r1 (If q2 ! q1 )
q1

l If both the charges are identical then,

r q1 N q2
d
1
q2
q1 m
 d 
o m
 r 
o

l If both the charges are unidentical then,

r q2 q1 N
o
d
1
q2
q1 m
 r 
o m
 d 
o

As shown in figure if the spheres of mass m are charged with


charge q then
l T T l
T T
tan T
Fe x
mg 2l
m q q m
1
x § Fe · § 2q 2l · 3
? x or ¨ 4 S  mg ¸
Fe Fe 2l ¨ mg ¸ x
¨ ¸
mg mg © ¹ © 0
¹

l If both the spheres are immersed in liquid of density U0 and if the distance between
them remains same then the density of the spheres is

KU0
U
K 1

l If the dielectric constant of liquid is K then,

U
K U  U0

l According to law of parallelogram, the resultant force on a charge due to two electric charges is,

F1 F F12  F22  2F1F2 cos T

where T = angle between F1 and F2 .


G G
F2 F2
T F2 sin T
D Also tan D .
F1  F2 cos T
F1

234
l If F1 = F2 = F’ then for different angle of T , magnitude of force is :

Angle T Force F

0D 2 F’

2  3
1
30D
2
F’


1
45 D 2 2 2
F’

60D 3 F’

90D 2 F’

120D F’


1
150 D 2 3 2
F’

180D 0

Principle of Superposition
The electric force on electric charge due to system of n charges is,

¦
n
G G 3 ˜ ri  r j
G qj G G
Fi kqi
j 1 | ri  r j |
j zi

Specific charge :

§e·
The ratio of electric charge and mass ¨ m ¸ is known as specific charge.
© ¹

Its SI unit is C kg 1 and dimensional formula is : M 1T1A1 or M Q


–1 1
l

(i) As the velocity of a particle increases, its charge remains same, but mass of the particle
m0
1  v2
m

increases. C2 .

So, as velocity increases, the specific charge of the particle decreases.

(ii) Specific charge of electron o  1.76 u 1011 C kg 1

Specific charge of proton o 9.580 u 107 C kg 1

Charge densities :
Charge distributed per unit dimension (length, area or volume) is called charge density.
l There are three types of charge densities.

235
(i) Linear charge density Ñ Charge distribution per unit length is known as linear charge
density.

O1
Q
l

SI unit o Cm 1 ; Dimensional formula o L1T1A1

(ii) Surface charge density : Charge Distribusion per unit area is known as surface charge density.

Vs
Q
A

SI unit o Cm2 ; dimentional formula o L2 T1A1

(iii) Volume charge density : Charge distribution per unit volume is known as Volume charge
density.

Uv
Q
V

SI unit o Cm3 ; Dimensional formula o L3T1A1

D
(8) The distance between two ions of same positive charge is 5A and electric force acting on them

is 3.7 u 10 N , then electron loss by each ion is .


9

(A) 2 (B) 3 (C) 1 (D) 4


(9) The earth and moon has same type of and equal magnitude of charges. To balance the
gravitational force between earth and moon, the required magnitude of charge is .

[ M e 6 u 1024 kg, M m 7.36 u 1022 kg ]

(A) 1/ 5.7 u 1013 C (B) 5.7 u 1013 Ab C

(C) 5.7 u 1013 C (D) 5.7 u 1013 Stat C


(10) +q and –q charges are put on diametric end points of circle of diameter d, then the force on
third charge +q which is on the center of circle is .

8Kq 2 2Kq 2 4Kq 2


(A) (B) (C) (D) 0
d2 d2 d

(11) If two unlike charge of same magnitude are placed at certain distance the force between them is
F. If 25 % electric charge is moved from one charge to another charge then the force between
them is .

(A) F (B) 4F
5 (C) 15F
16
(D) 9 F
16

236
(12) Two point charges placed at distance of 20 cm in air attracts each other with certain force. When a
dielectric slab of thickness 8 cm and dielectric constant K is introduced between these two charges
force of interaction becomes half of its previous value. Then the magnitude of K is .

(A) 1 (B) 4 (C) 2 (D) 2


(13) Two particles of mass 5 g and charge 10–7 C are placed on horizontal table at distance 10 cm.
When both the particle are in equilibrium position, the co-efficient of static friction
Ps .

(A) 0.15 (B) 0.19 (C) 0.18 (D) 0.2


(14) Two point charges q and 2q are placed in air at distance d. If third electric charge Q is kept on
the line joining two charges such that the resultant force on q and 2q becomes zero, then the
distance of charge Q from charge q is .


d d
(A) (B) 2 1 d (C) (D) 3 1 d
2 1 3 1

(15) Two point charges q1 and q2 are kept at distance 3 m . If sum of these charges is 20 PC and
repulsive force between them is 0.075 N , the magnitude of each charge is .

(A) 12 PC, 8 PC (B) 14 PC, 6 PC (C) 16 PC, 4 PC (D) 15 PC, 5 PC

(16) Positive charge on two conducting spheres is q1 and q2 . These two spheres are brought close
to each other such that after touching each other, they return back to their original positious.
How much will be the force between then in new situation ?

(A) Force remains same as before the spheres were in contact.


(B) Force becomes more than before the spheres were in contact.
(C) Force becomes less than before the spheres were in contact.
(D) Becomes zero.
(17) The similar spheres of mass 10–3 kg are suspended by silk strings of length 0.5 m. When both
the spheres are equally charged, they repel each other at 0.2 m distance, then the electric charge
on the each sphere is .

(A) 1.53 u 103 C (B) 2.15 u 106 C (C) 9.43 u 108 C (D) 2.36 u 106 C

(18) Two balls of same mass and radius are suspended by string of length 1 m. The mass and electric
charge on each ball is 15 g and 126 PC respectively. When both the spheres are in equilibrium,
the distance between them is 8 cm. Now if the charge on any one ball is reduced up to half then
the new distance between them is cm.

(A) 5.3 (B) 6.4 (C) 4.2 (D) 2.5

237
(19) Two spheres having same charge of 10 PC are suspended from a rigid support by a string of
length 1 m. In equilibrium, the angle between them is 60o then the tension produced in string is
N.
(A) 18 (B) 0.18 (C) zero (D) 1.8
(20) Charge Q is placed at each of the opposite vertices of a square. Charge q is placed at each of the

other two vertices. It the net electric force on Q is zero, then .


Q
q

(A) –1 (B) 1 (C) 2 2 (D)  1


2

(21) The distance between two equal charge Q is r. A third charge q is placed on line joining these
two charges such that all three charges remains in equilibrium. Then what is the position and
magnitude of q ?

(A) 2r , Q (B) r,Q


2 4
(C) r,Q
2 2
(D) 2r , 2Q

S
(22) In right angled triangle PQR, ‘ PQR . Also PQ 5 cm and QR 10 cm . 10 nC and
2

20 nC charges are placed respectively on point P and Q . If, due to this charges, the force

acting on 1 PC charge placed at point Q is 18 x mN , then x .

(A) 3 (B) 2 (C) 11 (D) 5

(23) The position vector of two 1 nC charges are 1, 1,  1 m and 2, 3, 1 m respectively. Then the
magnitude of coulombian force between this two charges is .

(A) 10 6 N (B) 10 3 N (C) 10 12 N (D) 10 9 N

(24) The specific charge of a steady electron is 1.76 u 1011 Ckg 1 . If it move with velocity v
c
2

(where c velocity of light) then its specific charge is C kg 1 .

(A) 2.0 u 1011 (B) 1.53 u 1011 (C) Zero (D) 2.6 u 1015

(25) The linear charge density on the circumference of a circle of radius a varies as O O 0 cos 2 T .

ª º
³
2S

The total charge on it is . «  cos 2 T d T S»


¬« ¼»
H int :
0

(A) Infinite (B) Zero (C) 2Sa (D) SaO0

238
(26) As shown in figure the linear charge density on the rim of the semi-circular wire is O DT
where D constant. Then the total charge on a semi-circular wire is .

a D S2
(A) aDS
2
(B) 2
a
2a D
(C) a D S (D)
dT
O a S2

(27) The charge density on a sphere of radius R is given by equation U(r ) Er 2 , then the total
charge on the sphere is .

2 SR3E 4 SR3E 4 SR5E


(A) 3
(B) 3
(C) 5
(D) Zero

(28) A square having length a has electric charge distribution of surface charge density V V 0 xy ,
then total electric charge on the square with respect to the Cartesian Co-ordinate system placed
at the centre of the square is .

V0a 4
(A) 4
(B) 4SV 0 a 2 (C) 2V 0 a 2 (D) Zero

Ans. : 8 (A), 9 (C), 10 (A), 11 (D), 12 (B), 13 (C), 14 (B), 15 (D), 16 (B), 17 (C), 18 (B),
19 (D), 20 (C), 21 (B), 22 (D), 23 (D), 24 (A), 25 (D), 26 (B), 27 (C), 28 (D)

Electric field :
The region of space around a system of electric charge, in which its effect can be experienced, is
known as Electric field.
l There are four types of electric field :
(i) Uniform electric field : The electric field, at every point of which a unit positive test
charge experiences the same electric force, is known as Uniform electric field. In
uniform electric field, the lines of force are parallel and equidistant. e.g. Electric field
between the plates of a parallel plate condenser.
(ii) Non - uniform electric field : The electric field, at different points of which a unit
positive test charge experiences different forces, is known as Non-uniform electric field.
(iii) Variable electric field : The electric field which changes with respect to time is known
as Variable electric field. E f (t )
(iv) Constant electric field : The electric field which does not depend on time is known as a
Constant electric field. E z f (t )
l The line integral of electric field along any closed path is zero. i.e. the electric field is a

v³ E ˜ dl
o o
conservative force field. 0.

239
o
Electric field intensity (E) Ñ

The force acting on a unit positive charge at a given point in an electric field of a point charge of a
o
system at charges is called Electric field or intensity of electric field (E) at that point.

o o
lim F
E q0 o0 q0

o o
l Force experienced by a charge q in an electric field of intensity F qE.

l According to Newton’s second law F ma .

? ma qE

?a
qE
m

l If charge is in motion in electric field then,

§ ·
Velocity after ‘t’ seconds is v v0  ¨ qE ¸ t .
© ¹
l m

1 § qE · 2
Distance travelled in ‘t’ seconds d v0t  ¨ ¸t
2© m¹
l

Time taken to fall through a height ‘h’ is t


2hm
l
qE

Electric field due to a point charge :

By Coulomb’s law, F
k Qq
r2

? q
F kQ
r2

The electric field due to point charge E


kQ
?
r2

o o
In vector form, E
kQ
r
r3

l The value of E depends on the magnitude of source charges and distance from that charge.
l The number of electric lines of force passing through unit area imagined around any point in
an electric field is defined as Intensity of electric field at that point.

240
l The electric fiux passing perpendicularly through unit normal area is known as Intensity of
o I
electric field. E o
A
l The direction of force acting on unit positive charge at a given point is the direction of electric
field at that point.
l SI unit of electric field : (i) NC1 (ii) Vm1
l Dimensional formula of electric field : [ E ] M1L1T 3A 1 or M1L1T 2Q1
G

l A positive charge like alpha-particle, proton and deuteron, experience a force in the direction
of electric field and a negative charge like electron experiences a force in a direction opposite
to the electric field.
l A charge q0 is kept in an electric field of strength E and experiences a force F then,

F
(i) E  q if field is diverging
0

F
(ii) E q0 if field is uniform

F
(iii) E ! q if field is converging
0

Electric lines of force :


Scientist Michael Faraday introduced the concept of electric field lines.
l An electric field line is a curve drawn in an electric field in such a way that the tangent to the
curve at any point is in the direction of net electric field at that point.
Characteristic of electric field lines :
(1) Electric field lines start from positive charge and end at negative charge.
(2) The tangent drawn at any point on the electric field lines shows the direction of electric field
at that point.
(3) Two field lines never cross each other. If two lines intersect at a point, two tangents can be
drawn at that point indicating two directions of electric field at that point which is not possible.
(4) Electric field lines of stationary electric charge distribution do not form closed loops.
(5) The separation of neighbouring field lines in a region at electric field lines indicates the
strength of electric field in that region.
(6) Field lines of uniform electric field are mutually parallel and equidistant.
l The electric field lines are geometrical representation of electric field and are not real. But electric
field is a reality. The field lines are actually are geometrical representation of electric field.

241
(29) A coin of mass 1.6 g is placed in an electric field of intensity 109 NC1 in vertical direction. For
equilibrium of coin, the number of electrons to be removed from the coin is .

(A) 9.8 u 107 (B) 6.25 u 109 (C) 1.6 u 1019 (D) 4.25 u 1010
(30) Mass of a bob of simple pendulum is 80 mg and the charge on it is 20 nC . It is suspended by a

string in a horizontal electric field of intensity 2 × 104 NC1 . In equilibrium, the angle made with
vertical direction and tension force arising in string is .

(A) 30D , 2.4 u 102 N (B) 45D , 1.57 u 103 N

(C) 27D , 8.8 u 104 N (D) 35D , 4.5 u 10 4 N

(31) If a particle of mass 1 g and charge 5 PC is moved with velocity 20 ms1 in direction opposite

of electric field of internsity 2 × 105 NC1 then how much distance is travelled by the particle
before coming to rest ?

(A) 1 m (B) 0.4 m (C) 10 cm (D) 0.2 m


(32) Two parallel conducting plates lie at a distance 20 mm. Potential of upper plate is +2400 V wrt
lower plate. If an electron is released at lower plate then how much time does it take to reach
upper plate ?

1.8 u 1011 Ckg 1


e
m

(A) 2 Ps (B) 1.4 ns (C) 1.7 ms (D) 2.7 Ps


(33) In Millikan experiment, a drop of charge Q remains stationary between the two plates having
2400 V potential difference. If the radius of drop becomes half and potential difference 600 V
then for equilibrium, the charge on drop is .

(A) Q
4
(B) Q
2
(C) Q (D) 3Q
2

(34) 10–8 C and –10–8 C charges are placed on any two vertices of equilateral triangle, then the

intensity of electric field at third vertix is NC1 . (length of sides of equilateral


triangle is 0.1 m).

(A) 3.6 u 104 (B) 7.2 u 104 (C) 9 u 103 (D) 3 u 109
(35) The length of each side of square PQRS is 5 m. If +50 C, –50 C and +50 C charges are placed
at vertices P, R and S respectively then the magnitude and direction of resultant electric field at
point Q is . (k = Coulombs, constant)
(A) 3 k, 30o (B) 2 k, 45o (C) 3 k, 25.5o (D) 2 k, 38.5o

242
(36) The magnitude of electric field at point P shown in figure is .

+q
A
EC
p
EB 2kq kq
(A) (B)
a a 2
a2
EA
3kq
+q +q (C) (D) Zero
a2
B a C

(37) An uncharged sphere of metal is placed in between two charged plates as shown in figure. The
lines of force look like.

(a) (b) (c) (d)

(A) a (B) b (C) c (D) d


(38) Some equipotential lines are as shown in figure. E1, E2 and E3 are the electric field at point 1, 2
and 3. Then .

(A) E1 = E2 = E3 (B) E1 > E2 > E3

(C) E1 > E2 , E2 < E3 (D) E1 < E2 < E3

(39) Three positive charges of equal value q are placed at the vertices of an equilateral triangle.
Which of the following will be resultant electric field lines ?

(a) (b) (c) (d)

Ans. : 29 (A), 30 (C), 31 (D), 32 (B), 33 (B), 34 (C), 35 (C), 36 (A), 37 (C), 38 (C), 39 (C)

243
Electric Dipole :
A system of two equal and opposite charges, separated by a finite distance is called
Electric dipole.

o
l Electric dipole moment ( p) of the system can be defined as follows :
o o
p q2 a
l SI unit of electric dipole moment = Cm (coulomb-meter)
o
l Dimensional formula of dipole moment > p @ L1T1A1 or L1Q1 .

l Electric dipole moment is a vector quantity and its direction is from the negative charge to
positive charge.
l The net electric charge on an electric dipole is zero  q  q 0 but its electric field is not
zero, since the position of the two charges is different.
o o
l If lim q o f and 2a ® 0 in p q 2 a , then the electric dipole is called a point dipole.
Electric field of a dipole :
To find out the electric field due to an electric dipole, placed at the co-ordinate system such that its
Z-axis coincides with the dipole and origin of the system coincides with the centre of the dipole.
l Electric field at the point on the axis of a dipole is,
o 2kpz

z
E( z ) pˆ
2
a 2 2

If z !! a then a 2 is not considered in denominator,


o 2kp
E ( z) pˆ
z3
l Electric field at a point on the equator of a dipole is,
o  kp

y2  a
E( y ) 3

2 2

o  kp
If y !! a then, E( y ) pˆ
y3
l If the point P lies on a line making an angle q with the axis of the dipole, then the intensity at

3cos 2 T  1 .
kp
a point P lying at a distance r from the centre of the dipole is E 3
r
244
G E p
E

T O
–q +q
If the direction of E is such a way that its makes angle E with OP then tan E 1 tan T
o
2
.

E axis
l For short (small) dipole E 2.
equater
r
p1 p2
o o
l As shown in figure, if p1 and p2 are placed at distance r then the force between two

6k p1 p2
dipole is F .
r4
o o
If p1 and p2 are parallel to each other then electric force
o o
p1 r p2 3k p1 p 2
F .
r4
o
o p2 o o
p1 If p1 and p2 are perpendicular to each other then electric force
r
r
2k p1 p2
F .
r4
l If a rod of length L and charge Q is bent in half circle then electric field at centre is

Q
E .
2 0 L2
Q
O
L
Torque acting on an electric dipole in a uniform electric field :
o o o
W puE +q oF o
q E
o
o E
W pE sin T T

When T 0D or 180 then W


D
0
K –q
T 90D then W max  pE  qE
K
F

T 270D then W min  pE

245
l In uniform electric field, the resultant force is zero. Only torque is experienced.
l In non-uniform electric field, it experience force and torque. When dipole is parallel to electric
field, torque is zero. Only force is experienced.
Work required for displacement of dipole in uniform electric field :
l The work done for displacement dT of a dipole in uniform electric field is
dW Wd T pE sin T d T

T2
? Total work W ³ pEsin T d T
T1

pE >  cos T @T2


T
?W
1

\W  pE > cos T2  cos T1 @

\ W pE cos T1  cos T2

(40) Electric field intensity at the centre of electric dipole is .

 Kp 2Kp
G G
(A) (B) Infinite (C) Zero (D)
a3 a3

(41) Two electric dipole of same dipole moment 6.2 u 103 C cm are placed on a line in such a
way that their axes are is same direction. If the distance between the centre of both dipole

is 108 m , then electric force between them is N.

(A) 21 u 1039 (B) 2.1 u 1034 (C) 21 u 1037 (D) 2.1 u 1017
(42) A unit positive charge is placed on an axis of electric dipole and its distance from the centre is
0.1 m. It experiences 0.025 N force, when it is placed at 0.2 m distance, the force experienced
by it is 0.002 N then the length of dipole is .

(A) 0.05 m (B) 0.2 m (C) 0.1 m (D) 0.4 m


(43) The charge q, q and –2q are placed on a vertices of equilateral triangle ABC. If the length of
each side is l then resultant dipole moment of this system is .

(A) 2l (B) 2ql (C) 3 ql (D) 4ql


(44) An electric dipole coincides with X-axis and its midpoint is placed at the origin O. A point P is 20
S
cm away from the origin and OP makes an angle with the x-axis. If the electric field near the
3
point P makes an angle T with axis, then the magnitude of T is .

246
S 3S
(A) (B) 2
3

(C) tan 1
3
(D) S  tan 1 3
2 3 2


o
(45) An electric dipole having dipole moment p 107 5iˆ  ˆj  2kˆ Cm placed in a uniform electric


o
field E 107 iˆ  ˆj  kˆ Vm 1 then magnitude of torque is Nm.

(A) 8.6 (B) 5 (C) 7.6 (D) Zero


(46) An electric dipole placed in a uniform electric field of intensity 4 u 105 NC–1 at angle 60° with

the electric field, experiences torque 8 3 Nm . If length of dipole is 4 cm then magnitude of


charge will be .
(A) 3 & (B) 1 mC (C) 2 & (D) 2 mC
8
(47) Dipole moment of an electric dipole is 2 u 10 Cm . The electric field intensity at a point of
distance 1 m and make an angle 60° with the centre of dipole is NC1 .
(A) 300 (B) 238.1 (C) 429.5 (D) 255.2
(48) An electric dipole consists of two opposite charges 1 & , each separated by a distance 2 cm is

placed in an electric field of 105 Vm1 . The work done for rotation of this dipole from equilibrium

to 180D is J.

(A) 4 u 103 (B) 2 u 103 (C) 103 (D) 5 u 103


(49) Three point charges +q, –2q and +q are situated at points 0, a, 0 , 0, 0, 0 , a, 0, 0 respec-
tively. The magnitude and direction of the dipole moment consisting this charges is .
(A) 2 qa , in  y direction

(B) a 0, 0, 0 and a, a , 0
2 qa , In direction of line joining points

(C) qa, In the direction of line joining points 0, 0, 0 and a, 0, a


(D) 2 qa , in,  x direction

(50) An electric dipole consists of charges r10 & , each separated by a distance 5 mm . The electric field
intensity at the points 15 cm distance on axis and 15 cm distance on equator is NC1 .

(A) 2.66 u 105 , 1.33 u 105 (B) 4.4 u 105 , 2.2 u 105
(C) 2.44 u 105 , 1.22 u 105 (D) 4.6 u 105 , 2.3 u 105
Ans. : 40 (A), 41 (B), 42 (C), 43 (C), 44 (D), 45 (A), 46 (B), 47 (B), 48 (A), 49 (B), 50 (A)

247
Continuous distribution of charges :
l The continuous distribution of electric charge can be of three types :
(1) Linear charge distribution :
The force acting on a charge due to linear charge
Z
distribution is,
o o
O ( r c) | dl c | § o o ·
³
o
o F kq o o 3 ¨© r  r c ¸¹
| r  rc|
L
l
p
r
o q
o o
O ( r c) | dl c | § o o ·
r

³
O o
X
Y ? Electric field E k o o 3 ¨© r  r c ¸¹
l | r  rc|

(2) Surface charge distribution :


The force acting on a charge due to surface charge
Z o distribution is,
da c
o o
V ( r c) | da c | § o o ·
³
o
F kq o o 3 ¨© r  r c ¸¹
o s | r  rc|
rc o q
r
o o
O V ( r c) | da c | § o o ·
³
Y o
X \ Electric field E k o o 3 ¨© r  r c ¸¹
s | r  rc|

(3) Volume charge distribution :


The force acting on a charge due to volume charge
Z dV' distribution is,

o
U ( r c) dv c § o o ·
³
o
o
F kq o o 3 ¨© r  r c ¸¹
rc
v | r  rc|
o q
r
o
U ( r c) dv c § o o ·
³
Y o
X
? E
\ Electric field k o o 3 ¨© r  r c ¸¹
v | r  rc|

l A conducting wire of length L carries total charge q which is uniformly distributed on it, then the
electric field at a point located on the axis of the wire at a distance a from the nearer end is

ª º
« a L  a »
1 q
4S 0 ¬« ¼»
E

l An arc of radius r subtends an angle q at the centre. A charge is distributed over the arc such
that the linear charge density is O . The electric field at the centre is,

248
kO
r ¬  sin T i  cos T  1 j ¼
ª ˆº
G
E ˆ

l A charge Q is uniformly distributed on the circumference of a circular ring of radius a. The


electric field at a distance x from the centre is, A
r
kQx a
T
a
E 3
P
2
 x2 2
x

l If point P is at very large distance x !! a then electric field, E kQx kQ

0  x
3
2 2 x2

Which is a equation of point charge. It shows that at very large distance, the charge on the
ring behaves like a point charge.

(51) Surface charge density on the surface of charged sphere is 0.7 Cm-2. When the charge increases
by 0.44 C the surface charge density is increased by 0.14 Cm-2. The initial charge and radius of
the sphere is .

(A) 2 C, 1 m (B) 2.2 C, 0.5 m (C) 1.5 C, 1 m (D) 2.5 C, 0.5 m


(52) 64 small drops of radius 0.02 m and charge 5 & are combined to form one big drop. If the
charge is not leak then the ratio of initial and final charge density on the drop is .
(A) 2 : 4 (B) 1 : 2 (C) 1 : 4 (D) 1 : 1
(53) A charged wire is bent in half circle arc of radius a. If the linear charge density is O , then the
electric field at the center of arc is .

O O O
(A) 2S  a (B) 2S  a 2 (C) 4S2  a (D) Zero
0 0 0

(54) Let there be a spherically symmetric charge distribution with charge density varying as

U( r )
U0 5  r
4 R up to r = R and U(r ) 0 For r > R, where r is the distance from the origin.

The electric field at a distance r r  R from the origin is given by

4U0 r ª 5 r º U0 r ª 5 r º 4SU0 r ª 5 r º U0 r ª 5 r º
(A) 3 0 «¬ 4  R »¼ (B) 3 0 «¬ 4  R »¼ (C) 3 0 «¬ 3  R »¼ (D) 
4 0 «¬ 3 R »¼

(55) The linear charge density on a thin circular ring of radius r is q q0 cos T . Here q0 is constant
and T is angle made in anti clock wise direction by maximum electric charge density from
diameter. The electric field at the centre of ring is .

q0 q0 q0 q0
(A)  r (B) 4  r (C) 2  r (D) 3  r
0 0 0 0

249
(56) The radius of thin semi-circular ring is 20 cm . It is uniformly charged with charge 0.7 n C .

Then the electric field at the centre of ring is NC1 .


(A) 10 (B) 75 (C) 50 (D) 125
(57) Charge Q is uniformly distributed on a circumference of ring having radius a. If an electron
placed on a centre of ring is displaced very small then it executes simple harmonic motion with
frequency f = .

Q eQ
(A) 2S (B) 2S
1 1
4S 0 ema 2
4S 0 ma3

Q
(C) (D)
eQ
4S 0 ma 4 S 0 ema3

(58) A charged wire is bent in the form of a semi circular arc of length ‘l’. If the charge on wire
is Q then electric field intensity at the centre is .

Q SQ Q
(A) (B) (C) 4 S  l 2 (D)
Q
2 0 l 2
4 0 l 2
0 4 S 0 l

Ans. : 51 (B), 52 (C), 53 (A), 54 (D), 55 (B), 56 (A), 57 (B), 58 (A)

l Electric flux :
The concepts of electric flux relates the electric field with its source.
“The flux linked with any surface is the surface integration of the electric field over the given
surface.”

³
o o
I E ˜d a
Surface

SI unit : (1) Nm2C1 (2) Vm

Electric field lines (flux) enters in to closed surface is


negative and electric field lines (flux) come outer is positive.

? resultant flux I 0

I 0
The flux of positive charge
o
I
E
o EA cos 0D
A

I EA

I ! 0 positive flux.

250
o
E
o The flux of negative charge
A

I EAcos180D

I  EA

o I  0 negative flux.
E

o
A o o Flux I EA cos90D
E AA
I 0
o
o
Flux I
E
A EA cos 0

o o I EA
E&A

Calculation of flux in different cases :


(1) The charge Q on center of a cube :

Total flux I
Q
0
l

Q
The flux passing from any one surface I
Q
6 0
l

The flux passing from any one point of vertices. I


Q
8 0
l

The flux passing from any one edge I


Q
l
12 0

(2) The charge on centre of any one side of cube :

q l The flux passing through the system of two cube

I
q
0

The flux passing throgh any one cube I


q
l
2 0

q
The flux passing through any one side of cube I
q
l
10 0

251
(3) The charge placed on any vertices of cube :

q
l To consider the charge q on center of cube another
seven cube is required.
l Thus the flux passing through the system of eigth

cube f = 0
q

? The flux passing through given cube f = 8 0


q

The flux passing through any one side of cube f = 24 0


q
q l

(4) The flux related with the surface of semi-sphere I Sr 2 E


Gauss’s Law :
l Gauss’s law is one of the fundamental laws of nature.
“The total electric flux associated with any closed surface is equal to the ratio of the net
electric charge enclosed by the surface to 0 ”

¦q
³ E ˜ da
oo
l Flux associated with any closed surface I 0 .

l Gauss’s law implies that the total electric flux through a closed surface is zero if no charge is
enclosed by the surface.
l Gauss’s law is true for any closed surface, no matter what its shape or size.
l The surface that we choose for the application of Gauss’s Law is called Gaussian surface.
l Gauss’s law is useful towarde a much easier calculation of electric field when system has
some symmetry.
Application of Gauss’s Law :
(1) Electric field due to linear charge distribution :

P l Electric field at distance r due to an infinitely long


straight uniformly charged wire,
G
r

o O o 2kO rˆ
rˆ or E
2 S 0 r
E
r

Where O
E
linear charge distribution

ED
1
r

252
(2) Electric field due to a uniformly charged infinite plane sheet :

l Electric field due to a uniformly charged infinite plane sheet is E V .


2 0

l This electric field is independent of the distance of the point from the plane. It depends
only on V .
(3) Electric fied due to two parallel plane sheet :

A B

I II III
EA
EA
EA EB
EB
VB
EB
VA

 E A  EB  V A  VB
1
For region - I E
2 0

For region - II V A  VB
1
2 0
E

For region - III V A  V B


1
2 0
E

If V A V and VB  V then,
For region - I E 0

V
For region - II E
0
For region - III E 0
Note : I case of conducting metal plate of definite thickness the total charge enclosed ¦ q 2V A .

2VA
Electric field E ˜ 2 A
0
?

V
? E 0

(4) Electric Field Due to a unifrom charged Thin Spherical Shell :


(i) For a point lying Inside a shell

¦q
³ E ˜ da
oo
0
r 0
R
q
o
?E 0

253
Thus, electric field inside the charged spherical shell is zero.
(ii) For a point Lying outside the shell :

1 q
E
E 4S 0 R 2

E Put, q VA V4 SR 2

4SR 2 .V
1
ED
r2 1
4S 0
E
E 0 r2
0 r
r R V R2
E
0 r 2

(5) Electric field intensity due to uniformly charged sphere :

o o
da
For point lying inside the sphere :
E
l

r Imagine spherical Gaussian surface of radius r ’ r ’  R


0
to determine the electric field at a point p ’ at a distance r ’.
Gaussian R
sphere inside Gaussian sphere
the surface outside the
surface
U
q q
Sr ’3 ˜ U
4
v 3 q’
4
3
SR 3

Sr ’3 ˜
4 q
SR 3
q’
3 4
3

q’ q ˜
r ’3
R3
The flux linked with the Gaussian surface

³ E ˜d a
o o
0
q’

? E ˜ 4Sr ’2
r ’3
R 3 0
q

? E ˜ 3
q r’
4S 0 R

Putting the value of q ,


Ur ’
E
3 0 Thus, E v r ’.

254
l For point lying outside the sphere :

Consider a Gaussian surface of radius r r ! R ,

E
³ E ˜d a
o o
0
q
Emax
1
? E ˜ 4Sr 2
Ev
0
q
r2
r
v
E

O r ? E Ÿ E v 2
1 q 1
4S 0 r
r R
2
r

(59) A hollow cylinder of radius 1 cm is placed in a uniform electric field of magnitude

2 u 104 NC 1 in such a way that its axis is parallel to electric field, then flux linked with
o
E
cylinder is .

(A) 2 u 104 S Vm (B) 2 u 102 S Vm (C) 0.02 u 103 NC 1 (D) zero


(60) The electric field in a region is given by the following equation :

ª3 ˆ 4 ˆº
« 5 i  5 j » u 2 u 10 NC
o
1
¬ ¼
3
E

The flux passing through a rectangular of 0.2 m2 area placed in yz plane inside the electric field
is Nm2C-1.
(A) 240 (B) 120 (C) 2.4 × 102 (D) 3 × 103
(61) A copper wire having linear charge density O is passed through a cube of length a, then the
maximum flux linked with the cube is .

Oa 2 Oa 6 Oa 2 3 Oa
(A) 0 (B) 0
(C) 0
(D) 0

(62) The inward and outward electric flux for a elosed surface are respectively 5 u 105 and

4 u 105 MKS unit. Then how much charge is inside the surface ?

(A)  8.85 u 107 C (B) 8.85 u 107 C (C) 8.85 u 107 C (D) 6.85 u 107 C
(63) As shown in figure the component of electric field Y
produced due to a charge inside a cube is 0.1 m 0.1 m

0 and E z 0 then the charge


1
Ex 600 x 2 , E y
O X
inside the cube a .
Z
(A) 600 & (B) 60 &
(C) 7 & (D) 6 &
255
(64) The charge Q and 2Q are enclosed by two concentric spheres A and B respectively.

(i) The ratio of electric flux linked with both sphere is .

(ii) If sphere A is fill with substance of dielectric constant K = 2, how much flux linked with it ?

2Q
B
Q

Q Q
(A) (i) 1: 3, (ii) 5  (B) (i) 1: 4, (ii) 4 
0 0

Q Q
(C) (i) 1: 3, (ii) 2  (D) (i) 2 :1, (ii) 
0 0

(65) A disk of radius having a uniformly distributed charge of 6 C is placed in the x-y plane with
a
4


its centre at 2a , 0, 0 . A rod of length a carrying a uniformly distributed charge 8 C is placed

on the x-axis from x a


4 to x 5a
4
x . Two point charges 7 C and 3 C are placed at

a4 , 4a , 0 and 34a , 34a , 0 , respectively. Consider a cubical surface formed by six surfaces
x r a2 , y r a2 , z r a
2 . The electric flux through this cubical surface is ______________.

2 C
(A) (B) (C) (D)
2C 10C 12C
0 0 0 0

(66) 8q charge is placed on any one vertex of a cube. The flux linked with this cube is
.

q q q q
(A) 8 0 (B) 4 S 0 (C) 
6 0 (D) 0

(67) An infinitely long wire of linear charge distribution O is passing through any side of cube of
length “a”, then the total flux passing through cube is .

Oa Oa Oa Oa
(A) 0 (B) 2 0 (C) 4 0 (D) 6 0

256
(68) The electric charge density on two parallel very long straight wire is 2 u 104 Cm1 respectively.
If the distance between these two wire is 0.2 m, then due to charge of first wire the force on
unit length of second wire is N.

(A) 72 u 102 (B) 8.4 u 109 (C) 9 u 109 (D) Zero

(69) The linear charge density on infinitely long straight wire is O Cm1 . If an electron moving round
in perpendicular plane to the wire and its centre is on the wire then the kinetic energy of electron
is .

O
(C) 0
eO eO eO
(A) 2S  (B) 2S  (D) 8S 
0 0 0

(70) Two wires of linear charge density O passing through a sphere of radius R and a cube of
sides R so that the flux linked with them is maximum. Then the ratio of flux of sphere to
the cube is .

1
(A) (B) (C) (D)
2 3
2 2 3 2

(71) Electric charge is uniformly distributed on a long straight wire of radius 1 mm. The charge per 1 cm
length of wire is Q C. Another cylindrical surface of radius 50 cm and length 1 m symmetrically
encloses the wire. The total electric flux passing through the cylindrical surface is .

(A) 0 0
Q 100Q 10 Q 100 Q
(B) (C) S  (D) S 
0 0

(72) The linear charge density on a infinitely long wire is


1 cm1
3
. Then the electric field intensity at a

point 18 cm perpendicular to the wire is N C 1 .

(A) 0.66 u 1011 (B) 3 u 1011 (C) 0.33 u 1011 (D) 1.32 u 1011
(73) Separation between two long conducting parallel plates is 2 cm . electron starts from rest and
moves from one plate to another plate in 2 V then electric charge density on the plate
is Cm2 .

(A) 2 u 1013 (B) 2.52 u 1013 (C) 1.52 u 1013 (D) 3.2 u 1013
(74) An electric dipole is prepared by taking two electric charges of r 5nC separated by distance
2 mm . This dipole is kept near a line charge distribution having density 4.5 u 104 Cm1 in such

a way that the negative electric charge of the dipole is at a distance 2.5 cm perpendicular to the
wire. The force acting on the dipole N.
(A) 0.12 (B) 0.5 (C) 1.5 (D) 0.25
257
(75) The radius of gold nucleus (Z = 79) is 7 u 1015 m . If the volume charge density on nucleus is
U and electric field on the surface of nucleus is E then electric field at the centre of nucleus
radius is .

E
(A) E (B) 2 E (C) (D)
E
3 2

(76) A ball of mass 1 mg and charge 20 nC is suspended by a string. When a uniformly charged

large plate is brought near the ball, string makes angle 30D with plane of plate. Then surface
charge density on plate is .

(A) 2.5 u 109 (B) 1.22 u 109 (C) 1.5 u 108 (D) 3.5 u 1012
(77) A large sheet carries uniform surface charge density V . A rod of length 2 l has a linear charge
density O on one half and O on the other half. The rod is hanged at midpoint O and makes
angle T with the normal to the sheet. The torque experienced by the rod is .

VOl VOl 2 VOl 2 VOl


(A) 0 cos T (B) 2 0 cos T (C) 2 0 sin T (D) 0 sin T
2 2

(78) An electron placed at distance d from a uniformly charged long plate is projected parallel to plate
with initial velocity u. If electron collide with plate after travelling distance l in horinzontal
direction then the surface charge density on the plate is .

d 0 mu 2d 0 mu d 0 mu 2 4d 0 mu 2
(A) (B) (C) (D)
el el el el 2

(79) A rectangular frame of sides 25 cm u 15 cm is placed perpendicular to uniform electric field

of 2 u 104 NC1 . If this frame is bent into circular frame then flux linked with it is

Nm2C1 .
(A) 750 (B) 1019.1 (C) 800 (D) 2015.5

(80) A particle of mass 9 u 10 5 g is held at some distance from very large uniformly charged

plane.The surface charge density on the plane is 5 u 105 Cm2 . What should be the charge on
the particle so that the particle remains stationary even after releasing it ?

(A) 1.6 u 1019 C (B) 1.56 u 1013 C (C) 6.25 u 1018 C (D) 2.52 u 1012 C

Ans. : 59 (D), 60 (A), 61 (D), 62 (A), 63 (C), 64 (A), 65 (A), 66 (D), 67 (C), 68 (A),
69 (B), 70 (C), 71 (B), 72 (C), 73 (B), 74 (A), 75 (D), 76 (A), 77 (C), 78 (D),
79 (B), 80 (B)

258
Assertion - Reason type Question :
Instruction : Read assertion and reason carefully, select proper option from given below.
(a) Both assertion and reason are true and reason explains the assertion.
(b) Both assertion and reason are true but reason does not explain the assertion.
(c) Assertion is true but reason is false.
(d) Assertion is false and reason is true.
(81) Assertion : Two charged particle A and B move in uniform electric field as shown in figure. The ratio
of charge and mass of particle B is greater than that
of particle A. Neglect the gravitational effect.
Reason : The vertical acceleration of particle A is greater A
than that of particle B.
(A) (a) (B) (b) B
(C) (c) (D) (d) d

(82) Assertion : If dielectric substance having dielectric constant K is put between two electric
charge then electric field is reduced.

Reason : According to equation E m electric field becomes .


Ea 1
K K
(A) (a) (B) (b) (C) (c) (D) (d)
(83) Assertion : Electric field lines cross each other.
Reason : In uniform electric field, electric field lines are parallel to each other.
(A) (a) (B) (b) (C) (c) (D) (d)
(84) Assertion : If proton and electron are placed in uniform electric field, their accelerations are
different.
Reason : The electric force on unit positive charge is independent of mass.
(A) (a) (B) (b) (C) (c) (D) (d)
(85) Assertion : Electric flux coming out and going inside from closed surface are 3 KVm and
8 KVm respectively. Electric charge enclosed by close surface is 0.53 & .

Reason : From Gauss's theorem I


Q
0 , equation Q I 0 can be used to verify this

statement.
(A) (a) (B) (b) (C) (c) (D) (d)
(86) Assertion : Two spherical shells have radii r1 amd r2 respectively. Their surface charge
densities are equal. Thus, electric field intensities near their surface is also same.

Reason : Surface charge density =


Electric charge
Surface area
(A) (a) (B) (b) (C) (c) (D) (d)
259
(87) Assertion : When a high energy X-Ray beam is made incident on a small metal ball suspended
in uniform electric field, the ball experiences some deflection.

Reason : X-Ray produces photo electron, so metal ball becomes negatively charged.

(A) (a) (B) (b) (C) (c) (D) (d)


(88) Assertion : A charge is put at midpoint of line joining two identical charges. For equilibrium of

§Q·
this system, the magnitude of this charge must be ¨ 4 ¸ .
© ¹

Reason : For equilibrium of any charge, the forces applied on it must be equal in magnitude and
opposite in direction.

(A) (a) (B) (b) (C) (c) (D) (d)


(89) Assertion : When a substance is negatively charged, its mass is slightly decreased.

Reason : Due to displacement of electron, the mass of substance is changed.

(A) (a) (B) (b) (C) (c) (D) (d)


(90) Assertion : When two substance attract each other, then maybe they are not charged.

Reason : Due to induction, charged substance attract neutral substance.

(A) (a) (B) (b) (C) (c) (D) (d)

Ans. : 81 (A), 82 (A), 83 (D), 84 (B), 85 (A), 86 (B), 87 (C), 88 (B), 89 (D), 90 (A)
Comprehension Type Questions :
Passage (I) :
The distance between two horizontal parallel plates is 1.5 cm and electric field between them is

105 Vm1 in downward direction. A oil drop of mass 4.9 × 10–15 kg and radius 5 × 10–5 m is placed
stationery between these two plate. Density of air is negligible compare to oil. Co-efficient of viscosity
is 1.8 × 10–5 Nsm 2 .
(91) Number of excess electron on the oil drop .
(A) 2 (B) 3 (C) 4 (D) 5
(92) If the direction of electric field is inverted then initial acceleration of oil drop is .

(A) 4.9 ms2 (B) 9.8 ms2 (C) 19.6 ms2 (D) Zero
(93) Terminal velocity of drop is nearly ms1 .
(A) 2.7 × 10–5 (B) 3.7 × 10–5 (C) 4.7 × 10–5 (D) 5.7 × 10–5
Passage (II) :
The nuclear charge ‘Ze’ is non uniformly distributed within a nucles of a radius R. The charge
density r(r) (charge per unit volum) depend only on the radial distance r from the center of the
nucleus as shown in figure. The electric field is along radial direction.

260
U( r )

r
a R
(94) At r = R electric field is .
(A) independent of a (B) directly proportional to a
(C) directly proportional to a2 (D) inverselay proportional to a
(95) For a = 0, the magnitude of d (the maximum magnitude of U ) .

3Ze2 3Ze 4Ze Ze


(A) (B) (C) (D)
4SR
3 2
2 SR 3SR 3SR 2
(96) Inside the nucleus, normally electric field depends linearly on r. For this, .

2R
(A) a = 0 (B) a (C) a = R (D) a
R
2 3
Passage - III
Two point particle of mass m are connected at the end of light rod of length l. These two
particles have charge +q and –q respectively. This arrangement is put into uniform electric field
at small angle T .
o
A +q E

T
0

B –q

(97) Magnitude of torque applied on rod is .


(A) qEl cos T (B) qEl sin T (C) qEl (D) Zero
(98) When rod becomes free, it rotates with angular frequency.

§ qE · 2 § 2 qE · 2 § qE · 2 1 § qE · 2
1 1 1 1

(A) ¨ ¸ (B) ¨ ¸ (C) ¨ ¸ (D) ¨ ¸


© ml ¹ © ml ¹ © 2ml ¹ 2 © 2ml ¹

(99) When rod becomes free, the minimum time required for becoming parallel to electric
field is .

S § ml · 2 § ml · 2 § 2ml · 2 § ml · 2
1 1 1 1

(A) ¨ ¸ (B) 2S ¨ ¸ (C) 2S ¨ ¸ (D) 2S ¨ ¸


2 © 2 qE ¹ © qE ¹ © qE ¹ © 2qE ¹

Ans. : 91 (B), 92 (C), 93 (D), 94 (A), 95 (B), 96 (C), 97 (B), 98 (B), 99 (A)

261
Match the columns :
Match appropriately the column-1 with column-2 :

(100) Column-1 Column-2

(a) Electric field of a point charge is (p)


1
r
proportional to _ at any point.

1
(b) Electric field of a short electric dipole is (q)
r2
proportional to _ at any point.

1
(c) Electric field of linear charge distribution is (r)
r3
proportional to _ at any point.

(A) a – q, b – r, c – p (B) a – r, b – q, c – p
(C) a – p, b – q, c – r (D) a – r, b – p, c – q

(101) Column-1 Column-2

(a) If Q1 0 and Q2 z 0 then (p) E z 0 and I z 0

(b) If Q1 z 0 and Q2 0 then (q) E will change but I will not

(c) If Q1 changes (r) E z 0 but I z 0

(d) If Q 2 changes (s) Both E and I will change

(A) a – s, b – p, c – q, d – r (B) a – r, b – p, c – q, d – s
(C) a – q, b – s, c – p, d – r (D) a – r, b – q, c – s, d – p
(102) Column-1 Column-2
(a) Electric field due to a uniformly charged (p) Zero
infinite plane sheet of thickness d

V
(b) Electric field of a point lying inside the (q) 0

sphere

Ur
3 0
(c) Electric field of a point lying inside a (r)

shell
(A) a – r, b – p, c – q (B) a – p, b – r, c – q
(C) a – q, b – r, c – p (D) a – p, b – q, c – r
262
(103) Two plates of surface charge density V1 and V2 are placed parallel to each other.

Column-1 Column-2

(a) V1 V2 V (p) In area I and II, E 0

(b) V1 V2 V (q) In area II, E 0

(c) V1 V and V 2 V (r) In area I, II and III, E 0

(A) a – p, b – q, c – r (B) a – p, b – p, c – r

(C) a – q, b – q, c – p (D) a – q, b – q, c – r

Ans. : 100 (A), 101 (B), 102 (C), 103 (C)

Matrix matching questions

Note : The question below have some statement/option. shown in Column-I and Column - II. Match
this statement/option correctly. Show the answer by filling the circle of matrix as given.

(104) By introducing a di-electric substance of constant K between two charges, its......

Column-1 Column-2

(a) Electric Charge (p) Becomes times


1
K

1
(b) Electric Field (q) Becomes times
K2

p q r p q r
Ans. :
a a

b b

c c

263
Difference between Electric field and Electric potential
l Electric field around an electrical device can be expressed in two ways :

o
(1) Electric field E (2) Electric Potential (V)

l Electric field is a vector quantity, while electric potential is a scalar quantity.


l Both the quantities can be defined near any point in electric field and both are interlinked (related) to
each other.
Electric Potential : The work required to be done in bringing a unit charge q from infinite distance
to a given finite point in the electric field is called Electric Potential (V) .
l Electric Potential at a point at a distance r in the given electric field i.e. integration of electric
field from infinite distance to distance r against the field is given by.

r o o
V  ³ E˜d r
f

l Electric Potential at infinite point is zero.

? V Ÿ
Work done(W) W
l Electric Potential V q W qV
Charge( q )

l SI unit = JC1 or Volt (V)

l CGS unit = Statvolt or abvolt

1 ab Volt (1 emu) 108 Volt


1 State Volt (1 esu) 299.8 300 Volt

l Dimension > V @ M1L2 T 3A 1 or M1L2T 2Q1

l Potential is a (comparative) relative quantity. It can be Ve , Ve or zero on a good


conductor.

l The absolute electric potential of earth is negative, because earth having 1 nC m2 negative

charge. But by taking this potential as a reference its magnitude is considered zero.
Factors affecting potential :
l Following factors affect the potential of any conducting substance :
(1) Magnitude of charge : Electric potential is directly proportional to magnitude of electric
charge.

264
(2) Area of a conductor : If magnitude of charge is variable then, potential of

conductor is V v
1
.
A

(3) Presence of a conductor in vicinity of charged conductor :


When a conductor is brought near charged conductor, then Potential of charged
conductor decreases.

(4) Medium around conductor : Medium around conductor affects potential as V


’ V
;
K
Where K dielectric constant.

Potential difference :

Po o
Potential at point P in electric field, VP  ³ E˜d r
f

Qo o
Potential at Point Q , VQ  ³
f
E ˜d r

Q o o
Potential difference between points P and Q : VQ  VP  ³ E ˜d r
P

Two types of Electric Potential :


(1) Positive potential : The work done against electric field in bringing unit positive charge from
infinity to a point in electric field, then potential of that point is called Positive potential.
l If the material has resultant positive charge, then also potential is taken positive.
l If a positively charged particle is connected to earth, then electrons from earth flow into that
particle. In such situation also, potential is considered positive.
l Negative potential : The work done in direction of electric field in bringing a unit positive charge
from infinity to a point in electric field, then potential at that point is called Negative Potential.
l If the resultant charge of particle is negative, then potential is considered negative.
l If a negatively charged particle is brought in contact with earth, then electrons from particle
will flow in to earth. In such situation also, potential is considered negative.
l The direction of positive potential is from higher charge to a lower charge while that of negative
potential is from lower charge to higher charge.
l Generally in practice, potential of positive charge is considered positive while potential of
negative charge is considered negative.

265
l Graph showing variation of potential :

+q –q +q +q
Y Y

V V

0 x X 0 x X
o
l If position vector of source charge q is r , then potential difference is given as,

§ ·
VQ  VP kq ¨ G G  G ¸
1 1
¨ rQ  r rP  r ¸
© ¹
G

Potential for a point charge :

Potential at point P ,
Y
f
o Po o
dr
P VP  ³ E˜d r
f

o fo o
q ? VP ³ E ˜d r
r
X
V P

o kq
but, electric field due to point charge, E rˆ
r2

f
? VP ³
kq
dr
P r2

f
? VP ³
1
kq dr
r r2

f
ª 1º
? VP kq «  »
¬ r ¼r

1 q
or VP
kq
VP
r 4S 0 r

Thus, potential due to a point charge is V v .


1
r

266
Zero potential due to system of charges
l If the charges are like, then zero potential is not acheived at finite distance.
l If two charges are unlike and having diffrent magnitude then zero potential can be obtained at the
points on a closed curve.
l Two points having zero potential are obtained on the line of joining of two charges, one among them
is lies between two charges and another is outside. Both points are nearer to lower charge.
Both near the lower charge.
l Q1  Q2 for,

For point on the line joining two charges,

d r
1
d Q2
Q1 Q2
r Q1

For point outside the line joining two charges,

P Q1 Q2 r
1
d Q2
d r Q1

Work done in electric field Ñ


The work done to move a charge q from point P having potential V1 to point Q having
potential V2 is,

W q'V
?W q V2  V1

l Now, kinetic energy of charge q due to potential difference.

K qV

? mv 2 qV
1
2

?Q
2 qV
m

? Momentum of electrically charged particle is p 2qVm .

l Electric Potential gradiant : ddrV The potential difference per unit length is known as Electric
potential gradiant.

Its SI unit is Vm1 .


(105) If work done to take 4C charge located near a point having –10 V potential, to a point having
potential V is 100 J then V = V.

(A) 10 (B) 15 (C) 20 (D) 5


267
(106) How much should be a sphere of radius 14 cm charged so that its surface charge density is
1 & P2 ?

(A) 12,420 V (B) 15,200 V (C) 15,820 V (D) 20,000 V


(107) 200 PC charge is uniformly spread on a conducting wire. The wire is then wound in a circle of
radius 10 cm, then potential at the centre of circle is V.
(A) 18 × 106 (B) 12 × 106 (C) 9 × 109 (D) 15 × 1010
(108) Two charges 30 nC and 20 nC are seperated by 15 cm. At which points on the line joining two
charges, the potential will be zero ?
(A) 45 cm, 200 cm (B) 40 cm, 100 cm (C) 80 cm, 150 cm (D) 9 cm, 45 cm
(109) In an electric field, the potential about points P and Q are 10 V and –4 V respectively, then what
is the work done to take 100 electrons from P to Q ?
(A) 2.24 × 10–16 J (B) –19 × 10–17 J (C) 9.6 × 10–17 J (D) –2.24 × 10–16 J
(110) As shown in figure, charges across the vertices of the square A and D and B and C are
o
exchanged, the electric field E and electric potential V about centre O will,
q q
A B
O

D C
–q –q

o o o
(A) E remains constant, V changes (B) Both E and V changes
o o
(C) Both E and V remains constant (D) E changes, V remains constant
(111) Two infinite long plates 1 and 2 are kept at a Y
distance 0.1 m from each other having potential
difference V2  V1 20 V . What will be the 0.1m X
velocity of an electron located on inside surface
of plate 1 from steady state when accelerated
1 2
towards plate 2 ?

(A) 32 u 1019 m/s (B) 2.66 u 106 m/s (C) 7.02 u 1012 m/s (D) 1.87 u 106 m/s

(112) What is the potential difference between two points A 2, 2 m and B 2, 0 m situated in

the electric field generated due to a point charge 103 & located at the origin ?

(A) 4.5 V (B) 9 V (C) 2 V (D) Zero

268
o
(113) Electric field in a region is E 30 x 2 iˆ , then potential difference VA  V0 Where

V0 Potential at origin point.

VA Potential at point A located at x = 2m


(A) 80 V (B) –80 V (C) 120 V (D) –120 V
(114) The work done to move a charge 5 C from point A to point B against electric field is 20 J. If the
electric potential at point A is 10 V, then what is the potential at point B ?
(A) Zero (B) 6 V (C) 14 V (D) 2.5 V
(115) Find the work done to move charge q from point x to point y as shown in figure.
+Q x r y –Q

a a

2kqQr 2kQqa
(A) a(a  r ) (D) r (r  a )
kQq
(B) r  2a (C) r
Qq

(116) The potential on “n” point drops having equal magnitude is V volt. They join together and form a
bigger drop, then find the potential on it.

(A) V
n
(B) Vn (C) Vn 12 (D) Vn 23

(117) The potential and electric field intensity of point P situated at some distance from electric charge
Q C is 600 V and 150 NC1 , then distance of point P from Q is m.
(A) 4 (B) 2 (C) 3.2 (D) 6.5
(118) As shown in figure, a square having side a has charges +q, +q, –q and –q on vertices of square
ABCD. It E is the midpoint of side BC, then what is the amount of work to be done to move a
charge e from centre of square O to point E ? .
A D
+q –q

a
o (
qe
A) 4S 
0
2 1
(B) Zero
+q
B a E C
–q
(C)
qe
4S 0
4 2 1 (D)
qe § 1 ·
S 0 a ¨© 5  1¸¹
2
a
(119) The sides PQ and QR of a right angle triangle PQR is 25 cm and 60 cm. A sphere of radius
2 cm and potential 9 × 105 V is placed at point Q. The work done to take 1 C charge from
R to P is W = .
(A) 2 kJ (B) 25 kJ (C) 42.12 kJ (D) 38.9 kJ
Ans. : 105 (B), 106 (C), 107 (A), 108 (D), 109 (A), 110 (D), 111 (B), 112 (D), 113 (B),
114 (C), 115 (A), 116 (D), 117 (A), 118 (B), 119 (C)

269
Electric Potential due to electric dipole

equator P l Electric Potential about point P

r 1 q 1 q
VP 
4 S 0 r 4 S 0 r
r
M r
q ª1 1º
«  r »
B 4S 0 ¬  ¼
T
r
A
axis
–q O +q
q ª r  r º
2a
4S 0 «¬ r r »¼

l Point P is at larger distance r !! 2a , So we can take AB || OP || BP .


? r r r
and r  r AM 2 a cos T
q § 2 a cos T ·
? V( r )
4 S 0 ¨© r 2 ¸¹

1 p cos T
? V( r )
4 S 0 r2
o o
l writing OP as unit vector r̂ i.e. p ˜ rˆ p cosq.
o
o 1 p ˜ rˆ
? V(r )
4S 0 r 2

l Potential on axis of dipole : Here T 0 or T S

? V r
4S 0 r 2
1 p

l If charge near given point is +q, then potential will be V and if charge is –q, then potential will be –V.
S
l Electric Potential at equator : At a point on equator, T . ? V 0
2
o o
l At any point, potential depends upon the angle between r and p .

The potential produced due dipole decreases with distance as .


1
l
r2
Electric Potential generated due to system of charges.
(i) q1
q2
K
r2 As shown in figure q1 , q2 , ... qn charges are at distance
r1
K
q3 r1, r2 , ... rn from any point, then electric potential at
p r3
K
point P is ...... .
rn
K
qn

270
k q1 k q 2 kqn
V r1  r2  ...  rn

ªq q º
? V K « r1  r2  ...  rn »
q
¬ 1 2 n ¼

¦
n
or V
1 qj
4S 0 rj
j 1

(ii)
As shown in figure if the position vectors of charge
Y o o o
q1 , q2 , ... qn are r1 , r2 , ... rn respectively and the
P o
position vector of point P is r then the total electric
q2
q1 r2
K
potential at point P is,
G
r q3
r1
K
r3
K
kq1 kq2 kqn
V   ... 
o o o o o o
X r  r1 r  r2 r  rn
rn
K

¦
n
Z ?
qn qj
V k
o o
j 1 r  rj
Potential due to continuous charge distribution :
(i) Potential due to an infinite line charge distribution having density O and length l at
distance r from point P,
G

Odl
³ o o
1
4S 0
V
l r  rc

(ii) Potential due to a charged plane sheet having surface charge density V and area A at
distance r from point P.
G

VdA
³ o o
1
4S 0
V
A
r  rc

(iii) Potential due to a charged object having volume charge density U and volume V at distance
r from point P,
G

UdV
³ o o
1
4S 0
V
V r  rc

Potential due to a shell having equal charge distribution :


(i) Potential at distance r from a shell having radius R and total charge on sphere is q,
G

1 q
4 S 0 r Where r > R.
V

271
(ii) On the surface of shell V Where r
1 q
R.
4S 0 R

(iii) Inside the shell, charge is zero. So work done to move that charge inside the shell is zero.
Thus, all points are equipotential. So potential is,

Where r  R.
1 q
V
4S 0 R

Distribution of electric Electric field and Graph


Charge Electric potential
E
1. Point charge Q
Q
E k 2
r
o Q r
V
E V k r
o P
r

Q r
kO
2. Line charge Ex r (sin D  sin E)

kO
r (cos E  sin D )
Ey
Ey
O Ex
D
l E
P ª r 2  l 2 1 º
2kO log « »
« r2  l2 1 »
V
¬ ¼

k Qr
E 3
(r 2  R 2 ) 2
On the centre of plane
r 0 then E centre 0

r
R
3. Circular charged ring r
2
E
2kQ
E max
3 3 R2
r !! R
Q r
kQ R
R
E 2
r2
P
G kQ
V
E V
(r 2  R 2 ) 2
1

r !! R

V
kQ r
r

272
Distribution of electric Electric field and Graph
Charge Electric potential
2kO
4. Arc shaped charged E centre sin T
R
rod Special cases

O (i) when T 45D


(quarter ring)

T P 2kO
E E centre
R
R
(ii) when T 90D (half ring)

2kO
E centre
R

(iii) when T 135D

( 4 th ring)
3

2kO
E centre
R
Vcentre 2kOT (T in radian)

V ª º
«1 »
r
5. Continuosly charged ring 2H 0 «
r 2  R 2 »¼
E
¬

r o 0 , near E
V
V 2H0
,

It will behave as charged shell


P
G
E for points near the ring
V ª
V 1 r2  R2  r º
2H 0 «¬ »¼

E
6. Infinite charged plane sheet At any point
V
E
2H0
r
Vr V
P V
2H 0
C
G
E

273
Distribution of electric Electric field and Graph
Charges Electric potential
V
7. Two infinite charged EA 
H0

plane sheet EB 0
V V
V
EC
H0
A B C

kQ
8. Charged conducting sphere Ea (r ! R)
r2

kQ
Eb (r R)
R2 E
R (r  R)
b a
Ec 0
r
C R
(r ! R)
kQ
Va r

kQ
Vb (r R) V
R
r
( r  R) R
kQ
Vc
R
kQ
9. Contiuosly charged Ea (r ! R)
r2
kQ
conducting sphere Eb (r R)
R2
kQr
Ec (r  R) E
R3
r
R (r ! R)
kQ
Va R
c b a r

kQ
Vb (r R)
R

kQ(3R 2  r 2 ) V
Vc (r R)
2R 3

r
3
R
Vcentre Vb
2

274
Distribution of electric Electric field and Graph
Charges Electric potential

2kO
10. Infinite long charged E out side ( r ! R)
r E
conducting hollow cylinder Ein side 0 (r  R)
O r
2kO R
Esurface (r R)
R

P Vout side 2k O log r (r ! R)


r
E Vin side 2k O log R (r  R)

Vout side 2k O log R (r R)

R is radius of cylinder

2kO
11. Infinitely long continuosly E out side r ( r ! R)

2kOr
charged conducting solid cylinder Ein side (r  R) E
R2

2kO r
Esurface
R
(r R) R
O
Vout side 2kO log r (r ! R)

§ r2 ·
2kO ¨ 1  2 ¸ ( r  R)
P ¨ R ¸
© ¹
Vin side
r E

Vout side 2kO log R (r R)

R radius of cylinder

(120) Q charge is uniformly distributed on a thin ring of radius R. If, initially, electron is steady at point
A which is quite far from centre and axis of ring, then find the velocity when electron passes
through the centre of ring.

kme
(A) (B) (C) (D)
2k Qe k Qe k Qe
mR m QR mR

(121) A quadruple is shown in figure. What is the potential at a point which is at distance r from the
axis of quadruple ?

275
+q a –2q a +q p

2qa 2 2qa 2

2 qa 2 qa 2
(A) 4S 0 r r 2  a 2 (B) 4S 0 r r 2  a 2 (C) 4S 0 r 3
(D) 4S 0 r 3

(122) A solid sphere of radius R is uniformly charged. At what distance from the surface, the electric
potential is half of the potential at centre of sphere ?

R 4R
(A) (B) R (C) (D)
R
2 3 3

(123) The charge per unit length of an arc ring having radius R is O . What is the electric potential at
its centre ?

kO kO k SO
(A) (B) (C) (D) kSO
2SR R R

(124) Two dipoles of dipole moment 5 u 1012 Cm are placed in such a way that their axis are parallel
to co-ordinate axis and intersect at the origin, Then potential at point 20 cm away and making an
angle 30° with axis is V.
(A) 1.536 (B) 1.12 (C) 1.25 (D) 2.12
(125) The potential at a point on the bisector of a thin rod of length 2 l and at a distance “a” from the
centre of thin rod is . (Linear charge density = O )

O l 2  a2 O l 2  a2  l
(A) S 0 ln (B) 4 S 0 ln
l 2  a2 l 2  a2  l

O l 2  a2
(C) S 0 ln (D) Zero
l 2  a2

(126) The electric potential for half sphere having radius R and surface charge density
s is .
VR VR R VR
(A) 4 0 (B) 0 (C) 4V 0 (D) 2 0

(127) Two identical plane S1 and S2 having surface charge density V1 and V 2 V1 ! V 2
are placed at distance d on a line normal to both plane. A line of length a ( a  d ) make an

angle 45D has charge q . The work done by field for motion of the charge perpendicular to field.
W= .

V1  V2 0 q V1  V2 a V1  V2 a q V1  V2 a
(A) 2qa
(B) 2 0
(C) 2 0 q (D) 2 0

276
(128) The total charge on an insulator ring of radius 0.5 m is 1.11 u 1010 C , which is distributed
unequally on circumference. Then magnitude of electric field surrounding this

³
o o
 E ˜ dr =
l 0
V. (l = 0 is the centre of ring.)
l f
(A) –1 (B) +2 (C) –2 (D) Zero
(129) As shown in figure A, B and C are concentric shells of radius a, b and c respectively.
(a < b < c). Their surface charge densities are V ,  V and V respectively. What is the electric
potential on the surface of shell A ?
C
B
A
a

V b
V c
V
V V V V
(A) 0 a  b  c (B) 0 a  b  c (C) 0 b  a  c (D) 0 a  b  c

(130) The radius of two concentric metal shells are R1 and R2 respectively and electric charge on
them are Q1 and Q2 . The surface charge density V of both the shell is equal. Find the electric
potential at the centre.

V § R1 · V V V § R2 ·
(A) 0 ¨ R ¸ (B) 0 R1  R 2 (C) 0 R1  R 2 (D) 0 ¨ R ¸
© 2¹ © 1¹
(131) The radius of a conducting hollow sphere is “a”. If the potential defference between two points,
one at distance “a” from centre and other at distance “3 a” from centre is V, then electric field
at distance 3 a from centre is .

(A) (B) (C) (D)


V V V V
6a 3a 4a 2a
(132) The distance between two points A and B is 2 L. +q and –q charges are placed at points
A and B respectively. The midpoint of distance AB is C. What is the work done for +Q charge
to move in semicircle arc CRD ?
qQ qQ  qQ qQ
(A) 2S  L (B) 6S  L (C) 6S  L (D) 4S  L
0 0 0 0

(133) The distance between two thin rings of radius R is d. The charge on these rings are +Q and –Q
respectively. The potential difference between centre of these two rings is .

Q ª1 º
(B) 4S 0 « R  »
1
(A) Zero
«¬ R 2  d 2 »¼

ª1 º
(D) 2S 0 « R  »
QR Q 1
(C) 2S  d 2
0 «¬ 2 »
R d ¼
2

277
Ans. : 120 (A), 121 (B), 122 (C), 123 (D), 124 (A), 125 (B), 126 (D), 127 (D), 128 (B),
129 (D), 130 (C), 131 (A), 132 (C), 133 (D)
Equipotential surface : The surface on which all the points have same potential is called
equipotential surface.
l The potential difference between any two points on equipotential surface is zero.
l The work done to move a charge on equipotential surface is always zero.
l Equipotential surfaces never intersect each other.
l Electric field on the equipotential surface is always normal to it.
l The surface of any charged conductor can be considered as equipotential surface as charge is
equally distributed on it’s surface.
l For point charge q, equipotential surfaces are spherical surfaces drawn with q as its centre.
l It is not necessary that equipotential surfaces are spherical because equipolential surfuce due to
linear charge distribution is cylindrical.
Relation between Electric field and Electric potential

Non-uniform electric field, E


dV
l 
dr

l For uniform electric field, E V


d
ŸV Ed

Relation between electric field and electric potential at a point in space E 


dV
l
dr

§ wV ˆ wV ˆ wV ·
Ingeneral (In cartesian co-ordinate system) E ¨ i  j kˆ ¸
© wx wy wz
G
¹
l

(134) Equipotential surface are shown in figure, then electric field intensity .
Y
cm
10V 20V 30V

300
O X
10 20 30 cm

(A) 200 Vm 1 , at 120D angle with x-axis (B) 100 Vm 1 , at 50D angle with x-axis

(C) 50 Vm 1 , In x-direction (D) 100 Vm 1 , In y-direction

(135) Two equipotential surfaces are placed parallel near to each other at distance as shown
in figure. The work done to bring a point charge q from surface A to surface B
is .

278
1 q 1 q
(A) 4S  r (B) 4S  2
0 0 r
A B

(C)  4S  r
1 q
r (D) Zero
0

(136) In the electric field of a point charge q, a certain charge is carried from point A to B, C, D and
E. Then the work done .
A

(A) is least along the path AB


(B) is least along the path AD

O
(C) is zero along any one of the path
B +q E
AB, AC, AD and AE
C D (D) is least along the path AE
(137) Point charge ‘q’ moves from point P to point S along the path PQRS in a uniform electric field E
pointing co-parallel to the positive direction of the x-axis. The co-ordinates of the points P, Q, R
and S are a, b, 0 , 2a, 0, 0 , a,  b, 0 and 0, 0, 0 respectively. The work done by the field
in the above process is .
Y o
P
(B) qEa
E
(A) qEa
S
Q X (C) qEa 2 (D) qE (2a)2  b 2

R
o

(138) An electric field is represented by E i in a region. Then magnitude of electric potential in
x3
this region is Assume the electric potential zero at infinite distance.
2A A A
(A) 2 (B) 2 (C) Zero (D)
x 2x x3
o
(139) What is the electric potential of electric field E y iˆ  x ˆj ?

(A) V  xy  C (B) V  ( x  y )  C (C) V  ( x 2  y 2 )  C (D) V C

(140) The electric potential at point ( x, y, z ) is V  x 2 y  xz 3  4 . Electric field intensity at this


point is .
o o
(A) E 2 xy iˆ  ( x 2  y 2 ) ˆj  (3xz  y 2 ) kˆ (B) E z 3 iˆ  xyz ˆj  z 2 kˆ
o
(C) E (2 xy  z 3 ) iˆ  xy 2 ˆj  3z 2 x kˆ
o
(D) E (2 xy  z3 ) iˆ  x 2 ˆj  3xz 2 kˆ
279
(141) The electric potential at point ( x, y, z ) is V 4 x 2 V , then electric field at point (1, 0, 2)
is Vm1 .
(A) 16, in +X direction (B) 8, in +X direction
(C) 8, in –X direction (D) 16, in –X direction
(142) The potential at a point X due to some charges situated on the X-axis is given by

volt.
20
V( x )
x2  4
The electric field E at x 4 Pm is given by .

(A) V P -1  in  ; direction (B) V P -1  in  ; direction


10 5
9 3

(C) V P -1  in  ; direction (D) V P -1  in  ; direction


5 10
3 9
(143) The graph of electric potential o distance R is shown in figure. At R 5 m distance the
electric field is .
V
5
4
3
(A) 2.5 Vm 1 (B) 2.5 Vm 1
2
(C) 2 Vm 1 (D) 2 Vm 1
1 5 5

0 R
1 2 3 4 5 6
(144) The electric potential at point (x, z) in xz plane is given by V  Kxz , then at distance r from
origin the electric field intensity E v .

1
(A) r 2 (B) (C) r (D)
1
r r3
(145) The relation between electric potential V and distance y is V 5  4 y 2 , then the force on the
electric charge 2 & at distance y 0.5 m is N.

(A) 4 u 106 (B) 2 u 106 (C) 6 u 106 (D) 8 u 106


(146) Some equipotential surface are shown in figure. The electric field intensity at each surface is
Vm 1 .

30cm
20cm
3 6
10cm (A) (B)
r2 r2
60V 9
30V (C) (D) Zero
r2
20V

280
(147) The linear charge density O is given on a arc of radius r. If the angle made by the arc at origin
S
is , then electric potential at centre is .
3

O O O O
(A) 4  (B) 8  (C) 12  (D) 16 
0 0 0 0

Ans. : 134 (A), 135 (D), 136 (C), 137 (B), 138 (B), 139 (A), 140 (D), 141 (C), 142 (A),
143 (A), 144 (C), 145 (D), 146 (B), 147 (C)

Electric Potential due to a system of charges

l Potential energy of point charge q is U p qVp (at any point p in electric field)

l Potential energy of system of two point charges :

k q1q 2
U r
q1 q2
1 q1q2
r U
4 S 0 r (In vaccum)

1 q1q2
U
4 S 0 K r (In dielectric medium)

l Potential energy of system of three point charges :

k q1q 2 k q 2 q3 k q1q3
U r12  r23  r13

l Potential Energy of system of ‘n’ point charges :

1 ª q1qn  q2 qn  qn1qn º
4S 0 «¬« rn  r1 rn  r2 rn  rn1 »»
¼
Un G G G G G G

N ( N  1)
l Total pairs of electric charge due to ‘N’ electric charges is .
2

l Relation between work and potential energy : If any electric charge performs motion from one point
to another in an electric field, the work W U f  Ui .

281
Potential energy of an electric dipole in an external electric field

As the electric field is only in X-direction,


B+q
'V (VB  VA )
 
2a 'x
E
T AC

VB
–q
A C E
2a cos T
> ' VA 0@

? VB  E 2a cos T .
? Potential energy of +q at B, is
U qVB
U  E q 2a cos T
o o
U E ˜ p

l If the axis of the dipole is normal to the electric field, then T S


2
and

? U Ep cos S 0
2

l If the axis of the dipole is parallel to the field then T 0


? U min  pE
Effect in the case of a metallic conductor placed in an external electric field.
(i) A steady electric charge distribution is induced on the surface of the conductor.
(ii) The net electric field inside the conductor is zero.
(iii) The net electric charge inside the conductor is zero.
(iv) On the outer surface of the conductor, the electric field at every point is locally normal to the
surface.
o
0 at every point inside the conductor, E  dr thus dr ? V
dV
(v) Since E constant.
dV
0

(vi) If there is a cavity inside the conductor then even when the conductor is placed in an external

electric field ( o
E ), the net electric field inside the conductor is zero and also inside the cavity is
zero. This fact is called electrostatic sheilding.

(148) on the vertices of an isosceles right angled triangle Q, +q and +q Q


charges are placed as in figure. If the total electrostatic energy of the
whole system is zero, Q = .

q 2q
(A) (B)
1 2 2 2

(C) 2q (D)  q +q


a
+q

282
(149) On the vertices of a cube of sides b, –q charges are placed. The electric potential energy of
+q charge placed at the centre of cube is .

4 2q 2 8 2q 2 8 2q 2 4 q 2
(A) S  b (B) 4S  b (C) S  b (D)
0 0 0 3 S 0 b

(150) Two charges q1 and q2 are placed 30 cm apart, as shown in figure. A third charge q3 is
moved along the arc of a circle of radius 40 cm from C to D. The change in the potential energy

q3 C
k q3
of the system is 4 S  then K is = .
0

40 cm
(A) 6q2 (B) 8q1

D
q1 q2
(C) 8q2 (D) 6q1 A
30 cm 10 cm
B
(151) Three charges –q, Q and –q are placed on a stright line at same distance. If the total potential
q
energy of the system is zero, then = .
Q

(A) 4:1 (B) 1:4 (C) 2:1 (D) 1:2


(152) Three charges  q,  2q and Q are placed on the vertices of equilatral triangle. If total potential
energy of system is zero, then Q = .

q 2 q q 2 q
(A) (B) (C) (D)
3 3 3 3

(153) Three point charges q, 2q and 8q are placed on a line of length 9 cm. If the potential energy of
system is minimum then the distance between charges is .

(A) 0.03 m, 0.06 m (B) 2 cm, 7 cm (C) 0.05 m, 0.04 m (D) 0.07 m, 0.02 m
(154) Four charges +q, –q, +q and –q are placed at vertices ABCD of a square of side r. The
potential energy of the system is .

(A) Zero (B)


kq 2 2 4 (C)
kq 2 2
(D)
kq 32
r r r

(155) Two charges 5 nC and 2 nC are placed at 2, 0, 0 cm and x, 0, 0 cm . If the electric


potential energy of the system is 0.5 - then x = cm.

(A) 10 (B) 30 (C) 20 (D) 50

283
(156) The length of electric dipole is 4 cm. If it is placed at angle 60o in unifrom electric field then
potential energy U = J. magnitude of charge r 8 nC and electric field

E 2.5 u 1010 NC1 .


(A) –4 (B) 2 (C) –8 (D) 6
(157) An electric dipole consists of two opposite charge 1 & separated by a distance 2 cm. The dipole
is placed in an electric field of 105 NC1 . The work done for displacement of dipole from

equilibrium to 180D is W = .

(A) 2 u 10 3 J (B) 5 u 10 2 J (C) 7 u 10 6 J (D) 4 u 10 3 J


(158) Two equal point charges placed on x-axis at distance x = –a and x = +a. A point charge
Q is at origin. When the charge Q travel a distance x on x-axis the electric potential energy
difference is propotional to .

(A) x (B) x 2 (C) (D) x


3 1
3
(159) An insulated solid sphere of radius R have positive charge density U . The potential energy
difference to bring a charge q from centre to the surface of sphere is .

UR q qU U 0
(A) 6 0 (B) 3  (C) Zero (D) 3q
0

Ans. : 148 (B), 149 (D), 150 (C), 151 (A), 152 (B), 153 (A), 154 (B), 155 (C), 156 (A),
157 (D), 158 (B), 159 (A)

Parallel plate capacitor Spherical capacitor Cylindrical capacitor


+Q –Q
ba

b
A a l

d
2SH0l
Surface charge density V Capacitance : Capacitance C
Q
§b·
l l l

log e ¨ ¸
A
§ ab · ©a¹
4SH0 ¨ ¸
© ba ¹
l The electric field between C In Presence of dielectric

the two paltes in C.G.S . C material of dielectric


ab
ba
constant K is
V Q V
E
H0 H0 A d l electric potential
2SH0 Kl
Q §1 1· Cc
 §b·
electric potential V 4SH 0 ¨© a b ¸¹ log e ¨ ¸
l
V
©a¹
Ed

284
Parallel plate capacitor Spherical capacitor Cylindrical capacitor
l Energy stored l If dielectric substance of Special Note :
2
dielectric constant K fill When two plates of
1 Q 1
U CV 2 QV
2 2C 2
l The force on one plate due between the area of two different cross section area
to other plate is Concentric sphere, then
is given and they consist a
2 2
capacitor then only effective
Q 1 CV QE
Capacitance :
2H 0 A
F
2 d 2
cross section area is
§ ab ·
C’ 4SH0 K ¨ ¸
©ba¹ consider.

l electric energy density l electric potential


Q §1 1·

UH H0 E 2 4SH 0 K ¨© a b ¸¹
1 V
2
H0 A
l Capacitance C Spacial Case :
d
b
In C .G.S . C
A
4 Sd a
When the dielectric A
l A
substance of dielectric
constant K is fill between l The induced Charge inside

two plates, Electric field E’ the sphere is Q ’  Q


E a
K b
and Capacitance C’ KC and capacitance
§ b2 ·
KH 0 A C’ 4SH 0 ¨
¨ b  a ¸¸
© ¹
C
d

Combinations of capacitors
Series Combination of Capacitors :
C1 C2 C3
+Q –Q +Q –Q +Q –Q
l The charge on every capacitor
has the same value and equal to
the charge of battery. Q
l The potential difference between V1 V2 V3
the two plates of different
capacitors is.
+ –
? V = V1 +V2 +V3
V
l If the effective capacitance of this combination is C S , then

1 1 1 1
 
CS C1 C 2 C3

C1C 2 C 3
? CS =
C1C 2 +C 2 C 3 +C 3 C1

285
l Electric potential V v 1
C and energy stored U v 1
C.
l If two capacitors are connected in series, then the effective capacitance between them is
C1C 2 Multiplication
CS =
C1 +C2 Addition
.

§ C2 · § C1 ·
l Electric potential V1 = ¨ ¸ V and V2 = ¨ ¸ V.
© C1 +C2 ¹ © C1 +C2 ¹
l If n Capacitors of equal capacitance ‘C’ are
connected in series, then equivalent capacitance
C
is CS n and potential difference between two

ends of the each capacitor is V’ n .


V

As shown in figure, if at equal distance ‘d’ ,‘n’ plates are placed, then it is considered as ( n  1) Capacitor
connected in series.

AH0
? Equivalent Capacitance C’
C
( n  1) d ( n  1)
.

Parallel combination of Capacitors :  Q1  Q1


l In such a combination the potential difference (V)
between the plates of every capacitor is the same and C1
is equal to the potential difference.  Q1  Q2  Q2
l Electric charge Q on every capacitor is different,
Therefore total charge Q Q1  Q2  Q3 Q  Q2
C2
 Q3  Q3
l If effective capacitance is C P then C P = C1 +C 2 +C 3  Q3
l Electric charge on capacitor Q v C and energy stored C3
U v C.
l If two capacitors are connected in parallel combination
V
then equivalent capacitance is C P = C1 +C 2 .
§ C1 · § C2 ·
l Electric charge Q1 ¨ ¸ Q and Q2 ¨ ¸Q.
© C1 +C 2 ¹ © C1 +C 2 ¹
l If n capacitors of equal capacitance ‘C’ are
connected in parallel the equivalent capacitance
is C P nC and electric charge on each
Q
capacitor is Q ’ n .
As shown in figure, if at equal distance ‘d’, ‘n’ plates are
placed, then it is considered as ( n  1) capacitor connected
in parallel.
H A
? Equivalent capacitance C’ ( n  1) 0 (n  1) C .
d
286
Parallel plate capacitor

Air medium between Partially filled dielectric dielectric medium between


to plates medium between two plates two plates
t
K
C K
A A
A C
d
d d
H0 A
H0 A ª 1º KH 0 A
C
C d  t «1  » C’ CK
d ¬ K¼ d

Separation of distance Separation of area.


d l
2 air 2 l
K air
K
d d
ª 2K º ª K  1º
C’ «
¬ K  1 »¼
C’ «
¬ 2 »¼
C C

Separation of distance Separation of area.

A A K1 A1
K1 K2
K2 A2
d1 d2 d
Behaviour : Series connection Parallel connection

ª º H0
C’ H0 A « »
K1K 2
equivalent capacitance
¬ K1d 2  K 2 d1 ¼
C’ (K1A1 + K 2 A 2 )
d

Special case : If d1 = d 2 = then If A1 = A2 = then


d A
2 2

H0 A ª 2K1K 2 º ª 2K1  K 2 º H 0 A ª K1  K 2 º ª K1  K 2 º
« » « »C
d ¬ K1  K 2 ¼ ¬ K1K 2 ¼ d «¬ »
¼
«
¬
»C
¼
C’ C’
2 2

287
l Mediums of different dielectric constant between the plates :

+Q –Q
Resultant capacitor
H0 A
K1 K2 K3
C
 2  3 A
d1 d d
d1 d2 d3 K1 K 2 K 3 E
d d
l In dielectric slab of different dielectric constant and different thickness insert between two plates of
Capacitor then.

A H A
equivalent C’
§ t ·
0

Capacitance d  (t1  t2  t3  ...)  ¨ 1  2  3  ... ¸


t t
© K1 K2 K3 ¹
t1 t2 t3
d

l Conducting slab of thickness t between two plates :


t equivalent Capacitance
K=o H0 A
(d  t )
C’
E=0
Note : If thickness of conducting plate is negligible then t  0 .
H0 A
C’ C
d d
l n capacitors of equal capacitance connected in parallel the equivalent capacitance Cmax nC

and when connected in series equivalent capacitance is C min C


n

?
Cmax CP
n2
Cmin CS

l Between two plates of parallel plate Capacitors a medium of dielectric constant K is insert and
battery

Physical quantity in air medium removed Connected


electric charge Q Q’ Q Q ’ KQ

Electric field E
E
E’ E’ = E
K
Capacitance C C’ KC C’ KC

electric potential
V
V’ V’= V
K

Energy stored U’ U
K U’ KU

288
l Special Case : As shown in figure if capacitance of each capacitor is ‘C’ then.

8 Capaciter
16 Capaciter

A B

Equivalent Capacitance A B
C AB 2C Equivalent Capacitance

C AB 1 n (n  1) C
2
Where n is number of capacitors connected in
parallel
l If we want to get equivalent capacitance ‘C’ at voltage ‘V’, then ‘n’ capacitors of equal
capacitance ‘ C ’’ and voltage ‘ V’’ should be connected.

CV 2
Where n
C’V’2
l When air is the medium between two plates of the parallel plate capacitor, then the force acting on
them is
2 2
Q 1 CV
2 0 A
F
2 d

l Equivalent potential of two charged capacitors :


(i) If same charged plates are connected then,

Q1  Q2 C1V1  C 2V2 Q1’  Q2’


V , V
C1 +C 2 C1 + C2 C1 + C 2

(ii) If different charged plates are connected then,

Q1  Q2 C1V1  C2V2 Q1’  Q2’


V , V
C1 +C2 C1 + C2 C1 + C 2

l If capacitors of capacitance C1 and C2 are connected in parallel, then there charge distribution

Q1’ C1
is C2 .
Q2’

l When two capacitors are connected in parallel then total charge on them is conserved. If V1 = V2 ,
then total energy is conserved , otherwise it decreases.
l Radius of two spherical conductors is r1 and r2 and electric charge on them is q1 and q2 , then in
air medium if they connected with a copper wire the eqvivalent capacitance is C 4S 0 r1  r2 .

289
The loss of energy when two charged capacitors are connected Uloss V1 –V2 2 .
C1C2
l
2(C1 +C2 )

l Simple electric circuits

Two capacitors connected in series V1 = V


C2 C1
l , V2 = V .
C1 +C2 C1 +C 2

l Three capacitors connected in series

C2 C3V C1C3V C1C 2 V


V1 =
C1C 2 + C2 C3 + C1C3
, V2 =
C1C2 + C2C3 + C1C3
, V3 =
C1C2 + C2 C3 + C1C3
.

l Charging and discharging of R – C series connection :


Charging :

§ t ·
instantaneous electric potential V V0 ¨1  e CR ¸ V = maximum voltage
© ¹
l 0

§ t ·
instantaneous electric potential Q Q0 ¨1  e CR ¸ maximum charge
© ¹
l Q0

§ t ·
instantaneous current I 0 ¨1  e CR ¸
© ¹
l I

l W CR is called time constant. Its unit is second.


Discharging :
t
l instantaneous potential V = V e RC
0

t
l instantaneous charge Q Q0 e RC
t
l instantaneous current I  I 0 e RC

l At charging 1 W (one time constant) after capacitor V’ 63.212 % V .

After 2 W V 86.466 % V, 3 O After V’ 95.021 % V and after f W it gets 100 % V .

l At discharging after 1 O time the voltage on capacitor is V’ 36.78 % V .


l Kirchoff’s law of circuit with capacitors :
l At branch point ¦ q 0

For closed circuit, ¦ ¦E


q
l
C
C
l Sign convention : + – + –
–E +E
q q
'V 'V
C C

290
(160) In an isolated parallel plate capacitor the electric charges on the surfaces of the plates are
Q1, Q2 , Q3 and Q4 as shown in figure. If the capacitance is C. What is the potential difference
between the plates ?

B
Q1 Q4
A Q2 Q3

Q1  Q2  Q3  Q4 Q2  Q3 Q1  Q2  Q3  Q4 Q2  Q3
(A) 2C
(B) 2C
(C) 2C
(D) 2C

(161) In the figure, what is the equivalent capacitance between A and B ?


2 PF
A
(A) 2 ) (B) 1 )
2 PF (C) 3 ) (D) 4 )
1 PF 1 PF
B
2 PF
(162) The area of each plate shown in the figure is A and the distance between consecutive plates is
d, then the equivalent capacitance between points P and Q is .
P 3A 0 4A 0
1
2 (A) (B)
5d 2d
3
4 5A 0 5A 0
Q (C) (D)
d 3d
(163) 4 capacitors of 9 P F are connected as shown in figure. The equivalent capacitance between
points A and B is PF .

B
A C1 (A) 18 (B) 9

C3 (C) 15 (D) 4.5


C2
C4
(164) Multiplication of equivalent capacitance of capacitors made by four plates as shown in figure .
1 1
2 A 2
C
3 3
4 4 D
B
(A) C (B) 1 (C) C 2
(D) 9 C
4
2

(165) The area of each plate of a parallel plate capacitor is A and the
separation between the plates is a. One plate is connected with battery a
of V volt and the negative terminal of battery is grounded. If the second
plate of capacitor is ground then the charge on plates of capacitor V
is .

0 AV 3 0 AV 2 0 AV 0 AV
(A) a (B) a (C) a (D) 2a
2

291
(166) 80 & charge is applied on the upper plate of 4 ) capacitor as shown in figure. In steady
circuit the charge on upper plate of 3 ) capacitor is &.
80 PC
(A) 80 (B) 40
4 PF
(C) 48 (D) 32

2 PF 3 PF

(167) Two parallel plates placed at distance 1 cm are connected to a DC source of potential difference
X. If a steady proton at a centre of two plates move at angle 45o in presence of field, then
X = .

(A) 1 u 1015 V (B) 1 u 1010 V (C) 1 u 107 V (D) 1 u 109 V


(168) As shown in figure 2 ) capacitor is charged. When switch S is in position-2 the percentage
loss of stored energy of capacitor is .
1 2
(A) 0 % (B) 20 %
8 ) (C) 75 % (D) 80 %
2 )

(169) The dimension of two capacitor A and B are same as shown in figure. A dielectric subtance of dielectric
constant K = 3 is placed between two plate of capacitor -B. The potential difference between plates A
and B is respectively.

10V
(A) 7.5 V, 2.5 V (B) 2.5 V, 7.5 V
A B
(C) 2 V, 8V (D) 8 V, 2 V

x y
(170) The distance between two plates of parallel plate capacitor is t. The capacitance is 100 pF. Now a
metallic slab of thickness t
3
is placed between two plates then its new capacitance is .

(A) 100 pF (B) 150 pF (C) 200


3
pF (D) 100
3
pF

(171) The area of each plate of a parallel plate capacitor is A and the separation between the plates is
a. If a dielectric slab of dielectric constant K and thickness t (t  d ) is insert between the two
plates then new capacitance of the capacitor .

A 0 0 A A 0 A 0
(A) d  t 1  1 (B) d  t 1  1 (C) d   1  1 (D) d  t 1  1
K K K K

292
(172) In the figure below the capacitance of every capacitor is 3 ) . The equivalent capacitance
C between A and B is .

A B (A) 9 ) (B) 1 )
P Q 3

D (C) 1 ) (D) 12 )

(173) For the system given below the equivalent capacitance between A and B is .
A

C2 C2
C C C
C C C (A) 5 1 (B) 2 1

B (C) 3 C
2 (D) 3 1 C2
(174) n1 capacitors of capacitance C1 are connected in series with a battery of 4 V and are charged.
n2 capacitors of capacitance C2 are connected in parallel with a battery of 1V volt. If the energy
stored in both type connection is same then C2 = .

2C1 16n2C1 2n2C1 16C1


(A) n1n2 (B) n1 (C) n1 (D) n1n2

(175) n drops of capacitance C are combine to form a big spherical drop. The capacitance of this big
drop is .

(A) n 3 C (B) n C (C) n 2 C (D) n3 C


1 1

(176) When a dielectric slab of thickness t d


2
placed between two plate of parallel plate capacitor its

capacitance becomes 4
3 times then the initial. Where d = distance between two plates. Then the
magnitude of dielectric constant of slab is .
(A) 4 (B) 8 (C) 2 (D) 6
(177) A capacitor of capacity C has charge q and stored energy is W. If the charge is increased to
2q, the stored energy will be .

(A) 2W (B) 4W (C) W


2
(D) W
4

(178) Two different dielectric substance are placed between a parallel plate capacitor as shown in
figure. Then the ratio of capacitance of this capacitor and a capacitor without dielectric
is .
l l
2 2
K1K 2
(A) K  K (B) K1  K 2
1 2
K1 K2
K1  K 2
(C) 2
(D) 2 K1  K2

293
(179) Four capacitors of equal capacitance are connected with 10 V battery as shown if figure. If point
B is ground then potential of point A and C is .

+ 10V
–
(A) 10 V,  10 V (B) 5 V,  10 V
B (C) 0 V, 0 V (D) 5 V,  5 V
A C

(180) In a variable capacitor n plates are placed in such a way that two consecutive plates are
sparated by distance d then its capacitance is .

(A) ( n  1) C (B) n C (C) (D) zero


C
n
(181) The distance between two parallel plates of area A is d. A copper sheet is placed above a dielectric
slab of dielectric constant K = 2. The equivalent capacitance of this arrangment is .

C3 d/3 A 0 2A 0
(A) (B)
Cu C2 d/3 d d
K=2 d/3
3A 0 5A 0
(C) (D)
C1
d d
(182) A parallel plate capacitor filled with three different dielectric A
A/2 A/2
materials having dielectric constants K1, K 2 and K3 as
d/3
shown in figure. The resultant capacitance is .
C1 K2 C2

K1 2d/3
K3 C3
A K 2 K3 2 0 K1K 2 A
(A) 2 d K1  2K 2
(B) 2 d K 2  3K 3 B

3 0 AK 2 K 3 A 0 ª 3K 2 K3  K 3K1  2K 2 K1 º
(C) 2d K 3  2K 2
(D) 2d «¬ K 3  2K 2
»
¼

Ans. : 160 (D), 161 (A), 162 (A), 163 (C), 164 (C), 165 (A), 166 (C), 167 (D), 168 (D),
169 (A), 170 (B), 171 (D), 172 (C), 173 (A), 174 (D), 175 (A), 176 (C), 177 (B),
178 (C), 179 (D), 180 (A), 181 (B), 182 (D)
Assertion - Reason type Question :
Instruction : Read assertion and reason carefully, select proper option from given below.
(a) Both assertion and reason are true and reason explains the assertion.
(b) Both assertion and reason are true but reason does not explain the assertion.
(c) Assertion is true but reason is false.
(d) Assertion is false and reason is true.
(183) Assertion : An electron in rest position travels from a point having electric potential 10 V to
the point having 30 V potential, then its kinetic energy is 3.2 × 10–18 J.

Reason Ñ Kinetic energy E q 'V e V2  V1 1.6 u 1019 30  10 3.2 u 1018 J.


(A) (a) (B) (b) (C) (c) (D) (d)

294
(184) Assertion : The capacitance of parallel plate capacitor is decreases as the plate is increase.
A 0
Reason Ñ The capacitance of parallel plate capacitor is C
d
(A) (a) (B) (b) (C) (c) (D) (d)
(185) Assertion : Due to uniform linear charge distribution on infinitely long straight wire the
electric potential at point r is propotional to log r.
Reason Ñ Ev 1
r and E  ddrV .
(A) (a) (B) (b) (C) (c) (D) (d)
(186) Assertion : As shown in figure the effective capacitance between points 1PF
A and B is 1.5 ) .
1PF
1PF

Reason Ñ Two capacitors in first line are in series and this connection is
A
parallel to third capacitor.
(A) (a) (B) (b) (C) (c) (D) (d)

Ans. :183 (A), 184 (D), 185 (A), 186 (B)

Comprehension Type Questions :

The electric potential due to point charge r q , at a distance r is V r r . Where


kq

K 9 u 109 MKS. work done to transfer a electric charge q from point A to B is


W q VB  VA . This work can be observed as kinetic energy/potential energy of charge.

Potential energy of charges q1 and q2 placed in air at a distance r is U r J . The


k q1q 2

magnitude may be positive, negative or zero depands upon the sign of charges.
(187) Potential at 1 m due to a electric charge 1 PC placed in air is V.
(A) 103 (B) 9 × 103 (C) 9 × 106 (D) 3 × 103
(188) Potential at mid point of dipole prepared by taking two electric charges of r10 & separated by
distance 1 cm is .
(A) zero (B) 10 9 (C) 10 V (D) 100 V
(189) Work done to bring 10,000 PC charge from A to B at 250 V potential is 2 J. Then the potential
at point A is V.
(A) 10,000 (B) 10–2 (C) 500 (D) 50
(190) Find out the work to travel two electrons from 1 m distance to 2 m distance placed in air. Where
e is electric charge and k is constant of electric.
Ke2  Ke 2
(A) Ke2 (B) (C) (D) zero
2 2
(191) Two protons of charge e and mass m are placed at distance 1 m in air. If K is constant of
electric force the velocity gain by proton when it is free .
(A) e k
m (B) 2e k
m (C) e
2
k
m (D) zero

Ans. : 187 (B), 188 (A), 189 (D), 190 (C), 191 (A)

295
4&5 Magnetic Effect of Current and Magnetism
(1) Biot-Savart’s Law
Biot Savart’s Law is used to determine the magnetic field any point due to a current carrying
conductor
o
o P 0 I dl u r
I dB
4S r2
Idl T
P P0 dl sin T
dB
r2
and for entire conducting wire,
I
o o P0 I
³d B ³
dl sin T n
4S
B
r2
o
T 0 or S sin T 0 ? B 0
T 90q sin 90q 1 ? B maximum
SI unit of B = Wbm–2 or Tesla
1 tesla = 104 Gauss.
Where P 0 = Magnetic permeability of vacuum
= 4 S u 107 T m A1 or Wb A 1 m1 or H m1 or N A2
(2) For conducting wire,

(A) For a wire of finite length

T1 T 2 P0 I
B
4 Sd
>sin T1  sin T2 @
d
P0 I
4Sd
>cos D1  cos D2 @
D1 D2
I I

(B) For a wire infinite length T1 T2 90

or D1 D2 0q

P0 I
? B ? BD I
2 Sd d

(C) For a wire half infinite length. T1 0 and T 2 90q

P0 I P0 I
B
4S d
0  1 4 Sd

322
(3) For Rings

r a o radius
a
x o distance of given point on axis from center.
0 x P

a  x2 2
1
2
r

P 0 Ia 2
(A) For number of turn N 1;
2 a 2  x2 2
B 3

P 0 NIa 2
(B) For number of turn N N ;B
2 a 2  x2 2
3

P 0 NI
(C) At center of ring x 0; B
2a

P 0 N Ia 2
(D) If x ! ! a ; B
2 x3

P 0 NI
(E) At a distance on axis equal to radius of ring. B 5
2 2. a

(F) The ratio of magnetic field at center of ring and at any point on axis of ring

3
BCenter § x2 · 2
BAxis ¨1  2 ¸
© a ¹

(1) A copper rod carries a DC current. The magnetic field associated with the current will be ..........

(A) Only inside the rod (B) Only out-side the rod

(C) Both inside and outside the rod (D) Neither inside, nor inside the rod

(2) A wire carrying a current I is bent in to a circle of radius r as shown in figure. The net magnetic
field at center O of the circular loop is ...............

r
P0 2I P0 2I
0 (A) S 1 (B) S 1
4S r 4S r

I I (C) Zero (D) Infinite

323
(3) Magnetic field at point P situated at perpendicular distance D from one end of wire of length L
and carrying current I is

P0 I
P I P0 I L2
(A) 0 (B) (C) 4 SD (D) infinite
4 SL 4 SD L2  D2

(4) An equilateral triangle loop of length ‘a’ is carrying current I in anticlock wise direction. Magnetic
field produced at center of the triangle is .

9 P0 I P0 I 3P 0 I 5 2P0 I
(A) (B) (C) (D)
2S a 3 3S a 2S a 3S a

(5) An electric charge of 35 & is moving with speed 2 u106 ms 1 along a path shown in figure.
Then magnetic field produced at point P is 7.

G (A) Zero (B) 242.5


r
50 mm
(C) 2425 (D) 2524
q 35 F 600
I
G
v

(6) quadrant will behave like North pole.

Y
I
Second quadrant First quadrant

Xc X (A) First (B) Third


I
Third quadrant Forth quadrant (C) Second (D) Forth

Yc
(7) A current of 6 A passes through the wire shown in figure. Then magnitude of magnetic field at

point O is T. The radius of arc is 0.2 m. P0 4S u 107 TmA –1

(A) 1.41 u 104 (B) 1.41 u 105


O (C) Zero (D) 1.41 u 103

6A

(8) As shown in figure, current I passes through hexagon having side a. Magnetic field at the center
of it is .
324
P0 I 3P 0 I
(A) (B)
I a 3 3S a Sa

3 3P 0 I P0 I
(C) (D)
Sa 3S a
(9) On connecting a battery to the two ends of a diagonal of a square conducter frame of side a, the
magnitude of the magnetic field at the center will be .
P0 2P0 4P0 I
(A) zero (B) (C) (D)
Sa Sa Sa
(10) 108 T magnetic field produced at point P situated at 4 cm perpendicular to very long wire
carring current I. How much magnetic field will be produced at a distance 12 cm. perpendicular
to the same wire ?
(A) 1.33 u 10 8 (B) 1.11 u 104 (C) 3 u 103 (D) 9 u 102

(11) In hydrogen atom, an electron revolving in the orbit of radius 0.53A with speed of 6.6 u 1015
0

revolution per second. Magnetic field at the center B = T.


(A) 0.125 (B) 1.25 (C) 12.5 (D) 125
(12) Two linear conductors AOB and COD are mutually perpendicular. Currents passing through them
are I1 and I 2 respectively. Point P lies at perpendicular distance a from point 0 of ABCD plane.
Magnetic field intensity at point P is .

(A)
P0
I1  I 2 (B)
P0
I1  I 2 (C)
P0
I12  I 2 2 (D)
P0
I12  I 2 2
1/ 2 1/ 2

2S a 4S a 2S a 2S a
(13) Electric current of 5 A passes through A current carrying straight wire. A point lying at a distance
10 cm from wire on perpendicular bisector of wire, makes angle 60° with both ends of wire.
Then intensity of magnetic field arising at that point is T.
(A) 3 P 0 (B) 3.98 P 0 (C) 39.8 P 0 (D) Zero
(14) Magnetic field lines associated with a very long straight current carrying wire will be .
(A) Along the length of the wire (B) centripetal
(C) Circular in the plane perpendicular to straight wire (D) Hyperbola
(15) Which of the following graphs shows the variation of magnetic induction B o distance (r) from
very long straight current carrying wire ?

(A) (B) (C) (D)


B B B B

r r r r
325
(16) The magnetic field due to a straight conductor of unifrom cross section of radius a and carrying
steady current is represented by graph.
B B B B

(A) (B) (C) (D)

a r a r a r a r

Ans : 1 (C), 2 (B), 3 (C), 4 (A), 5 (C) 6 (B), 7 (B), 8 (B), 9 (A), 10 (A), 11 (C), 12 (C),
13 (B), 14 (C), 15 (B), 16 (A)

Ampere’s Circuital Law


The line integral of magnetic field on a closed curve (loop) in a magnetic field is equal to the

product of a algebraic sum of the electric current ¦ I enclosed by that closed loop and the

permeability P 0 of vacuum.

v³ B˜ dl P0 ¦ I
o o
\

§ P0 I ·
Magnetic field inside the conductor at a distance r from the axis of wire is B ¨ 2S a 2 ¸ r , r  a
© ¹

N
Magnetic field at a point inside a solenoid of infinite length, B P0 nI where n
l

sin D1  sin D 2
P 0 nI
Magnetic field at a point inside solenoid of finite length, B
2

§ N ·
Magnetic field produced in a toroid, B P0 ¨ ¸I
© 2S r ¹
P0 nI

(17) Two coplaner and concentric coils of 20 turns each having radii of 40 cm and 80 cm are carrying
current 0.4 A and 0.8 A in opposite direction respectively. The net magnetic field at the center is
T.

(A) 4 P 0 (B) 2 P 0 (C) P0 (D) P0


10 5
4 4
(18) When a steady current carrying straight wire turned into one circular loop, the magnetic induction at
the center of loop due to current is B. If the same wire is turned into n loops to make a circular
coil, the magnetic intensity at the center of this coil for same current will be .
(A) nB (B) n 2 B (C) 2nB (D) 2n 2 B

326
(19) If ratio of magnetic intensities at center and at distance x from center on axis of current carrying
circular ring of radius R is 8:1 then x = .
R 2R
(A) 3R (B) (C) 2 3 R (D)
3 3

(20) Magnetice field is B1 at centre of current carrying coil of radius a and B2 at a distance a on its

B1
axis from centre, then ratio B = .
2

(A) 2 :1 (B) 1 : 2 2 (C) 2 2 : 1 (D) 1 : 2

(21) Two concentric rings carry current I1 and I 2 . If the ratio of their radii is 1 : 2 and ratio of

I1
magnetic field at centre is 1 : 3 . Then I .
2

(A) (B) (C) (D)


1 1 1 1
4 6 2 3

(22) Current I passes through solenoid having radius a and length L. Magnetic field produced at the
end point of solenoid is

2P0 nIL P0 nIL 2 P 0 nIL


(A) (B) Zero (C)
2 L  a (D) L2  a 2
1
L a
2 2 2 2 2

(23) Magnetic field at the mid point of axis of solenoid having radius 1.0 m and length 2.0 m
is .
P 0 nI P 0 nI P 0 nI
(A) (B) (C) 2 P 0 nI (D)
2 2 2 2

(24) Two similar coils are kept mutually perpendicular such that their centres coincide. At the centre,
what is the ratio of the magnetic field due to one coil and the resulant magnetic field by both
coils if the same current flows through them ?

(A) 1: 2 (B) 1: 2 (C) 2 :1 (D) 3 :1


(25) A wire is wound on solenoid having 10 A current capacity. If length of solenoid is 80 cm and
having cross section radius 3 cm. Then length of required wire is Take B 2T .

(A) 1.2 u 102 (B) 4.8 u 102 (C) 2.4 u 103 (D) 6 u 103

(26) Ampere’s circuital law is equivalent to .


(A) Bio Savart’s law (B) Coulomb’s law (C) Faraday’s law (D) Kirchoff’s law
(27) The dimensional formula of magnetic intensity B is .

(A) M1L2 A 1 (B) M1T 2 A1 (C) M 2 T A 2 (D) M 2 LT 2 A 1

327
(28) Which one is the correct graph between the magnetic induction (B) along the axis of current
carrying long solenoid and distance x from one end of solenoid ?
B B B B
(A) (B) (C) (D)

x x x x

Ans. : 17 (C), 18 (B), 19 (A), 20 (C), 21 (B), 22 (C), 23 (D), 24 (A), 25 (C), 26 (A), 27 (B)
28 (A)
l Force on a charged particle in magnetic field.

If a particle carrying a positive charge q and moving with velocity v enters in a magnetic field
B then it experiences a force F which is given by the expression.
o §o o·
F q ¨ v u B¸
© ¹

?F Bqv sin T
If charge is negative
o §o o· § o o·
 q ¨ v u B¸ q ¨B u v ¸
© ¹ © ¹
F

\ F Bqv sin T
Force on charged particle will be zero
(1) If B 0
(2) If particle is neutral then q 0
(3) If charge particle is static then v 0
(4) If charge particle moving parallel or anti parallel to magnetic field then T 0 or S
l Lorentz Force : When the moving charged particle is subjected simultaneously to both electric
o o
field E and magnetic field B . So the Lorentz force acting on it

o ª o § o o ·º
q «E  ¨ v u B ¸»
¬ © ¹¼
F

mv p 2mK 1 2mv
l Cyclotron : Radius of circular path of charged particle. r
Bq Bq Bq B q

If charged particle accelerated by voltage V and obtain kinetic energy K then

p mv 2mK 2mqV

328
Angular frequancy of charged particle, Zc
Bq
m
2S m
Periodic time of charged particle, T
Bq
(29) A particle of mass m has an electric charge q. This particle is accelerated through a potential
difference V and then entered normally in a uniform magnetic field B. It performs a circular

motion of radius R. The ratio of its charge to the mass is . ( is also called
q q
m m
specific charge.)

(A) (B) (C) (D)


2V V VB mV
2 2
BR 2BR 2R BR
(30) A proton, a deuteron ion and an D - particle of equal kinetic energy perform circular motion normal
to a uniform magnetic field B. If the radii of their paths are rp , rd and rD respectively then

Here q d q p , md 2m p .

(A) rD rp  rd (B) rD rd ! rp

(C) rD ! rd ! rp (D) rD rd rp
(31) Maximum force acting on electron moving in magnetic field of 5 u 105 T with velocity
4 u 104 ms 1 is N.
(A) 1.6 u 1019 (B) 3.2 u 1019 (C) 1.6 u 1017 (D) 3.2 u 1017
(32) If Lorentz force acting on charged particle is zero and electric field is 5Vm –1 then
o o
|Bu v | .
(A) Zero (B) Infinite (C) 5 (D) 10
(33) Force acting on moving proton having velocity of 10 i ms  1 in magnetic field of 5 j T will be
.
(A) 5 u 1018 k (B) 2 u 1018 k (C) 8 u 1018 k (D) 10 u 1018 k
(34) A proton is moving perpendicular to a uniform magnetic field of 5 T with 2 MeV kinetic energy.
The magnetic force acting on proton is N.
(mp 1.6 u 1027 Kg , qp 1.6 u 1019 C )
(A) 8 u 1011 (B) 16 u 1011 (C) 8 u 1012 (D) 16 u 1012
o
(35) A proton having velocity v 2i  3 j m s 1 is moving in magnetic field of B 2i  3 j T .
o

Magnetic force acting on proton is N.


(A) 1.6 u 1019 (B) 9.1 u 1031 (C) Zero (D) infinite
(36) A particle having 2 C charge passes through magnetic field of 4k T and some uniform electric
field with velocity 25 j ms 1 . If the Lorentz force acting on it is 400 i N . The electric field in
this region is  Vm1 .
(A) 200 i (B) 200 k (C) 100 i (D) 10 j

329
(37) A proton (mass = 1.67 u 1027 kg and charge = 1.6 u 1019 c ) enters perpendicular to a magnetic
field of intensity 2 T with a velocity 3.4 u 107 ms1 . The acceleration of the proton is
ms2
(A) 6.5 u 1015 (B) 6.5 u 1013 (C) 6.5 u 1011 (D) 6.5 u 109
(38) A deutron of kinetic energy 50 KeV is describing a circular orbit of radius 0.5 meter in a plane
o
perpendicular to magnetic field B . The kinetic energy of the proton that describes a circular orbit
o
of radius 0.5 meter in the same plane with the same B is KeV.
(A) 25 (B) 50 (C) 100 (D) 200
(39) Two electron having same velocities v and moves parallel to each other at distance r. The ratio
of magnetic force and electric force acting on them is .

v c v2 c2
(A) (B) (C) 2 (D) 2
c v c v
(40) Path of charged particle entering perpendicular to magnetic field will be .
(A) circular (B) linear (C) elliptical (D) parabolic

Ans. : 29 (A), 30 (A), 31 (B), 32 (C), 33 (C), 34 (D), 35 (C), 36 (C), 37 (A), 38 (C),
39 (C), 40 (A)

l Force acting on current carrying wire of length l placed in uniform magnetic field
o o o
F I l uB
o o
?F BIl sin T where q is angle between l and B .
l Force between two parallel current carrying conductors and seperated by a distance y.
o P0 I1 I 2 l
|F|
2S y

Force per unit length,

F P 0 I1 I 2
l 2S y

If conductors carry current in same direction, then the force between them will be attractive.
If conductors carry, current in opposite direction, then force between them will be repulsive.
o o o
l Torque acting on current carrying loop, suspended in a uniform magnetic field, 0u%
o o
where M NI A = Magnetic dipole linked with coil.

(41) The magnetic dipole moment of a current carrying loop is independent of .


(A) Magnetic field in which it is lying (B) Number of turns
(C) Area of the loop (D) Current in the loop
330
(42) A rectangular loop carrying current I is situated near a long straight wire such that the wire is
parallel to one of the sides of the loop and is in the plane of the loop. If a steady current I is
established in wire as shown in figure, the loop will .
I

(A) Rotate about an axis parallel to the wire (B) Move away from the wire or towards right
(C) Move towards the wire (D) Remain stationary
(43) A conducting circuler loop of radius r carries a constant current I. It is placed in a uniform
o o
magnetic field B such that B is perpendicular to the plane of the loop. The magnetic force
acting on the loop is .
o o o
(A) Ir B (B) 2S rI B (C) zero (D) S r 2 IB
(44) A circular coil of radius 4 cm and of 20 turns carries a current of 3 ampere. It is placed in a
magnetic field of intensity of 0.5 T. The magnetic dipole moment of the coil is
ampere m2
(A) 0.15 (B) 0.30 (C) 0.45 (D) 0.60
(45) Two thin long parallel wires seperated by distance b are carrying current I amp each. The
magnitude of the force per unit length exerted by one wire on the other is .
P0I 2 P0I 2 P0 I P0 I
(A) (B) (C) (D)
b 2
2S b 2S b 2S b2
(46) A close loop PQRS carrying a current is placed in a uniform magnetic field. If the magnetic force
on segment PS, SR and RQ are F1 , F2 and F3 respectively and are in the plane of the paper and
along the directions shown, the force on the segment QP is .
Q
P
(A) F3  F1  F2 2 (B) F3  F1  F2
2
F3

(C) F3  F1  F2 (D) F3  F1  F2 2
F1 2

S R
F2
(47) As shown in the figure, two very long straight wires are kept parallel to each other and 2A
current is passed through then in the same direction. In this condition, the force between them is
F. Now if the current in both of them is made 1A and direction are reversed in both, then the
force between them .
(A) Will be and attraction
F
4

(B) Will be and repulsive


F

2A
2
2A 1A 1A
(C) Will be and attractive
F
2
first
(D) Will be and repulsive
F
4

331
(48) As shown in the figure 20 A, 40 A and 60 A current are passing through very long straight wires
P, Q and R respectively in the direction shown by the arrows. In this condition the direction of
the resultant force on wire Q is .

(A) towards left of wire Q


(B) towards right of wire Q
20 A 40 A 60 A (C) normal to the plane of paper.
(D) in the direction of current passing through Q
P Q R
(49) As shown in the figure, a straight wire PQ of length 2 m carrying 2 A current is placed parallel
to a very long wire at a distance of 2 m. Find the force acting on wire PQ. If the current passing
through the long wire is also 2 A.

Q
(A) 6 u 107 N (B) 16 u 107 N
I1 4 A I2 4 A
y=4m (C) 16 u 108 N (D) Zero

(50) A conducting wire of 4 m length is used to form circular loop. If it carries a current of 1.0 A it’s
magnetic dipole moment will be = Am2

S S 4
(A) 2S (B) (C) (D)
2 4 S

(51) Dipole moment of a coil is 2i  3 j  5k . If the coil is suspended in the uniform magnetic field

having magnitude 5kT torque acting on it will be = .


(A) 35 (B) 117 (C) 25 (D) 135
(52) An electron moves with a constant speed v along a circle of radius r. It’s magnetic moment will
be (e is the charge of electron)

(A) e v r (B) (C) S r 2 v (D) 2S ev


evr
2

(53) A circular coil having N turns is made from a wire L meter long. If a current of I is passed
through this coil suspended in a uniform magnetic field of B tesla, the maximum torque that can
act on this coil = .

BIL2 BIL2
(A) (B) zero (C) (D)
ILB
2S N 4S N 8S 2 N

332
(54) 1 A current carrying circular loop having radius 20 cm is kept in XY plane as shown in the figure
Torque acting on loop is Nm.
Y o
(A) 0.15 (B) 0.25
B 2T

(C) 0.35 (D) 0.55


X
I

(55) As shown in figure a rectangular coil having one turn is kept in uniform magnetic field of

j T . Torque acting on it will be Nm.


0.05 
2
Z

0.5 A (A) 11.32 u 104 k (B) 22.64 u 104 k

0.08 m (C) 5.64 u 105 k (D) Zero


I
Y
X 0.04 m

(56) A loop carrying current I lies in the XY plane as shown in the fig. The unit vector k is out ward
and perpendicular to the plane of the paper. The magnetic moment of the current loop is
.
Y

§S · 2 
(A) I a 2 k (B) ¨  1¸ a I k
a X ©2 ¹
0
§S · 2 
(C)  ¨  1¸ a I k (D) 2S  1 a 2 I k
©2 ¹

(57) Straight conducting wire of length 0.5 m and carrying current 1.2 A is placed perpendicular in
uniform magnetic field of 2 T. Magnetic force acting on it will be N.
(A) 2.4 (B) 1.2 (C) 3.0 (D) 2.0
(58) Two very long parallel wire seperated by 10 cm and carrying current 10 A in same direction.
Force acting on unit length of one wire due to other will be N.
(A) 2 u 104 N Attractive (B) 2 u 104 N Repulsive
(C) 2 u 107 N Attractive (D) 2 u 107 N Repulsive
(59) A small coil of N turns has an effective area A and carries a current I. It is suspended in
o
a horizontal magnetic field B such that its plane is perpendicular to B. Find the work done
in rotating it by 180q about the vertical axis .

(A) NIAB (B) 2 NIAB (C) (D) 4 NIAB


2NIA
B
333
(60) A square coil 20 cm ´ 20 cm has 100 turns and carries a current of 1 A. It is placed in a
uniform magnetic field B = 0.5 T with the direction of magnetic field parallel to the plane of the
coil. The magnitude of the torque required to hold this coil in this poisition is Nm.
(A) zero (B) 2 (C) 10 (D) 40

Ans. :41 (A), 42 (C), 43 (C), 44 (B), 45 (B), 46 (D), 47 (A), 48 (A), 49 (B), 50 (D),

51 (B), 52 (B), 53 (C), 54 (B), 55 (C), 56 (B), 57 (B), 58 (A), 59 (B), 60 (B)

Galvanometer
Use to detect and measure small electric currents. If the coil becomes steady after a deflection q,
Deflecting torque = Restoring torque.
NIAB KT

I ( K o effective torsional constant of the spring)


K
?I
NBA
?IDI

I
Current sensitivity Si
NBA
I K
The current sensitivity of a galvanometer is definned as the deflection produced in the
galvanometer per unit current flowing through it.
Ammeter
Use to measure electric current
The small resistance joined in parallel to a galvanometer to convert it into an ammeter is called a
sShunt.

Shunt = S
GI g
I  Ig

To convert a galvanometer’s rangh by n time I nI g necessary shunt S


G
n 1

§ G ·
Current passing through shunt I s I¨ ¸
© G S¹

§ S ·
Current passing through galvanometer I g I¨ ¸ where, I o net current
©SG ¹
Voltmeter
Use to measure p.d. between two ends of conductor
To convert a galvanometer into a voltmeter, a resistance of high value is joined in series with it.

G
V
Rs
Ig

334
To increase voltage capacity by n times, necessary series resistance R s (n  1) G
Voltage sensitivity : Sv
Voltage sensitivity of a galvanometer is defined as the deflection produced in the galvanometer
per unit voltage applied to it.
I NBA
Sv
V KR
(61) Resistance of galvanometer is G.. If shunt required to make its range n times is S, then
n= .

(A) (B) 1  (C) 1  (D)


G G G S
S S S G

(62) Resistance of galvanometer is G.. If series resistance required to make its voltage capacity n
times is Rs, than Rs = .

(A) Gn (B) (n–1) G (C) (n+1) G (D)


G
n 1
(63) Resistance of galvanometer is G. What will be resistance of voltmeter after making its voltage
capacity n times ?

(A) nG (B) (n–1) G (C) (n+1) G (D)


G
n 1
(64) Resistance of DC ammeter is 10 : and its current capacity is 20 mA. Resistance required to
convert it in to volt meter measuring 3V p.d. is :.
(A) 110 (B) 120 (C) 130 (D) 140
(65) 0.5 % of the total current in ammeter passes through galvanometer. If resistance of galvanometer
is G, resistance of ammeter is

(A) (B) (C) 119 G (D) 200 G


G G
200 104

(66) What will be the shunt required to pass 10 % of the main current through moving coil
galvanometer having resistance 99 : :.
(A) 10 : (B) 9.9 : (C) 9 : (D) 11 :
(67) A galvanometer of resistance 25 : giving full scale deflection for a current of 10 miliampere is
to be converted into a voltmeter of range 100 V by connecting a resistance of ‘R’ in series with
galvanometer. Value of resistance R is :.
(A) 10,000 (B) 10,025 (C) 975 (D) 9975
(68) An Ideal battery of 100 V is connected in series to a 20 : resistor. A galvanometer of 5 : is
used to measure current in the circuit. Error in measurement will be .
(A) 0.5 A (B) 1 A (C) 2 A (D) 3 A

335
(69) A galvanometer of resistance 200 : is connected to a shunt of 20 : to form an ammeter. On
connecting this ammeter to a battery of 10 V and a resistor of 4 : in series, A
electric current passes through ammeter.

(A) (B) (C) (D)


55 77 122 177
122 55 55 22
(70) On connecting a shunt of 12: in parallel to a galvanometer, its deflection decreases from 50
division to 20 division. Then resistance of galvanometer is :.
(A) 18 (B) 26 (C) 30 (D) 36
(71) Resistance of a galvanometer is G. On passing electric current Ig, it shows full scale deflection. A
shunt S1 , is required to convert this galvanometer in to an ammeter of range 0 to I. If shunt S2 is

required to convert this galvanometer into an ammeter having range 0 to 2I, then S =
S1
.
2

2I  I g 1 § I  Ig ·
(A) (B) 2 ¨ 2 I  G ¸ (C) 2 : 1 (D) 1 : 1
I  Ig © ¹
(72) A galvanometer of resistance 50 : shows full scale deflection of 30 division when it is
connected in series with 3 V battery and 2950 : resister. To obtain 20 division deflection for
same galvanometer, the value of series resistor required is :.
(A) 4450 (B) 5050 (C) 5550 (D) 6050
(73) A galvanometer has resistance of 15 : and gives full scale difflection for 4 mA current. To
convert it into a ammeter of range 0 to 6 A, .
(A) 10 m : resistance connected in parallel with galvanometer
(B) 10 m : resistance connected in series with galvanometer
(C) 1 : resistance connected in parallel with galvanometer
(D) 0.1 : resistance connected in series with galvanometer
(74) A voltmeter of resistance 1000 : giving full scale deflecation for a current of 100 mA is to be
converted into an ammeter of range 1A. The value of shunt S is :.
(A) 10000 (B) 9000 (C) 222 (D) 111

(75) If a galvanometer of resistance 25 : is shunted by 2.5 : , then


Ig
I
I g = current passing through galvanometer, I = net current

(A) (B) (C) (D)


1 1 3 4
11 10 11 11
(76) The ( o T ) graph for a current carrying coil is .
t
(A) (B) (C) (D)

0 90 180 90 180 90 180 90 180


T T T T

336
Ans. : 61 (C), 62 (B), 63 (A), 64 (D), 65 (A), 66 (D), 67 (D), 68 (B), 69 (A), 70 (A),
71 (A) 72 (A), 73 (A), 74 (D), 75 (A), 76 (A)
Magnetism and Matter :
o
Magnetic dipole moment of current carrying loop m IA

Pole strength of magnet p where F = Force, B = magnetic field


F
B


Magnetic dipole moment of bar magnet mb 2 Pl direction of mb is from the south pole to the
JG G

north pole
P 0 p1 p2
The magnitude of force of attraction or repulsion between two magnetic poles F
4S r 2
The force acting between two small bar magnet lying on same axis x distance apart from
3P 0 m1m2
each other F
2 Sx 4

o 2P0 mz
The magnetic field at a point z on the axis from the center of bar magmet B( z )

l
m
4S z  l 2 2 2

o 2P 0 m l
if z !!! l , the value of B ( z ) m
4 Sz 3
The magnetic field on the equaterial point at a distance y from the centre of dipole of a bar magnet
o P0 m l


B ( y) 3
m
4S y  l 2 2 2

o P0 m l
if y !! l , the value of B ( y ) m
4 Sy 3

The torque acting on a magnetic dipole of magnetic moment m in uniform magnetic field ( B ) :
o
o o o
W muB

The periodic time, T 2S I


mB
o o
The potential eneregy, U m<B mBcos T
o
The work done on the magnetic dipole placed in uniform magnetic field ( B ) and rotating by
(displacing) angle T

W mB 1  cosT
mB cosT1  cos T 2
mB cosT 2  cos T1

337
Gauss’s law for magnetic field :

v³ B < d a
o o
The net magnetic flux associated with closed surface, 0

o
(77) A bar magnet of length l, pole strength p and magnetic moment m is split into two equal pieces

each of length . The magnetic moment and pole strength of each piece is respectively
l
2
and .
o p o o
o
(A) m, (B) (C) (D) m, p
m m p
,p ,
2 2 2 2
(78) When a bar magnet is cut into two equal parts parallel to the length which of the following
physical quantity does not change ?
(A) pole strength of poles (B) magnetic dipole moment
(C) intensity of magnetic field (D) moment of inertia
(79) A large magnet is broken into two pieces. so that their lengths are in the ratio 2:1, the pole
strengths of the two parts will have ratio .
(A) 1 : 2 (B) 2 : 1 (C) 4 : 1 (D) 1 : 1
(80) The unit of pole strength of magnet is (where Q is charge and v is velocity)

v 1
(A) Qv (B) (C) Q (D) Qv
Q
v
(81) Point A and B lie on axis of bar magnet of length 3 cm at a distance 24 cm and 48 cm from
center of bar magnet on opposite sides. Ratio of magnetic field at point A and B is .
(A) 8:1 (B) 4:1 (C) 3:1 (D) 1: 2 2
(82) If magnetic field at two points lying on equatorial line and axis of small bar magnet are same
then ratio of its distance from center of magnet is .
(A) 2–3 (B) 23 (C) 2 13 (D) 2 13
(83) Force acting on north pole of magnet of polestrength 3200 Am. lying 10 cm away from south
pole of bar magnet of polestrength 40 Am is N.
(A) –1.28 (B) 1.28 (C) 1.28 u 107 (D) 1.28 u 107

(84) A magnet of magnetic moment 0.1 Am2 is placed in a uniform magnetic field 0.36 u 104 T . The
force acting on its each pole is 1.44 u 104 N . The distance between two poles would be
cm.
(A) 1.25 (B) 2.5 (C) 1.8 (D) 5.0
(85) The magnetic dipole moment of steel wire of length L, is m. It is bent from the middle and
arranged as 600 . So the new magnetic dipole moment will be .

(A) (B) (C) m (D) 2 m


m m
2 2
338
(86) A straight wire of length l and magnetic dipole moment m is bent in form of a semi circle. Hence
new magnetic dipole moment is .

(A) (B) (C) (D)


m 2m 3m 4m
S S S S

(87) A straight wire of length l and magnetic dipole moment m is bent in the form of a circle. Its two
ends makes angles 600 at the centre. Hence new magnetic dipole moment is .

(A) (B) (C) (D)


m 2m 3m 4m
S S S S

(88) Magnetic field of current carrying coil at a distance 10 cm on axis from centre is 10 4 T .

If diameter of coil is 1 cm then magnetic dipole moment will be Am2 .


(A) 0.5 (B) 1.0 (C) 1.5 (D) 2.0

(89) A closely would solenoid of 6 cm, having 10 turns cm–1 and area of cross-section 3 u 104 m 2

carries a current of 1.0 A. The magnetic moment m of the solenoid is Am2 .

(A) 1.8 u 102 (B) 0.3 u 102 (C) 1.6 u 102 (D) 3.6 u 102

(90) The dimensional formula of magnetic field (B) in MLT and C (Coulomb) is given as .
(A) M1T 1 C 1 (B) M1T 2 C1 (C) M1L1T 1C 1 (D) M 1T 2 C 2

(91) Force between two identical bar magnets whose center are 4 cm apart is 4.5 N when their axis
are in the same line. If seperation is increased to 24 cm, the force between them is N.

(A) 0.37 u 102 (B) 0.6 (C) 1.2 (D) 2.4

(92) Two idential bar magnets with length 10 cm and mass 50 g. are arranged freely with their like
poles facing each other in a inverted vertical glass tube. The upper magnet hangs in the air above
the lower one so that the distance between the nearest pole of the magnet is 3 mm. pole strength
of the poles of each magnet will be Am.

S (A) 6.64 (B) 33.2


(C) 11.1 (D) 99.6
N
N

(93) Two short magnets of equal dipole moments M are arranged perpendicularly such that their
centres coincide (fig.) The magnitude of the magnetic field at a distance d from the centre on the
bisector of the right angle is

339
m
P
N
(A) (B)
om o 2m
4 d3 4 d3

S 0 N m
(C) (D)
o2 2m o 2m
4 d3 4 d3

(94) Two short magnets of magnetic moment 1000 Am2 are placed as shown at the corners of a
square of side 10 cm. The net magnetic induction at P is T.

(A) 0.1 (B) 0.2

(C) 0.3 (D) 0.4

P
2
(95) Two magnets A and B are identical and are arranged as shown in the figure. Their length is
negligible in comparison to the separation between them. A magnetic needle is placed between
the magnets at point P which gets deflected through an angle T under the influence of magnets.
The ratio of distance d1 and d2 will be .

T
(A) 2 tanT (B) 2 tan T
1/ 3 1/ 3
P
(C) 2cotT (D) 2cotT
1/3 1/3

d1 d2

(96) A loop of radius 4 cm and 20 turns carries a current 3 A. If it is placed in magnetic field of 0.5
T, the potential energy of dipole in most stable position is J.
(A) –0.15 (B) 0.15 (C) –1500 (D) 1500
(97) A short bar magnet placed with its axis at 30° with a uniform external magnetic field of 0.25 T
experience a torque of 4.5 u 102 Nm . Magnetic moment of the magnet is JT 1

(A) 0.18 (B) 0.36 (C) 0.54 (D) 0.72


(98) A bar magnet is held perpendicular to a uniform field. How much angle by which it is should be
rotated so that the value of torque becomes half of the original value of torque .
(A) 300 (B) 450 (C) 600 (D) 750

340
(99) A bar magnet with magnetic dipole moment m rotates and makes an angle T with the intensity
of magnetic field H, the work done in this process is .
(A) mH cos T (B) mH (1  cosT ) (C) mH sin T (D) mH (1  sin T )

(100) A magnet of magnetic dipole moment 5.0 Am 2 is lying in a uniform magnetic field of 7 u 104 T

such that its dipole moment vector makes an angle of 300 with the field. The work done in
increasing this angle from 300 to 45 is about J.
0

(A) 5.56 u 104 (B) 24.74 u 104 (C) 30.3 u 104 (D) 5.50 u 103
(101) A circular coil having 50 turns and radius 4 ´ 10–2 m carries a current of 2 A. It is placed in
uniform magnetic field of intensity of 0.1 Wbm–2. The work done to rotate the coil from the
equillibrium position by 180q is J
(A) 0.1 (B) 0.2 (C) 0.3 (D) 0.4
(102) The moment of inertia of magnetic needle is 8 u 106 Kgm2 and its magnetic dipole moment is

101 Am2 . The value of magnetic field if it performs 10 oscillations in ten second is T.

(A) 3.15 u 103 (B) 1.35 u 103 (C) 3.15 u 105 (D) 1.35 u 105
(103) The period of oscillation of two magnets in the same field are in the ratio of 2:1. If their moment
of inertia are equal, the ratio of their magnetic moments is .
(A) 1 : 2 (B) 1 : 4 (C) 2 : 1 (D) 4 : 1
(104) The period of oscillation of a magnet is 2 sec. When it is remagnetised so that the pole strength
is 4 times, its period will be sec.
(A) 1 (B) 2 (C) 4 (D) 8
(105) Rate of change of torque t with deflection T is maximum for a magnet suspended freely in a
o
uniform magnetic field of induction B when
(A) T 0 (B) T 450 (C) T 600 (D) T 900

(106) A magnet freely suspended in a vibration magnetometer is heated so as to reduce it’s magnetic
moment by 36 % by doing this, its perrodic time .
(A) Increase by 36 % (B) Increase by 25 % (C) Decrease by 25 % (D) Decrease by 64 %

(107) Two magnet are held together and allowed to oscillete in earth’s magnetic field. With like poles
together and unlike poles together periodic time are 4 s and 6 s respectively. The ratio of their
magnetic moment is .
(A) 6 : 4 (B) 30 : 16 (C) 2.6 : 1 (D) 1.5 : 1
Ans. : 77 (B), 78 (C), 79 (D), 80 (A), 81 (A), 82 (D), 83 (B), 84 (B), 85 (B), 86 (B),
87 (C), 88 (A), 89 (A), 90 (A), 91 (A), 92 (A), 93 (C), 94 (A), 95 (C), 96 (A),
97 (B), 98 (C), 99 (B), 100 (A), 101 (A), 102 (A), 103 (B), 104 (A), 105 (A),
106 (B), 107 (C)

341
l If B is the magnetic field at any place on the earth
Bh Bcos T
Its horizontal component B h Bcos T I

Its vertical component Bv Bsin T


Bv Bsin T o
Where I o Angle of dip. B

Here B Bh 2  Bv 2

tan T
Bv
Bh

l Magnetic susceptibility :

Fm Where_ M o Intensity of magnetization


M
H

H o Magnetic intensity

l Permeability

P P 0 (1  F m )

? Pr 1  Fm
P
P0 Where Pr o Relative permoability

According to Curie’s Law

cP 0
Fm ? Fm D
M 1
H T T

(108) The magnetic dip angle at a certain place where the horizental and vertical components of earth’s
magnetic field are equal is .
(A) 0o (B) 30o (C) 45o (D) 90o
(109) At a place on Earth, the horizontal component of Earth’s magnetic field is 3 times its vertical
component. The angle of dip at this place is .
(A) 0o (B) 30° (C) 45° (D) 90°
(110) The angle of dip at a given place in magnetic merridian is 30o , then the angle of dip in the plane
perpendicular to the magnetic meridian is rad.

S S S
(A) 0 (B) (C) (D)
3 6 2
(111) At a certain place on the earth, the horizontal component of magnetic field is 73.2 % more than
the vertical component. The angle of dip at this place would be .
(A) 30o (B) 45o (C) 60o (D) 90o

342
(112) The magnetic dip angle at two places are 30o and 45o . Calculate ratio of horizontal components
of earth’s magnetic field at the two places.
(A) 3: 2 (B) 1 : 2 (C) 1 : 2 (D) 1 : 3

(113) A small bar magnet of magnetic dipole moment 1.6 Am2 is placed in a magnetic meridian in such
a way that its north pole remains in north direction. At this time, if neutral point is obtained at
distance 20 cm, then horizental component of earth magnetic field Bh T.

(A) 1 u 105 (B) 2 u 105 (C) 3 u 105 (D) 4 u 105

(114) The Earth’s magnetic field at some place on magnetic equator of Earth is 0.4 u 104 T . Estimate
the magnetic dipole moment of the Earth. Consider the radius of earth at that place to be
6400 Km.
(A) 1.05 u 1020 Am2 (B) 1.05 u 1021 Am2 (C) 1.05 u 1022 Am2 (D) 1.05 u 1023 Am2
(115) A bar magnet is placed with its south pole towards geographic north. The neutral point is situated
at distance of 40 cm from the center of the magnet. The length of the magnet is 20 cm.
The horizontal component of the earth’s magnetic field is 3.2 u 105 T . The pole strength of the
magnet is Am.
(A) 5 (B) 10 (C) 25 (D) 45
(116) Two short bar magnets of length 1 cm each have magnetic moments 1.20 Am2 and 1.00 Am2
respectively. They are placed on a horizontal table parallel to each other with their N poles
pointing towards the south. They have a common magnetic equator and are seperated by a
distance of 20 cm. The value of the resultant horizontal magnetic induction at the mid-point of the
line joining their centers is T. Bh 3.6 u 105 T .

(A) 3.5 u 104 (B) 5.8 u 104 (C) 3.6 u 105 (D) 2.56 u 104
(117) Relative permeability of substance is 0.075. Its magnetic susceptibillity is .
(A) 0.925 (B) – 0.925 (C) 1.075 (D) –1.075
(118) A toroid wound with 100 turns/m of wire carries a current of 3A. The core of toroid is made of
iron having relative magnetic permeability P r 5000 under given conditions. The magnetic field

inside the iron is T. Take 0  ×10-7 TmA -1

(A) 0.15 (B) 0.47 (C) 1.5 u 102 (D) 1.88

(119) A magnet of 1.2 Am2 magnetic dipole moment having dimension of 0.15 m u 0.02 m u 0.01 m .

Then intensity of magnetization M is Am1 .

(A) 10 4 (B) 2 u 104 (C) 4 u 104 (D) 8 u 104

(120) A magnet has coercivity of 3 u 103 Am 1 . It is kept in a 10 cm long solenoid with a total of 50
turns. How much current has to be passed through the solenoid to demagnetize it ?
(A) 0.1 A (B) 0.6 A (C) 6 A (D) 10 A

343
(121) A magnetic field of 1600 Am1 produces a magnetic flux 2.4 u 105 Wb parallel to length of an iron

bar of cross sectional area 0.2 cm2 . The susceptibillity of iron bar will be .
(A) 298 (B) 596 (C) 1192 (D) 1788
(122) The susceptibillity of a paramagnetic substance at 73o C temperature is 6 u 103 then the
suseptibillity at 173q C temprature will be .
(A) 1.2 u 102 (B) 1.8 u 103 (C) 3 u 103 (D) 4.5 u 103

(123) A magnet in the form of a cylindrical rod has a length of 5 cm and a diameter of 2 cm. It has a
uniform magnetization of 5 u 103 Am 1 . Its net magnetic dipole moment is JT 1 .

(A) 7.85 u 102 (B) 8.75 u 102 (C) 5.78 u 102 (D) 7.58 u 102
(124) A magnetic needle vibrates in the vertical plane perpendicular to the magnetic meridian. The time
period of vibration is found to be 2 sec. The same needle is then allowed to vibrate in the horizontal
plane and the time period is again found to be 2 sec. Then the angle of dip is .
(A) 0o (B) 30° (C) 45o (D) 90o
(125) A bar magnet suspended in earth magnetic field and oscillating in horizontal plane with periodic
time T. If a wooden bar having moment of inertia equal to bar magnet is tied with bar magnet
then periodic time of system will be .

(A) (B) (C) (D)


T T T
3
2T
2 2

(126) A bar of iron has size of 5cm u 1cm u 1cm and density of 7.78 u 103 kgm3 . If each atom of iron
has atomic dipole moment of 1.8 u 1023 Am2 . Then magnetic dipole moment of iron in saturation
magnetization state will be Am2 .
(A) 4.75 (B) 5.74 (C) 7.54 (D) 17.54
(127) Two magnet of equal dipole moment are arranged perpendicularly at their center. The periodic
time of oscillation of system at anywhere on earth is T. Periodic time of each magnet will be 
.
(A) (B) 2 4 T (C) 2 14 T (D) 2 3 T
1 1
2T

(128) The graph of susceptibility o temperature for a diamagnetic substance is .


(A) (B) (C) (D)

F F F F

0
0 0 0
T T T T

344
(129) The most appropriate magnetization M versus magnetising field H curve for a paramagnetic
substance is   .

M C
(A) A (B) B
A
(C) C (D) D
H
E
D

(130) The F o T1 graph for an alloy of paramagnetic nature is shown in fig. The Curie constant is  K.
c

(A) 57 (B) 67

(C) 77 (D) 97

1
1 3 1 T
10 K
T

Ans. : 108 (C), 109 (D), 110 (D), 111 (A), 112 (A), 113 (B), 114 (D), 115 (D), 116 (D),
117 (B), 118 (D), 119 (C), 120 (B), 121 (B), 122 (A), 123 (A), 124 (C), 125 (C),
126 (C), 127 (C), 128 (D), 129 (A), 130 (A)

Assertion - Reason type Question :

Instruction : Read assertion and reason carefully, select proper option from given below.

(a) Both assertion and reason are true and reason explains the assertion.

(b) Both assertion and reason are true but reason does not explain the assertion.

(c) Assertion is true but reason is false.

(d) Assertion is false and reason is true.

(131) Assertion : Cyclotrom does not accelerate electron

Reason Ñ Mass of the electron is very small.


(A) a (B) b (C) c (D) d

(132) Assertion : The magnetic field produced by a current carrying solenoid is independent of its
length and cross sectional area.

Reason Ñ The magnetic field inside the solenoid is uniform.


(A) a (B) b (C) c (D) d

345
(133) Assertion : A circular loop carrying current lies in xy plane with its center at origin has a
magnetic flux in negative Z-axis
Reason Ñ Magnetic flux direction is independent of the direction of current in the conductor.
(A) a (B) b (C) c (D) d
(134) Assertion : A direct current flowing through a metalic rod produces magnetic field only outside
the rod.
Reason Ñ There is no flow of charge carriers inside the rod.
(A) a (B) b (C) c (D) d
(135) Assertion : The poles of a magnet can never be seperated.
Reason Ñ Atoms themselves are magnets.
(A) a (B) b (C) c (D) d
(136) Assertion : When the radius of a circular wire carrying current is doubled, its magnetic moment
becomes four times.
Reason Ñ The magnetic moment of the loop depend on the area of loop.
(A) a (B) b (C) c (D) d
(137) Assertion : Steel is not attracted by a magnet
Reason Ñ Steel is not a magnetic substance.
(A) a (B) b (C) c (D) d
(138) Assertion : The force between two small bar magnets lying on the same axis is inversely
proportional to square of distance between them.
Reason Ñ The force between two poles of a magnet is inversely proportional to the square of
the distance between them.
(A) a (B) b (C) c (D) d
Asn. : 131 (A), 132 (B), 133 (C), 134 (D), 135 (A), 136 (B), 137 (C), 138 (D)
Comprehension Type Questions :
Paragraph -I Read the following paragraph and give the answers to the questions

A B

I 300
O

b D
C
A current loop ABCD is kept on the plane of the paper as shown in the figure. The arcs BC
(radius = b) and DA (radius = a) of the loop are joined by two straight wire AB and CD.

346
A steady current I is flowing in the loop. Angle made by AB and CD at the origin O is 30 . Another
o

straight thin wire with steady current I, flowing out of the plane of the paper, is kept at the origin.
(139) The magnitude of the magnetic field (B) due to arc AD at the origin ‘O’ is .
P0 I P0 I P0 I
(A) zero (B) (C) (D)
24a 4S a 12S a
(140) The magnitude of the magnetic field (B) due to the arc BC at the origin ‘O’ is .
P0 I P0 I P0 I
(A) zero (B) (C) (D)
24b 4S b 12S b
(141) The magnitude of the magnetic field (B) due to the loop ABCD at the origin O is .
P0 I § b  a · P0 I § b  a · P0 I § b  a ·
(A) ¨ ¸ (B) ¨ ¸ (C) ¨ ¸ (D) zero
2S © ab ¹ 24 © ab ¹ 4S © ab ¹

(142) Due to the presence of the current I at the origin, .


(A) the forces on AB and DC are zero
(B) the forces on AD and BC are zero

I1 I ª º
(C) the magnitude of the net force on the loop is given by « 2 b  a  a  b »
4 ¬ 3 ¼

P 0 II1
(D) the magnitude of the net force on the loop is given by b  a
24 ab
Paragraph -II
Advanced countries are making use of powerful electromagnets to move trains at very high
speed. These trains are called Maglev trains (abbreviated from magnetic levitation.) These trains
float on a guideway and do not run on steel rail tracks.
Insteand of using an engine based on conventional fuels like LPG, CNG, Deisel they make
use of magnetic field forces. The magnetized coils are arranged on the guideway which repel the
strong magnet placed under train’s carriage. This helps train move over the guideway, a
technique called Electrodynamic suspension. When current passes in the coils of guideway, a
typical magnetic field is set up between the under carriage of train and guideway which pushes
and pulls the train along the guideway depending on the requirement.
The lack of friction and its aerodynamic style allows the train to move at very high speed.
(143) The force which makes maglev move is .
(A) Gravitational (B) Magnetic (C) Nuclear forces (D) Air drag
(144) The advantage of maglev train is .
(A) More friction (B) More pollution (C) Less pollution (D) Less friction
(145) The levitation of the train is due to .
(A) Mechanical force (B) Electrostatic attraction
(C) Electrostatic repulsion (D) Magnetic repulsion
Ans. :139 (B), 140 (B), 141 (B), 142 (B), 143 (B), 144 (D), 145 (D)

347
Match the columns :
(146)

Column-1 Column-2

(a) Biot-Savart’s law (p) gives direction of induced magnetic field

(b) Law of right hand thumb (q) gives intensity of induced magnetic field

(c) Fleming’s left hand rule (r) gives direction of induced current

(d) Fleming’s right hand rule (s) gives direction of force due to magnetic field.

(A) a – s, b – r, c–q d–p


(B) a – p, b – q, c–r d–s
(C) a – q, b – s, c–r d–p
(D) a – q, b – r, c–s d–p

(147)

Column-1 Column-2

P0I
(a) Magnetic field due to a straight very long wire and (p)
2r

carrying current I at a point at perpendicular


distance r from the wire

P0 I
(b) Magnetic field due to a circular coil carrying (q)
4S r

current I and radius (r), at its center.

2 P0I
(c) Magnetic field due to a circular coil of radius r and (r)
4S r

carrying current I at a point on its axis at a distance


r from it’s centre.

P0 I
(d) Magnetic field at a centre of current carrying (s) 4 2r

ring having are length r and having radius r.

(A) a – r, b – s, c – p, d–q
(B) a – r, b – p, c – s, d–q
(C) a – p, b – q, c – s, d–r
(D) a – s, b – p, c – r, d–q

348
(148) Two wires each carrying a steady current I are shown in four configurations in column I. Some
of the resulting effects are described in column II. Match the statments in column I with the
statements in column II.
Column - I Column - II
(a) (p) The magnetic fields (B) at P due to the
P
currents in the wires are in the same
direction.
(b) P (q) The magnetic fields (B) at P due to the
currents in the wires are in opposite
direction.
(c) P (r) There is no magnetic field at P.

(d) (s) The wires repel each other.


P

(A) a – p, b – r, c – q, d–s
(B) a – q, b – p, c – r, d–p
(C) a – p, b – p, c – r, d–q
(D) a – s, b – p, c – s, d–q

(149)

Column - I Column - II

(a) Moving coil galvanometer (p) having very small resistance.

(b) Ammeter (q) having medium resistance.

(c) Voltmeter (r) having very high, medium or

very small resistance.

(d) Avometer (s) having very high resistance.

(A) a – p, b – q, c – r, d–s
(B) a – p, b – q, c – s, d–r
(C) a – q, b – p, c – r, d–s
(D) a – q, b – p, c – s, d–r

Ans. : 146 (D), 147 (B), 148 (B), 149 (D)

349
6&7 Electromagnetic Induction and Alternating Current

Magnetic flux ( ) )
T
  he magnetic flux linked through any surface placed in a magnetic field is the number of magnetic
field lines crossing this surface normally.
o o
dA B

v³ B ˜ dA
o o
T Magnetic flux )

BA cos T
o o
where, T Angle between B and A .

Some cases : o o
A A
T
o o
A A

o o o o
Plane Perpendicular to B Perpendicular to B Angle T with B Parallel to B

T 0 T 180q T T T 90o

Magnetic flux ) BA )  BA ) BA cos T ) 0

Flux ( ) ) Maximum Maximum

and positive and negative

Unit of Magnetic Flux

SI unit weber (Wb)

MKS unit tesla-meter2

CGS unit gauss- cm 2 , maxwell.

Other units :

jule volt - coulumb volt - second, ohm coulumb, henry - ampere


,
Nm
,
ampere
,
A ampere

Dimentional Formula :

ª¬) º¼ ª¬ M1 L2 T 2 A 1 º¼

350
Equations :
o o
) B˜A BA cosT
o o o and o .
B Magnetic field, A area vector, T Angle between A B
If coil having N turns, ) NBA cosT
If coil rotating is uniform magnetic field with constant angular speed then magnetic flux linked
with coil at time t,
) NBA cos t
Magnetic flux linked with current I carrying coil laying in uniform magnetic field.
) NBA cos t
P0 I
where, B current carrying straight wire having infinite length
2S a
P0 I
B current carrying circular loop having radius a
2a
P0 NI
B current carrying solenoid having length of l and N turns.
l

If surface is spherical then ) ³ B ˜ da


o o
l

l A rectangular loop of length L and breadth b is placed near a very long wire carrying
current I, the side of the loop nearer to the wire is at a distance a from the wire,
magnetic flux linked with the loop.

P0 I b ª L  a º
) ln «
2S ¬ a »¼

(1) Two coplanar and cocentric coil of radius 100 cm and 1 cm respectively. If 1 A current passing

through big coil then magnetic flux linked will small coil is . P0 4S u 107 TmA1 .

(A) 0.02 Wb (B) 2 u 1010 maxwell (C) 0.02 maxwell (D) 2 u 1010 Tm2
(2) A rectangular loop of length 2 cm and breadth 1 cm is placed near a very long wire current
carrying 10 A and in plane of wire. If the side of loop nearer to the wire is at a distance of 2 cm
from wire then magnetic flux linked with loop is G cm2 .
(A) 2 u 108 (B) 2 (C) 1.386 u 108 (D) 1.386
(3) A circular loop of radii 10 cm having 10 turns placed in uniform magnetic field of 2 u 104 G.
Initially plane of loop is perpendicular to field. Now it is rotate with uniform angular speed of
2S rad s 1 . At which time the flux linked with loop becomes half of maximum time ?

(A) s (B) s (C) s (D) s


1 1 1 1
2 12 6 4

351
(4) Two coil of equal surface area having turns 10 and 20 are lies in a uniform magnetic field with
its plane perpendicular to the field. If both coils are rotates with constant angular speed 1 and

2 respectively. The magnetic flux linked with coils are equal at time t then 2
= .
1

(A) (B) (C) (D)


1 1 1 2
3 6 4 3

(5) A solenoid of 10 cm length and cross section diameter 5 cm having 1000 turns carrying20 mA
current. A circular plane of 2 cm radius is kept near to end of solenoid and its plane
perpendicular to axis of solenoid. The magnetic flux associated to circular plane is ...... H < A .
(m0 = 4p ´ 10–7 Tm A–1. p2 = 10 )
(A) 3.2 u 108 (B) 3.2 u 107 (C) 3.2 u 109 (D) 3.2 u 106
(6) A rectangular plane of length 2 cm and breadth 4 cm is kept horizontally and magnetic field of
0.3 T applied in direction inside at 30q with perpendicular upward direction to rectangular plane.
Magnetic flux linked with plane is ...... Wb.
(A) 8 u 105 (B) 8 u 106 (C) 1.2 u 104 (D) 12 u 106

Ans. : 1 (C), 2 (D), 3 (C), 4 (B), 5 (B), 6 (C)

Electromagnetic induction and Faraday's law :


Electromagnetic induction : The phenomenon in which electric current is induced in a conductor
by varying magnetic field is called electromagnetic induction.
Faraday's law : The magnitude of the induced emf produced in a close circuit ( or a coil ) is
equal to the negative of the time rate of change of magnetic flux linked with it.
l Equations of induce emf :
d )
dt
d)
1 When N turns
dt
 N ) 2  )1
When varing magnetic flux.
+t
 NA B2  B1
When varing magnetic field.
+t
–NBA cosT 2  cosT1
When changing in T
+t

l F NBA cos t when coil or loop rotate with constant angular speed
d
? NBA cos t
dt
Negative sign indicate presence of Lenz's law

352
If only area A is changing, then  1% FRV T
dA
l
dt

If only magnetic field is changing, then  1$ FRV T


dB
l
dt

l If only angle T is changing, then

(cos t = +NBA VLQ t (Induced emf is in form of AC voltage).


d
 1%$
dt

Induced current (I) Induced charge (q) Induced power (P)

1 d ) N N2 § d ) ·
2
e2
I dq Idt d) P ¨ ¸
R R dt R R R © dt ¹

(R resistance)
Lenz's force and uses of Lenz's law :
When induced current passing through conducting loop laying in magnetic field it experiance a force.
o o o
F I l u B , is called Lenz force.

The direction of induced emf or current in a circuit is such as to oppose the cause that produces
it. The direction of magnetic field due to induced current is oppose to causes megnetic field.

The various position of relative motion between the magnet and the coil

Position of magnet

Direction of Anticlockwise Clockwise Clockwise Anticlock wise


induced current direction direction direction direction

Behaviour of face As a north As a south As a south As a north


of the coil pole pole pole pole

Type of magnetic force Repulsive Attractive Repulsive Attractive


developed. force force force force
Magnetic field linked Cross ( … ) Cross ( … ) Cross ( … ) Cross ( … )
with the coil and it's Increases Decreases Increases Decreases
progress as viewed
from left

353
If the coil move towards I current carrying straight infinite wire or I
increasing with time then induced current anticlock wise shown in
figure. The coil is move away from wire or current I decreases
i
with time, the induced current in clockwise direction.

Two coils carrying currents I1 and I 2 , placed with their planes


parallel and approach each other shown in figure.

I1 I2 (i) If I1 and I 2 are both clockwise (or anticlockwise) then


both I1 and I 2 will decrease. If the currents I1 and I 2 are in
opposite sense both the currents will increase.

(7) A conducting circular loop of surface area 5 u 103 m2 is placed perpendicular to a magnetic field

which varies as B 0.10T sin ª¬ 100 S s 1 t º¼ . Find the charge flowing through any cross

section area during the time t 0 to t 5 ms . Resistance of loop is 10 : .

(A) 5.0 mC (B) 500 mC (C) 5 C (D) 50 &

(8) A solenoid of diameter 20 cm has 500 turns per unit length. At a centre of this solenoid, a coil of
100 turns is wrapped closely around it. If the current in solenoid changes from zero to 2 A in
1 ms. Calculate the induced emf developed in the coil.

(A) 3.95 mV (B) 3.95 V (C) 3.95 u 103 V (D) 39.5 V

(9) A circular loop of diameter 50 cm and 10 turns is placed with its plane perpendicular to uniform
magnetic field of 0.4 T. Find an induced emf two condition given below for t = 0 to t = 20 ms,
when loop is rotated with constant angnlar speed of 100 rads–1.

(i) Loop rotated about an axis, passing through its centre and perpendicular to its plane ...... .

(ii) Loop rotates about is diameter ...... .

(A) (0, 7.85) V (B) (0, 0) V (C) (7.85, 0) V (D) (78.5, 78.5) V

(10) Resistance of conducting coil having 8 turns is 8 : . A galvenometer of eight time resistance of
coil is connected with coil. If entire system moves in 4 ms in field having magnetic flux
12 u 10 5 wb to 18 u 105 wb , then induced current in circuit is ...... .

(A) 1.6 (B) 1.6 u 106 (C) 1.6 u 103 (D) 1.6 u 104

354
(11) A wire in form of a semicircle of radius 5 cm rotates about the diameter with an angular
frequency of 10 S rad s–1 in uniform magnetic field of 2 T. The axis of rotation is perpendicular
to the field. If the total resistance of the circuit is 4 : , then mean power generated per period of
rotation will be ......  W. (Take S 2 10 )
(A) 7.81 u 103 (B) 7.81 u 106 (C) 78.12 u 103 (D) 7.81 u 105
(12) A thin semicircular conducting ring of radius 5 cm is free falling vertically in a horizontal magnetic
field of 5 u 103 T as shown in figure. At the position PQR, the speed of the ring is 20 cms 1 the
potential different across P and R point is ...... and point ...... . having high potential.
Q
o
(A) O and Q (B) 1 u 104 V and R
v

(C) O and R (D) 1 u 104 V and P


P R

(13) A square loop of side a is placed such that its plane is the same as that of a very long wire carrying
a current I. The centre O of the loop is at a distance x from wire. The loop is given velocity v so
that loop moves far away from wire then induced emf in loop will be ...... P V . Take; a = 2 cm,
I=1 A, P0 4S u 107 TmA1 , v 50 cms1 , x 5m

(A) 1.6 u 103 (B) 1.6 u 106 (C) 1.6 u 102 (D) 1.6 u 106
(14) A conducting coil of resistance 225 : , 250 turns and area of 1 u 102 m 2 lies with its plane
normal to a uniform magnetic field of 0.3 T. If area vactor of coil rotates from 0° to 90° in
500 ms then average power produced will be ...... mW.
(A) 1 (B) 100 (C) 10 (D) 0.1
(15) A coil having radius 4 cm and 150 turns lies with its area vector make 45q angle with magnetic

field of intensity 4 u 107 . If a coil rotates in 1.41 s and area vector make angle 135q
maxwell
cm 2
with magnetic field then average induced emf developed is ...... .
(A) 72 S u 103 V (B) 72 S V (C) 301.4 mV (D) 301.4 V
(16) As shown in figure, a long wire kept vertically on the plane of paper carries electirc current 2A.
A conducting ring having diameter of 4 cm moves towards the wire with velocity of 2 cm s1
with its plane coinciding with the plane of paper. Find the induced emf produced in the ring when
it is at a perpendicular distance 2 m from the wire.

I v a
(A) 2.512 V (B) 2.512 u 106 P V
(C) 2.512 mV (D) 2.512 u 106 V
r

Ans. : 7 (D), 8 (B), 9 (B), 10 (C), 11 (A), 12 (B), 13 (D), 14 (C), 15 (D), 16 (B)

355
Motional EMF due to translatory motion :
o
A conducting rod of length l moving with a uniform velocity v perpendicular to the uniform
o
magnetic field B shown in figure.

P Conducting electron experiance a magnetic force


G in direction from P to Q, so,
v
end P of rod becomes positive while end Q of rod becomes
negative.
Q An electric field is set up within the rod which is oppose
motion of electron.

When magnetic force Fm and electric force Fe in equillibrium,


Fm Fe
Ÿ Bev Ee

? electric field E Bv
induced emf (l %vl
o
l If rod is moving by making an angle T with the direction of magnetic field B .
induced emf %vl VLQ T
l When conducting rod PQ, length of l and mass of m starts sliding from the top of an inclined
plane as shown, it moves with velocity v. Perpendicular to its length but an angle (90  T ) with
o
the direction of magnetic field B. Both rails connected with R.

o l Induced emf between P and Q, emf %v VLQ   T l


?
B
%vl FRVT

Bvl c os T
Induced current I (Direction from Q to P)
Q R
o
v
The rod will move down with constant velocity (terminal
P velocity)
T R
BIl cosT mg sin T (frictional force zero)
substituting value of I
o
mgR sin T
Terminal velocity Vt
v
B2l 2 cos 2 T

When a semi circular conducting rod of radius R is move


R o
with a velocity v in a magnetic field B as shown in figure the
P Q
induced emf between end P and Q, emf %v5

356
P A U shaped conducting frame is placed in a magnetic field B
in such a way that the plane of the frame is perpendicular to the
field lines. A conducting rod is supported on the parallel arms of the
frame perpendicular to them and is given a velocity at time t = 0.
G
v
Induced emf %vl

Q %vl
Induced current I ( R is resistant of loop)
R R

B 2l 2 v
The magnetic force acting on rod to opposite direction of its velocity, Fm BIl
R

Necessary mechanical power to keep constant velocity of rod PQ, Pm


B2 l 2 v 2
Fm v
R

B2 l 2 v 2
Electric power Pe I 2R Hence, Pm Pe (Mechanical power = Electric power)õ
R

Motion of conducting rod in a vertical plane. If PQ


conducting rod length l and mass m is released from rest (at
t=0) as shown in figure. Force acting on PQ rod. Weight mg
P Q downward direction.
Lenz force Fm BIl (upward direction)
G
v

When rod achieve terminal velocity,

B 2 vt 2 l 2
mg BIl ( R is resistance)
R

? Terminal velocity Ÿ vt
mgR
.
B2l 2
When a rod PQ of length l pivoted at one end P is
Z rotated with angular velocity Z in a magnetic field B as
shown in figure the induced emf between its end P and Q.
P Q
% l2 .
1
rifid end emf
2

(17) A conducting bar of 5 m length is allowed to fall freely from a 100 m high tower keeping it
aligned along east west direction. If rod remain horizontal in its motion then intensity of electric
field produced at t = 4s is ...... Vm1 . Take angle of dip 60° and horizontal component of earth's

magnetic field if 0.7 G, g 10 ms 2 .

(A) 1.4 (B) 14 (C) 1.4 u 103 (D) 0.14

357
(18) A U shaped conducting frame is placed in uniform magnetic field of intensity 2 T in such a way
that the plane of the frame is perpendicular to the filed. A conducting rod having length
10 cm and mass 40 g is supported on the parallel arms of the frame and is given a velocity v0 at
§o o·
time t = 0 ¨ v A B ¸ . If resistance of rod is 10: and velocity v t at time t = t. If ratio of
© ¹

ª vt º
« » 0.3679 then t = ...... s.
¬« v0 ¼»
(A) 0.1 (B) 2.718 (C) 1 (D) 10
(19) A square metal wire loop of side 10 cm and resistance 2 : is moved with a constant velocity
40 u 103 ms1 in a uniform magnetic field of induction B = 2 Wb m2 as shown in figure. The
magnetic field is perpendicular to the plane of the loop and directed into the paper. The loop is
connected to a network of resistors each of the value r ’ : . what should the value of r ’ so that
a steady current of 1 u 103 A flows in the loop.
magniefic field perpedicular to the plane of paper and pointing inwand
o
A rc (A) 3 : (B) 12 :
v
rc
rc

rc (C) 6 : (D) 8 :
B rc
Metal loop

(20) A metal rod CD, length of 5 cm moves with a velocity 2ms 1 parallel to very long straight current
carrying wire AB, as shown in figure. The nearest end of CD rod is at 5 cm from AB wire as shown
in figure. The emf induced between the end C and D is ...... . and D behaves as a ...... electrode.
P 4S u 107 TmA 1
0
D
(A) 2.77 9 , positive (B) 2.77 mV, positive
b = 10 cm
G
v
C (C) 0.277 9 , negative (D) 2.77 V negative

5cm = a

A I = 1A B
(21) A metal rod PQ of length 0.3 m slides on parallel rails AB and CD, each rail having resistance of
0.02 : cm—1. A resistance R 17 : is connected between end of rails A and C. The whole
system kept perpendicular to the magnetic field B of 3.5 u 104 T . A variable force F is applied to
rod PQ, so that it is accelerated and moves x distance apart right side from resistance R and
obtain velocity of 2 ms1 . If induced current in loop is 100 $ then distance x ...... cm.
(neglect the friction force).
(A) 10 (B) 50 (C) 100 (D) 150

358
(22) A metal rod PQ of mass m = 50 g. and of negligible resistance slides on two parallel metal rails AB
and CD seperated by a distance of 50 cm. The rail have negligible resistance and one side ends are
connected by resistance R of 10: . The rails and rod are located in a region of uniform magnetic
field of 2T and direction in to the plane of loop ACPQ. The rod is given an initial velocity of 4 ms 1
to the perpendicular of magnetic field. The distance obtain by rod before it comes to rest after long
time is ....... . (neglect friction force).
(A) 2 m (B) 4 m (C) Infinite (D) 6 m
(23) P An equilateral triangular loop PQR of side, a is at the edge of a uniform
o magnetic field B at t = 0 as shown in figure. It is pulled to the right with
v
R a constant velocity v and its edge R leaves the region of magnetic field
at t = to. Which of the graphs represents the variation of induced
Q current I with time ?

(A) i (B) i

t1
O t O
t1 t

(C) i (D) i

O t1 t O t t1
(24) When a rod of length 1m pivoted at one end is rotated horizontally with constant angular velocity
of 5 rad s 1 at one place. If vertical magnetic field of earth at such place is 0.2 G then 50 9
induce emf developed after ...... rotation is completed by rod.

(A) 0.5 (B) (C) (D) 1


1 3
2 4
(25) A wire in the form of a circular loop of radius 8 cm lies with its plane normal to a uniform
magnetic field of 0.4 T. If the wire is pulled to make a square shape in the same plane in time
t = 2 s, the induced emf in the loop will be ...... .
(A) 4.32 u 104 V (B) 8.64 u 104 V (C) 4.32 mV (D) 8.64 mV
(26) A square conducting loop of side 40 cm, mass 50 g and resistance 15 : free falls vertically and
entering in uniform magnetic field of 2 T to directed perpendicular to the plane of loop. The
height d through which the loop fall, so that it attains terminal velosity on entering the region of
magnetic field, then d = ...... (take g = 10 ms 2 )
(A) 4 (B) 20.7 (C) 13.8 (D) 6.9

359
(27) A uniform metal rod is moving with a uniform velocity v parallel to a long straight wire carrying a
current I. The rod is perpendicular to the wire with its ends at distance r1 and r2 with r1  r2
from it. The emf induced in the rod is ...... .

§r · P 0 Iv § r2 · ª r2 º
ln ¨ 1 ¸ ln ¨ ¸ «1  »
P 0 Iv P0 I v
(A) Zero (B) (C) (D)
2S © r2 ¹ 2S © r1 ¹ 4S ¬ r1 ¼

(28) o Figure shows a conducting loop being pulled out from a magnetic
B o
field with a velocity v . Which of the four plots shown in bellow
o
v figure may represent the power delivered by the pulling agent as a
function of the velocity v.

P (D)
(C)
(B)

o (A)
d O
x B v
o
(29) v A rectangular loop is being pulled at a constant speed v through
x a region of certain thickness d, in which a uniform magnetic
field B is set up. The graph between position x of the right
(l < d) hand edge of the loop and the induced emf e will be ...... .

(A) (B)

x x

(C) (D)

x x

(30) A simple pendulum of mass m and conducting wire of length 1 m oscillating under gravity
through an angle 30° from its equillibrium position. The earth's magnetic field component
perpendicular to simple pendulum is 0.35 u 10 –4 T . Maximum potential difference induced across
the pendulum is ...... mV ( take g 10 ms2 )
(A) 57 (B) 114 (C) 28.5 (D) 85.5
Ans. : 17 (C), 18 (D), 19 (C), 20 (C), 21 (C), 22 (A), 23 (C), 24 (D), 25 (B), 26 (D),
27 (C) 28 (A), 29 (D), 30 (A)

360
Self Inductance :
l Inductance is that property of electrical circuits which oppose any change in the currert in the
circuit.
l Inductance is the electro-magnetic analogue of mass (m) in mechanics
l Its symbol is L.

Nm
2
weber tesla m
Units of L : =
amp amp (amp)2
joule coulomb volt volt sec
= = = ohm sec
(amp) 2
(amp) 2
amp
But practical unit is henry (H)
Dimensional formula ¬ª L º¼ M1L2 T 2 A 2

N)
Equation : L N = number of turns, ) = linked magnetic flux per turns
I
I = current, L = Inductance
Remember : Magnetic flux I v³ BA cosT .

P0 N 2 r
l Circular loop : L N = number of turns, r = radius of coil
2
P0 N 2 A
l For solenoid : L A = cross section area of solenoid
l
N = number of turns, l = length of solenoid
P0 1 2r
l For Torroid : L r = Axial radius of torroid
2
2 2 P0 N 2 a
l Current carrying square loop, L a = length of square loop
S
ªr º
ln « 2 »
P0
For coaxial cylinder : L r2 ! r1 : r1 and r2 are radius of cylinder
¬ r1 ¼
l
2S

Self induscent : /
dI
l
dt
Self induced current opposes any change in the current in a circuit.

I m
 I o
emf emf

I Increasing in I dareasing in

Energy stored in induction Energy stored per unit volume of long solenoid
called magnetic energy density

1 2 B2
2P0
U LI B
2

361
l Mutual inductance :
If two conducting coils are kept close to eachother and a steady current I passed through
one coil, magnetic flux links with the other coil ) MI .
Where, M = mutual inductance of the system formed by two coils.
If the current flowing through the current carrying coil is changed an emf is induced in the
second coil.

= time rate of changing current, M = mutual inductance


dI1 dI1
H2  M 21
dt dt
Note : Units of self inductance and mutual inductance are same.
Equation :
The system of two co-axial solenoid wounding on eachother of length l and having tarns N1 and N2 .

P0 N1 N2
M A where A = common cross section area
l

l The system of two cocentric and co-planner circular loop having radius R and r r   R

P 0S r 2
M
2R
l The system a small circular coil of radius r at the centre of a large rectangular coil of sides a
and b with a, b ! ! r .

2P0 r 2 a 2  b2
M a, b ! ! r .
ab
l A rectangular loop of length L and width b placed at a distance a from a long straight wire
shown in figure.

P 0 Ib § L  a · I
ln ¨ ¸
© L ¹
M
2S
a b

l Connection of inductor :

* Series connection : Ls L1  L2

(when two coil are situated far away to eachother)


I Ic

L1 L2

* When two coils one situated closed to eachother and current passing through both coil in same
direction.

362
o Ls L1  L 2  2M L1 L2

I I
* When two coils are situated closed to eachother and current passing through both coil in
opposite direction
o Ls L1  L 2  2M
I
L1

L2

l Parallel connection :


1 1 1 L1 L 2 L1
L L1 L2
or L P
L1  L2 (where M = 0)
I1
(Both coils far eway from each other)
* Both coils situated closed to eachother I
L2
L1 L 2  M 2 A B
LP
L1  L 2 r 2M

l Relation between M, L1 and L2

M k L1 L2
Where k = co efficient of connection

k=
Magnetic fulx linked with secondary coil
magnetic flux linked with primary coil

0 d k d1
l Growth and decay of current in L – R circuit.
L R

K2

H K1
Ideal inductor of inductance L and ideal resistor of resistance R connected in series with
cell of emf H shown in figure.

l If switch K1 is closed at time t = 0 with switch K2 open.

The current start increasing in inductor with time t is given by

§ t ·
o It I 0 ¨1  e W ¸ Where W L
time constant. Its unit is s.
© ¹ R

After a long time t f the current attains a steady value I 0


E
l
R

363
§ 1·
At t W, I I o ¨1  ¸
© e¹
l 0.632 I 0

Graph of I o t

I
I0

t
Decay of current

At time t = o , let I o be the current in the circuit. If K2 is closed (with K1 open) the
E
l
R
current decaye as

W time constant
t L
I I0 ˜ e W
R
t W,
I
Io I0
I 0.368 I 0
e

Graph of I o t
to
Note : Here inductance and resistor are ideal means inductor having zero resistance and resistor having
zero inductance.

(31) A solenoid 1.0 m long and 5 cm diameter has 1000 turns. Another solenoid of 100 turns is tightly
would over the first solenoid. When the current in the first solenoid changes from 0 to 5A in 10

ms. The induced emf in the second selonoid will be ...... 0  × 10-7 TmA -1

(A) 1.25 mV (B) 0.125 mV (C) 12.5 mV (D) 125 mV

(32) An ideal inductor of inductance 5 H and pure resistor of resistance 100 : are connected in
series to a battery of emf 6 V of negligible internal resistance through a switch. The switch is
closed at time t = 0. Time is taken for the current to rise to 50% of the maximum ....... and after
0.1 s potential different across the inductor will be ....... .

(A) 34.6 ms, 8 mV (B) 69.3 ms, 80 mV (C) 69.3 ms, 0.8 mV (D) 34.6 ms, 0.8 V

(33) An inductor of inductance 100 mH and resistor of resistance 24 : are connected to a 18 V DC


source when steady current flows in circuit at that time energy stored in inductor and dissipated
power in resistor respectively are ...... J and ...... W.

(A) 2.8, 135 (B) 0.14, 6.75 (C) 0.028, 13.5 (D) 1.4, 67.5

364
(34) An inductor of inductance of 100 mH and resistor of resistance 50 : are connected in series to
a 2 V DC battery. After some times the current attains a steady value. The battery is now short
circuited the time required for the current to fall of half the steady value is ...... .
(A) 1.386 s (B) 13.86 s (C) 1.386 ms (D) 13.86 ms
(35) A cross section of co-axial cable having length 100 m and central wire radii of a = 1 mm shown
in figure is normal to the palne of paper. Current of 2 A is passing through the central wire as
well as cylindrical layer of co-axial cable in mutually opposite direction. The magnetic flux linked
between area of two wire and self induction of cable are ...... Wb and ...... + respectively.

0  ×10-7 TmA-1

central wire
a (A) 6.44 u 105 , 32.2 (B) 1.61 u 105 , 161
outor wire
(C) 6.44 u 103 , 3.22 (D) 1.61 u 103 , 0.805
b

(36) A solenoid of length 2 m and 2000 turns having diameter is 6 cm. If 2 A steady current passing
through solenoid then magnetic energy and density of magnetic energy link with it will be ...... J

and ...... Jm -3 respectively. 0  ×10-7 TmA-1

(A) 28.4 u 103 , 5.02 (B) 14.2 u 103 , 2.51

(C) 7.1 u 103 , 1.25 (D) 1.42 u 103 , 0.251

(37) A square big loop made from thin wire of length 20 m. Another small square loop of length 0.4
cm is kept coplanes and concentric with big loop. If 2A current passing through big loop then

mutual inductance of the system will be ...... . 0  ×10-7 TmA-1

(A) 1.44 u 107 H (B) 14.43 mH (C) 3.6 u 101 mH ` (D) 3.6 u 106 +

(38) A conducting small loop of diameter 10 cm and having 10 turns is placed coplaner and concentric
with big loop of diameter 10 m. If 2 A current passing through big loop then mutual inductance of

the system is ...... . 0 S u 7 7P$1 S 2 

(A) 100 mH (B) 1 u 103 H (C) 1 u 108 H (D) 1 u 103 mH

(39) A toroidal ring has cross section radius 4.0 cm and axial diameter 40 cm is having wounding

turns 3 u 104 . The self inductance of ring will be ....... . 0 S u 7 7P$1

(A) 0.57 H (B) 4.52 H (C) 5.1 H (D) 0.452 H

365
(40) A coil having 100 turns length of 10 cm and radius of cross section area is 2 cm. When 1 Amp
current passing through coil linked magnetic flux is 5 u 105 Wb . Then stored energy density in
coil will be ...... Jm 3 .
(A) 0.5 (B) 5 (C) 1.99 (D) 19.9
(41) The self inductance of a loop having radius 10 cm and 1000 turns will be ......
. 0 S u 7 7P$ 1  S 2 
(A) 2 mH (B) 0.2 H (C) 20 mH (D) 2 H
(42) Two coils of self inductance L1 and L2 are placed closer to each other so that total flux is one coil
is completely linked with other. If M is mutual inductance between them, then M = ......

L1 L2
L1
(A) L1 L2 (B) L (C) (D)
2
L1 L2
2

(43) An inductor L, a resistance R and two identical bulbs B1 and B2 are connected to a battery
through a switch shown in figure gives the correct description of the happening when the switch
k is closed ...... .
L B1

R B2

(A) B1 and B2 light up together with equal brightness all the times.
(B) B2 lights up earlier and finally B1 shines brighter than B2 .
(C) B1 light up earlier and finally both the bulbs acquire equal brightness.
(D) The bulb B2 lights up earlier than B1 and finally both the bulbs shine equally bright.
(44) In the fig. two coplaner and coaxial loop of radius 5 cm, their centers aepart 5 m from eachother.
The mutual inductance of system when 2 A current passing through one loop is ......
+ . 0 S u 7 7P$ 1  S 2 
1 2
(A) 107 (B) 105
a I (C) 106 (D) 103
O1 O2

l
(45) If current of L – R, DC circuit in 4 s is 75 % of steady current. Then ratio of

= ...... s.
L
R
(A) 1.44 (B) 2.88 (C) 5.76 (D) 3.84

Ans. : 31 (D), 32 (D), 33 (C), 34 (C), 35 (A), 36 (B), 37 (D), 38 (C), 39 (B), 40 (D),
41 (C) 42 (D), 43 (C), 44 (A), 45 (B)

366
AC Generater / Dynamo
l Principle : Electro magnetic induction
Construction : A coil consist of large number of turns N of insulated Copper wire wound over a soft
iron core.
l Uniform magnetic field produced by permenent magnetic pole.
o
l The coil rotates about its axis with constant angular velocity . Area vector is A .
l The two ends of coil are connected to slip ring and induced emf produced between two
brushes. Which are passed against the slip ring. where
Magnetic flux at time t, )t NAB cos t = angular slip, N = number of turns
d ) t
Induced emf V NAB VLQ t A = surface area of coil
dt
Vm sin t B = magnetic field
Vm NAB
Voltage V changing with time according to sine function. If at time t = 0 the plane of coil
parallel to magnetic field the indus emf given by, V Vm cos t .
l The direction of the induced emf of the current in the coil Induced current (I)
is detemine by the Fleming's right hand rule. magnetic field
o
(B)

direetion of velocity
o
(v )

(46) A coil having N turns and surface area A is rotates about it axis with constant angular velocity of
S
50 rad s 1 in uniform magnetic field B. The magnetic flux linked with it at time t is given by
2
)t NAB cos t . During what minimum time the voltage becomes maximum ?
(A) 2 u 103 (B) 2 u 102 (C) 2 (D) 2 u 101
(47) If 2 A current passing through an AC generator of 40 W power and terminal voltage is 200 V.
Then produced emf ...... V.
(A) 160 (B) 220 (C) 240 (D) 180
(48) In AC generator Induced emf is maximum at time t = 0, The induced emf becomes zero at
minimum time t = 50 ms. Then angular speed of AC generator is ...... rad s 1 .
(A) 10 S (B) 5 S (C) 100 S (D) 50 S
(49) The value of the AC voltage of a generator V = Vm = 4 V at t = 0. At time t 21S second the
voltage V = 3.464 V. The voltage keep decreasing up to zero. The frequency of the generator is
...... H Z .
(A) 1 (B) 10 (C) 0.5233 (D) 60
(50) A voltage produced in AC dynamo is given by V 120 sin 100 S t cos 100 S t . Where t is in
second and V is in volt. Maximum voltage and frequency of it will be ...... and ...... respectively.
(A) 120 V, 100 Hz (B) 120 V, 50 Hz (C) 60 V, 100 Hz (D) 60 V, 50 Hz
Ans. : 46 (B), 47 (B), 48 (A), 49 (C), 50 (C)

367
Eddy Currents
Whenever a solid conductor is kept in a region of varying magnetic field the magnetic flux linked with
the conductor changes and induced emf is produced by induction. As a result circulatory currents are inducced
in the plane normal to the direction of flux. These currents are distributed through out the conductr. These
are known as Eddy currents. because of their circulatory nature.
Eddy currents were first observed by physicist Foucault.
When a conductor rotates in a uniform magnetic field, then also eddy currents are produced
in it.
Disadvantage of Eddy Current :
l The electric energy dissipate in the form of heat energy in metallic plate due to eddy current.
l A metellic plate is allowed to oscillate like a simple pendulum between two pole pices of a
strong magnet. The oscillations of the plate is damped is calleel electromagnetic damping.
Uses of Eddy Current :
l Eddy currents are undisiarable although it is used in induction Furnace, Speedometer, Electric
brakes, Electric power meters.
l To reduce the effect of eddy currents a laminated core instead of a single solid piece of iron
is used. Many time ractenguler slots one made metal plate to reduce of Eddy current.
l DC Moter
Principle : A current carrying coil placed in the magnetic field experience a torque. The torque
rotates the coil.
Construction and Figure :

B C

N S

F
A D
R1 R 2
B1 B2

l ABCD Armature coil R1 R 2 Slipring comutator


B1 and B2 = carbon brushes N,S = Strong magnetic poles.
l Working :
o o §o o·
Force on any arm of the coil is given by FI ¨ l u B ¸ , from figure force on AB will be perendicular
© ¹
to plane of the paper and pointing inwords. Force on CD will be equal and opposite. So coil rotates
in clockwise seen when viewed from top in figure. The current in AB reverses due to communication
keeping the force on AB and CD in such a direction that the coil continues to rotate in the same
direction.

368
l Back emf Ñ
Due to the rotation of armature coil in magnetic field a back emf is induced in the circuit. It is
given by (  I5
back emf (  I5 H = Applying DC Voltage
R = Resistance of armature coil
E NBA VLQ t Induced emf
back emf , D or
? H k (B, N and A are constant.)
Current in the DC motor :
E (k
I
R R

When motor is just switched on  so e = 0, Hence I = maximum


E
R
When time starts a large current flows through the motor which may burn it out. Hence a starter
is used for starting a DC motor safely.
Efficiency of DC motor Ñ

Efficiency K
Pmechanical Pout e Back emf
Psupplied Pinput E Supply Voltage
l Use of DC Motors :
They are used in electric locomotives, electric cars, rolling mills, electric cranes, electric lifts,
DC drills, fans and blowers, centrifugal pumps and air compressor etc.
(51) Which of the following is not an application of eddy currents .
(A) Induction furnace (B) X–ray crystallography
(C) Galvanometer damping (D) Speedometer of automobiles.
(52) The pointer of a dead – beat galvanometer gives a steady deflection because .
(A) Its frame is made of abonite.
(B) Its pointer is very light
(C) Its magnet is very strong
(D) Eddy currents are produced in the conducting frame over which the coil is wound.
(53) Eddy currents are produced when .
(A) A metal is kept in varying magnetic field.
(B) A metal is kept in the steady magnetic field.
(C) Through a circuler coil currents is passed.
(D) A circular coil is placed in a magnetic field.
(54) When the speed of a DC motor increases the armature current ..............
(A) Increases (B) Does not change
(C) Decreases (D) Increases and decreases continously.

369
(55) Back emf 2 V of DC dynamo of rotational velosity is 1 . If rotational velocity tripple then back
emf will be .
(A) 2 (B) 6 (C) 0.66 (D) 18
(56) A DC generator with fixed field excilation developes as emf of 100 V. When oprating at 1500
rpm. At what speed must it rotate to develop 120 V in rpm.
(A) 1200 (B) 800 (C) 750 (D) 1800
(57) The armature of DC motor has 20 : resistance. It draws current of 1.5 ampere, when run by
200 V DC supply. The value of back emf induced in it will be V.
(A) 250 (B) 220 (C) 170 (D) 180
(58) An electric motor operates on a 50 V supply and a current of 7 A. If the effeciency of the
motor is 30 %. The resistance of the winding of the motor is :.
(A) 9.4 (B) 2.9 (C) 5 (D) 8

Ans. : 51 (B), 52 (D), 53 (A), 54 (C), 55 (B), 56 (D), 57 (C), 58 (C)


Assertion - Reason type Question :
Instruction : Read assertion and reason carefully, select proper option from given below.
(a) Both assertion and reason are true and reason explains the assertion.
(b) Both assertion and reason are true but reason does not explain the assertion.
(c) Assertion is true but reason is false.
(d) Assertion is false and reason is true.
(59) Assertion: No induced emf is developed across the ends of a conductor, if it is moved parallel
to a magnetic field.
Reason Ñ No force acts on the free electrons of the conductor.
(A) a (B) b (C) c (D) d
(60) Assertion : A rectangular loop and a circular loop are moved with a constant velocity from a
region of magnetic field out into a field free region. The field is normal to the loops.
Then a constance emf will be induced in the circular loop and a time-varying emf
will be induced in the rectangular loop.
Reason Ñ The induced emf is constant if the magnetic flux changes at a constant rate.
(A) a (B) b (C) c (D) d
(61) Assertion : A magnetised iron bar is dropped vertically through a hollow region of a thick
cylindrical shell made of Copper. The bar will fall with an acceleration less than
g, the acceleration due to gravity.
Reason Ñ The emf induced in the bar causes a retarding force to act on the falling bar.
(A) a (B) b (C) c (D) d
(62) Assertion: A coil is connected in series with a bulb and this combination is connected to a d.c.
source. If an iron core is inserted in the coil, the brightness of the bulb will increases.
Reason Ñ The reactance offered by the coil to d.c. current is zero

(A) a (B) b (C) c (D) d

370
(63) Assertion : If the current in a straight conductor increases from A to B the diraction of the
current induced in the coil will be anticlockwise.
Reason Ñ According to Lenz's law, the direction of the induced current is such that it oppose
the change which produces it.

(A) a (B) b
A B
(C) c (D) d
(64) Assertion : An emf can be induced between the two ends of a straight Copper wire when it
moved through a uniform magnetic field.
Reason Ñ As the straight wire moves through the magnetic field the magnetic flux through the
wire changes.
(A) a (B) b (C) c (D) d
(65) Assertion : Three identical coils A, B and C are placed with their planes parallel to one
Induced currout another. Coils A and C carry equal
currents as showing in figure. If
coil A is moved towards B, with
coils B and C fixed in position the
induced current in B will be in the
anticlockwise direction.
A B C
Reason Ñ The direction of the induced current is given by Lanz's law.
(A) a (B) b (C) c (D) d
(66) Assertion : A coil of metal wire is kept stationary in a non-uniform magnetic field. An emf is
induced in the coil.
Reason Ñ Whenever the magnetic flux through a metal coil changes an emf is induced in it.
(A) a (B) b (C) c (D) d
(67) Assertion : If a conducting rod moves in X-axis and to parallel to Y-axis in uniform magnetic
field B pointing in the positive Z direction the end of rod near to X -axis gets positive
charge.
Reason Ñ The free electrons in the rod experiance a force in positive Y direction.
(A) a (B) b (C) c (D) d

Ans. : 59 (A), 60 (D), 61 (A), 62 (D), 63 (D), 64 (C), 65 (A), 66 (D), 67 (A)

Comprehension Type Questions :


Paragraph 1 :
Two long parallel horizontal rails, distanse d apart and each having a resistance O per unit
lenght, are joined at one end by resistance R. A perfectly conducting rod MN of mass m is free
to slide along the rails without friction, shown in figure. There is a uniform magnetic field of
induction B normal to the plane of the paper and direction into the paper. A variable force F is
applied to the rod MN such that as the rod moves, constant current flows through R.
371
M

x
o
R F d

N
(68) The magnitude of the induced emf in the loop is =

§ 2O x · § R ·
(A) Bvd ¨ ¸ (B) Bvd ¨ ¸ (C) Bvd (D)
1
© R ¹ © 2O x ¹
Bvd
2

(69) The current in the loop is I =

2Bvd Bvd
(A) (B) (C) (D)
Bvd Bvd
R 2O x R  2O x R  2O x
(70) The velocity of the rod MN is v =

B2 d 2 § 2O x · § R ·
¨1  ¸ ¨ 1 ¸
B2 d 2
(A) (B)
2O m © R ¹ R © 2O x ¹

§ R · § 2O x ·
log e ¨ 1 ¸ log e ¨ 1 ¸
B2 d 2 B2 d 2
(C) (D)
2O m © 2Ox ¹ 2O m © R ¹

Paragraph 2 :
A pair of parallel horizontal conductiog rails of negligible resistance shorted at one end is
fixed on a table. The distance between the rail is L. A conducting massless rod of resistance R
can slide on the rail without friction. The rod is tide to a massless string which passes over a
pully fixed to the edge of the table. A mass m tide to the other end of string, hangs vertically. A
constant magnetic field B exists perpendicular to the table. The system is released from rest.
(71) The acceleration of the mass m moving in the downward direction is

§ B2 L2 v · § B2 L2 v ·
(A) g (B)
B2 L2 v
(C) ¨  ¸ (D) ¨  ¸
© mR ¹ © mR ¹
g g
mR

(72) The terminal velocity attained by the rod is =

(A) g (B) (C) (D)


mgR mgR
gR
BL B2 L2

(73) The acceleration of mass m when the velocity of the rod is half of the terminal velocity is

(A) g (B) (C) (D)


g g g
2 3 4

372
Paragraph 3 :
An infinitesimally small bar magnet of dipole moment M is pointing on X-axis at a distance x
from origin O and moving with a speed v in the x-direction. A small closed circular conducting
loop of redius a, resistance R and negligible self inductance lies in the yz palne with its centre at
x = 0 and its axis coinciding with the x-axis.
(74) The magnitude of magnetic field at a distance x on the axis of the short bar magnet is

P0 M P0 M P0 M P0 M
(A) (B) (C) (D)
2S x 2S x 2 2S x3 2S x 4
(75) If x 2a , the magnetic flux through the loop is

P0 M P0 M P0 M
(A) P 0 M (B) (C) (D)
2 4a 16a
(76) If x 2a , then associated magnetic flux f in loop,

3P 0 Mv 3 P 0 Mv 1 P 0 Mv 1 P 0 Mv
(A) 16 a 2 (B) (C) (D)
32 a 2 8 a2 16 a 2
(77) If x 2a the magnetic moment of the loop is

3S P 0 Mv 3S P 0 M v SP 0 M v 3S P 0 Mv
(A) 32 R (B) 8R (C) (D)
2R 4R
Paragraph 4 :

Two co-axial circular coils of radii R and are seperated by a distance x = 3 R and
R
100
carry currents I1 = 2I and I2 = I respectively.
(78) The magnetic field at the center of the smaller loop due to current I1 = 2I in the bigger loop is

P0 I P0 I P0 I P0 I
(A) (B) (C) (D)
R 4R 8R 3R
(79) The magnetic flux (F) linked with the smaller loop is

5SP 0 IR 3S P 0 Ir SP 0 IR
(A) u 10 5 (B) S P0 IR u 105 (C) u 105 (D) u 10 5
4 4 2
(80) The mutual inductance of the pair of coil is

) )
(A) (B) (C) (D) zero
2)
I I 2I
(81) If M and m are the magnetic moment of the bigger and smaller loops respectively then the ratio

is
M
m
(A) 10 4 (B) 2 u 104 (C) 10 2 (D) 2 u 102

Ans. : 68 (C), 69 (D), 70 (D), 71 (C), 72 (D), 73 (B), 74 (C), 75 (D), 76 (B), 77 (A),
78 (C), 79 (A), 80 (A), 81 (B)

373
Match the columns :
(82) In column 1 are listed unit's of some quantites. Match their dimantional formula listed in column 2.

Column-1 Column-2

(a) newton (p) M1 Lo T -2 A -1


ampere-meter

(b) henry (q) M1 Lo T -2 A -1

(c) weber (r) M1 L2 T -2 A -2

(d) tesla (s) M1 L2 T -2 A -1

(A) a – q b–p c–r d–s


(B) a – s b–p c–r d–q
(C) a – r b–q c–p d–s
(D) a – p b–r c–s d–q
(83) In column-1 are listed two inductors of inductance 10 mH and 40 mH are connected in different
types of connection. Equivalent inductance of such connection are listed in column-2. Match
column-1 and column-2. Constant of connection k = 0.3.

Column-1 Column-2

(a) Flux linked with the both inductor in same (P) 38 u 10


3
H

direction and nearest to eachother. When


L1 and L 2 connected in parallel

(b) Flux linked with the both inductor in opposite (Q) 8 u 10


3
H

direction when L1 and L 2 connected in series.

(c) Flux linked with the both inductor in same direction (R) 62 u 10
3
H

when L1 and L 2 connected in series

(d) Both inductor connected in parallel and far away (S) 0.8 u 10
3
H

from eachother.
(T) 5.8 u 10
3
H
(A) a – S, b – P, c – Q, d–R
(B) a – T, b – P, c – R, d–Q
(C) a – R, b – T, c – S, d–Q
(D) a – P, b – T, c – Q, d–R
Ans. : 82 (D), 83 (B)

l
374
A.C. Current
l AC GENERATOR :

Principle : Electro magnetic induction

Construction : A coil having N turns will rotates about it axis which is parallel to its palne and
perpendicular to uniform magnetic field is setup.

l The two end of the coil are connected to an external circuit by means of slipring and brushes.

l When the coil rotates in the uniform magnetic field. It cuts the magnetic lines of force and
flux linked with the coil changes accorded to cos t (At time t 0 ,T 0 ),

The emf induced V BAN VLQ t

V sin t
m

Current I I sin t
m

V
Where I m m
R

l Hence the induced emf and induced current in the coil varies with time as per the function
of sine. Here voltage obtain between brushes is called AC voltage.

l AC voltage or current increases-decreases and its direction change with time according to
the function.

Time with V = Vm sin t I I m sin t V = Vm cos t I I m cos t


referance to periodictime

t 0 V 0 I 0 V = Vm I Im

T S
t
2Z
V Vm I Im V 0 I 0
4

T S
t
Z
V 0 I 0 V = Vm I Im
2

3T 3S
t V = Vm I Im V=0 I 0
4 2

2S
t T V=0 I 0 V = Vm I Im

375
V
V
I
I
T T
2 4

O t O T t
T
3T
T 3T T 4
S
2
2
4

Remember : The average value of AC current and voltage is zero during single periodic time (T).
l L–C–R series AC circuit :
An ideal inductor (L) having zero ohmic resistance, a capacitor with capacitance (C)
and a resistor (R) with zero inductance are joined in series with the source of A.C voltage
called AC series circuit.
For this circuit,

At some time t current passing through the circuit I (t ) , the rate of the change of the
dQ
dt

d 2Q
current
dI
dt dt 2

Charge Q Idt

The potential difference between two ends of resistor is VR It R

d 2Q
The potential difference between two ends of inductor VL L L
dI
dt dt 2

Potential different between two ends of capacitor is Vc


Q
C

Vm cos t 9L  9C  9R

The differential equation of current :

dI R 1 Vm
LC ³
 I  I dt cos t
dt L L

The differential equation for the charge :

d 2Q Vm
 
R dQ Q
cos t
dt 2 L dt LC L

l This differential equation resemble with the equation

d2y Fo
  sin t .
b dy k
y
dt 2 m dt m m

376
Equivalence between the Mechanical and Electrical Quantities :
Number Mechanical Quantity Electrical Quantity

(1) Displacement (y) Electric change (Q)

§ dy · § dQ ·
(2) Velocity ¨ v¸ Eleatric current ¨ I¸
© dt ¹ © dt ¹

(3) Resistive coefficient (b) Resistance (R)

(4) Mass (m) Inductance (L)

§1·
(5) Force constant (k) Inverse of capacitance ¨ C ¸
© ¹

§ k · § 1 ·
(6) Angular frequency ¨¨ m ¸¸ Angular frequency ¨¨ LC ¸¸
© ¹ © ¹

(7) Periodic Force Periodic Voltage

l Complex current for the L–C–R Series Circuit


Differential equation of complex current :

LC ³
 i
Vm
e jZt
di R 1
idt
dt L L
The solution of this equation is called complex current is given by
i I m ˜ e jZ t

jZt jZt
? i
Vm e Vm e

R j / § 1 ·
R j¨ / ¸
j
& © &¹

Above equation resembles with Ohm's law equation I


V
.
R

In the equation, R  j /  =
j
&
impedance of L-C-R series AC circuit. Its unit is Ohm ( Z complex number)
l Equations of Impedance,
l j / =L Inductive reactance of inductor | ZL | XL /.

j
l ZC Capacitive reactance of the capacitance and it's value
&

&.
| ZC | XC 1

l Ohm's law for complex current, complex voltage and impedance

Vm ˜ e jZt Vm j t G
i e
|Z | ˜ e
jG
|Z|

377
ª cos t  G  j VLQ t  G ¼º
| Z| ¬
Vm

§ 1 ·
R ¨ /
2
Where | Z | ¸
© &¹
2

Real Current :

t G
Vm
I cos
|Z|

Vm Vm
Where I m
§ 1 ·
|Z| 2
R +¨ /– ¸
© &¹
2

Here G is phase difference between voltage and current


l Geomatrical representation of Z.

Impedance Z = R + j / 
j
&

j
In this equation R is real part of complex, j / and are imagenary part.
&
Remember : The componants which are connected in circuit, mention them in complex
plane and then decide | Z | and phase different for the circuit.

Imaginary A
axis H
/
/  1&
|
|Z
OD R
G
O OA / ;L
1 R D Real axis 1
& OF & XC
F
OD /  1&

The impedance | Z | represent by point H.

§ 1 ·
? OH R2  ¨ / 
2

¸
© &¹
|Z|

ª§ 1 ·º
«¨ /  & ¸ »
« © ¹»
Phase different G
1
« »
tan
« »
R
¬ ¼

378
G uS
Time different must be in radian unit G
0
t ( rad )
180
Remember :
Voltage leads the current when is in first quadrant. If is in fourth quadrant
current leads the voltage
To obtained real current equation for the given circuit put the value of magnitude of Z and

phase different in the equation I


Vm
cos t
|Z|

rms value of current/AC voltage


l rms value of AC voltage V = Vmcos t and V = Vmsin t given by

Vm
Vrms 0.707 Vm
2

Note : voltage of AC source voltage = Vrms

l I I m cos t or I I m sin t is given by

Im
I rms
0.707 I m
2

The average value of AC voltage or current


l The average value AC voltage or current over an interval of one period is zero.

T

1
T ³0 m
V V sin t

l Average value of V = Vmsin wt on half period of cycle

³ Vm sin t dt
T
2 2Vm
 Vm
2
V
T 0

63.7 % (Vm )

Series Resonance for L–C–R


For a definite angular frequency ( 0 ) of the voltage, value of rms current becomes maximum.
This is called the series resonance in L–C–R, AC series circuit.
At series resonance,

Resonance angular frequency S f 0


1
l 0
LC

l Imaginary part of impendance becomes zero.


379
§ ·
i.e. ¨ /  & ¸ 
1
© ¹

l Impedance value becomes minimum | Z | = R

l I rms becomes maximum and hence

Vrms Vrms
I rms (max.)
|Z| R

l Phase differance between V and I becomes zero (  ).

l Power factor, cos .

Average power P (Hence power loss maximum).


Vm I m
l
2

The equation of average power and power factor equation for L–C–R series AC circuit :

Real power P = m m cos


V I
9rms I rms FRV
2

Here, cos = power factor.

R R


cos
R  /  1&
|Z| 2 2

Some more quantities for AC voltage/AC current :

§ 1·
Admitance Y Ñ Reciprocal of impedance is Known as admittance ¨ y ¸
© Z¹

1 Im I rms
Y
Z Vm Vrms

It's unit is mho.


l Susceptance (S) Ñ The reciprocal of reactance is defined as susceptance

1
S
X

l It is of two types.

1 1 1
(i) Inductive susceptance, SL
XL / S f/ and

& S f&
1
(ii) Capacitive susceptance, SC Xc

380
(1) Resistive circuit (R-circuit)

It

V I
Phaser diagram
V = V m cos t

Equation of current I I o cos Z t

Vm
Peak value of current Im =
R
Phase different between voltage and current 
Power factor cos 

Power Vrms I rms (maximum power loss)


Vm Im
P
2

Time different between voltage and current ' t 0

(2) L–C–R series circuit VL


R L C
VL – VC V

It VL – VC
I
VR
VC
V = Vm cos t Phaser diagram

I m cos t r where
Vm
(1) Equation of current I Im
|Z|

Equation of Voltage : V = V2R + VL – VC


2

§ 1 ·
2
Impedance of circuit : | Z | = R +¨ /± ¸
2

© &¹
2 2
R + XL – XC =


1
Phase difference : tan
VL – VC XL – XC &
VR R R

l If net reactence is inductive circuit behaves as LR circuit.


l If net reactance is capacitive circuit behaves as CR circuit.

l At resonance X L = X C 0 Ÿ XL X C , this is the condition of resonance.

l Half power frequencies and band width : The frequencies at which the power in the circuit
is half of the maximum power (the power at resonance) are called half power frequencies.

381
l The current in the circuit at half power frequencies
P(max)
I rms
I 0.707 I rms .
2
l There are two half power frequences, P(max)
P
P
1 called lower half power frequency, the circuit is 2

capacitive

2 called upper half power frequency. It is greater 1 0 2

than 0, at this frequency the circuit is inductive.

l Band width ( ' ) : The difference of half power frequencies 1 and 2 is called band width

( ' ) and ' = 2


± 1.

For series resonant circuit it can be proved ' .


R
L
l Q factor :
l The characteristic of a series resonant circuit is determined by the Quality factor (Q-factor)

of the circuit.
l It defines sharpness of I rms o curve at resonance. When Q factor is large, the
sharpness of resonance curve is more and vise-versa.
l Q-Factor is also defined as,
Max. energy stored
Q-facter 2S u
Energy dissipation

2S Max. energy stored Resonant frequnecy


u =
T mean power dissiption Baund width

VL 0
/ 9C 1
= = or =
VR R VR 0
&5

Q-factor
1 L
=
R C
I rms

R O Ÿ Q Factor infinite

Value of R is very low, Ÿ Q factor is large


Value of R is low Ÿ Q factor is normal
Value of R is large Ÿ Q factor is very low

382
L-Circuit L
(1) Current : I Im cos t  S2
It
Vm Vm Vm
(2) Peak current : I m XL
=
/ S f/

(3) Phase difference between voltage and current q RU S2


V = Vm cos t

(4) Power factor : cos 


(5) Power : P = 0

(6) Time difference ' t


T
4

(7) Phase diagram : Voltage leads the current by S


2 .
V V
900
or

900
I I

? XL = / S f/ (Where S f )
Ÿ XL D f (L = constant).

XL

Here inductive reactance increses linearly with increase in frequency, therefore an inductor is
called ‘low pass filter’.
C
C-Circuit :

§ S ·
I m cos ¨ t  ¸
It
(1) current : I
© 2 ¹

9m S f&
Vm
(2) Peak current : I m = Vm &
V = V m cos t
Xc

§ S ·
(3) Phase difference between voltage and current : q ¨ or  ¸
© 2 ¹

(4) Power factor : cos G 0


(5) Power : P = 0

383
(6) Time difference : 't
T
4

S
(7) Phase diagram : current leads the voltage by .
2
I
90 0 I
or

V 900
V

Since, X C
1 1 Xc
& S f &

Ÿ XC D
1
f (C = constant)
f

Here capacitives reactance decreases non-linearly with increases in frequency, therefore capaci-
tor is called high pass filter.
RC-Circuit :
R C VR
G I
VR VC VR IR
VC I Xc
VC V
Phaser diagram
V = V m cos t

(1) Applied voltage : V = VR2 + VC2

§ 1 ·
R2 + ¨ ¸
2

(2) Impedance : Z = R +
© &¹
2
X C2 =

(3) Current : I I m cos t 


Vm Vo
(4) Peak current : I m
Vm
Z R  X
2 2
L R2 
1
4S 2 f 2 C2

(5) Phase difference :


XC
WDQ 1 WDQ 1
1
R &5

(6) Power factor : cos


R
R + X C2
2

(7) Current leads the voltage in phase by d

384
RL-Circuit :

R L

VR VL VR IR VL V
I VL IX L
G
VR I
V = V m cos t
Phaser diagram

(1) Applied voltage : V = VR2 + VL2

(2) Impedance : | Z | = R 2 + X L2 = R2 + 2 2
/ 5 2  S 2 f 2 /2

(3) Current : I I m cos t 

Vm Vm Vm
(4) Peak current :
Im
Z R  XL
2 2
R  4S f L
2 2 2 2

(5) Power factor : cos


R
R 2 + X L2

XL /
(6) Phase different : WDQ WDQ
1 1
R R
(7) Voltage leads the current in phase by d

LC-Circuit :
L C
VL

VL = IX L ,


VL VC
V = VL - VC
I
VC = IX C VL  VC

900
I
V = Vm cos t

(1) Applied voltage : V = VL  VC Phaser diagram

(2) Impedance : | Z | = X L  X C = X /!
VC 1
( &
ÜëËõ)

§ S·
(3) Current : I I m cos ¨ t r ¸
© 2¹

Vm Vm Vm
(4) Peak current : X L  XC
Im = =

Z 1
&

385
(5) Phase different :  q
(6) Power factor : cos 

(7) If / ! 1& the current legs behind the voltage in phase by S


2 .
If /  1& the voltage legs behind the current in phase by S
2 .

RLC - Parallel circuit :


IV
Vm
It( R) = Vm G
IR IL IC R
V = Vm cos t

Vm
R L C
It(L) = Vm S L
XL
s

Vm
I t (C ) = Vm SC
XC

IC
Phaser diagram
I IC  I L
IC  I L

IR VL
IL

From phaser diagram, current I


I R  IC  I L
2 2

Phase different WDQ


1 IC  I L tan 1
SC  S L
IR G

Admiltance ( Y ) :

§V · §V V ·
2

= ¨ m¸ +¨ m – m¸
2
Vm
¨X ¸
|z| © R ¹ © L XC ¹

§1· § 1 1 ·

2

? Y= = ¨ ¸ +¨ ¸
2

¨X ¸
1
G 2  SL  SC
2
©R¹ © L XC ¹

Z

l Resonance :

IC I L Ÿ I min IR

Ÿ Ÿ SC  SL 0Ÿ ¦S
V V
= 0
XC X L

386
V
Z max R
IR

Phase different 

Power factor cos  (maximum)

1
Resonance frequency f
2S LC

l Parallel LC circuit :
(I) L R

It C

V = Vm cos t

1 L
(1) Zmax
Ymin CR

(2) Minimum current of circuit : I min


CR
Vmax u
L

Susceptance S Ñ SL = SC Ÿ X = X Ÿ impedence X f
1 1
(3)
L C

 2
R2
(4) Resonant frequency :
1
0
LC L

(5) Q factor
1 1

 2
CR 1 R
2

LC L

(II) If inductance has no resistance, if R = 0 then circuit becomes parallel LC circuit as shown
in figure.

L
C
It

V = Vm cos t

Here condition of resonance : I C IL Ÿ Ÿ X L = XC


V V
=
XC XL

387
L–C oscillating : C
A B
+ –
I
L
C + –

L
(a) (b)

In given circuit when key (A) is closed and key (B) is open the cell charges the capacitor, then
on removing key (A) and key (B) closed the charged capacitor connected to L and circuit
behaves L-C circuit mention in figure.
Here, resistance of inductor is zero (Ideal Inductor) for this circuit.

d 2Q Q
Diffrential equation, 2
dt LC

Angular frequency Z o
1
LC

Equation for charge at time t, Q Qo cos o


t

Qo Charge on C, initially

Equation for charge at time t


I  Qo o
VLQ o
t

l Here charge oscillating within time according to function cosine.

The electric energy of capacitor reduced to 0 from in time t 0 to t


2
l
Q T
2C 4

The magnetic energy in inductor is increasing from zero to maximum in time t


1 2
l LI 0
2

to t T
4

l The current and voltage varing with time shown in below graph.
I Q

S S

O
4 2
3S T t
4

388
(84) The value of frequency f Hz will be the impedence of given below circuit is
maximum value . C 4P F

R
L = 2500 H

(A) 0.628, zero (B) 0.628, infinite (C) 62.8, zero (D) 62.8, infinite
(85) The value of impendence Z for given below circuit at a source voltage frequency of 50 Hz will
be :.
L = 50 H C 1P F

R 10 :

f 50 H Z

(A) 10 (B) 100 (C) 50 (D) 5


(86) The series combination of R (:) and capacitor C (F) is connected to an AC source of V volts

and angular frequency . If the angular frequency is reduced to the current is founded to be
5
reduced to one-half without changing the value of the voltage. Determine the ratio of the
capacitive reactance and the resistance.
(A) 1.11 (B) 0.90 (C) 0.77 (D) 1.30
(87) An inductor L and resistor R connected in parallel with an AC source of V = 200 cos 344t
shown in circuit below. Hence voltage to current.

L = 63.7 mH

V R 20 :

(A) lags 45° (B) leads 60° (C) leads 45° (D) lags 60°

(88) A coil of inductance 0.16 H, Resistance R 20 : and capacitor of capacitance C are conneeted in
L–C–R series circuit. If resonance frequency is 72.70 Hz then capacitor C =
and | Z | .
(A) 30 )  : (B) 30 pF, 20 : (C) 20 )  : (D) 30 )  :
(89) An Ac voltmeter connected with 50 Hz AC source it read 200 V, maximum voltage during
periodic time will be .
(A) 28.2 V (B) 2.82 V (C) 282 V (D) 0.282 V

389
(90) A resistant of 100 : resistance and inductor of 14 H inductance are connected in series. If AC

current of Hz frequency pass through circuit then voltage is to curront.


50
S

(A) leads 60° (B) lags 60° (C) leads 45° (D) lags 45°
(91) If 1 ) capacitor applying ac voltage of V 200 sin100t (V) then reading of ameter connected
in circuit will be mA.
(A) 14.18 (B) 20 (C) 40 (D) 30
(92) Capacitive reactance of 25 : capacitor is C ) and it connected in circuit with AC supply of

Hz frequency the value of capacitance will be


400
.
S

(A) 25 ) (B) 50 ) (C) 400 ) (D) 100 )


(93) A 200 : resistor and coil of self inductance 1H are connected in series with ac source of

Hz frequency. The time different between voltage and current will be ms.
200
S

(A) 1.37 (B) 1.60 (C) 2.74 (D) 3.20


(94) An inductor having negligible resistance and 50 mH self inductance and 500 pF capacitance are
connected in AC circuit. The resonance frequency will be .
(A) 31.8 Hz (B) 31.8 kHz (C) 31.8 MHz (D) 31.8 GHz
(95) The phaser diagram of circuit given in figure is .

L C

(A) (B)
VC
VC
I
VL
VL

(C) (D)
VL VC

I I
VC VL

390
(96) The phase diagram of given circuit is .

S
R

L
(A) (B)

V
IR IL
V
IL
IR

(C) (D)
IR IL
IL IR
V V

(97) An ideal resistor and ideal inductor are connected in series with 100 V A.C. source. If voltmeter
read same reading across resistor or inductor then the reading of it .
(A) 50 V (B) 70.7 V (C) 88.2 V (D) 100 V
(98) L.C.R. circuit having impedence of 110 : and phase different between current and voltage 60°
is applying AC voltage V 200 sin 100 t circuit power will be W.
(A) 100 (B) 110 (C) 90.90 (D) 200
(99) A voltmeter reading across capacitor C in mantion below circuit is V.

230 V, 50 H Z

R L C

VR 10 V VL 250 V VC ?

(A) 15.8 (B) 20.3 (C) 10.3 (D) 18.3

(100) A coil is connected with AC source of 120 V and frequency 50 Hz, current passing through coil
is 5A and consumed power is 108 W. Then resistance of circuit will be .
(A) 24 : (B) 10 : (C) 12 : (D) 4.3 :

391
(101) In a serice resonant L–C–R circuit the voltage across R is 10 volt and R = 1k :
with C 2 ) . The resonant frequency  rad V 1 . At resonance the voltage across
L is .
(A) 250 V (B) 4 u 103 V (C) 25 V (D) 40 V

(102) In L.C.R series circuit R = 100 : , L = 0.5 H and C = 10 × 10-6 F . If 50 Hz AC supply


connected to the circuit the impedence will be :.
(A) 1.8765 (B) 18.76 (C) 101.3 (D) 189.9
(103) The resonant frequency of a circuit is f. If the capacitance is made 16 times the initial values
then the resaonant frequency becomes .

(A) (B) 2f (C) 4f (D)


f f
2 4
(104) The resistance of RL AC circuit is 10 : and applied voltage Vm across the circuit at

 UDG V . If current in the circuit is then value of L will be .


Im
1
2
(A) 0.5 (B) 0.707 (C) 0.8660 (D) 1.73
(105) The following series L–C–R circuit when driven by an emf source of angular frequency
70 k rad s 1 , the circuit effectively behave like .
L 100 + &  ) 5  :
L C R

(A) RC (B) LR (C) LCR (D) LC


(106) In the circuit shown below the AC source has voltage V 200cos W volts with
2000 rads . The magnitude of the current will be A.
1
Z

6:

5 mH, 4 : 50 )
(A) 20 (B) 10 (C) 2 (D) 1

§ 200 · § 10 3 ·
(107) An inductance of ¨ ¸ , a capacitance of ¨¨ S ¸¸ F and a resistance of 10 : are
© S ¹ © ¹
mH

connected in series with an AC source 220 V, 50Hz. The phase angle of circuit is
(A) 90° (B) 60° (C) 30° (D) 45°

392
(108) The ratio of impedance and resistance for L–C–R series circuit where,
V 110 2 cos (2000t  25q) V and I 10 2 cos (2000t  20q) A will be .
(A) 2 (B) 1 (C) infinite (D) 2
(109) The resistance of 100 : and the coil of inductance of 0.5 H are connected in series with AC
source 240 V, 50 Hz. Time different between maximum voltage and maximum current will
be . (take V Vm sin t )
(A) 3.2 ms (B) 6.4 ms (C) 3.2 s (D) 1.60 ms
(110) A capacitor of capisitance 100 F and a resistor of resistance 40 : connected in series with AC
source 110 V – 60 Hz. The time different between maximum voltage and maximum current
(At t = 0 Þ V = O V)
(A) 0.75 ms (B) 2.88 ms (C) 3.10 ms (D) 1.55 ms
(111) Which one of the following curves is represent the variation of current (I) with time (t) when
key close in given circuit.

K
L

(A) (B)
I I

t t

I I
(C) (D)

t t
(112) Which one of the following curves is represent variation of voltage (V) with time (t) w hen key
close in circuit of Question No-111.
(A) (B)
V V

t t

V V
(C) (D)

t t
393
(113) An AC source of varible frequency f is connected to an LCR series circuit. Which of the graph
represent the variation of current (I) in the circuit with frequency ( f ).
(A) (B)
I I

t t

(C) (D)
I I

t t

(114) A resister of resistance R 10 : and inductor of inductance of L = 25 mH are connected with


AC source of 50 Hz frequency. The Q factor of circuit will be .
(A) 0.5 (B) 0.393 (C) 0.785 (D) 1

(115) An AC supply of 50 Hz is connected with L–C–R series connection. If L = 2H and phase


S
different between current and voltage is rad then C = P F.
4

(A) 0.5 (B) 5 (C) 2.5 (D) 0.25

(116) A bulb filament having an inductance, is connected first with DC voltage and then AC of same
voltage. It will be more shine brightly with .
(A) AC (B) Equally both

(C) DC (D) brightly for AC source only

(117) An ameter and AC voltage V 4 cos 1000 t V connected in series with L = 3 mH and
R 4 : then ameter reading will be A.

(A) 56 u 10 3 (B) 0.56 (C) 5.6 u 103 (D) 5.6

(118) In an AC circuit the direction of current change in 1 u 10 2 s then frequency f of AC current


will be .
(A) 60 (B) 31.4 (C) 50 (D) 6.28

Ans. : 84 (A), 85 (A), 86 (D), 87 (C), 88 (A), 89 (C), 90 (C), 91 (A), 92 (B), 93 (C),
94 (B), 95 (C), 96 (A), 97 (B), 98 (C), 99 (B), 100 (D), 101 (C), 102 (D), 103 (D),
104 (C), 105 (A), 106 (A), 107 (D), 108 (D), 109 (A), 110 (D), 111 (D), 112 (D),
113 (D), 114 (C), 115 (B), 116 (A), 117 (B), 118 (C)

394
Transformer :

input R1 output

l A device in which AC voltage can be increased or decreased.

l Principle : Electro magnetic induction.

l It is used only for AC voltage.

l This devices could not change the frequency of AC voltage.

The coil which is connected with AC source is called primary coil (P) and out put voltage
obtain between two ends of coil is called secondary coil (S)

l Both coil connected by magnetic force line.

l The resistance is infinite between primary and secondary coil.


Equations :

s
1s 9s Ip
= = r
p
1p 9p Is

For primary coil : For secondary coil,

p Induced emf s
Induced emf

N p = Number of tarns NS = Number of turns

Vp = Applying input voltage


Vs Output voltage across R L

IS Current
Ip Current

r Transformation ratio

r is transformation ratio
r ! 1 step-up transformer more out put voltage.

r  1 step-down transformer less out put voltage.

Here phase different between VS and VP is S .

395
Step-up Transformer Step-down Transformer

Symbol Symbol

P S P S

VS > VP VS < VP

NS > N P NS < N P

S
! P S
 P

IS  IP IS ! IP

RS > R P RS < R P

r !1 r 1

For ideal transformer :


Input power I P VP = Output power I S VS

Pin = Pout
Efficiency of transformer (K )
Pout
K u 100
Pin

VS I S
u 100
VP I P

Efficiency of ideal transformer is 100 %.


For practically used transformer,
Pin = Pout + P
loss

§ Output · § Loss ·
Input-Power = ¨ ¸  ¨ Power ¸
© Power ¹ © ¹
Here power lost in transformer due to heating of coil, leakage of magnetic flux of
eddy current
Uses :
Voltage regulator, Induction furnace, power transmission etc.

(119) A step down transformer is used to reduce the main supply of 220 V to 10 V. If the primary
draws 5 A and secondary 88 A current, calculate the efficiency of the transformer .
(A) 8.8 % (B) 80 % (C) 88 % (D) 8 %

396
(120) A transformer has an efficiency of 75 %. The power input is 4 kW at 100 V. If the secondary
voltage is 200 V, calculate the ratio of current in the primary and secondary. .
(A) 7.5 (B) 0.75 (C) 1.5 (D) 2.66
(121) The primary of a transformer has 400 turns while the secondary has 2000 turns. The power
output from the secondary at 1000 V is 12 kW. If the resistance of the primary is 0.9 : and
that of the secondary is 5 : and the efficiency of the transformer is 90 %, calculate the power
loss in the primary coil and is the secondary coil will be W and W.
(A) 2000, 310 (B) 400, 72 (C) 4000, 720 (D) 800, 144
(122) A transformer is used to step up 6.6 kW, 220 V to 4.4 kV with 80% efficiency. If primary
coil have 1000 turns then number of turns and current in secondary coil are
and .
(A) 2 u 103 , 12 A (B) 2 u 10 4 1.2 A (C) 2 u 10 4 1.2 A (D) 2 u 103 1.2 A
(123) A 250 V potential different generat by generator of 25 kW is transmit through transmition line of
1 : then power loss will be %
(A) 40 (B) 25 (C) 10 (D) 20
Ans. : 119 (B), 120 (D), 121 (C), 122 (C), 123 (A)
Assertion - Reason type Question :
Instruction : Read assertion and reason carefully, select proper option from given below.
(a) Both assertion and reason are true and reason explains the assertion.
(b) Both assertion and reason are true but reason does not explain the assertion.
(c) Assertion is true but reason is false.
(d) Assertion is false and reason is true.
(124) Assertion : A variable capaciter is connected in series with a bulb and this combination is
connected to an AC source. If the capacitance of the variable capacitor is
decreased the brightness of the bulb is reduces.
Reason Ñ The reactance of the capacitor increases if the capacitance is reduced.
(A) a (B) b (C) c (D) d
(125) Assertion: A bulb connected in series with coil is connected to AC source. If a soft iron core is
introduced in coil, the brightness of bulb will be reduced.
Reason Ñ On introducing soft iron core in the coil the inductance increases.
(A) a (B) b (C) c (D) d
S
(126) Assertion: The alternating current lags behind the emf by phase angle of , when AC flows
2

through an inductor
Reason Ñ The inductive reactance increases as the frequency of AC source decreases.
(A) a (B) b (C) c (D) d
(127) Assertion: A capacitor of suitable capacitance can be used in an AC circuit in place of the choke coil
Reason Ñ A capaciter blocks DC and allows AC only.
(A) a (B) b (C) c (D) d

397
(128) Assertion : An alternating current does not show any magnetic effect.
Reason Ñ Alternating current varies with time.
(A) a (B) b (C) c (D) d
(129) Assertion : The division are equally marked on the scale of AC ammeter
Reason Ñ Heat produced is directly proporational to the current.
(A) a (B) b (C) c (D) d
(130) Assertion : Average value of AC over a complete cycle is always zero.
Reason Ñ Average value of AC is always defined over half cycle.
(A) a (B) b (C) c (D) d
Ans. : 124 (A), 125 (C), 126 (C), 127 (B), 128 (B), 129 (D), 130 (B)

Comprehension Type Questions :


Passage I :
The AC generator which is one of the most important application of the phenomenon of
electromagnetic induction converts machenical energy into electrical energy. A rectanguler coil
consisting of a large number of turns of copper wire wound over a soft iron core is rotated
between the pole of a permanent strong magnet. The magnetic flux through the coil changes
continously with time thus producing induced emf called AC voltage,
is given by,
V = Vm sin W

When a load resistor R is connected across the terminals a current I flows through the circuit.

V Vm
I = sin t I o VLQ t
R R

(131) In an AC generator, a coil of area A and having N turns rotates in a magnetic field B. The
magnetic flux through the coil is .....................
(A) Maximum equal to NAB when the plane of the coil is perpendicular to the magnetic field.
(B) Zero when the plane of the coil is parallel to the field.

(C) NAB when the plane of the coil makes an angle of 60° with the field.
1
2

(D) NAB when the plane of the makes an angle of 30° with the field.
1
4

(132) In an AC generator, initially (i.e. at t = 0) the plane of the coil is normal to the magnetic field.
Which graph shown in figure represents the variation of induced emf with time.

(A) (B)

t t

398
(C) (D)
t t

(133) The emf of an AC generator is given by H § S ·


100 sin ¨100S t  ¸ where e is in volt and t in second.
© 3 ¹

(A) The peak value of the emf is 100 2 volts.


(B) The frequency of rotation of the armature is 50 Hz.
(C) At start (ie. at t = 0) the plane of the armature makes an angle of 60° with the magnetic field.
(D) At start, the plane of the coil is perpendicular of the field.
Passage II :
An L–C–R series circuit with 100 : resistance is connected to an AC source of 200 V
and angular frequency 300 rad s–1. When only the capacitor is removed, the current leads the
voltage by . When only the inductor is removed, the current leads the voltage by .
(134) The impedance of the L–C–R circuit is :.
(A) 200 2 (B) 100 (C) 200 (D) 100 2
(135) The current in the circuit is .
(A) 2A (B) 2 (C) 2 2 A (D) 1 A
(136) The power dissipated in the circuit is W(  q )õ
(A) 200 W (B) 100 W (C) 50 W (D) 800 V
Passage III :
An L–C–R circuit consists of an inductor, a capacitor and a resistor driven by a battery and
V connected by two switches K1 and K2 as shown in
figure. At the time t = 0 switch K1 is closed and K2 is left
R K1
C open. The maximum charge capacitor plate can be hold is
q0 . When K 2 is closed and K1 is open the charge
L ocillating LC circuit. (Inductor is an ideal inductor)
K2

(137) When switch K2 is open then circuit behave to as RC circuit and time constant for this circuit
= RC .
qo
(A) at time t W the charge on the capacitor plates q .
2

(B) at t 2W q  .
qo 1  e
2

(C) at t 2W q qo 1  e
1

(D) work done by the battery is half the energy dissipated in the resistor.

399
(138) At time t = 0 when the charge on the capacitor plates is q1 switch K1 is opened and K2 is closed.
The maximum charge the capacitor hold is q0. choose the correct statement from the following.
ª t Sº ª t Sº
(A) q qo cos «  » (B) q qo cos «  »
¬ LC 2¼ ¬ LC 2¼

d 2q d 2q
(C) q  LC (D) q q
1
dt 2 LC dt 2

(139) At an instant of time t = 0 when the capacitor has been charged to a voltage V, switch K1 is
opened and K 2 is closed,. Then,
(A) at t = 0, the energy is stored in the magnetic field of the inductor.
(B) at t > 0 , there is no exchange of energy between the capacitor and the inductor.
(C) at t > 0 the current in the circuit flows only in one direction.

(D) the maximum value of the current in the circuit is C


L
V

Ans. : 131 (D), 132 (C), 133 (C), 134 (B), 135 (C), 136 (C), 137 (B), 138 (C), 139 (D)

Match the columns :

(140) An inductor of inductance L = mH and resistor of resistance R = 10 are. connected in


100

series with AC source of V = 200 sin 100 t . Then match column I and column II.
Column-1 Column-2
(a) Maximum value of steady current is A. (p) 0.02
(b) Phase different between current and voltage will be degree (q) 14.14
(c) Current in circuit at t = 0 will be A. (r) 45
(d) At s the circuit current becomes first time zero (s) 30
(t) 20
(A) a o r b o t c o p d o q
(B) a o p b o q c o r d o t
(C) a o t b o r c o q d o p
(D) a o q b o t c o p d o s
(141) Column-1 is listed type of circuit and Column-2 listed for powerfactor of circuit. Match them
eachother :
Column-1 Column-2

(a) LCR series AC circuit / (p) 0


1
.
&
(b) LCR series AC circuit at resonace position (q) 1
(c) L–R series AC circuit (r) CR

R
(d) Only Capacitive AC circuit (s) Z

400
(A) a o s b o q c o s d o q
(B) a o s b o q c o p d o r
(C) a o q b o q c o s d o s
(D) a o s b o q c o s d o p
(142) For L–C–R series AC circuit column-1 listed variation of component and column-2 listed
variation in current. Match column-1 and column-2 :
Column-1 Column-2
(a) If R increaes (p) I decreaes
(b) If increaes (q) I increaes

(c) Ifõ X L decreaes (r) first I increase then decrease

(d) Ifõ Z increaes (s) Not sure about it.


(A) a o p b o s c o s d o p
(B) a o q b o s c o r d o p
(C) a o p b o p c o s d o p
(D) a o s b o p c o s d o p
(143) A series LCR circuit with L = 5H, C = 80 ) and R 40 : is connected to a variable
frequency 230 V AC source. Column-1 listed quantities and Column-2 listed the value of
quantities at resonance frequency. Then match Column-1 and Column-2.

Column-1 Column-2

(a) Impedance | Z |  : (p) 230 V

(b) I rms A (q) 1437.5 V

(c) VRrms (r) 0

(d) VLrms (s) 1206.5 V

(e) VLrms + VCrms (t) 5.75

(u) 80
(w) 40
(A) a o w b o t c o p d o q e o r
(B) a o t b o p c o q d o w e o r
(C) a o p b o w c o t d o r e o q
(D) a o q b o t c o r d o p e o w

Ans. : 140 (C), 141 (D), 142 (A), 143 (A)

l
401
8 Electro Magnetic Waves

  Q
Gauss's Law for electricity, vÔ E <d a G
close surface
H0

 
Gauss's law for magnetism, vÔ B <d a 0
close surface

The magnetic force line always form closed loops.

Faraday's Law :

dI d Ë  Û
emf H =  = Ì vÔ B < d a Ü
dt dt ÌÍ surface ÜÝ

The varying magnetic field generate electric field.

 
Ampere's circuital Law : vÔ B < d l N0 I
line

³
o o § o o·
= P
¨' I J < d a¸
J <da
surface © ¹

Ampere Maxwell Law :

o o o o
dE
vvÔ³ B < d l P0 Ic  P0 H0 ³
dt
<da

P0 I c  P0 I d

' P0 I
P0 I c  I d
Ampere - Maxwell law shows that the total current passing through any surface of which the
closed loop is the perimeter is the sum of the, conduction current and the displacement current.
Where I c conduction current, I d displacement current, I = total current

Displacement current I d Ñ
Displacement current produced due to change of electric field or electric flux with time during
the procedure of charging or discharging of capacitor.
l When electric flux linked between two plates of capacitor become constant then displacement
current become zero.
l Displacement current and conduction current are equal during charging or discharging of capacitor.

402
l Magnetic field produced by displacement current like conduction current.

l Unit of displacement current and conduction current is same and is same as ‘A’ (Ampere)

³J <d a
o o
conduction current I c

dI E o o
Displacement current I d $ , where IE
dE
o o A<E
dt dt

Displacement current in integnal form.

o o
dE
Id o ³ dt < d a

Where permittivity of free space and Rate of change of electric field.


dE
o
dt

Hert'z Experiment
Induction coil
G
Q1 V E
e

M
G
E
G
Spark gap (S)
b
G
a

C D

N o
X Shows that B is perpendicular
Q2 Ve to the page directed into the page
at C and D
The produced electromagnetic waves travelling along the X direction shown in fig.
The spheres Q1, and Q2 constitute a capacitor while the rods behave as an inductor. Such an
arrangement can be considered equivalent to L-C oscillator circuit and known as Hertzian
Dipole. Dipole moment of it is p p 0 cos t

l The frequency of the generated electromagnetic waves is equal to the frequency of oscillation of the
electric changes.
l The energy of the electromagnetic waves is equal to the kinetic energy of the charges oscillating
between the two spheres.
l The electric field and magnetic field vectors oscillate in mutually perpendicular planes,
perpendicular to the direction of propogation of the waves.
o o
l The direction of propagation is that of E × B , it's magnitude (in free space) is 3 u 108 ms 1 .

403
(1) The direction of electric field and magnetic field are shown in figure (1) and (2) for a plane
electromagnetic wave travelling along X - direction. true option is ......
Y o Y
E o
B

o
C X
X
o o
fig. 2 C E fig. 1
o Z
Z B
(A) (1) right (2) wrong (B) (1) and (2) both right
(C) (2) right (1) wrong (D) (1) and (2) both wrong
(2) The electric field in Y direction and magnetic field in Z direction for an electromagnetic wave
passes in through the space. Which will be true option ?

§o o· o §o o· o §o o· o
(A) ¨ E u B ¸ < E (B) ¨ E u B ¸ < B 1 (C) ¨ E u B ¸ < B (D) None of above
© ¹ © ¹ © ¹
1 0

(3) The Hertzian dipole is shown in figure at time t. Which will be the Q1
o o
correct option given below for direction of E and B at point P.

Q2
(A) (B)
o
o E
B

o o
P P
E B

(C) (D)

o o
E E

o
B
P
P o
B

404
(4) The oscillation of charges of electric dipole shown in figure at time t which one is correct figure
of electric field lines and magnetic field line produce due to charges oscillation ?
(A) + (B) + (C) + (D) +

– – – –
(5) A sphere of mass 5 mg, having charge is hanged at one end of spring of force constant
Nm . (Shown in figure.) The frequency of emmited radiation will be
5 1 .
2 u 10
Rigid support
(A) (B) 2S Hz
1
Hz
2S

k (C) S Hz (D)
1
Hz
S

q
(6) For Hertzian dipole moment p po cos t which curve is true from given below, at time

and T.
T T 3T
t , ,
4 2 4

(A) po (B) po

O t O t

(C) P (D)
po po

O t O t

(7) From which is wrong characteristics of electromagnetic waves ?


(A) The maximum and minimum magnitude of electric field and magnetic field vectors produce at
same time and same point.
(B) The energy of electromagnetic wave is divide eqully in electric field and magnetic field.
(C) The electric field and magnetic field vectors oscillate in mutually perpendicular and also
perpendicular to the direction of propagation of the wave.
(D) Medium is not required for propagation of electromagnetic wave.
(8) Maxwell's equations indicated the fundamental basic of
(A) Only charge (B) Only magnet (C) Only mechanics (D) Both (A) and (B)

Ans. : 1 (A), 2 (C), 3 (C), 4 (B), 5 (D), 6 (A), 7 (C), 8 (D)

405
l Difference between Electromagnetic waves and Plane Electromagnetic Waves

Electromagnetic Wave Plane Electromagnetic Wave

o o
l The electric field ( E ) and magnetic field l The electric field E , and magnetic

o o
( B ) vector osccillate in mutually field B vector oscillate in mutually
perpendicular to each other and all perpendicular plane and perpendicular
possible perpendicular to propagation at propagation of direction along in
of direction. perticular direction
e.g. direction of propagnation in X- axis,
o
E vector in Y-axis
o
B vector in Z-axis

Here, 0 and
dE dE
0.
dx dr
l Unpolarized wave. l These waves are polarized wave.
l normally near to source area. l Normally far away from source.
l Cylindaric wave plate l Wave front are plane
l frequency is not constant l Constant frequency

Electric field Electric field


Wave length Magnetic
field Diraction
O= Wave length Magnetic of
Diraction of field Propoginationë
Propaginationë

l An Electromagnetic wave propagating along the X-direction


Y
G
E
direc
tion
G
of
G
prop
B
E
O agat
ion
G

X Velocity c
B G
Z E G

l Characteristic of this waves


B

o o
l The equation of an electric field E and magnetic field B for electromagnetic wave.

406
o
Electric field E E x i  E y j  E z k but, E x 0, E z 0 and

E y = Eosin ( t  kx so,
o
E Eo sin t  kx j
Where angular frequency and k wave vector
o
Magnetic field B B x i  B y j  B z k but B x 0, B y 0 and

Bz Bo sin t  kx so,
o
B Bo sin t  kx k
l The velocity of electromagnetic wave in vaccum (free space) :
1
C
P0 R

P0 4S u 10 7 NA 2 permeability of free space.

o 8.85 u 10 12 C 2 N 1m 2 permittivity of free space.


l The vector of the electromagnetic waves perpendiculer through away medium :
1 1 c
v
P
or v P0 Pr or v Pr K .
o r

Where P = permeability of medium


permittivity of medium
P
Pr relative permittivity of medium
P0

r Relative permeativity of medium


0

= K = Dielectric constant of the medium


l The refractive index of the medium :

P1 K Pr
c
n r
v
o o
l Relation between E and B :
E cB
l If electromagnetic wave propagating along positive X direction then E y cBz and its

propagating along negative X-direction, Then E y  cB z .

§o o·
Direction of vector C = Direction of ¨ E u B ¸
o
l
© ¹

o o o c o
l E  c c u B and B c
uE

o o
Where c c unit vector of c . It's magnitude is 3.0 u 108 ms 1
407
l The electromagnetic energy per unit volume (energy density) of electromagnetic waves :

Energy density associated with electric field, UH ( 2 rms


1
l o
2
2
B
l Energy density associated with magnetic field U B
rms
2P0

l Energy density associated with electromagnetic wave.


2
B
U ( or U
2 rms
o rms P0

l Intensity of electromagnetic wave :


Energy Power
Intensity I Time u Area Area

( 2 rms ˜ c
H 0 E02c
I 0
Uc
2

Maximum intensity I max E o Bo

l Equation of intensity in form of B rms :


2
cB rms
I
P0

E rms ˜ Brms
I
P0

l Linear momentum by electromagnetic waves on surface :


U
P
c
Where U = The energy of electromagnetic waves incident on a surface and it is completly absorbed.
c = velocity of wave.
If incident energy totally reflect by surface then linear momentum obtain to the

surface P
2U
, because change in momentum is P   P 2P .
c
l Pointing Vector :
A power passes through unit area in direction of propagating of wave is called pointing

vector (S) .
o

o o o
? S EuH

P P r Po
Wave impedance : Z
E
l H r o

o o
Where E Vector of electric field, H magnetic intensity

408
Energy E
l Momentum of wave P = Velocity c

o
l Radiation of pressure P
S
c
l Electro magnetic spectrum :
UD\V ;UD\V , Ultraviolet, Visible, Infrared, Microwave, Short radio wave, long radio wave
frequency f decrease (from UD\V to radio wave) (value is in decreasing order)
Wave length increases (from J -rays to radio wave) (value is in ascending order)
l Wave length range, production, detection and uses of different types of electromagnetic waves.
Type Wavelength Production Detection Uses
Range
Radio > 0.1m Rapid acceleration and Receiver's Used in radio and TV commu-
decelerations of aerials (conduc- nication system
electrons in aerials ting wire)
Microwave 0.1m to Klystron magnetron, Point contact Maglev train, RADAR, air
1 mm Gun diode. diodes craft, navigation, interceptor
vans, ovens
Infrared 1 mm to Vibration of atoms and Thermopile Infrared lamps are used in
(IR) 700 nm molecules Bolometer, physiotheraphy, infrared
infrared detectors are used in remote
sensing satellites for milletary
purpose, agriculture, remote
control of TV, video players
and wifi systems.
Visible Light 700 nm to Electrons in atom, emit The eye, photo- Used for visibility of objects
400 nm light when they move cells, photogra-
from one energy level phic film, photo
to a lower energy level. diode,
light dependen-
tresistor (LDR)
Ultraviolet 400 nm to Inner shell electrons in Solar cell, Pho- Lasik eye surgery, water
1 nm molecule moving from tocells, photo- purifiers, UV lamps are used to
one energy level to a graphic film kill germs.
lower level.
X-ray 1 nm to X-ray tubes or inner Photographic Used in medical applications
10-3 nm shell electrons of film to find the fracture in bones,
molecule Geiger tubes, as well as in a treatment of
Ionization certain types of cancer.
chamber.
Gamma  10 nm
3 Radioactive decay of - do - Are used in medicine to
rays the nucleus destroy cancer cells

409
¨¨ t  3 ux10
ª § ·º
(9) The electric field of an electromagnetic wave is given by E 8.284 « 7.54 u 106 ¸¸ » mVm
1

¬« © ¹ »¼
8

The energy density field will be .


(A) 318.5 u 10 19 J (B) 318.5 u 1019 Wm3 (C) 318.5 u 1019 Jm 3 (D) 318.5 u 1019 W

(10) The electric field of an electromagnetic wave with intensity 1.328 Wm2 is given by


o
E E o sin ªS 9 u 1014 t  3 u 106 º i Then X component of electric field E x will be
¬ ¼

( c 3 u 108 ms 1 ), ( o
 × 10 SI )
-2

(A) 100 (B) 10 10 (C) 0.1 (D) 1000

(11) The electric field of an electro magnetic wave is given by E 10 sin ª30 u 10 t  10 x º . Then
¬ ¼
14 7

radiation pressure will be .


(A) 4.42 u 10 8 Pa (B) 442 Pa (C) 4.42 u 1010 Pa (D) 442 × 1010 Pa

(12) Radiation pressure on earth's surface by sunlight of average intensity 1480 Wm2 is incident on

surface of earth is (take c 3 u 10 ms )


8 1

(A) 49.3 u 106 (B) 49.3 u 105 (C) 4.93 u 106 (D) 4.93 u 105

(13) The permeability of medium having refractive index 1.5 and dielectric is 2, will be

TmA1 . P0 4S u 10
7
TmA
1

(A) 0.45 S u 107 (B) 5S u 107 (C) 5S u 107 (D) 4.5S u 107

(14) An average intensity of electromagnetic energy is proportional to square of amplitude of wave. In


this statement the dimensional formula of proportional constant will be .

(A) M1 L2 T 3 A 1 (B) M 1 L2 T 3 A 2 (C) M1 L2 T 3 A 2 (D) M 1 L2 T 3 A1

(15) If 50 W radiation energy incident on one surface and it completely absorbed by surface then
magnitude of E rms and B rms will be Vm1 and T.

(A) 15. 5 u 108 (B) 21. 7 u 108 (C) 18. 6 u 108 (D) 27. 9 u 108

(16) The energy of an electro magnetic waves which are passes through a volume DV, associate with
this volume, then fregnency of this energy's oscillation is .
(A) zero (B) half the frequency of the wave
(C) the frequency of the wave (D) double the frequency of the wave

410
(17) The electric field in an em wave is given by E
¬«
c ¼»
50 sin ª Z t  x º . Then the energy contained

in a cylinder of cross-section 20 mm2 and length 50 cm along the X-axis is J.

(A) 4.5 u 1012 (B) 7.5 u 1012 (C) 5 u 1012 (D) 5.5 u 1012

(18) The intensity of the sunlight on the earth is 1380 Wm -2 . Assume this light to be a plane

monochromatic wave. Then the amplitude of the magnetic field in the wave is T.

(A) 3.4 u 10 6 (B) 5 u 104 (C) 4.2 u 106 (D) 2.6 u 104

(19) An em-wave passing through vaccum is described by E E o sin kx  t . Which of the

following is indipendent of the wave length .

(A) (B) k (C) (D) k


k

(20) If magnetic monopole existed then which of the following Maxwell's equation would be
modified ?

   
(A) vÔ E ¹ d a
qm
(B) vÔ E ¹ d l d  
B ¹d a
H0 dt Ô

³( ˜d a
o o
(C) vÔ B ¹ d a (D) P0  P0i
  d
0 o
dt

(21) A long straight wire of resistance R, radius a and length l carries a constant current I. The
pointing vector for the wire will be .

(A) 2ISRal (B) I R (C) IR (D) I R


2 2 2
al al 2 Sal

(22) Micro waves are used for communication and in RADAR because .
(A) They have short wave length (B) Its very less diffraction

(C) Its more diffraction (D) Its propagination with high speed

(23) Give the name of the devices which produced visible light.

(A) Klystrons (B) Magnetrons

(C) Gunn diodes (D) Incondencent lamp

Ans. : 9 (B), 10 (B), 11 (C), 12 (C), 13 (D), 14 (B), 15 (D), 16 (D), 17 (D), 18 (A),
19 (A), 20 (A), 21 (D), 22 (B), 23 (D)

411
Assertion - Reason type Question :

Instruction : Read assertion and reason carefully, select proper option from given below.

(a) Both assertion and reason are true and reason explains the assertion.

(b) Both assertion and reason are true but reason does not explain the assertion.

(c) Assertion is true but reason is false.

(d) Assertion is false and reason is true.


(24) Assertion : When an electromagnetic wave going through vaccum is described as
E E o sin kx  t , then Z is independent of the wavelength
k

Reason Ñ Z
k
is speed of the wave.
(A) a (B) b (C) c (D) d
(25) Assertion : Displacement current goes through the gap of a capacitor whenever the charge
of the capacitor increases or decreases.

dI E
Reason Ñ Displacement current I d P0
dt
(A) a (B) b (C) c (D) d
(26) Assertion : The energy contained in a small volume through which an em wave is passing
oscillates with the frequency of the wave.

Reason Ñ Energy density of the wave is given by


1
o
E2
2
(A) a (B) b (C) c (D) d
Ans. : 24 (a), 25 (a), 26 (D)

Comprehension Type Questions :


Passage-I :
A light beam travelling in the X-direction is described by the electric field
§ x·
300 sin ¨ Z t  ¸ Vm 1 . An electron is allowed to move along the Y-direction with a speed of
© c¹
Ey

2 u 10 7 ms 1 .

(27) The maximum magnetic field is .

(A) 9 u 1010 T  Z direction (B) 9 u 1010 T  Z direction

(C) 10 6 T  Z direction (D) 10 6 T  Z direction

(28) The maximum electric force on the electron is N.

(A) 4.8 u 1017 (B) 3.6 u 1017 (C) 2.4 u 1017 (D) 1.2 u 1017

412
(29) The maximum magnetic force on the electron is N.

(A) 4.8 u 1018 (B) 3.2 u 1018 (C) 6.4 u 1018 (D) 1.6 u 1018

Passage-II :

The magnetic field in a plane em wave is given by B


ª
200 sin « 4 u 10 S
¬
15

1 §
¨t
©
 x ·º
¸ PT
c ¹ »¼

If c 3 u 10 ms , then answer the following questions :


8 1

(30) The maximum electrical field is NC .


–1

(A) 2 u 10 4 (B) 6 u 104 (C) 5 u 104 (D) 3 u 104

(31) The average energy is Jm .


3

(A) 18 u 103 (B) 21 u 103 (C) 24 u 103 (D) 16 u 103

(32) Magnitude of pointing vector of electromagnetic wave is A T –1 s –1 .

(A) 9.55 u 106 (B) 3.17 u 106 (C) 4.75 u 106 (D) 6.34 u 106

Passage-III :
A 2000 W bulb is kept at the centre of a spherical surface at a distance of 20 m from the surface.
The working efficiencey of the bulb is 2 % and consider it as point source. Give answer the following
question :

Ho 8.85 u 10 SI and c 3 u 108 ms 1


12

(33) Maximum magnitude of electric field E o for electromagnetic wave is .

(A) 1.73 NC-1 (B) 2.45 NC-1 (C) 7.96 NC-1 (D) 7.13 NC-1

(34) Intensity of electromagnetic wave is Wm


2 .

(A) 1.73 u 103 (B) 2.45 u 103 (C) 7.96 u 103 (D) 7.13 u 103

(35) Force acting on surface N.

(A) 7.5 u 10 8 (B) 1.33 u 107 (C) 2.65 u 107 (D) 2.45 u 107

(36) Density on surface Jm


3 .

(A) 1.33 u 1010 (B) 2.65 u 1011 (C) 7.50 u 108 (D) 2.65 u 1010

Ans. : 27 (C), 28 (A), 29 (B), 30 (B), 31 (D), 32 (A), 33 (B), 34 (C), 35 (B), 36 (B)

413
By Avinash More

9 & 10 Optics

Ray optics :
Reflection of light from plane and spherical Surfaces :
Mirrors
(1) Plane (2) Spherical
(i) Concave (f Negative) (ii) Convex (f positive)
For plane mirror : Radius of curvature R f , forcal length f f , mgnification m 1,
– (object distance) = Image distance
u v
If object is moving with velocity v away / towards the mirror then image moves with velocity
2v away / towards mirror.
angle of Incidence (i) = angle of reflection (r)
When angle between two plane mirrors is T , then number of images for object placed
between them,
360q
(1) n  1 , Where even integer which is independent of position of object.
360q

(2) If odd integer


360°

Two possibilities :
(i) If object is placed on bisector of angle between two mirror then take above equation.
360q
(ii) If not placed as above then n

q
360q
(3) For two parallel mirrors, \ n f
0

i=r Deviation

G
i r
G
T

Reflection once Reflection twice


q  i q  

414
By Avinash More

For spherical Mirrors :


(1) R 2 f
(2) Gauss's equation


1 1 1 uv
Ÿ f =
f u v u+v

u = object distance, v = image distance


(3) Lateral magnification
(Transverse magnification)

height of image; h height of object


h’
m h’
h

v

u

f
u f
=

v f
=
f

use sign convension


m is negative for real image, m is positive for virtual image

(1) Light ray incident on plane mirror at angle of 30°. Then find the angle of deviation for incident
ray so that it is reflected from second mirror placed at 60° angle with first mirror.
(A) 120° (B) 240° (C) 150° (D) 90°
(2) Light ray is incidenting on plane mirror. If mirror is rotated at angle T then the reflected ray will
be rotated at angle .

(A) 2 (B) (C) 3 (D)


2

(3) The angle between two mirrors should be kept . so that for both mirrors incident and
reflected rays remains parallel to each other.
(A) 45° (B) 60° (C) 90° (D) 30°
(4) PQ is incident ray and RS is reflected ray of light. Both are parallel to each other. Then which
mirror should be kept on right side so that it may possible ? there may be one or more reflections
are possible by mirror.
P Q

S R
415
By Avinash More

(A) plane mirror (B) convex mirror


(C) plane and concave mirror (D) concave mirror
(5) on the axis of concave mirror of focal lenght f a small linear object of length b is placed at
distance u from the pole. what will be volume of image ?

§ f · 2
2 2

(C) ¨ ¸b
§ f · §u  f ·
(A) ¨ ¸ b
©u  f ¹
(B) u  f 2
b
©u f ¹
(D) ¨ ¸ b
© f ¹

(6) An object is moving with constant velocity vo on the principal axis of concave mirror towards the

mirror. If the verocity of image is vo then find object distance for that.

(A) 2 R (B) 3 R (C) R (D)


R
2

(7) For object distances u1 and u2 from pole of concave mirror the magnification obtained is same,
then focal length of mirror is .

u1  u2 u1  u2
(A) 2 u1  u2 (B) (C) u1  u2 (D)
2 3
(8) The real image obtained by concave mirror is n times larger than object. If focal length of
mirror is f, then object distance will be .

n  1
n  1
f f
(A) f (B) f (C) n  1 (D)
n n
(9) A candle is placed at 24 cm from the surface of convex mirror and a plane mirror is placed such
that the virtual images by both mirrors coinsides. If object is at 15 cm from plane mirror then
focal length of convex mirror will be cm.
(A) 8 (B) 5 (C) 10 (D) 12
(10) A square plate of sides 3 cm is placed at 20 cm from concave mirror. The focal length
of concave mirror is 15 cm. Then the area of image formed by concave mirror will
be cm .
2

(A) 124 (B) 81 (C) 144 (D) 169

Ans. : 1 (B), 2 (A), 3 (C), 4 (D), 5 (A), 6 (C), 7 (B), 8 (A), 9 (A), 10 (B)

Refraction, Total Internal Reflection and it's uses :

sin T1
(1) sin T 2
n21

n2 v1
n1 v2

1 = angle of incidence in medium - 1

416
By Avinash More

2 = angle of refraction in medium – 2

n 1 = absolute refractive index of medium – 1

n 2 = absolute refractive index of medium – 2

v1 = velocity of light in medium – 1

v2 = velocity of light in medium – 2

n21 = Refractive index of medium – 2 with respect to medium – 1

(2) absolute refrective index of medium


c
n n
v
c = velocity of light in air (Vaccume)
v = velocity of light in medium

(3) If n2 ! n1 , then sin 1


! VLQ 2

? T1 ! T2 , deviation T1  T2

(4) If n1 ! n2 , then sin 1


 VLQ 2

? T1  T2 , deviation T2  T1

(5) For transparent slab

? n21 u n12
1
(i) n21
n12 1

(ii) For transparent slab


n31 n32 u n21

n 51 n54 u n43 u n32 u n21

(6) Lateral shift

;
n2 > n1
t sin 1
 2
x =
cos
§ ·
2

¨  ¸  if q1 is very small
¨ ¸
2

© ¹
t 1
1

t thickness of homogeneous medium


(7) Real depth and virtual depth

hi n rarer
(i) (seeing from rarer medium)
@

=
ho n denser

417
By Avinash More

hi Virtual depth of object


ho Real depth of object

§ 1·
Shift = ho  hi ¨1  ¸ ho
© n¹

hi n denser)
(ii) ho
=
n rarer) (seeing from denser medium)

Shift d hi  ho
n  1 ho
(iii) If different immissible liquids are in beaker then virtual depth of bottom.

  ................
d1 d2
n1 n2

n1 , n2 ,.... and d1, d2 .... are refrective indices and real depth of respective
liquids.
(8) (i) Total internal reflection

n1 ! n2 , C = critical angle
n2
sin C
n1

1
sin C n2 1, n1 n
n
(ii) The radius of circular path of vision of fish which is in water
h
depth of fish
n 1
r
2 , h

(iii) Area of circular path of vision, A Sr


2

Sh
2

n 1
2

30°
(11) As shown in the figure the light ray incident at angle
of 30q on surface of air and oil–1. Then after Air
passing through oil–1, oil–2, glass medium it enter to
oil – 1
water. Find angle of refraction of ray in water.
Refrective index of glass and water are 1.51 and oil – 2
1.33 respectively. glass
water

(A) sin (B) sin (C) sin (D) sin


1 1 1 1 1 1 1 1
3.02 1.51 2.66 1.33

418
By Avinash More

(12) A light ray is propagating from space to the medium having refractive index n. If angle of
incidence is twice the angle of refraction then angle of incidence will be .

(A) 2 cos (B) 2 sin (C) 2 sin 1 n (D) cos


1 n 1 n 1 n
2 2 2

(13) A light ray is incidenting on a plane slab of refrective index n and thickness t at very small
incidence angle , then lateral shift in this case is .

tT tT n 1 tT n
(A) (B) (C) n  1 (D) t n T
n n

(14) A plate is placed on the surface of liquid. Refrective index of liquid is 3 . Source of light is at
5

depth 4 m form the surface of liquid. The minimum diameter of plate should be m so
that light can't comes out.
(A) 12 (B) 8 (C) 9 (D) 6

(15) The depth of a container is t. In this container the oil with retrective index n1 is filled up to half

depth and for remaining half depth water of refractive index n2 is filled. What will be virtual

depth of an object placed at the bottom of container ?

t n1  n2 2t n1  n2 t n1 n2
(A) (B) (C) n  n (D) 2 n  n
2t n1n2
2 n1n2 n1 n2 1 2 1 2

(16) A light ray enter to denser medium from air. If reflected and refracted rays are propagating
normal to each other then angle of incidence at medium is .

(A) sin tan C


1 1
(B) cos1 tan C (C) tan sin C
1 1
(D) sin -1 cos C

(17) The refractive index and critical angle of glass with respect to air is n and C respectively. Light
ray is entering at angle of incidence C from air, if angle of refraction is r then sin r .

1 1 1
(A) (B) (C) (D)
1
3 2
n n n n

(18) A light rays takes time t1 sec to cover distance d in air and t2 sec to cover distance 5d in

medium then critical angle of medium with respect to air is .

(A) tan (B) sin t (C) sin t (D) tan t


1 10t
1 1 1 5t 1 1 t 1 1 t
t2 2 2 2

Ans. : 11 (C), 12 (A), 13 (B), 14 (D), 15 (A), 16 (C), 17 (D), 18 (B)

419
By Avinash More

The deviation and dispersion of light by prism :


(i) Equation for prism
ie A i angle of incidence, e angle of emergence
A = angle of prism angle of deviation

(ii) A r1  r2 r1 Angle of refraction at first face r2 Angle of incidence on second face

(iii) If G Gm (minimum angle of deviation)

A  Gm
then i e ? i
2

r1 r
A
r r2
2
(iv) Refractive index of prism

A  Gm
sin
n 2
A
sin
2

(v) For a thin prism (A < <) G m A n  1

(vi) Angular dispersion, G v


A nv  1 , Gr
A nr  1
(vii) T Gv  Gr nv  nr A
Gv  Gr nv  nr
(viii) Disperssive power, D
G n 1

Gv  Gr nv  nr
where, G , and n
2 2
(ix) For maximum angle of deveation, i 90q

(x) For no emergence, r2 ! C

(19) The refractive index of material of prism is cot A


2
where A is angle of prism. what will be minimum

angle of deviation by this prism ?

(A) 180° – 2A (B) 180° – A (C) 90° – A (D)


A
2
(20) On the prism having angle of prism 60° when light is made incident at angle 50° it suffer
minimum deviation. The minimum angle of deviation will be .
(A) 60° (B) 55° (C) 40° (D) 45°
(21) Angle of prism is 60°. What will be minimum angle of deviation for this prism ? Refractive index
of prism is 2.
(A) 45° (B) 30° (C) 60° (D) 35°
420
By Avinash More

(22) The refractive index of prism is 2 . The minimum angle of deviation and angle of prism is same
for that prism. Then angle of prism will be .
(A) 30° (B) 45° (C) 60° (D) 90°
(23) A horizontal ray is incident on the prism having angle of prism 4° and refractive index 1.5 plane
mirror is placed behind this prism as shown in figure, then find total angle of deviation for ray.

P
M (A) 4° clockwise (B) 178° clockwise
(C) 8° clockwise (D) 2° clockwise

(24) As shown in the figure one side is silvered for prism ABC with refractive index 2 . Angle of
incidence is 45°. The refracted ray is reflected by the surface AB in the same direction, then
‘CAB .
A

(A) 20° (B) 10°


n 2
(C) 30° (D) 25°

B
C

(25) A ray of light propagating inside prism parallel to the base incident on the hypotenuse of prism of
right angled prism. If refractive index of material of prism is n then for total internal reflection by
hypotenuse the maximum value of the angle of base should be .

1 § n  1 ·
(A) cos (B) sin ¨ ¸
1 1
n © n ¹

(C) tan (D) sin


1 1 1 1
Tmax n n

Ans. : 19 (A), 20 (C), 21 (B), 22 (D), 23 (B), 24 (C), 25 (A)

l Equation of lens, Magnification, power of lens, combination of thin lenses in contact :


Refraction at a spherically curved surface.
n2  n1
 
n1 n2
(From rarer to denser medium)
u v R
421
By Avinash More

n2 n1 n1  n2
  (From denser to rarer medium)
u v R

n1 Refractive index of medium in which object is placed. n2 Refractive index of second medium.

u = object distance, v = Image distance, R = Radius of curvature.

For lens

1 1 § n2  n1 · § 1 1 ·
(1) General equation of lens :  u  v ¨
¨ n
¸
¸
¨
¨R
 ¸
¸
© 1 ¹ © 1 R2 ¹

§ 1 1 ·
n  1 ¨¨  ¸
¸
1
(2) Lens - Maker's equation, f
© R1 R2 ¹


1 1 1
(3) Gauss, equation : f v u

(4) Newton's equation : x1 x2 f1 f 2

(5) Lateral magnification : m


v
u

f v
f
f
f u

v2  v1
(6) Longitudinal magnification : m u2  u1

For small object

§v· § f · § f v·
2 2

¨ ¸
2

¨ ¸ ¨ ¸
dv
©u¹ © f u¹ © f ¹
m
du

Ai
(7) Arial magnification : ms Ao
Ai Area of image A o = Area of object

? ms
2
m

(8) Power of lens

1
f unit is D, m
P , 1

422
By Avinash More

(9) Combination of Lens :

1 1 1
(i) f
=
f1
+
f 2 (Two lens)

1 1 1 1
f n (Generalized)
= + + ...+
f f1 f2

(ii) Equivalent power (Far lenses in contact) : P = P1 + P2 + ...+Pn

(iii) For lenses with seperation

1 1 1 d
f
= +
f1 f 2

f1 f 2 d distance between lenses

Power, P = P1 + P2  dP1 P2

(iv) Magnification for lenses in contact.

m m1 u m2 u .... u mn

(10) Combination of lens and cenvex mirror

f R 1 v  d
2 2

d distance of lens from mirror

(11) Relationship between velocity of object and velocity of image. If object is moving from large
distance toward lens, then velocity of image is,

§ f ·
2
vi ¨ f  u ¸ vo
© ¹

Initially vi increases showly then increases repidly.

(12) For silvered lens

If one surface of lens is silvered then it behave as mirror so it's focal length.

1 1 1

f f1 fm

f1 Focal length of lens due to which refraction takes place.

fm Focal lenght of mirror due to which reflection takes place.

423
By Avinash More

(i) For plano convex lens.

f1 fm
f
R R
n 1
f1 fm
2

? f R
2n

(ii) For convex plano lens

f1
f
fm

f
R
n 1
f1 fm

? f R
n 1
(iii) For cenvex lens

f
f1 fm

R ? fm R
2 n  1
f1
2

? f R
n 1

424
By Avinash More

(13) Displacement method :


When distance between object and screen D ! 4 f than by keeping it constant if convex
lens of focal length of f is placed between object and screen then real image at length I1 is
formed on the screen. When lens is displaced by distance x the image of length I2 obtained on
screen then.

(i)
D2  x2
f
4D
(ii) length of object = I1 I2
(14) Division of lens in equal parts and combination
(i) In the direction of optical axis :
Division Combination

2f 2f
f
P P P
2 2

f f
(ii) If the direction of principal axis :
Division Combination

f,P f,P

f f
2

where f is focal lenght of main lens and P is it's power


(26) If lens is displaced towards object by 20 cm then the magnification remains same so focal length
of lens will be cm.
(A) 17.5 (B) 18.5 (C) 16.5 (D) 15.5
(27) Convex lens form real image of object on the screen placed at constant distance. Lens is moved
on principal axis away from screen with constant velocity v 0.5 ms 1 . object is also given
proper velocity to keep image on the screen. When height of object is twice the height of image
at that time find velocity of object.
(A) 1.5 ms 1 away from screen (B) 1.5 ms 1 towards the screen

(C) 2.5 ms 1 towards the screen (D) 2.5 ms 1 away from screen

(28) An object is place on principal axis at 10 cm on left hand side of cenvex lens L1 having focal
length 20 cm. Another lens L2 of focal length 10 cm is placed at 5 cm on right hand side of first
lens co-axially. Find the distance of final image from second lens and magnification.

(A) (B) (C) (D)


50 cm, 3 70 cm, 3 50 cm, 4 25 cm, 3
3 4 3 4 3 3 3 4

425
By Avinash More

(29) For plano cenvex lens the cenvex surface is silvered. It's radius of curvature is R. Find the focal
length of cencave mirror formed by it. n = 1.5

(A) – R (B) 4R (C) 2R (D) 3R

(30) Two thin prism of angle of prism A and refractive index n are placed so that their bases are in
contact with each other. This system behave as convex lens. As shown in figure parallel rays are
made incident on it then find it's focal length.

A
(A) n 1 A (B) n 1
h hA

2h
(C) n 1 A (D) n  1
2h 2h A

(31) As shown in figure (i) a thin convex lens having focal lenght of 10 cm is cutted in two equal
parts. These two parts are arranged as shown in figure (ii). An object of height 1 cm is placed at
distance 7.5 cm from this system then height of image will be cm.
B

(A) 1 (B) 2
A (ii) (C) 0.5 (D) 4

(i)

(32) The power of convex lens in air is + 10 D. The refractive index of convex lens is 1.5. If on left side
of lens is air and on right side of lens is water of retractive index 1.33 then find the power of lens.
(A) 2.42 D (B) 3.67 D (C) 4.42 D (D) 6.70 D
(33) The power of convex lens in air is +5 D. If it is immersed in water then it's power will
be .
(A) 4.25 D (B) 1.25 D (C) 2.25 D (D) 3.25 D
(34) For two symmetric convex lens A and B focal lengths are same but radius of curvature are
different so that RA 0.9 RB . If n A 1.63 then nB ?

(A) 1.7 (B) 1.6 (C) 1.5 (D)


4
3
(35) Convex lens of focal length 25 cm and concave lens of focal length 20 cm are placed co-axially
at distance d from each other. If the resultant power of system is zero then find d.
(A) 5 m (B) 5 cm (C) 3 cm (D) 0.5 m
(36) The distance between an object and screen is 1 m. From the possible positions of lens for one position is at
40 cm at that time the image of object is obtained on the screen, then the power of lens is .
(A) 5 D (B) 6 D (C) 2 D (D) 4 D
Ans. : 26 (A), 27 (B), 28 (C), 29 (D) 30 (A), 31 (B), 32 (D), 33 (B), 34 (A), 35 (B) 36 (A)

426
By Avinash More

Microscope and Astronomical Telescope (Reflecting and Refracting) and their magnifying power
(1) Simple microscope (Magnifying lens)

(i) magnification : m 1 D (Image is obtained at D.)


f

1  Da (Distance between eye and lens is a)


f

D = Distance of distinct vision


f = Focal length of lens

(ii) f (If image is formed at infinite distance)


m D

(If eye is at distance a from lens)


D a
f

(2) Compound Microscope :

Resultant Magnification, m mo u me

L u D
fo fe

L = Tube length, D = Distance between eye piece and image, fo = Focal lenght of objective,

fe = focal length of eyepiece

fe ! fo

Final image obtained is enlarged, virtual and inverted.


(3) Astronomical Telescope :

(i) Magnification : m
D

fo ! fe
fo
fe

§ fe ·
(ii) ¨1  D ¸
fo
mD
fe © ¹
D angle subtended by the object with objective or eye
Angle subtended by the final image with eye.
optical length f0  fe

Tube length L t fo  f e
Final image formed is small, virtual and inverted.
(4) Terestrial Telescope
– To observe distant object from Earth
– Three lens : objective, eye piece and Erecting lens.
– Final image formed is small virtual and eracted.

427
By Avinash More

magnification m
fo
fe

L f o  fe  4 f
f focal length of eracting lens
(5) Galelian Telescope :
– It is one type of terestrial telescope but it's vision range is small
– Convex lens is objective
– Concave lens is eyepiece
fo
m
fe

L fo  fe
(6) Reflecting Telescope :

magnification, m
fo
fe

fo Focal length of main concave mirror


fe Focal lenght of eyepiece
(7) Defect of vision
(A) Near sightedness (myopia)
Removel - concave lens
(i) f d f focal length, d distance of defect
(ii) Person can see up to distance x and what to see up to distance y then

xy
f
x y
y!x

x y
P
xy

(B) Far sightedness : (Hypermetropia)


Removel - convex lens
If person can't see object away from d and object is at distance D from eye then to
see that object.
dD dD
f
d D , P
d D

(C) Astigmatism
Removel - cylindrical lens
(D) Press Biopia
Removel - Bifocal lens

428
By Avinash More

(37) In compound microscope focal length of objective is 1 cm and focal length of eye piece is 5 cm
and both lenses are at 12.2 cm from each other. The object should be placed from objective at
distance cm so that final image is formed at the distance of distinct vision and the
resultant magnification by instrument will be .
(A) 6, 22 (B) 8, 44 (C) 8, 34 (D) 6, 32
(38) Focal length of eyepiece of telescope is 5 cm. Final image is formed at very large distance. At
that time magnification is 10. If image is formed at distance of distinct vision 25 cm at that time
find the magnification.
(A) 60 (B) 50 (C) 10 (D) 12
(39) Astronomical telescope is adjusted for infinite distance. If objective is replaced by slit of length x. The
image formed by eyepiece of this slit is of length y, then magnification of telescope will be .

(A) x  y
x
(B) x  y (C) y (D)
y
x

(40) Diameter of moon is 3.5 u 106 m and it is at distance 3.8 u 108 m from Earth. It is observed
with the telescope having objective focal length of 2 m and eyepiece focal length of 10 cm then
image will be formed at angle .
(A) 11q (B) 21q (C) 31q (D) 41q
(41) For Galelian telescope focal length of objective and eyepiece 50 cm and 5 cm respectively. If in
normal condition it is used for large distance then it's magnification will be and tube
length will be .
(A) 5, 25 (B) 10, 55 (C) 10, 40 (D) 10, 45 cm
(42) A person can see the objects easily between 20 cm to 75 cm. If he wear the spects of power
1 D then give range of his vision.
(A) 25 cm to 300 cm (B) 25 cm to 200 cm (C) 30 cm to 200 cm (D) 30 cm to 300 cm
Ans. : 37 (B), 38 (D), 39 (C), 40 (A), 41 (D), 42 (A)

Wave optics
Interference, young's experiment of two slits and equation of width of fringe

(1) Type of wave front Indensity Amplitude

ID AD
1
(i) Spherical wavefront
1
r2 r

ID AD
1
(ii) cylindrical wavefront
1
r r

(iii) Plane wavefront I D rq A D rq


r distance from the source

(2) (i) Phase difference


k r2  r1 = k (Path difference), k
2S
O
2S
(ii) Phase difference = (time difference),
T
429
By Avinash More

(3) Average intensity of light at a point due to superposition of two harmonic waves,
emitted from two point like sources.

I I1  I 2  2 I1 I 2 cos 2
 1
(i) In coherent sources

1
z 2 , I I1  I 2

(ii) Coherent sources

1 2
 I2  I1 Constant

I I o cos 2 , Phase difference


2


k r2  r1
Path difference r2  r1

(4) Methods to obtain coherent sources


(i) By division of wavefront (Young's Experiment)
(ii) By division of Amplitude (Reflection by thin film layers)
(5) Condition for constructive interference

phase – difference, k r2 – r1 = 2n ½°
path – difference, r2 – r1 = n
¾ n = 0,1, 2,3, ....
°
¿

Intensity I Io 4 I’, I’ intensity of both waves I1 I2


I’

(6) Distructive Interterence

Phase – difference k r2  r1 = 2n  1 ½
°
¾ n = 1, 2, 3, ....
Path – difference r2  r1 = 2n  1 °
2 ¿

Intensity I 0

(7) Young's two slit experiment

(i) Path difference r2  r1 d Distance between two coherent sources

d sin T D Distance between slit and screen

d tan T x Distance of fringe from centre of screen

angular distance
dx
T
D

(ii) Constructive Interference in Young's experiment (For bright fringes)

430
By Avinash More

d sin T nO

nO, n
dx
0,1,2,3,....
D

(iii) Destructive interfrence in young's experiment (Dark fringes).


O
d sin T 2n  1
2

O
2n  1
dx
, n 1, 2,....
D 2

(iv) Distance between two consecutive bright or dark fringes in Young's experiment.
OD
x
d

OD
(v) Width of fringe
x
2 2d

(vi) In Young's experiment if n2 th bright fring of light with wavelength O2 superpose


to n1 th bright fringe of wave length O1 then,

n1 O1
dx
D

n2 O2
dx
D

? n1 O1 n2 O2

(vii) In Young's experiment If transparent sheet of thickness t and refractive


index n is placed in the path of one ray,

(a) Optical path difference between two rays nt  t n  1 t


(b) If fringe is displaced by x, then path difference n 1 t

n 1 t
dx
D

n 1 t
D
x
d

x is called Lateral shift.


The fringes will shifts towards the light rays in the path of which the sheet
is placed, that shifting does not depends on the order of fringe and wavelength.
(viii) For any wavelength absent on screen against the slit (Distructive interference)
431
By Avinash More

2
O
d
2n  1 D
n 1,2,3,....

d2 d2 d2
Absent wavelengths, O , , , ...
D 3D 5 D

(ix) Distance between fringes ' x


(a) Distance between nth and mth bright fringes

OD
(i) ' x n  m n!m
d
(b) Distance between nth bright and mth dark fringes

§ 1 · OD
'x ¨n  m  ¸
© 2¹ d
n!m

§ 1 · OD
(ii) 'x ¨m  n  ¸
© 2¹ d
m!n

(x) Visibility of fringes


I max  I min
I max  I min
V

2 D ª I1 º
«I D»
D 1 ¬ 2 ¼

2 I1 I 2
I1  I 2

(a) If I min 0, V 1 (maximum)

(b) If I max 0, V 1

(c) If I max I min , V 0

(8) Interference by thin film


(i) For reflecting light

(a) 2 P t cos r 2n  1 O2 n 1, 2 , 3,... (constructive Interference)

(b) 2 P t cos r nO , n 1, 2 , 3,... (Destructive Interference)


r angle of reflection
(ii) For refracting light
(a) 2 P t cos r nO (constructive Interference) n 1, 2, 3,...

(b) 2 P t cos r 2n  1 O2 (Destructive Interference) n 1, 2,3,...

432
By Avinash More

(9) Lord's mirror

Path-Difference S2 P  S1 P n O (Destructive Interference) n 1, 2, 3,...

2n  1 O2 (Constructive Interference)

n 0,1, 2, 3,...
(10) Fresnel Biprism
Fresnel biprism is made by joining base of two prisms having small prism angle.
d Distance between two coherent sources, a Distance of slit from biprism,
b Distance of screen from biprism D ab Distance of screen from slit,
D Prism Angle,
n Refractive index of material of prism

OD
width of fringes
d
Where, D ab

d 2a n  1 D

(11) Newton's Ring


Placing convex lens on the plane glass plate and incidenting light normally on it the
circular rings of different radius are observed.
R = Radius of curvature of cenvex surface.
At the centre, dark and then successive bright and dark rings are observed.

(i) radius of nth dark ring, rn OR n, n 0,1, 2, 3,...

rn D n

(ii) radius of nth bright ring rn 2n  1 OR n 1, 3, 5, ...


2

rn D 2n  1

(iii) Diameters of nth dark ring Dn and n  P th ring is Dn  P then wavelength

Dn2  P  Dn2
O
4 PR

(12) Doppler effect for Light


f real frequency, O real wavelength
f’ virtual frequency, O ’ virtual wavelength
v Velocity of light source with respect to steady listener
c velocity of light

433
By Avinash More

(i) If listener is steady and source of light is moving towards listener then O ! O ’

'O O v (Violet shift)


c

(ii) If listener is steady and source of light is moving away from the listener then O ’ ! O

Doppler shift ' O O v (Red shift)


c

(iii) Doppler Broadening


When gas is filled in electric discharge tube then the range of spectrum due to
random motion of atoms of gas.

' f v f Ÿ r' f rv f
c c

'O O Ÿ r' O
v
rv O
c c

(13) Radar
For microwaves transmitted towards the plane and reflected from it, the frequency difference,

' f 2v f
C

(43) In Young's experiment of two slit the maximum intensity is I 0 . The distance between two slits is
d 5O, O wavelength of monochromatic light. Then find the intensity at a point on the
screen in front of the slit Distance between slit and screen D 5d .

(A) I 0 (B) 0 (C) (D) 2 I 0


Io
2

(44) As shown in the figure from the centre of a circle of large radius at same distances two point
like coherent sources S1 and S 2 are placed. Where d 2O , O wavelength of emitted light,
then find the possible values of T for which on the semi circle the intensity is maximum.

T (A) 20°, 50°, 150° (B) 30°, 80°, 120°


O
(C) 45°, 90°, 170° (D) 30°, 90°, 150°
S1 S2
d 2O

(45) The monochromatic, collimated light beam is incidenting in Young's experiment, which makes an

angle T sin 1 O with a normal to the plane of slit, then find the intensity at the centre of screen.
2d

(A) I 0 (B) 2I 0 (C) 0 (D) 4I 0

434
By Avinash More

(46) Find the ratio of intensity of central bright fringe to the intensity at position situated at distance
one fourth of the distance between two successive fringes.

(A) 2 (B) (C) 4 (D) 16


1
2
(47) In Young's experiment when a glass plate of retractive index of 1.5 is placed in the path of one
ray of the rays forming interference, the fring pattern is displaced by y. When this plate is

replaced by sheet of mica having same thickness then fringe pattern is displaced by y . Then
3
2
find the refractive index of second plate.
(A) 1.5 (B) 1.75 (C) 1.25 (D) 1.00
(48) In young's experiment of two slits the light of wavelength 600 nm is used. If a transparent plate
of thickness 1.8 u 105 m having refractive index 1.6 is placed in the path of one ray then how
many fringes of same type will be displaced.
(A) 18 (B) 9 (C) 36 (D) 12
(49) In Young's experiment the distance between two slits is 0.055 cm and distance of screen from slit
is 100 cm. Then find the distance between second bright fringe at upper side of centre to the

third dark fringe at lower side of centre. Wavelength of light is 4000 A .


D

(A) 0.3 cm (B) 0.5 cm (C) 0.4 cm (D) 0.6 cm


(50) As shown in the figure two coherent sources S1 and S 2 are placed on Y-axis. The wavelength of
light emitted from it is O . Distance between two sources is d 2O , then at what distance on positive
X-axis first dark fringe will be obtained.
5O 3O
S1 (A) (B)
12 12

7O 15O
(C) (D)
S2 12 12
X

(51) The ratio of intensities of light emitted from two coherent sources is 4, then find visibility of fringes.

(A) (B) 4 (C) 9 (D)


3 4
5 5

D
(52) In young's experimnt the width of fringe is 2 mm when wavelength of light is 6000 A . If the entire
instrument is immersed in water having refractive index 1.33 then what will be width of slit ?
(A) 0.5 mm (B) 3.5 m (C) 1.5 mm (D) 2.5 m

Ans. : 43 (B), 44 (D), 45 (C), 46 (A), 47 (B), 48 (A), 49 (C), 50 (C), 51 (D), 52 (C)
Diffraction :
O
(1) Diffraction D O wavelength, d width of slit
d
(2) Types of Diffraction : (i) Fresnel (ii) Fraunhoffer
(3) Fraunhoffer Diffraction by single slit

435
By Avinash More

(A) Central (Zeroth order) maximum


(i) q

(ii) Maximum Intensity I0


(B) Minimum of nth order
nO
(i) sinT n , n 1, 2, 3, .... Tn angle of Diffraction for nth order minimum
d
O Wavelength
If Tn is very small then,
sin T n  T n | tan T n

nO
Tn
d
[Tn Angular width of nth order minima from central maxima]

xn
tan T n , xn Distance of nth minima from centre of central
D
maxima

nO xn
?
d D
OD nO f
(ii) xn n , D Distance between slit and screen
d d
f focal length of convex lens
(iii) D nS Intersity I = 0
S d sin T
Where D
O
(C) nth order maximum
O
(i) sin T n 2n  1 , n 1, 2,3,...
2d
Tn Diffraction angle for nth order maxima
T n is very small sin T n | T n | tan T n
(ii) Distance of n th order maxima from centre of central maxima,
O
xn 2n  1 D, D = f
2d

§ sin D ·
2
(iii) Intensity, I I0 ¨ ¸
© D ¹

S d sinT S
D ,D 2n  1 ,n 1, 2,3,...
O 2
Intensity decreases rapidly with increase in the order of maxima

436
By Avinash More

(4) Fresnel distance :


The distance up to which bending of light is very less.

d2
Zf d Linear dimension of obstacle
O
(5) Diffraction by Grating :
The device formed by equispaced paralled slits having equal width is called grating.
d ae a width of part of obstacle d grating element,
Total width e Width of slit
d=
ruling

d sin T nO , n 0,1, 2, 3,...


Total bright fringes 2n  1 (For bright fringes), T Diffraction angle
(6) In the diffraction due to circular obstacle the central ring formed is bright. Which is called
Airy's disc. In the surrounding of it bright and dark concentric rings are observed which is
called Airy's rings.
(7) (A) Rayligh's criteria for circular obstacle like lens
1.22 O
sin T | T O wavelength of light D Diameter of lens
D
1.22 O
(B) (i) Angular resolution of telescope, D min
D
1
(ii) Resolving power of telescope, RP D min

D
1.22 O
(C) (i) For microscope
1.22 O
dm f dm The minimum distance between two point
D
like object so that it can be seen distinet

1.22 O
2sin E O wavelength

f Focal length of objective


D Diameter of objective
(ii) If medium of large refractive index (n) is between objective and object then

2n sin E
RP
1.22 O
, n sin E Numerical Aperture

1
RP D
O

437
By Avinash More

(8) Malus law : I I 0 cos T , T


2
Angle between optic axis of two parallel Tourmaline plates

I0 Intensity of incident light, I Intensity of emerging light

S 3S
T , ......, then I 0 (Crossed)
2 2

T 0, S ......, then I I0

When unpolarized light passes through the polarizer then for emerging light, I ave
Io
2

(9) Brewster's law

Tp  r 90q Tp angle of incidence, r angle of refraction

n tan T p n Refrative index of transparent medium

(10) Methods to obtain polarized light


(i) Polaroid, e.g. Tourmaline plate (ii) Polarization by reflection
(iii) Double refraction, e.g. Nicol prism (iv) By scattering

(53) In the Fraunhoffer diffraction by a single slit the distance between first and fifth minima is
0.40 mm. The wavelength of light incident normal to slit is 550 nm and distance of screen from
slit is 50 cm, then find the angle for first order minimum.

(A) 1.5 u 10 4 rad (B) 2 u 10 4 rad (C) 3 u 10 4 rad (D) 2.5 u 10 4 rad

(54) In the Fraunhoffer diffraction by single slit the width of slit is 0.60 mm. Wavelength of light normal
to slit is 600 nm. Distance of screen from slit is 60 cm, then find width of central maximum.
(A) 1.2 mm (B) 0.6 mm (C) 2.4 mm (D) 4.8 mm
(55) In the Fraunhoffer diffraction by single slit the wavelength of light incidenting normal to slit is
D
5000 A , distance between screen and slit is 100 cm. First order minima is obtained at 5 mm
from central maximum then find the width of slit.
(A) 0.5 mm (B) 0.2 mm (C) 1 mm (D) 0.1 mm
(56) In the Fraunhoffer diffraction by a single slit the light of wavelength O is incidenting normally on
the slit of width d. Distance of screen from slit is D. If linear width of central maximum is half
then the width of slit, then d .

OD
(A) (B) OD (C) 4O D (D) 2O D
4

(57) Light of wavelength 600 nm incident on an obstacle and least bending of light is up to 15 m then
find the linear dimension of obstacle.
(A) 3 mm (B) 2 mm (C) 4 mm (D) 5 mm

438
By Avinash More

(58) In the Fraunhoffer diffraction by single slit, if wavelength of light incidenting normal to slit is

doubled, distance between slit and screen made three times and width of slit is made times
3
2
then width of central maximum will be .
(A) Three times (B) Four times (C) Double (D) Half

(59) Light of wavelength 800 nm incident normally on the grating having 1.25 u 105 . How many
line
meter
maximum number of bright fringes can be obtained on the screen kept at large distance ?
(A) 17 (B) 19 (C) 21 (D) 23

(60) For the telescope having objective aperture of 4.88 m, the wavelength of light is 6000 A , then
D

what should be minimum resolving angle ? 1 rad 2 u 105 sec.

(A) 3 u 102 second (B) 2 u 102 second (C) 2.5 u 102 second (D) 3.5 u 102 second

(61) In electron microscope the accelerated potential difference is increased from 10 kV to 90 kV


then new value of resoalving power will be .

(A) 4 R (B) 2 R (C) 3 R (D)


R
2

(62) Five polaroids are arranged such that the optic axis of each is making 30° with optic axis of
previous one. unpolarized light incident on first poleroid then how much part of light will emerge ?

(A) (B) (C) (D)


1 1 81 1
128 256 512 512

(63) The unpolarized light with energy 3 u 103 J incident on the polarizer of area 3 u 104 m 2 . If

polarizer is rotating with angular speed of 3.14 rad s 1 then find the energy emerging per
1 rotation.

(A) 47.1 × 10-4 J (B) 27.1 × 10-4 J (C) 37.1 × 10-4 J (D) 17.1 × 10-7 J

(64) Two polaroids are in crossed position and the intensity of emerging light is zero. If a third polaroid is
placed between these two polaroid at the half angle of angle between optic axies of those two, then
intensity of emerging light will be . Where I 0 is maximum intensity of incident light.

I0 I0 I0
(A) (B) (C) I 0 (D)
2 4 8

(65) velocity of light in air is 3 u 108 ms 1 and in glass is 2 u 108 ms 1 . If light ray incident at angle
of polarization then find angle of refraction.
(A) 37.7° (B) 27.7° (C) 17.7° (D) 47.7°

439
By Avinash More

Ans. : 53 (B), 54 (A), 55 (D), 56 (C), 57 (A), 58 (B), 59 (C), 60 (A), 61 (C), 62 (C),
63 (A) 64 (D), 65 (A)

Experimental Techniques :
(1) (i) Convex mirror (ii) Concave mirror (iii) To measure focal length of convex lens.
(2) Draw the graph of angle of deviation versus angle of incidence using equilateral prism.
From that deremine refractive index of materical of prism.
(3) Determine refractive index of a slab using travelling microscope.
Graphs

(i) Concave mirror, v o u (Both Negative) (ii) Concave mirror, o (Both Negative)
1 1
v u
u B O m u1
M O
450

C 1
v

v N
OB OC 1 1
f f
2 2 OM ON
(iii) Concave mirror, m o u , m magnification (iv) Convex lens, v o u (v positive
u distance from pole (Both Negative) u Negative)

v
m

mD
1
f
OB OC C
u 2 2

O u B O
u

(v) Convex lens Negative, (vi) Prism oi


1 1 1 1
o (
v u u v
positive)

1 1 N G
f
| OM | ON
n
1
G Gm
v
i=e
M m u1 O i

440
By Avinash More

(66) Which graph is v o u for concave mirror. Where f = focal length of concave mirror. Both
distances changes from zero to infinity.
(A) (B)

v v

f u
O O
f u

(C) (D)

v v
f
O O u
f u

(67) Focal length of concave mirror is 20 cm. The lateral magnification obtained by it is 4 then find
object distence.
(A) 30 cm (B) 25 cm (C) –25 cm (D) –30 cm
(68) In the experiment of combination of convex mirror and convex lens the distance between these
two is 10 cm. When object is placed at a certain distance it's image is formed at the same
position. When only convex lens is used the image is formed at 60 cm then focal length of mirror
will be cm.
(A) 20 (B) 30 (C) 15 (D) 25
(69) By the convex mirror having focal length 12 cm the image of an object is obtained one fourth of
length of object, Then find the distance between object and image. Linear object is on axis
normal to axis.
(A) 45 cm (B) 40 cm (C) 30 cm (D) 37.5 cm
(70) A bearn of parallel rays incident on convex lens. Their path is as shown in figure.

(A) n1 n2 ! n3 (B) n1  n2  n3
n1
(C) n1 ! n2 ! n3 n2  n3
n2 n3
(D) n1

(71) For convex lens the distance between object and real image is d. If lateral magnification is m
then find it's focal length.

1  m d
d md 1
(A) (B) (C) m 1  m 2 (D)
1  m 1  m
2
2 2

441
By Avinash More

(72) Refractive index of a prism having prism angle A is 3 . If it's minimum angle of deviation is A
then find it's angle of prism.
(A) 45° (B) 30° (C) 60° (D) 90°
(73) For thin convex lens object distance is 0.2 m and image distance is 0.5m. Image is formed on
other side of lens then it's focal length will be m.
(A) 0.143 (B) 0.243 (C) 0.343 (D) 0.443
(74) The depth of a well is 6.65m. If well is completely filled with water and refractive index of water is
1.33 then seeing from above the bottom of well will be observed shifted upward by .
(A) 3.65m (B) 5m (C) 1.65m (D) 12.65m

Ans. : 66 (B), 67 (C), 68 (D), 69 (A), 70 (A), 71 (B), 72 (C), 73 (A), 74 (C)
Assertion - Reason type Question :
Instruction : Read assertion and reason carefully, select proper option from given below.
(a) Both assertion and reason are true and reason explains the assertion.
(b) Both assertion and reason are true but reason does not explain the assertion.
(c) Assertion is true but reason is false.
(d) Assertion is false and reason is true.

(75) Assertion : For spherical mirrors the Gauss's equation is applicabale only when aperture of
mirror is small.
Reason : Laws of reflections are true only for plane mirrors.
(A) a (B) b (C) c (D) d
(76) Assertion : object is placed at focal point of concave mirror the image is obtained at intinite
distance.
Reason : Concave mirror behave as diverging surface.
(A) a (B) b (C) c (D) d
(77) Assertion : For the observer in denser medium the object observed in rarer medium is seen
uplitted.
Reason : This is observed due to refraction.
(A) a (B) b (C) c (D) d
(78) Assertion : The phenonenon in which white light gets divided in to it's constituent colours is
called dispersion of light.
Reason : The spectrum obtained by a prism made up of flint gless is wider, more dispersed
and more detaited as compared to one obtained by common crown glass.
(A) a (B) b (C) c (D) d
(79) Assertion : To rectity the defect of near sightedness convex lens is used.
Reason : For far Sightedness the image of distant object is formed behind ratina.
(A) a (B) b (C) c (D) d

442
By Avinash More

(80) Assertion : When light ray passes from glass to air at that time the critical angle for violet is
minimum.
Reason : Wavelength of violet colour is more than other colours.
(A) a (B) b (C) c (D) d
(81) Assertion : In Young's two slit experiment the interference is observed.
Reason : The effect produced by superposition of two or more waves is called interference.
(A) a (B) b (C) c (D) d
(82) Assertion : In Young's double slit experiment the path difference for first order maximum fringe
is O .

Reason : Path difference (phase difference)


O
2S
(A) a (B) b (C) c (D) d
(83) Assertion : Due to intense scattering of blue light sky seems bluish.
Reason : Intensity of scattered light inversly praportional to fourth power of wavelength
of light.
(A) a (B) b (C) c (D) d
(84) Assertion : Touemelive plate is a natural polarizer.
Reason : The device convert unpolarized light in to polarized light is called polarizer.
(A) a (B) b (C) c (D) d

Ans. : 75 (C), 76 (C), 77 (A), 78 (B), 79 (D), 80 (C), 81 (A), 82 (B), 83 (A), 84 (A)

Comprehension Type Questions :


paragraph :
The ray of light incidenting on transperent medium at an angle of 60° is reflected as totally
polarized light. For that only 15 percent components is reflected from incidenting V components.

(85) Find refractive index of transparent medium.


(A) 1.51 (B) 1.73 (C) 1.61 (D) 1.41
(86) What will be angle of refraction in transparent medium ?
(A) 30° (B) 60° (C) 45° (D) 50°
(87) For transperent medium what will be angle between reflected ray and refracted ray.
(A) 45° (B) 30° (C) 60° (D) 90°
(88) In refracted ray S components will be %.
(A) 85 (B) 15 (C) 100 (D) 70
Pragraph : Radius of curvature of concave mirror is 30 cm. Object is placed at 20 cm on principal axis.
(89) Focal length of concave mirror will be cm.
(A) 15 (B) –15 (C) –30 (D) 30
(90) Image distance will be cm.
(A) –20 (B) –30 (C) –40 (D) –60

443
By Avinash More

(91) Find magnification of image.


(A) 2 (B) 3 (C) –3 (D) –2
(92) Give the type of Image.
(A) Real, Inverted, enlarged (B) Virtual, erect, enlarged
(C) Virtual, erect, small (D) Real, Inverted, small

Paragraph : As shown in the figure a thin rod AB 6 cm


of length 6 cm is placed on the principal axis
18 cm
of concave mirror in such a way that it's P
B A F
real image BcAc is obtained. Focal length of
Ac Bc
mirror is 18 cm.

(93) Distance of Bc from the pole of mirror will be cm.

(A) 18 (B) 36 (C) 30 (D) 24

(94) Distance of A from the pole of mirror will be cm.

(A) 30 (B) 24 (C) 32 (D) 45

(95) Distance of A c from P cm.

(A) 27 (B) 30 (C) 24 (D) 45

(96) Length of image will be cm.

(A) 12 (B) 27 (C) 9 (D) 6

Paragraph : In Young's experiment distance between two slits is 0.1 mm and distance of screen from slit

is 1 m. If wavelength of light is 5000 A then,


D

(97) Find distance between two consecutive bright fringes.

(A) 5 cm (B) 5 mm (C) 10 mm (D) 10 cm

(98) Angular distance of third bright fringe from centre of central fringe will be rad.

(A) 0.15 (B) 0.075 (C) 0.030 (D) 0.015

(99) Find the distance of fourth dark fringe from centre of central fringe.

(A) 1.75 u 102 m (B) 1.75 mm (C) 3.5 u 102 cm (D) 3.5 mm

(100) Find the width of fringe.

(A) 0.25 mm (B) 2.5 mm (C) 5 mm (D) 0.25 cm

Ans. : 85 (B), 86 (A), 87 (D), 88 (C), 89 (B), 90 (D), 91 (C), 92 (A), 93 (B), 94 (A),
95 (D), 96 (C), 97 (B), 98 (D), 99 (A), 100 (D)

444
By Avinash More

Match the columns :


(101) Focal length of concave mirror is 10 cm then,
Column-1 Column-2
(A) a o p, b o q, c o r, d o s
Objict Distence Imege
(B) a o s, b o r, c o p, d o q
(a) 5 cm (p) enlarged, inverted, Real
(C) a o q, b o p, c o r, d o s
(b) 15 cm (q) Size of object, inverted, Real
(D) a o s, b o p, c o q, d o r
(c) 20 cm (r) small, Erected, Virtual

(d) 25 cm (s) Enlarged, Erected, Virtual

(102)

Column-1 Column-2

(a) Young's Double slit experiment. (p) Incoherent sources

(b) Sources having different angular frequencies. (q) Coherent sources

(c) Each point on wave front behave as source. (r) Principle of Superposition

(d) Displacement of particle is (s) Hygen's Principle

(A) a o p b o r c o q d o s
(B) a o q b o p c o s d o r
(C) a o p b o q c o r d o s
(D) a o r b o p c o s d o q
(103)

Column-1 Column-2

nv  nr
(a) Angular Dispersion (p)
n 1

(b) Thin prism (q) v


 r

(c) Dispersive power (r) A n  1 A’ n ’  1

(d) Dispersion without (s) $ n  

deviation for two prism


(A) a o q b o s c o p d o r
(B) a o p b o q c o r d o s
(C) a o p b o s c o q d o r
(D) a o s b o q c o r d o p

445
By Avinash More

(104)
Column-1 Column-2
(a) Brewstor's law (p) I I 0 cos T
2

(b) Snell's law (q) x1 x2 f


2

sin
(c) Malus law (r)
1
n21
sin 2

(d) Newton's equation (s) n = tan p

(A) a o p b o s c o r d o q
(B) a o q b o p c o r d o s
(C) a o s b o c c o p d o q
(D) a o s b o q c o r d o p
(105) Focal length of Convex lens is f. In column - I it's division and combination is given and in
column-2 it's focal length is given match properly.

Column-1 Column-2
(a) (p) 2f

(b) (q) f

(c) (r)
f
2

(d) (s) f

(A) a o s b o p c o q d o r
(B) a o p b o q c o r d o s
(C) a o q b o r c o s d o p
(D) a o r b o s c o q d o p

Ans. : 101 (D), 102 (B), 103 (A), 104 (C), 105 (A)

l
446
11 Dual Nature of Matter and Radiation
Name of scientist Discovery

Jean perin – Discovered an electron


Milikan – Charge of an electron

J.J.Thomson found the value of


e
–
m

Wilhem Rontgen - discovered X-rays


Henry Bacquerel and Madam curie - Radio activity
Hertz - photo electric effect

Methods of Emission of Electron Ñ


(1) Thermionic Emission : In this method, the current is passed through a filament of metal so
that it gets heated sufficiently and electrons get emitted from the metal.
(2) Field Emission Ñ when a metal is subjected to strong electic field of the order of
108 Vm –1 , electrons are pulled out of the metal surface.

(3) Photo electric emission Ñ when an electromagnetic radiation of enough high frequeney is
incident on a cleaned metallic surface, electrons can be liberated from the metal surface.
This phenomenon is known as the photoelectric effect and electrons so emitted are known
as photo electrons.

Work function (Threshold energy)


The minimum energy required to get emission of an electron ( to eject the free electrons
from metallic surface) is defined as work function of that surface of metal ( I0 ).

I0
hc
hf 0
O0 where f o threshold frequency, O0 threshold wavelength

I0 eV
eOo
hc 12375
Work function in electron volt, §D·
O0 ¨ A ¸
© ¹

(taking planck's constant, h 6.6 u 1034 Js )

Threshold Frequency f0 Ñ The minimum frequency of incindent light for the emission of

photo electrons from metallic surface is defined as threshold frequency f 0 .

447
for the emission of photo electrons, the frequency of the incident light f t f 0

Threshold wave length O0

For the emission of photo electrons from the given metallic surface, the wavelength of incident
light should be some maximum or less than that maximum wavelength. This maximum

wavelength is called the threshold wavelength O0

For emission of photo electrons, O d O0

(1) Effect of intensity of incident light Ñ

By Increasing the intensity of incident light (keeping frequency constant) the number of photo
electrons emitted and hence photoelectric current increases. but the maximum kinetic energy of
photo electrons do not change i.e. value of stopping potential remains unchanged.

? Intensity I v no. of incident photons


v no. of emitted photo electrons in 1 second

D photo electric current

(for a point like source)


1
D
distance 2

1
D distance (for a linear source)
ivI i
i

(current) 2I
I

(intensity) I V0 0 V

(2) Effect of potential Ñ


l when collector C is kept positive with respect to photo sensitive surface S, the emitted
photo electrons are attracted to collector C and amount of current passing through the
micro ammeter. At certain value of positive potential difference, when all the emitted
electrons are collected, increasing the potential difference further has no effect on the
current. This current is known as saturation current.
when the C is made negative with respect to S, on increasing this negative potential, the
number of photo electrons reaching the collector (value of photo electric current) gradu-
ally decreases.

448
l For some specific negative potential of the collector, even the most energetic electrons are
unable to reach collector, then photo electric current becomes zero. This minimum specific
negative potential of C with respect to S is known as stopping potential or cut off
potential V0

l If the value of stopping potantial is V0 , then the energy required for electron to cross this
potential barrier is, eV0

If maximum speed of photo electron is vmax , then eV0


1
mv 2max
2
(3) Effect of frequency Ñ
If frequency of incident light ! f 0 is increased, the maximum kinetic energy of emitted photo
electrons increased. i.e.the value of stopping potential is also increased but photo electric current
remains constant.
photo electric I 2 ! I1
current I2

f 3 ! f 2 ! f1 ! f 0
I1

saturation current (For


case-1 )
f3 f2 f1

V03 V02 V01 0 V


(4) Effect of photo sensitive surface Ñ
When the photo sensitive surface is changed by keeping frequency and intensity of incident light

constant, the graph of stopping, potential V0 o frequency f found to be straight line and
pasallel to each other. which intersects the X-axis (frequency axis) and the Y-axis at different
points. Which shows that the values of threshold frequencies are different for different metals
§h·
but slope of the graph ¨ ¸ is equal for all the metals.
©e¹

V0

metal-1
T T metal-2
( f 0 )1 ( f0 )2
§1 ·
¨ ¸1 § 1 ·
© O0 ¹ ¨ ¸
© O0 ¹2
f or
1
h( f0 )1 O
e
h( f0 )2
e
It is clear from the graphs, the value of threshold frequency and work function for metal–2 are
more than for metal–1
449
l Einstein's equation for photoelectric effect
The maximum kinetic energy of emitted photo electrons,

K max hf  I0

1 mv 2
2 max
hf  hf 0
h f  f0

1 mv 2 hc  hc § ·
hc ¨ 1  1 ¸ Ÿ vmax 2 hc (O0  O )
2 max
O O0 © O O 0 ¹ m OO0

This equation is called eEinstein's photo electric equation.

l Relation between stopping potential and frequency Ñ

According to definition of stopping potential,

1 2
mvmax eV0
2

§ ·
? eV0 h f  f0 hc ¨ 1  1 ¸
© O O0 ¹

? V0 hc § 1  1 · §
1237 5 ¨ 1  1 ¸
·
e ¨© O O0 ¸¹ © O O 0 ¹

In the graph of V0 o f

V0 h f  f hf
 0
hf
0
e e e

comparing above equation with equation of straight line

y mx  c , slope is found to be h and intercept on X-axis is f0 and intercept on


e

 hf 0
Y-axis is is obtained.
e

(1) The work functions for tungsten and sodium are 4.6 eV and 2.3 eV respectively. If threshold

wavelength for sodium is 5460 A , the value of threshold wavelength for tungsten is
D
.

(A) 10682 A (B) 2730 A (C) 526 A (D) 5892 A


D D D D

(2) The maximum velocity of photo electron emitted from surface of metal is 5 u 106 ms 1 . If specific

charge of an electron is 1.8 u 1011 Ckg –1 then the value of stopping potential is .
(approximately)
(A) 2 V (B) 3 V (C) 7 V (D) 4 V

450
(3) If the intensity of radiation incident on a photo cell be increased by four times, then the number
of photoelectrons and maximum kinetic energy of photoelectrons emitted become ...... .
(A) Four times, doubled (B) Four times, remains unchanged
(C) Doubled, remains unchanged (D) Remains unchanged, Doubled
(4) In a photo cell, with exciting wave length O the maximum speed of emitted photoelectron is v . If
3O
the exciting wave length is changed to the maximum speed of emitted photo electron will be ......
4

(A) greater than (B) (C) less than (D)


4 4 4 3
v v v v
3 3 3 2

(5) Light of wave length 0.6 Pm falls on a surface of metal causes the emission of photoelectrons.
for which the stopping potential is 0.5 V. With light of wavelength 0.4 Pm falls on the same
surface the stopping potential found to be 1.5 V. then the work function of surface of metal is
.
(A) 1.5 eV (B) 0.75 eV (C) 2.5 eV (D) 3 eV
(6) When a certain metallic surface is illuminated with light of wavelength O , the stopping potential
is 4 V0 . When the same surface is illuminated with light of wavelength 2O , the stopping

potential is V0 . The threshold wavelength for the surface is ...... .

O
(A) 6O (B) 8O (C) 3O (D)
4
(7) The frequency of incident light falling on a photosensitive surface is doubled, the value of stopping
potential will be ...... .
(A) Doubled (B) More than doubled
(C) halved (D) less than doubled.
(8) Sodium surface is illuminated by ultraviolet and visible radiation successively and stopping
potantial determined. This stopping potantial is ...... .
(A) Equal in both cases (B) More with visible light
(C) More with ultraviolet light (D) Varies randomly
(9) Light of two different frequencies whose photons have energies 1 eV and 5 eV respectively,
successively illuminates a metal of work function 0.5 eV . The ratio of maximum speed of the
emitted photo electron will be ...... .
(A) 1 : 4 (B) 1 : 1 (C) 1 : 3 (D) 4 : 1

(10) Light of wavelength O falls on a metal having work function O . Where O0 is threshold
hc
0

wavelength of surface of metal. Photo electric effect take place only if


(A) O t O0 (B) O d O0 (C) O 4O0 (D) O t 2O0

451
(11) When the point like sousce is kept 1m away from a photocell, photo electric current 16 mA is
obtained. When the same surface is kept 4m away, the photo electric current will be...... .
(A) 1 mA (B) 2 mA (C) 4 mA (D) 16 mA
(12) The threshold frequency for a metallic surface is f 0 . When radiation of frequency 2 f 0 is
incident on this surface,the maximum speed of emitted photoelectron is found 2 u 106 ms 1 .
When the radiation with frequency 5 f 0 is incident on that, the maximum speed of the emitted
photo electron will be ...... ms1 .

(A) 3 u 106 (B) 6 u 106 (C) 8 u 106 (D) 4 u 106

(13) The kinetic energies of photo electrons emitted from a metal are K1 and K2 , when it is irradiated
with lights of wavelength O1 and O2 respectively. The work function of the metal is

K 2 O2  K1O1 K1K 2
O1O2 K1  K2 O1O2 K2
(A) O1  O2 (B) O  O
1 2
(C)
O1  O2 1 2 1
(D) O  O K

(14) The work function for a metallic surface is I0 . Now this surface is successively illuminated with
the radiations of energy 5I0 and 10I0 respectively. The ratio of maximum speed of emitted
photo electrons will be ...... .
(A) 1 : 3 (B) 1 : 1 (C) 1 : 2 (D) 2 : 3
(15) The maximum kinetic energy of emitted photoelectron is 0.5 eV when the metal surface is
illuminated with the radiation of frequency 8 u 1014 H z . When the same surface is illuminated with

radiation of frequency 12 u 1014 H z , the maximum kinetic energy of emitted photoelectron is found
to be 2eV. Then the work function of the metallic surface is ...... .
(A) 3.5 eV (B) 0.5 eV (C) 2.5 eV (D) 3.85 eV
(16) The maximum speed of emitted photoelectrons, when light of wavelength O is incident on a
metallic surface having work function I 0 is ...... . where h planck's constant, c speed of
light in vaccum, m mass of electron.

§ 2 hc  OI0 · 2
1
2 hc  OI0
(A) ¨ ¸ (B)
© mO ¹ m


ª 2 hc  OI º2
ª 2 hc  I º2
1 1

(C) « » (D) « »
« » « »
0 0

¬ ¼ ¬ ¼
mO m

(17) When a metallic surface is illuminated with light of wavelenght O , the stopping potential is 3 V0
When the same surface is illuminated with light of wavelength 2O , the stopping potential is V0 . The
threhold wavelength for this surface is

(A) 4O (B) 3O (C) 6O (D) O


5
3
452
(18) The work function for aluminium surface is 4.2 eV . The wavelength of incident light for which
the value of stopping potential will be zero ......
h 6.6 u 10 34 Js, c 3 u 108 ms 1
D D D D
(A) 2694 A (B) 2946 A (C) 1854 A (D) 4268 A
(19) The value of threshold frequency for a certain metal is 3.3 u 1014 Hz . If light of frequency

8.2 u 10 Hz is incident on this surface, the value of stopping potential is .


14

h 6.63 u 10 34 Js, e 1.6 u 1019 C.


(A) 2.03 V (B) 3.68 V (C) 1.74 V (D) 4.06 V
(20) The frequency of incident light on a metallic surface is made three times, the maximum kinetic
energy of the emitted photo electrons will become .
(A) three times (B) less than three times
(C) 1/3 times the earlier value (D) more than three times
(21) When the light of frequency 5 u 1014 Hz is incident on a metallic surface having threshold

frequency 4 u 1014 Hz , the photoelectric current found to be 1.8 mA . When the frequency of
incident light is halved and the intensity is made three times, the value of photoelectrics current
will be ...... .
(A) 0.9 mA (B) 5.4 mA (C) 3.6 mA (D) zero
(22) The difference between the maximum kinetic energies of photoelectrons emitted from a metallic
D D
surface by light of wavelength 2000 A and 5000 A will be h 6.6 u 10 34 Js
(A) 3.71 eV (B) 5.94 eV (C) 7.42 eV (D) 2.97 eV
(23) A light with frequency 6 u 1014 Hz is incident on a metal surface whose work function is
1.59 eV .The maximum kinetic energy of photoelectrons emitted will be .....

h 6.63 u 10
34
Js
(A) 0.49 eV (B) 0.90 eV (C) 1.26 eV (D) 1.08 eV
(24) The maximum wavelength of radiation that can produce photoelectric effect in certain metal
having work function 3.2 eV will be ...... ( h 6.625 u 10 34 Js )
D D D D
(A) 1988 A (B) 2466 A (C) 2953 A (D) 3881 A
(25) The work function of a photo sensitive surface is 1.6 eV . In order to have the value of stopping
potential equal to 1 V for that surface, the wavelength of the incident light will be in the region of
...... . ( h 6.6 u 10
34
Js )
(A) X-ray region (B) infrared region (C) ultraviolet region (D) visible region

(26) How many photons of radiation of wavelength 5000 A have energy equals to the energy of a
D

photons of . radiations of wavelength 2.5u1013 m ?


(A) 2 u 106 (B) 4 u 106 (C) 8 u 106 (D) 0.5 u 106

453
Ans. Ñ 1 (B), 2 (C), 3 (B), 4 (A), 5 (A), 6 (C), 7 (B), 8 (C), 9 (C), 10 (B), 11 (A),
12 (D), 13 (A), 14 (D), 15 (C), 16 (C), 17 (A), 18 (B), 19 (A), 20 (D), 21 (D),
22 (A), 23 (B), 24 (D), 25 (D), 26 (A)

l Particle like nature of light (photon nature)


Photon is bundles or packet (quanta) of discrete energy. The energy of the smallest packet is
equal to hf .
Properties of photon Ñ

(1) The speed of photon in vaccum is same as speed of light 3 u 10 ms .


8 1

(2) Energy of a photon of frequency f is, E
hc
hf
O

where c velocity of light in vaccum, plank's constant, h 6.6 u 10 34 Js ,


l = wavelength of light.

E in eV |
taking h = 6.625 u 10
hc 12375 12400
eO §D· §D·
O ¨A¸ O ¨A¸
-34
Js
© ¹ © ¹

The energy of photon is not continuous but discrete like hf 2hf ,... which shows the
quantization of energy.
(3) Linear momentum of photon of frequency f is

muc uc
E E hf h
P
c 2 c c O

(4) Rest mass ( m 0 ) of photon is zero, but effective mass,

E hf h
m
c 2
c 2 cO

This mass is also known as kinetic mass of photon.

mass of particle moving with velocity v is, m , where m0 rest mass


m0

1  v2
2

but speed of photon in vaccum v = speed of light c

§ 2 ·
? m0 m ¨ 1 v ¸
¨ c 2 ¸¹
0
©

(5) Photon is electrically neutral.


(6) Photons are not effected by electric and magnetic field.
(7) Like a real particle, photon interacts with other particles obeying the law of conservation
of energy and momentum..
454
Number of emitted photons :
The number of photons emitted per second from a source of monochromatic radiation of
wavelength O and power P is given as,

P P PO
hc where E = energy of each photon.
n ;
E hf

Intensity of lght (I) Ñ


Energy crossing per unit area normally per second is called intensity (I) of incident light

power)
E P E
?I (' P
At A t
At a distance ‘r’ from a point source of power P, intensity is given by

P P 1
I 2
ŸIv
A 4 U r2

For a linear source, ÞIµ


P 1
I
2S rl r

where l length of a cylinder at a distance r from the source.

(27) If the efficiency of an electric bulb of 2 W is 20 %, what is number of photons emitted by it in


one second ? the wavelength of light emitted by it is 400 nm. ( h 6.6 u 10 34 Js )

(A) 3.46 u 1016 (B) 4.67 u 1017 (C) 2.52 u 1017 (D) 8.08 u 1017

(28) The monochromatic light of wavelength of 660 nm is produced from He–Ne LASER.Hence,
output power of 6 mW is obtained. If this light is incident on the target, what will be the number

of photons incident per second h 6.6 u 10


34
Js
(A) 4 u 1016 (B) 2 u 1016 (C) 3 u 1016 (D) 5.5 u 1016

D
(29) A source S1 is produsing 1014 photons per second of wavelength 3000 A . Another source S 2 is

producing 1.04 u1014 photons per second of wavelength 3120 A . Then the ratio of powers of
D

sources S1 and S 2 respectively is ...... .

(A) 1 : 1 (B) 1 : 1.02 (C) 1.04 : 1 (D) 1 : 2

(30) 12 u 1012 photons are incident on a surface in 10 s. This photons correspond to a wavelength

12 A . If the surface area of the given surface is 0.02 m 2 . Find the intensity of incident
D

radiations. Velocity of light, c 8 u 108 ms 1 , h 6.6 u 1034 Js

(A) 2.19 u 103 Wm2 (B) 3.48 u 103 Wm2 (C) 9.9 u 103 Wm2 (D) 6.62 u 102 Wm2

455
(31) Monochromatic light of wavelength 6000 A is incident normally on a surface of area 2 cm 2 . If the
D

intensity of light is 200 mWm 2 , find the number of photons being incident on this surface in one
second.
34 8 1
h 6.6 u 10 Js, c 3 u 10 ms
(A) 1.21 u 1014 (B) 3.88 u 1013 (C) 6.16 u 1014 (D) 4.54 u 1013

Ans. Ñ 27 (D), 28 (B), 29 (A), 30 (C), 31 (A)

l Matter Wave (Wave like nature of particle)


According to de-Broglie a moving material particle some times acts as a wave and some times
as a particle.
The wave associated with moving particle is called matter wave or de-Broglie wave and it
propagates in the form of wave packets with group velocity.
(1) de-Broglie wave length :
According to de Broglie theory, the wavelength of de-Broglie wave is given by,

O ŸOv v v
h h h 1 1 1
p mv 2mE P v E
where h planck's constant,
m mass of the particle, v speed of the particle, E = kinetic energy of particle.
(2) de-Broglie wavelength associated with the charged particle :
The kinetic energy of a charged particle accelerated through a potential difference of V

volt, E
1
mv 2 qV .
2

? Hence de-Broglie wavelength, O


h h h
p 2mE 2mqV

12.27 D 0.286 D
O = A , Oproton = A
electron V V
0.202 D 0.101 D
ODeutron = A, O = A
V v  Particle V
(3) de-Broglie wavelength associated with uncharged (netural) particle :
0.286 × 10-10 0.286 D
Oneutron =
E in eV E in eV
m A

Energy of thermal neutron at ordinary temperature

kT Ÿ O
3 h h h
2m kT
E
2 2mE 3 3mkT
2

where_ T = Absolute temperature, k Boltzmann's constant 1.38 u 1023 JK 1

6.62 × 10-34 25.17 D


?O A
u 1.38 × 10
thermal neutron -27 23 T
3 × 1.67 × 10 T

456
(4) Ratio of wavelength of photon and electron :

hc § hc ·
The wavelength of a photon of energy E is given by,, OP ¨' E ¸
E © O ¹

While the wavelength of an electron of kinetic energy K is given by,

h
Oe
2mK

Op 2
?
c 2mc K
2mK
Oe E E
2

(32) The de-Broglie wavelength of a neutron at 627° C is O . What will be its wavelength at 127° C

O
(A) O (B) 2O (C) O (D)
2 3
3 2 2
(33) If the kinetic energy of a free electron is made thrice, its de-Broglie wavelength will become.......

1
(A) times (B) 3 times (C) 3 times (D) times
1
3 3

(34) The de-Broglie wavelength of a neutron having energy 8 eV is ...... .( h 6.6 u 10


34
Js , mass

of neutron = 1.7 u 1027 kg )

(A) 1 u 1011 m (B) 1.8 u 1011 m (C) 2.2 u 1011 m (D) 0.6 u 1011 m

(35) A proton and a deutron have equal energies. The ratio of their de-Broglie wavelengths is ......

(A) 2 : 1 (B) 1 : 2 (C) 2 :1 (D) 1: 2

(36) A proton and an D  particle are accelerated through same potential difference of 200 V. If de-
Broglie wavelength associated with proton is 5200 A°, then the de-Broglie wavelength associated
with D  particle is ...... .

D
1300
(A) (B) 1300 2 A (C) 2600 A (D) 2600 2 A
D D D
A
2

(37) The linear momentum of an electron intially at rest, accelerated through a potential difference of
25 V is

(A) 5.4 u 1024 kgms 1 (B) 2.7 u 1024 kgms 1

(C) 1.2 u 1024 kgms 1 (D) 3.2 u 1024 kgms1

(38) If the kinetic energy of the particle is increased by 16 times, then the value of de Broglie
wavelength of particle is ...... .
(A) decreased by 75 % (B) increased by 75 %
(C) decreased by 67 % (D) increased by 67 %

457
(39) The de-Broglie wavelength of a proton accelerated through a potential difference of 450 V is
( h 6.6 u 1034 Js, charge of proton 1.6 u 1019 C , mass of proton 1.6 u 1027 kg )

(A) 0.14 u 1011 m (B) 0.2 u 1011 m (C) 0.26 u 1011 m (D) 0.09 u 1011 m

(40) The de Broglie wavelength of a particle moving with a velocity 2.25 u 108 ms1 is equal to the
wavelength of photon. The ratio of kinetic energy of the particle to the energy of the photon is
...... . (velocity of light is c 3 u 108 ms 1 )

(A) 8 (B) 8 (C) 8 (D) 18


7 5 3

(41) The kinetic energy of electron and proton is equal. Then the relation between their de-Broglie
wavelength is

(A) O p ! Oe (B) O p  Oe (C) O p 2Oe (D) O p Oe

(42) An electron and a proton have the same de-Broglie wavelength. Then the kinetic energy of the
electron is ...... .
(A) greater than the kinetic energy of proton (B) less than the kinetic energy of proton.
(C) equal to kinetic energy of proton (D) zero

(43) A body of mass 0.5 kg is moving with a velocity of 1000 ms 1 . The de-Broglie wavelength of
the body is ...... .

(A) 3.32 u 1027 A (B) 1.32 u 1026 A (C) 1.6 u 1027 A (D) 0.132 u 1026 A
D D D D

(44) A particle of mass 1 J has the same de-Broglie wavelength as an electron moving with a

velocity of 2 u 106 ms 1 . The velocity of the particle is .

(A) 1.82 u 1015 ms 1 (B) 3.6 u 1016 ms 1 (C) 3.6 u 1021 ms 1 (D) 9 u 102 ms 1

(45) The velocity of an electron having a wavelength of 10 A is ...... .


D

(A) 7.25 u 106 ms 1 (B) 7.25 u 105 ms 1 (C) 5.25 u 106 ms 1 (D) 4.25 u 105 ms1

(46) Two charged particle of mass 2m andõ 3m have charged 3q andõ 2q respectively. Now both
particles are accelerated through a same potential difference.Then the ratio of their de-Broglie
wavelength is ...... .

(A) 2 : 3 (B) 3 : 2 (C) 1 : 6 (D) 1 : 1

(47) De Broglie wavelength of a proton and D  particle is same. If proton is accelerated through a
potential difference of V volt, then D  particle should be accelerated through a potential
difference of ...... volt.

(A) 1 (B) 8 (C) 2 (D)


1
8

458
(48) The frequency of a photon is 1.5 u 1014 Hz. Its momentum will be ...... . kgms1
Plank's constant h 6.6 10 34 Js, velocity of light c 3 u 108 ms 1 .

(A) 3.3 u 10 28 kgms1 (B) 3.3 u 10 34 kgms1 (C) 3.3 u 10 30 kgms1 (D) 6.6 u 1028 kgms1
(49) An electron of mass m when accelerated through a potential diference of V volt has de-Broglie
wavelength O . The de-Broglie wavelength associated with a proton of mass M accelerated
through the potential difference of 4 V will be ....... .

(A) O M (B) O m (C) O m


(D) O m
2 m 2 M 2M 4 M

D
(50) A photon of wavelength 1.4 A collides with an electron. After the collision the wavelength of
D
proton becomes 2.0 A . Then the energy of scattered electron will be ...... .
(take h 6.63 u 1034 Js )
(A) 4.6 u 10 15 J (B) 4.6 u 10 16 J (C) 3.2 u 10 16 J (D) 2.3 u 10 16 J

(51) To reduce de-Broglie wavelength of an electron from 3 u 1010 m to 1 u 1010 m , its energy
should be ...... .
(A) increased to 9 times (B) increased to 3 times
(C) decreased to third part (D) decreased to nineth part
(52) The rest mass of an electron is m0 . It is moving with the velocity of 0.6 c, its mass m will be ...... .
where c = velocity of light in vaccum.
5m0 4m0 m0
(A) m0 (B) (C) (D)
4 5 6
(53) The potential diference through which an electron should be accelerated so its wavelength will
D
become 0.5 A ...... .
(A) 466 V (B) 747.0 V (C) 941.0 V (D) 602.0 V
(54) The chargless particle neutron has mass of 1.67 u 1027 kg and its kinetic energy is 0.04 eV ,
then calculate de-Broglie wavelength of neutron. h 6.62 u 10
34
Js
D D D D
(A) 1.80 A (B) 1.43 A (C) 2.86 A (D) 3.2 A
(55) De Broglie wavelength associated with an electron moving with the velocity of 105 ms 1 is ...... .

6.6 u 10 34 Js , mass of electron m


31
h 9 u 10 kg
D D D D
(A) 73.33 A (B) 7.33 A (C) 46.2 A (D) 146.66 A

Ans. Ñ 32 (C), 33 (A), 34 (A), 35 (C), 36 (B), 37 (B), 38 (C), 39 (A), 40 (C), 41 (B),
42 (A), 43 (B), 44 (A), 45 (B), 46 (D), 47 (D), 48 (A), 49 (B), 50 (B), 51 (A),
52 (B), 53 (D), 54 (B), 55 (A)

459
Davission and Germer Experiment Ñ
This experiment proves the wave like nature of an electron.
In this experiment using Bragg's law, from the formula 2d sin T nO , the wavelength

found to be 1.67 A . which is near to the de Broglie wavelength of electron ( O 1.65 A ).


D D

Which shows wave like nature of electron.


Heisenberg's Uncertainty principle Ñ
According to Heisenberg's uncertainty principle, If the uncertainty in the x - cordinate of the
position is 'x and uncertainty in the x- cordinate of its momentum is 'p , then

(in one dimension)


h
'x 'p t
2S

? ' x ' p t = where


h
=
2S
Now, If 'x o 0 , then Dp ® f
and 'p o 0 then, Dx ® f
Similarly, the uncertainty in the measurements of Energy and time for a particle using above
principle,
'E . 't t =

l If the radius of the nucleus is r then uncertainty in the position of proton inside the nucleus is
'x 2r d

Hence the uncertainty in momentum of proton is.

= = = h
'p
'x d 2r 4S r

For a particle if the uncertainties in the measurement of angular momentum and angular
displacement are 'L and 'T respectively.then from Heisenberg's uncertainty principle,
'L . 'T t =

(56) The correctness of velocity of an electron moving with velocity 50 ms 1 is 0.005 % The
accuracy with which its position can be measured will be ...... .

(A) 46 u 103 m (B) 46 u 104 m (C) 46 u 105 m (D) 46 u 106 m

(57) A proton and electron are lying in a box having unpenitrable walls, the uncertainty in their
momenta will be ...... .

(A) For proton is more, as compared to electron (B) For electron is more, as compared to the proton

(C) same for both the particles (D) directly proportional to their masses

460
(58) The maximum uncertainty in the position of proton is 6 u 108 m , then the minimum uncertainty

in its velocity will be ...... .( h 6.625 u 10


34
Js , mass of proton = 1.67 u 1027 kg )

(A) 1 mms 1 (B) 1 ms 1 (C) 1 cms 1 (D) 100 ms 1

(59) If the uncertainty in the position of an electron is 1010 m , then the value of uncertainly in its

momentum (in kg ms1 ) will be

(A) 1.054 u 1024 (B) 1.112 u 1024 (C) 1.054 u 1022 (D) 1.112 u 1022

Ans. Ñ 56 (A), 57 (C), 58 (B), 59 (A)


Assertion - Reason type Question :

Instruction : Read assertion and reason carefully, select proper option from given below.

(a) Both assertion and reason are true and reason explains the assertion.

(b) Both assertion and reason are true but reason does not explain the assertion.

(c) Assertion is true but reason is false.

(d) Assertion is false and reason is true.


(60) Assertion Ñ The work function of a metal is 2 eV . To have photo emission from the surface of
the metal, the maximum wavelength of incident photon is 6200 A°.

Reason Ñ Work function, )


hc
Omax

(A) a (B) b (C) c (D) d


(61) Assertion Ñ Light with frequency which is1.3 times the threshold frequency is incident on a photo
sensitive surface. Now, the frequency of incident light is halved and the intensity is
doubled, the photo electric current remains unchanged.
Reason Ñ Photo electric current is directly proportional to the intensity of incident light.
(A) a (B) b (C) c (D) d
(62) Assertion Ñ Proton is nearly heavier by 1840 times than an electron. A proton is accelerated
through a potential difference of 1 kV, Its kinetic energy becomes 1 keV
Reason Ñ Kinetic energy gained = (charge) × (potential difference)
(A) a (B) b (C) c (D) d
(63) AssertionÑ The kinetic energy of photoelectrons emitted from the photo sensitive surface
depends on the frequency of the incident light.
Reason Ñ Kinetic energy of emitted photoelectrons changes with the change in the frequency of
incident light.
(A) a (B) b (C) c (D) d

461
(64) Assertion Ñ On increasing the frequency of incident light, the number of emitted photons remains
constant.
Reason Ñ the number of emitted photo electrons does not depend on the frequency of incident light,
but depends on the intensity.
(A) a (B) b (C) c (D) d
(65) Assertion Ñ An electron and a proton are accelerated through same potential difference. The de
Broglie wavelength associated with an electron is more than the de- Broglie
wavelength associated with proton.
Reason Ñ The de-Broglie wavelength associated with the charged particle accelerated through a

potential difference of V volt is given by, O v 1


h
2mqV m (for equal value of qV ).
(A) a (B) b (C) c (D) d
(66) Assertion Ñ A radiation of monochromatic light (with enough high frequency) is incident on a
metallic surface. The kinetic energy of emitted photoelectrons is lying between
0 to K max .
Reason Ñ The value of work function changed with the depth from the surface of metal.
(A) a (B) b (C) c (D) d
(67) Assertion Ñ The de-Broglie wavelength associated with molecules is inversely proportional to the
square root of the absolute temperature.
Reason Ñ The value of vrms for moleaules depends on the absolute temperature.
(A) a (B) b (C) c (D) d
Ans Ñ 60 (A), 61 (D), 62 (A), 63 (A), 64 (A), 65 (A), 66 (C) 67 (A)
Comprehension Type Questions :
paragraph Ñ

The work function of ceisium metal is 2.14 eV . When radiation of frequency 6 u 1014 H z is
made incident on it, then photoelectrons are emitted. Answer the following questions Ñ
(68) Maximum kinetic energy of photoelectron ...... .
(A) 5.58 u 10 20 J (B) 3.34 u 1019 (C) 5.58 u 10 18 J (D) 3.34 u 10 20 J
(69) The value of stopping potential ...... .
(A) 0.236 V (B) 0.349 V (C) 1.03 V (D) 0.87 V
(70) maximum speed of photo electrons ...... .
(A) 155 u 103 ms 1 (B) 224 u 103 ms1 (C) 3.50 u 105 ms1 (D) 276 u 103 ms 1
(71) The value of threshold wavelength ...... .
D D D D
(A) 4647 A (B) 3288 A (C) 5789 A (D) 6134 A
(72) The value of threshold frequency ...... .

(A) 5.18 u 1014 Hz (B) 4.44 u 1014 Hz (C) 5.56 u 1018 Hz (D) 4.89 u 1014 Hz

462
(73) The variation of intensity (I)of incident radiation with photo electric current (i) can be shown by
...... . i i

(A) (B)

I I

i i

(C) (D)

I I

Ans. : 68 (C), 69 (B), 70 (C), 71 (C), 72 (A), 73 (A)

Graphical questions Ñ
(74) Which of the following graph represents the variation of particle momentum and the associated
de-Broglie wavelength
(A) p (B) p

O O
(C) p (D) p

O O

(75) According to Einstein's photoelectric equation, the graph between the kinetic energy of photoelec-
trons ejected and the frequency light is
K K
(A) (B)

f f

463
(C) K (D) K

f f

(76) In the graph shown below, V2 ! V1 then ...... .

where V potential difference, i o photoelectric current


i

(A) O1 O2 (B) O1  O2

(C) O1 O2 (D) O1 ! O2

O2

O1
V2 V1 V

(77) The anode voltage of a photocell is kept fixed. The wavelength of the light falling on the
cathode is gradually changed.The plate current (I) of the photocell varies as follows.

(A) I (B) I

0 O 0 O

(C) I (D) I

0 O O
0

Ans. : 74 (D), 75 (D), 76 (D) 77 (A)

464
Match the columns :
(78) Methods for eomission of electron are shown in column-1 and in column-2 methods to obtain it
are shown. Match the columns.

column-1 column-2

(a) Thermionic emission (p) By incidenting suitable light

(b) photo electric emission (q) By heating (by passing current through

filament)

(c) Field emission (r) By colliding accelerated electron beam on


the surface of the metal.

(d) Secondary emission (s) By appying strong electric field.

(A) a o q b o p c o s d o r
(B) a o p b o q c o r d o s
(C) a o r b o s c o p d o q
(D) a o s b o r c o q d o p
(79) In column-1 physical quantities related to photoelectric effect are shown. Join them with
appropriate physical quantitces given in column-2.

column-1 column-2

(a) saturation current (p) Frequency of incident light

(b) stopping potential (q) work function

(c) de Broglie wavelength associated (r) Area of photo sensitive surface

with photo electron

(d) Force exerted on photo sensitive (s) Intensity of incident light

surface due to incident radiation. (For constant frequency)õ

(A) a o s b o p, q c o p, q d o p, r, s
(B) a o r, p b o s, r c o r d o q
(C) a o p b o r c o r, s d o s
(D) a o s b o r c o q d o p
Ans. Ñ 78 (A), 79 (A)

465
12 & 13 Atom and Nucleus

D – Scattering experiment and Rutherford's atomic model :


– Radioactive source 83 Bi
214 emitts, D  particles of energy 5.5 MeV, incident on a thin gold foil of

thickness 2.1 u 107 m . Scattered D  particles observed on circular scintillation screen of


ZnS. [zinc sulphide]

– 105 D  particles scattered at  15q , 0.1% Scattered at ~150q , about only 1 out of 10
4

D  particle was scattered at 180q .


– Path of scattered D  particle can be determined with the help of Coloumb’s law and Newton’s

second law. Repulsive force acting between D  particle and gold nucleus is F
1 (2 e) (79 e)
4S 0
.
r2
The magnitude and direction of the force on D  particle continuously changes as it approaches the
nucleus.
– The perpendicular distance of the initial velocity vector of the D  particle from the centre of the
nucleus is known as the impact parameter (b).
– For head on collision b 0.
– The minimum distance of the D  particle from the centre of the nucleus (for b 0 ) is known as
the distance of closest approach.
– Radius of nucleus is about 10–15 m.

If 't' is thickness of foil and 'N' are No. of scattered D  particles then, = constant,
N
l
t

.
N1 N2
t1 t2

According to Rutherford, orbit of an electron is not circular but spiral and motion of an electron

ends inside the nucleus. In this case the atom cannot remain stable. Thus Rutherford’s atomic

model failed to explain the stability of atom.

(1) An D  Particle of 10 MeV is moving for a head on collision. What will be the distance of

closest approach from the nucleus of atomic number Z = 60 ?

(A) 1.44 u 1014 m (B) 2.88 u 1014 m (C) 0.53 u 1014 m (D) 1.728 u 1014 m

(2) An D  particle with some energy is moving for head on collision with nucleus of Z = 85, if the

distance of closest approach is 1.85 u 1014 m , find the energy of D  particle.

(A) 23.13 MeV (B) 13.2 MeV (C) 10 MeV (D) 20 MeV

466
(3) Distance of closest approach of an D  particle with energy 27 MeV is 1.10 × 10–14 m. Find
atomic no of an atom ?

(A) 100 (B) 103 (C) 105 (D) 90


(4) If thickness of foil in D  scattering experiment increases from 2 × 10–7 m to 2.5 × 10–6 m,
Find the increased number of scattered D  particles ?

(A) about 12 times (B) 100 times (C) remains constant (D) 10 times
(5) If number of scattered D  particles increased by 40%, what is the percentage change in
thickness of foil ?

(A) 40 % (B) 80 % (C) 10 % (D) 20 %

(6) If thickness of foil is t1 , number of scattered D  particles are 8500 and thickness of foil is t2 ,
No. of scattered D  particles are 27,500 then,

(A) t1 3.2 t2 (B) t2 3.2 t1 (C) t2 1.6 t1 (D) t2 t1

Ans. : 1 (D), 2 (B), 3 (B), 4 (A), 5 (A), 6 (B)

Bohr’s atomic model and energy levels of H-atom


Hypothesis-1 : Electron can revolve only in those orbits in which its orbital angular momentum is

an integral multiple of 2S . These orbits are known as stationary or stable orbits. In such orbit
h

electron does not radiate energy. 2S = and h Plank’s constant = 6.625 u 10 34 Js .
h

Hypothesis-2 : When electron transit from higher energy (Ei ) orbit to lower energy (E k ) orbit. It
radiates photon of frequency f. Similarly when electron absorbs a photon of frequency f, it makes
transition from lower energy state (E k ) to higher energy state (Ei ) . Ei  Ek hf .

nh
l mvr n=
2S

Ei  E k
hc
O
hf

l Radius of an orbit is,

n 2 h2 0
r where, n principle quantum no.
S m Z e2

r v
n2
0 permittivity of vaccum
Z
m = mass of an electron, Z = atomic number

467
For H – atom r v n2 .

l Kinetic energy for electron,

1 1 Ze2
K mv 2
2 8S 0 r

1 (Ze) (e)
Potential energy, 4S 0
U
r

1 Ze2
4S 0 r

Total energy (E) = K  U

1 Ze 2
8S 0 r
E

K
U
E
2

me 4 Z2
l by substituting value of r, E En , by substituting value of m, e, 0 , h
8 02 h2 n2

13.6 Z2  Z2
En eV , E n D
n2 n2

For H  atom Z 1 Ÿ E n
13.6 1
2
eV En v  .
n n2

Energy levels of H – atom :

E(eV)
0 n f
–1.51
n=3 n 3 (Second excited state)
–3.4
n=2 n 2 (First excited state)

–13.6 n 1 (Ground State)

468
l Transition of an electron from orbit with quantum number ni and energy Ei to orbit with
quantum number nk and energy E k , E i  E k hf

me4 § 1 1 ·
? f ¨  ¸
8 02 h3 ¨ n2 n i2 ¸¹
© k

§ 1 1 ·
? ¨  ¸
1 me4
O 8 02 C h3 ¨ n2 n i2 ¸
© k ¹

§ 1 1 ·
? R ¨  ¸
1
O ¨ n n i2 ¸
© k ¹
2

FÝë_ me4
= Rydberg constant = 10973700 m 1.
8 02 c h3
R

l Rydberg constant for an atom,

Rf
R atom
1
m
M atom

Rf 10973700 m1 , m mass of an electron, M mass of nucleus of atom.

l Frequency of electron in a orbit with principle quantum number ‘n’ for hydrogen atom,

Sme4
Z
me4
, angular frequency
4 02 h3 n3 2 02 h3 n3
f

Zv
2 Rc 1
f
3
n n3

(Periodic time) T v n3
1 2R
O n3

f D n 3 , O v n3

l If electron transit form n = 3 to n = 2 and n = 2 to n = 1

n=3
hf31 hf32  hf 21
f31 f32
? f31 f32  f 21
n=2
? 
1 1 1
f 21 O31 O32 O 21
n=1

469
Success of the Bohr model :

l Stability and energy of Hydrogen-like atoms can be calculated.

l  2
Atomic spectra of Hydrogenic atoms e.g. He , Li , Be
3

can be explain.

l It is useful for the confirmation of some principles, for the invention of “heavy hydrogen” or
“deuterium”.

Limitations :

l The orbits of an electron need not to be circular.

l There is an odd combination of classical and quantum mechanics.

l Unable to explain the relative intensities of the spectral lines.

l Fine structure of spectral lines can not be explained.

l Unable to explain the arrangement of electrons in atoms.

(7) The ratio of energies in Fourth and third excited state for Hydrogen atom is .

(A) 4 : 5 (B) 16 : 25 (C) 25 : 16 (D) 1 : 1


(8) Momentum of photon of red light with frequency 400 × 1012 Hz is .

(c = 3 × 108 ms–1)

(A) Zero (B) 8.8 × 10–28 kgms–1


(C) 11.65 × 10–6 MeV C–1 (D) insufficient information
(9) Calculate the radius of an electron in most external orbit, when Phosphorus atom is added in
Silicon (whose Di-electric constant = 12).

(A) 380.9 pm (B) 390.8 pm (C) 930.8 pm (D) 830.9 pm


(10) Linear speed of an electron in Hydrogen atom for ground state (first orbit) is .

(A) c
2
(B) c
11
(C) c
137
(D) c
274

(11) When electron transit from n = 5 to n = 1 in Hydrogen atom. Find the speed of emitted photon.

(A) 104 ms1 (B) 2 u 102 ms 1 (C) 4 ms1 (D) 8 u 102 ms 1

(12) If R, v, T and E are radius of orbit, speed of electron, periodic time of revolution and total energy of
an electron respectively. Which option is not directly proportional to quantum number ‘n’.

(A) vR (B) RE (C) v


E
(D) T
R

470
(13) Total energy of an electron in first excited state for Hydrogen atom is –3.4 eV. What is kinetic
energy of this electron ?
(A) 0 (B) 3.4 eV (C) –3.4 eV (D) 6.8 eV
(14) Angular momentum of an electron in Hydrogen atom for ground state is L1 and for fourth
excited state is L4 then L4  L1 = .

(A) 5L1 (B) 3L1 (C) 2L1 (D) 4L1


(15) Energy of photon in Hydrogen atom is 12.1 eV, its angular momentum is .

(A) 1.05 u 10 34 Js (B) 2.11 u 10 34 Js (C) 3.16 u 10 34 Js (D) 4.22 u 10 34 Js
(16) Kinetic energy of an electron in n = 1 for H-atom is 13.6 eV, Total energy of an electron for
n = 2 for He 2 is .

(A) 13.6 eV (B) 3.4 eV (C) 13.6 eV (D) 3.4 eV


(17) Find the ratio of orbital periodic time for an electron in n = 1 and n = 2.
(A) 1 : 2 (B) 2 : 1 (C) 1 : 4 (D) 1 : 8
(18) Ratio of orbital area for an electron in first excited state and ground state in Hydrogen atom
is .
(A) 2 : 1 (B) 4 : 1 (C) 8 : 1 (D) 16 : 1
(19) Energy of an electron in ground state for H – atom is (R = Rydberg’s constant) .

(A)  Rh
c
(B) 1
Rhc
(C) Rhc (D) hc
R

(20) Radius of first orbit in Hydrogen atom is 0.528 A , radius of second orbit is .
D

(A) 4.752 A (B) 2.112 A (C) 0.071 A (D) 0.142 A


D D D D

(21) Find the speed of photon during transition n = 5 to n = 1 in Hydrogen atom.

(A) 4.718 ms 1 (B) 7.418 ms 1 (C) 4.178 ms1 (D) 7.148 ms 1


(22) If principle quantum number n > 4 is not possible, then number of possible elements are .
(A) 4 (B) 32 (C) 60 (D) 64
(23) For energy levels A, B and C, E A  EB  EC . O1 , O 2 and O3 are wave lengths for A, B, and
C. Which option is true for a transistion in Figure.
C
O1 O 2
O1 O3 (A) O3 O1  O 2 (B) O 3 O1  O 2
B

O2 (C) O32 O12  O 22 (D) O1  O 2  O3 0


A
471
(24) Electron transit from fourth orbit to first excited state in Hydrogen atom, Find the frequency of

radiation. R 107 m 1
(A) (B) (C) (D) u 1015 Hz
3 3 9 3
u 105 Hz u 1015 Hz u 1015 Hz
16 16 16 4

(25) Total energy of an electron in excited state of H – atom is –3.4 eV. calculate the De-broglie
wavelength.

(A) 6.6 u 1010 m (B) 6.6 u 1011 m (C) 6.6 u 109 m (D) 6.6 u 1012 m

(26) Orbital angular momentum quantum number l = 7, What is orbital angular momentum ?

(A) 7h
2S
(B) 42h
2S
(C) 7 h
2S
(D) 56 h
2S

Ans. : 7 (C), 8 (B), 9 (A), 10 (C), 11 (C), 12 (B), 13 (B), 14 (D), 15 (B), 16 (C),
17 (D), 18 (D), 19 (C), 20 (B), 21 (C), 22 (C), 23 (B), 24 (C), 25 (A), 26 (D)

Hydrogen spectrum :
When Hydrogen electrically discharged at low pressure, atom excited and emits radiation of certain
wavelengths. The group of these radiations is called Hydrogen spectrum.
l Different series

§ ·
Lyman series : O
1 R¨1  1 ¸
©12
n ¹
2

where n 2, 3, 4 ...

n 2 o HD or LD line

n 3 o HE or LE line

n 4 o HJ or LJ line

l In Lyman series for maximum wavelength n = 2 and minimum wavelength n f.


l It is seen in ultraviolet region.

§ ·
l Balmar series : O
1 R¨ 1  1 ¸
©2 2
n ¹
2

where n 3, 4, 5, 6 ...

n 3 o HD for maximum wavelength o n 3

n 4 o HE for minimum wavelength o n f

l It is seen in visible region.

472
§ ·
l Paschen series O
1 R¨ 1  1 ¸
©32
n ¹
2

Where n 4, 5, 6, 7 ...

l n 4 o HD , n 5 o H E , ....

l for maximum O o n 4, for minimum O o n f


l In near infrared region.

§ ·
Brackett series O
1 R¨ 1  1 ¸
©4 n ¹
l 2 2

where n 5, 6, 7, 8 ...

l n 5 o HD , n 6 o H E , ....

l for O max o n 5, for O min o n f


l In infrared region.

§ ·
Pfund series : O
1 R¨ 1  1 ¸
©5 n ¹
l 2 2

where n 6, 7, 8 ...

l n 6 o HD , n 7 o HE , ....

l for O max o n 6, for O min o n f In far infrared region

n n  1
l When electron transits in lower orbit from nth orbit no. of emitted spectrat lines = .
2

§ 1 1 ·
R Z2 ¨ n 2  n 2 ¸ .
Emitted wavelength for any atonic spectra, O ¨ k i ¸
1
© ¹
l

1 H1 , deuterium 1 H 2 , ionized helium 2 He4 3 Li6


 
(27) Hydrogen and ionized lithium are

given. Their wavelengths are O1, O 2 , O 3 , O 4 respectively compare their wavelengths for a
transistion of an electron from n 2 to n 1.

(A) O1 2O 2 3O3 4O 4 (B) 4O1 2O 2 2O3 O 4

(C) O1 2O 2 2O3 O 4 (D) O1 O 2 4O3 9O 4

(28) Calculate the number of emitted spectral lines, when hydrogen atom having principle quantum
number = 4 moves from ground state to specific excited state.

(A) 3 (B) 5 (C) 6 (D) 2

473
(29) Find the ratio of minimum and maximum wavelength for H-spectra.
(A) 81.86 (B) 86.81 (C) 0.012 (D) 0.12
(30) Find the ratio of maximum and minimum wavelength for Brackett series.
(A) 3.6 (B) 0.36 (C) 78.2 (D) 2.78
(31) Maximum wave number in infrared series is m–1.
(A) 12.18 × 105 (B) 12.18 × 1010 (C) 18.12 × 105 (D) 8204
(32) Find the ratio of O of D  line, for Balmer and Lyman series.
(A) 27 : 5 (B) 5 : 27 (C) 1 : 4 (D) 20 : 27
(33) Wavelength of first line for Balmar series .

(A) 6563 A (B) 6365 A (C) 6563 m (D) 6563 cm


D D

(34) f1 and f 2 are frequency of last and first line of Lyman series. f3 is frequency of last line of
Balmar series then,

(A) f1  f2 (B) f 2  f1 (C) f3 f1  f 2 (D) f1  f 2


1
f3 f3 f3
2

(35) Minimum wavelength of Lyman series is 912 A , then maximum wavelength is A.


D D

(A) 1216 A (B) 1824 A (C) 2434 A (D) 3648 A


D D D D

Ans. : 27 (D), 28 (C), 29 (C), 30 (D), 31 (A), 32 (A), 33 (A), 34 (A), 35 (A)

Excitation and Ionization energy and potential


l Electron revolving in a stationary orbit absorbs specific energy and jumps to an orbit of higher
energy, absorbed energy is called excitation energy and corresponding potential is called
excitatoion potential.
0 n= f

–1.51
n=3
eV
12.09 eV
–3.39 n=2
eV
10.21 eV
–13.6
n=1
eV
energy (eV) = potential (V)
n 1o n 2 required energy = 3.39  ( 13.6) 10.21 eV
n 1o n 3 required energy = 1.51  (13.6) 12.09 eV

474
l The minimum energy required to remove an electron from an atom (to send is it n f ) is called
ionization energy and corresponding potantial is called ionization potential. Ionization energy
(eV) = Ionization patential (V) .
O n= f

1.51eV
–1.51 eV n=3

3.39eV
–3.39 eV n=2

13.6 eV
–13.6 eV n=1
l n 1o n f required energy = 0  ( 13.6) 13.6 eV

l n 2o n f 0  (3.39) 3.39 eV
Emission and absorption spectra
l During the transition of an electron from higher energy (Ei ) orbit to lower energy (Ek ) orbit,
the emitted radiation is called emission spectrum Ei  Ek hf .
l Intensity of such a spectral lines increases as atomic density increases and it decreases as
temperature increases.
l Wavelength of spectraline depends on atomic number.
l Radiation of continuous wavelength is incident on atomic gas to send electron from lower
energy orbit to higher energy orbit. In incident radiation certain wavelength absorbs, these
appear as dark lines in the spectrum, such a spectrum is known as “absorption spectrum”.
l For example, radiation emitted by the lower layer of photosphere in the Sun, certain
wavelength are absorbed hence dark lines are observed. These lines are called fraun
hoffer lines.
l X-ray : Discovered by Rontgen. wavelength between 0.001 to 1 nm.
l X-ray emitted when there is a collision between electron and anode of Cu, Tungston and M o .
Electrons accelerated with 20  40 kV .
l Relative intensity

KD

X-ray
continuous spectrum
KE

Omin

O (1012 m)
30 40 50 60 70 80 90

475
l All the wavelengths (frequencies) emitted in X – ray radiation causes continuous spectrum.
l During the head on collision of electron with anode, radiation of minimum wavelength and
maximum frequency emitted.

O min where V = potential required to accelerate the electron.


hc
eV

l Characteristic X – ray spectrum :


KE

KD

n=1(K)

n=2(L)
n=3(M)

l Incident electrons penetrate deep into the atoms of the anode and knock out the electron
from the atom, from the inner shells, which creates vacancies. The electron from outer shells
experience transition to these vacancies and fill them. The radiation of definite frequencies are
emitted during such transition.

l The radiation is called KD , if it is emitted when electron of K – shell ( n 1 ) is thrown out

and the vacancy is filled by the electron from L – shell ( n 2 ). Similarly KE during the

transition from n 3 to n 1 , LD during the transition from n 3 to n 2 . The X – ray

spectrum formed by such lines is called the chracteristic spectrum.

l Due to screening of the charge of the nucleus Z o Z  1 .

m e4 (Z  1) 2
Energy 
802 n2 h2
En

13.6 (Z  1) 2
 eV
n2

l For KD radiation : E L  E K hf K D

§1 1 ·
? 13.6 (Z  1) 2 ¨  u 1.6 u 1019 J

©1 2 ¹
hf kD
2

13.6 u 1.6 u 1019 § 3 ·


? fk ¨ ¸ (Z  1)
2
6.62 u 10 34 © 4 ¹

476
13.6 u 1.6 u 10 19 u 3
? f (Z  1)
6.62 u 1034 u 4

? f C (Z  1)

? f CZ  C equation of straigth line.

where C
1
4.965 u 107 Hz 2

f o Z graph is straight lines and its slope is C.

Moseley’s expt. work :


l Moseley suggested that the elements should be arranged with respect to their atomic numbers,
which establish the relation between chemical properties and position of elements in the
periodic table.
l Missing positions in periodic table filled up with appropriate elements.
l Rare earth (Lanthanide) elements, elements coming after Uranium, are arranged properly.

l Information regarding to charge of nucleus can be obtained with the help of K radiation.

Other useful information :

O d 4A radiation, whose penetration is high, called hard X – ray.


D
l

O ! 4A radiction, whose penetration is low, called soft X – ray.


D
l

l Only 1 % energy of incident electron converts in energy of X – ray 99 % energy, waste in


form of Heat. So, arrangement of cooling is required in coolidge tube.
l If f is a fraction of kinetic energy of an electron, converts in X – ray then wavelength of

hc
emitted X – ray is, O f eV0 .

K
radiation : EL  EK
(L o K)
l hfK

radiation : EM  EK
K
(M o K)
l hf K

L
l radiation : E M  E L hf L
(M o L)

(36) Excitation energy in third orbit for Hydrogen atom is eV.


(A) 1.51 eV (B) 3.4 eV (C) 0.66 eV (D) 0.85 eV

477
(37) Energy required to send an electron from second orbit to third orbit is 47.22 eV for a given atom,
then its atomic number is .

(A) 1 (B) 3 (C) 4 (D) 5


(38) Excitation potential of Helium atom in second orbit is .

(A) 7.55 V (B) 21.7 V (C) 13.2 V (D) 10.21 V


(39) For an atom excitation potential in first is ‘V’ volt, its ionization potential is volt.

(A) 1
4
(B) 3
4
(C) 4
3
(D) 5
4

(40) Energy required to remove an electron from first excited state in Li  is eV.

(A) 122.4 (B) 30.6 (C) 13.6 (D) 3.4


(41) If accelerating potential in X – ray tube increases, .

(A) Intensity of X – ray increases. (B) minimum wavelength of X – ray increases.


(C) minimum wavelength of X – ray decreases. (D) Intensity of X – ray decreases.

(42) Wavelength of X – ray photon is 3.3 A , its corresponding energy is .


D

(A) 7.5 keV (B) 3.8 keV (C) 5.5 MeV (D) 3.7 MeV

(43) Minimum wavelength in X – ray tube of potential 4 MV is A.


D

(A) 1 (B) 0.0062 (C) 0.0031 (D) 10–5


(44) Changing of position of molecules and proper place of molecules can be explained with the help
of .

(A) Moseley’s law (B) Mendeleev’s law (C) Crompton effect (D) Hund’s law

(45) Wavelength K X-Ray is 0.76 A , then find the atomic number of element of anode.
D

(A) 20 (B) 60 (C) 41 (D) 80

(46) In two different emission of K radiation atomic number of target nucleus are 65 and 81. Find
the ratio of their wavelengths.

(A) (B) (C) (D)


1 1 2 25
4 16 5 16

(47) If accelerating potential in X – ray tube increases, speed of X – ray .

(A) Increases (B) Decreases (C) Does not change (D) nothing can be said

478
(48) Operating voltage in X – ray tube is 66 kV, in continuous spectrum of X – ray .
(A) 0.01 nm and 0.02 nm wavelengths remains present.
(B) Both the above wavelengths are absent.
(C) 0.01 nm wavelength would be present and 0.02 nm wavelength would be absent.
(D) 0.01 nm wavelength would be absent and 0.02 nm wavelength would be present.
(49) Maximum radiated frequency in X-ray tube is ‘f’ and operating voltage is ‘V’ volt. If operating

voltage becomes 2 , find maximum radiated frequency.


V

(A) (B) f (C) 2f (D) 4f


f
2
(50) Wavelength of K radiation from Z = 41 is O , wavelength of K radiation from Z = 21
is .
O
(A) 4 O (B) (C) 3.08 O (D) 0.26 O
4
(51) Electron beam of 80 keV energy is incident on Tungston in X – ray tube. If energy of an electron
in K – shell of Tungston is –72.5 keV then .

(A) Presence of continuous spectrum with minimum wavelength 0.155 A is observed.


D

(B) Continuous spectrum of all the wavelengths is observed.


(C) Characteristic spectrum of X – ray of tungston is observed.

(D) Continuous spectrum of minimum wavelength 0.155 A and characteristic spectrum of


D

X – ray is observed.
(52) Gragh of Intensity of X – ray o wavelength in Coolidge tube is as shown in Figure. O c = minimum
wavelength and O K = wavelength of, K . If Accelerating potential increases then .

Oc Ok O

(A) Oc Increases (B) O K decreases


(C) OK – Oc Increases (D) Oc , O K decreases but O K  Oc remain constant.
(53) Ionization energy of H–atom is 13.6 eV. H–atom in the ground state is excited with the help of
radiation of 12.1 eV energy. Number of radiated spectral lines are .
(A) One (B) Two (C) Three (D) Four

479
(54) Wavelength of K radiation in H-atom during the emission of X-ray is 0.32 A , then wavelength
D

of K radiation is .

(A) 0.21 A (B) 0.27 A (C) 0.34 A (D) 0.40 A


D D D D

(55) If Z is atomic number of element, Frequency of characteristic spectrum of X – ray is directly


proportional to .

(A) Z 2 (B) (Z  1) 2 (C) Z (D) 1


Z

Ans. : 36 (C), 37 (D), 38 (A), 39 (C), 40 (B), 41 (C), 42 (B), 43 (C), 44 (A), 45 (C),
46 (D), 47 (C), 48 (D), 49 (A), 50 (A), 51 (D), 52 (C), 53 (C), 54 (B), 55 (B)

Atomic mass and the constitution of Nucleus :


l The entire mass and entire positive charge are concentrated at the central region, known as
nucleus, includes protons and neutrons.
l Proton and neutron are is also called nucleon.
l Atomic number (Z) = number of protons (P)
l Atomic mass number (A) = [P + n]
l neutron number (N) = A – Z

Symbol for element Z X A or ZX


A
l

l The 12th (twelfth) part of the mass of unexcited 6 C12 atom is called 1 amu.

1u 1 amu 1.66 u 1027 kg

l mass spectrometer is used to measured mass of an atoms accurately.


l Isotops having same chemical propertics but different mass. same number of protons but
different number of neutrons.
l For example in case of Cl, the proportion of 34.98 u is 75.4 % and 36.98 u is 24.6 %. Hence
the mass of Cl atom is obtained from its weighted average.

75.4 u 34.98  24.6 u 36.98


mass of Cl Atom =
100

= 35.47 u

l The nuclei for which the neutron number (N = A – Z) is same are called isotones.
l For nuclei Z, A, N are same but radio active properties are different called isomers.
l For nuclei atomic mass number (A) are same, called isobars.

480
l e.g. of isotops : 92 U
235
, 92 U
238

l e.g. of isotops : 36 Kr
86
, 37 Rb
87

l e.g. of isobars : 82 Pb
214
, 83Bi
214

l
35 Br
80 having a pair of isomers.

Naclear forces, nuclear radius and stability of nucleus

l P–P, P–n, n–n strong nuclear force.

l It is also called quark-quark force because P and n are made up of quarks.

l Types of quarks : up, down, charm, strange, top, bottom.

l Nuclear forces depends on orientation of spin.

Average radius of nucleus is given by R R0 A 3 where R0


1
l 1.1 fm

l Density of nucleus is about 2.3 × 1017 kgm–3, 1014 times than density of water.
l As atomic number increases, coloumbian force increases multiplyingly, to balance it number of
neutron increases so that strong nuclear force icreased.

For e.g. 6C
12 P 6, n 6 but in 92 U
235
P 92 , n 143

It is essential condition for the stability of nucleus.

l Mass - energy : 1 u 1.66 u 1027 kg ; 1 eV 1.6 u 10 19 J

1u 931.48 MeV o mass-energy equivalency

E m c2

(1 u ) (3 u 108 ms 1 )2

(1.66 u 1027 ) (9 u 1016 ) 931.48 u 106


1.6 u 1019 931.48 MeV

Binding energy of nucleus

Mass of the nucleus is always less than the total mass of its constituents in the free state. This
decrease in the mass is called mass defect,

M  Zmp  Nmn

'm Zmp  Nmn  M

481
l The energy equivalent to mass defect is called binding energy of nucleus. Eb = (' m) c2 .

l When it is divided with number of nucleons, we get average binding energy per nucleon-

§ Eb ·
¨ A ¸ , this energy is actually measurement of stability of the nucleus.
© ¹
E bn

l Ebn is an average energy per nucleon to release all the constituent particles from the nucleus.

l Average binding energy per nucleon (Ebn ) is measurement of stability of the nucleus.

l In case of deutron, 1 H 2 its mass is 2.0141 u and sum of the mass of proton and neutron in
free state is 2.0165 u. ' m 2.0165  2.0141 0.0024 u , the energy equivalent to this mass
defect is 0.0024 u 931.48 2.24 MeV. Which is called binding energy.

l Hence, binding energy of 1 H 2 is 2.24 MeV.

l Thus to liberate proton and neutron from 1 H 2 , 2.24 MeV energy has to be supplied to it from
outside. Conversely if one proton and one neutron coalease 2.24 MeV energy is emitted out.
MeV
For 1 H 2 E bn
2.24
l 2
1.12 .
necleon
Ebn

10 Fe

A
0 50 100 150 200 250

§ ·
Ebn is maximum ¨ | 8.8 ¸ for the nucleus of (Fe) > A 56@ .
MeV
© ¹
l
nucleon

l Ebn is small for A < 30 and A > 170.

l For 30  A  170 Ebn is almost constant. These nuclei are the most stable. It is due to the
saturation property of the nuclear forces.

l When heavy nucleus ( A ! 170 ) gets divided in two lighter nuclei, energy released, this
process is called nuclear fission.

l Two lighter nuclei (A  10) are fused to form a heavier nucleus and energy is produced, this
process is called nuclear fusion.

482
l The energy to be supplied to separate a nucleon from a nucleus is called separation energy.

l Mass defect per nucleon is called packing fraction (f).

'm
Ÿ E bn f c 2 or f u 931.48
MeV
f .
A necleon

(56) For the same separation of 1 fm in the nucleus, the force acting between two proton is F1,
between two neutron is F2 and between proton and neutron is F3 then,

(A) F1 < F2 < F3 (B) F2 < F1 < F3 (C) F1 < F2 = F3 (D) F1 = F2 < F3

(57) 1040 deuterons are present in a star initially. Energy released according to reaction given below :

1H
2
 1H 2 o 1H 3  p and 1 H 2  1H 3 o 2 He 4  n

If emitted power is 1016 W. The time to destroy quantity of deuteron is .

(A) 106 s (B) 108 s (C) 1012 s (D) 1016 s

m 1 H2 2.014 u, m( P) 1.007 u , m (n) 1.008 u, m (2 He4 ) 4.001 u .

(58) Reaction of nuclear fission, 92 U


236
o X117  Y117  n  n . For X and Y E bn 8.5 MeV

and E bn 7.6 MeV for U 236 then produced energy MeV.

(A) 2000 MeV (B) 200 MeV (C) 20 MeV (D) 2 MeV
(59) Packing fraction (f) = .

(A) A
MA
(B) AM
A
(C) M
MA
(D) MA
A

(60) If u = 1 amu and A = atomic mass number If mass of an atom is Au then A = .

(A) 1 (B) 12 (C) 16 (D) between 1 to 110


(61) Mass in the proton is completely converted in energy, then energy is MeV.

(A) 9310 (B) 931 (C) 10078 (D) 100


(62) A and B are Isotops, B and C are Isobars. dA , dB and dC are the densities of their nuclei
then .

(A) d A ! d B ! dC (B) d A  d B  dC (C) d A dB dC (D) d A d B  dC

(63) The energy required to separate a nucleon from a nucleus is called .

(A) Binding energy per nucleon (B) Binding energy


(C) Reaction energy (D) Separation energy

483
(64) Calculate the released energy Q in nuclear fusion. 1 H2  1H 2 o 2 He4  Q

m 1 H2 2.0141 u , m 2 He4 4.0024 u .

(A) 12 MeV (B) 6 MeV (C) 24 MeV (D) 48 MeV

(65) If velocity of light becomes 23 , the energy releases in nuclear fission decreases in multiple

of .

(A) 2
3
(B) 4
9
(C) 5
9
(D) 5
9

(66) Mass of two isotops of Boron 5B


10 and 5 B11 are 10.01294 u and 11.00931 u respectively.
Mass of an atom of Boron is 10.811 u. Find the proportion of these two isotops.
(A) 30 %, 70 % (B) 40.12 %, 59.88 % (C) 72.05 %, 27.95 % (D) 19.90 %, 80.10 %
(67) Mass defect in nuclear fission is 0.03 %. Energy released in fission of 1 kg mass is .

(A) 2.7 u 1013 J (B) 27 u 1014 J (C) 0.27 u 10 13 J (D) none of these

(68) In a nuclear reaction given below, Z X A .

7N
14
 2 He4 o ZX
A
 1H1

(A) 7N
16 (B) 7N
17 (C) 8 O16 (D) 8 O17

(69) For 8O ,
16 Ebn = MeV.u , m p 1.007825 amu , mN 1.008665 amu . M 8 O16
= 15.9949 amu .,
(A) 7.973 (B) 79.73 (C) 0.79 (D) none of these

(70) Find the number of electron, proton and neutron in 12 g, 6 C12 .

(A) 6 u 1023 (each) (B) 12 u 1023 (each)

(C) 18 u 1023 (each) (D) 36 u 1023 (each)

(71) Mass-defect for the nucleus with Z = 2 and A = 4 is 0.04 u. Calculate the binding per nucleon.
(A) 931 MeV (B) 93.1 MeV (C) 9.31 MeV (D) 0.04 MeV
(72) Energy produced due to destruction of proton is 3724 MeV in a nuclear reaction, then these number
of protons = .
(A) One (B) Two (C) Three (D) Four

484
(73) Atomic mass number of Helium and sulpher are 4 and 32 respectively. The radius of Sulphur
nucleus is times the radius of Helium nucleus.
(A) 2 (B) 4 (C) 8 (D) 12

(74) 1H
2
 1H 3 o 2 He
4
 1
0n . Binding energies of 1 H 2 , 1H 3 and 2 He 4 are a, b and
c (MeV) respectively. Energy releases in the reaction = MeV.

(A) a  b  c (B) c  a  b (C) c  a  b (D) a  b  c


(75) E bn for the nucleus of 3 Li 7 and 2 He
4 is 5.60 MeV and 7.06 MeV respectively. Energy
released = MeV.

3 Li
7
 p o 2 2 He4

(A) 19.6 (B) 17.3 (C) 8.6 (D) 2.4


(76) Energy releases in a given fusion = .

1H
2
 1H 2 o 1H 3  1H1

(A) 1 erg (B) 1 eV (C) 4 MeV (D) 4 keV

(77) Energy released in each fission of U 235 is 200 MeV. Output power in nuclear reactor is
1.6 MW. Find the rate of fission. (number of fission per second).
(A) 5 × 1016 (B) 1017 (C) 1.6 × 1013 (D) 1019

Ans. : 56 (C), 57 (C), 58 (B), 59 (D), 60 (B), 61 (B), 62 (C), 63 (D), 64 (C), 65 (C), 66 (D),
67 (A), 68 (D), 69 (A), 70 (D), 71 (C), 72 (D), 73 (A), 74 (C), 75 (B), 76 (C), 77 (A)

Natural Radio activity

l Becquerel found that radiations of certain specific properties are emitted naturally from
Uranium, this phenomenon is called narural radio activity. Those radiations were initially
known as Becquerel rays.

l Madam curie separated two new elements from the ore of Uranium called pitch blende. They
were named Polonium and Radium, their activities are several times that of Uranium.

l The emission of radioactive radiations is spontaneous, instantaneous and continuous. It is not


affected by temperature, pressure, electric field and magnetic field.

l One can not stop the emission of radioactive radiations or can not change the rate of
emission.

l Heavy element emits radioactive radiation to become stable from unstability.

l D  particles : D  particle is a nucleus of - 2 He 4 with 2-proton and 2 neutron. Charge is +2 e.

485
l Almost the nucleus for which Z > 83 emits D  particles.

ZX
A
o Z 2Y
A 4
 2 He4 ( D )

e.g., 92 U
238
o 90Th
234
 2 He4 ( D )

l The disintegrating nucleus is called the parent nucleus and newly formed nucleus is called
the daughter nucleus.
l In the emission of D  particle, atomic number decreases by 2 and atomic mass number
decreases by 4.

l E  particle : E  particles are electrons emitted from nucleus. Their velocity depends on the
nuclide emitting them. In other reaction of E  decay positrons are also emitted.

l e = E and e = E .

l When a neutron converts into proton and electron is produced in the nucleus which can not

live in a nucleus so emits as E

n o p  e  X (anti neutrino)

ZX
A
o Z 1Y
A
 e E  X

Atomic number increases by one and Atomic mass number does not change.

l When a proton converts in to a neutron positron, is emitted as E . Atomic number decreases by

one and atomic mass number does not change.

p o n  e   X (neutrino)

ZX
A
o Z 1Y
A

 e  E  X

l J  rays : They are electromagnetic waves.


l Photon of J  rays emitted during the transition of nucleus from higher energy state to lower

energy state, wavelength O .


hc
E

l When nucleus emits D and E particles it is in a excited state, according to need, by emitting
photon of J  ray comes to a stable state.

l e.g., Due to emission of E 27 Co


60
converts into 28 Ni
60 , during this emits J  rays of
1.17 MeV and 1.33 MeV step by step.

486
l All these raido active radiations affect the photographic plate, produce fluorescence.

D E J

Relative ionizing power : 10000 100 1


Relative penetration power : 1 100 10000
l Energy value in D  emission

§ A ·
Q KD ¨ ¸ Where, kD = kinetic energy of D , A = atomic mass number
© A4¹

Nuclear Reactions
l By bombarding suitable particles of suitable energy on a stable element, transformed into
another element is called artificial nuclear reaction. Energy gain or loss is denoted as
Q – value.
A  a o B  b  Q

7N
14
 D 2 He 4 o 8O17  1 H1  Q

A o Target nucleus o 7N
14

a o projectile particle o D 2 He4


B o product nucleus o 8O17

b o product (emitted) particle o 1H


1

Q > m A  ma  m B  m b @ c2

Q ! 0 exoergic reaction, Q  0 endoergic reaction


l In nuclear reaction momentum, electric charge and energy each one is conserved.
l Q  value of reaction = energy equivalent to decrease in mass in the reaction = increase in
the kinetic energy.
Nuclear Fission :
Neutron is good projectile because it is charge less and does not have to face the coloumb repulsive forces.
l Disintegration of nucleus in which enormous energy is produced. This process was named
nuclear fission.

e.g., 92 U
235
 0 n1 o 92 U
236
o 51Sb
133
 41Nb
99
 4 0 n1  Q

92 U
235
 0 n1 o 92 U
236
o 56 Ba
144
 36 Kr
89
 3 0 n1  Q

92 U
235
 0 n1 o 92 U
236
o 54 Xe
140
 38Sr
94
 2 0 n1  Q

487
l Fission fragments having Z values between 36 to 56 and A values between 90 to 95. They

converts into stable nuclei with emission of E and J .

l Energy of incident neutron is about 2 MeV.

l Energy produced is about 200 MeV.

Nuclear chain reaction and Nuclear reactor

l With the help of neutrons produced nuclear fission, more nuclei is accomplished. So, we get
more energy and more neutrons. A series of such processes is called nuclear chain reaction.
If such a process is properly controlled, then energy can be obtained continuously at steady
rate. Nuclear reactor is the illustration of this.

Difficulties encountered in nuclear reactor and their removal :

l To stop neutrons from escaping reflecting surfaces, moderators like water, heavy water,
Graphite and Beryllium are used.

l In a chain reaction enormous heat energy is produced and the temperature is likely to become
106 K. Hence coolants like water, liquid sodium are used.

l The ratio of number of neutrons produced at any stage to the number of neutrons incident is called
multiplication factor (K). It is a measure of the growth of number of neutrons.
If K = 1 the reactor is said to be critical. If K > 1 is said to be super critical in such a condition
explosion can take place, if K < 1 said to be subcritical, the process slows down and eventually stops.

l In order to control the value of K, rods of Boron and Cadmium are kept, called control rods.

Reactor : As a fuel 3 % 92 U
235 and remaining 92 U
238 .

l
92 U
235
 0 n1 o 93 Np
239
 1e
0
 X

93 Np
239
o 94 Pu
239
 1e
0
 X

Plutonium is intense radio active and fissionable by slow neutron.

l In Pressurised Water Reactor normal water is used as moderator and also as coolent.

l Water is pushed into the core of the reactor at temperacure 600 K and 150 atm pressure,
generated steam operates the turbine, which produces electric power.

Thermonuclear Fusion in Sun and other Stars :

l The sun emitting energy at the rate of 3.8 u 1026 Js 1 .

l When two proper light nuclei are fused at a very high temperature to form a heavy nucleus,
enormous energy is produced, such process is called thermonuclear fusion.
488
l Proton - Proton cycle :

1
1H  1H1 o 1H 2  1e
0
 X  0.42 MeV

1 e
0
 1e
0
o 2 J  1.02 MeV

1H
2
 1H1 o 2 He3  J  5.49 MeV

2 He
3
 2 He
3
o 2 He 4  1H1  1H1  12.86 MeV

l First three reaction ocur twice.


l Total produced energy = 2 u 0.42  2 u 1.02  2 u 5.49  12.86 26.7 MeV.

(78) What is produced in a consecutive step in a given reaction ?

A4 A4
ZX
A
o Z 1Y
A
o Z 1T o Z 1T

(A) D , E, J (B) E, D , J (C) J , D , E (D) D , J , E

(79) Find the number of D and E  particles in the conversion 92 X


235
o 88Y
219 .

(A) 4, 4 (B) 5, 5 (C) 6, 6 (D) 4, 8


(80) If PD , PE , PJ are penetrating power of D , E, J then .

(A) PD PE PJ (B) PD ! PE ! PJ (C) PD  PE  PJ (D) PD PE  PJ

(81) D and E particles are emitted from the ends A and B respectively in a wire,
then .
(A) Electric current flows from A to B
(B) Electric current flows from B to A
(C) Electric current is not produced.
(D) Electric current flows toward the mid-pt. from every side.
(82) During the radio active emission of element having atomic number 90 and atomic mass number

232, final product is 82 Pb


208 . Find the emitted number of D and E – particles.

(A) D 3, E 3 (B) D 6, E 4 (C) D 6, E 0 (D) D 1, E 6

(83) Nuclear reaction : 5B


10
 2 He 4 o 7 N13  _________

(A) Proton (B) neutron (C) electron (D) D


(84) 6C
12 absorps neutron and emits E  particle. Find the final product.

(A) 7N
14 (B) 5B
13 (C) 7N
13 (D) 6 C13

489
(85) Arrange the ionizing power of D , E and J in descending order.

(A) J , D , E (B) J, E, D (C) D, E, J (D) E, J , D


(86) Energy produced in nuclear fission is 200 MeV. If output power of reactor is 5W then find the
rate of fission.

(A) 1.56 u 1010 s 1 (B) 1.56 u 1011 s 1

(C) 1.56 u 10 16 s 1 (D) 1.56 u 1017 s 1

(87) D  particle is emitted by nucleus ZX


A. Final nucleus emits E . Then find the atomic number
and atomic mass number of final nucleus.
(A) Z – 3, A – 4 (B) Z – 1, A – 4 (C) Z – 2, A – 4 (D) Z, A – 2

(88) Three D  particles and one E  particle are emitted by 86 R n


236 , then for product nucleus X
.
(A) Z = 83, A = 224 (B) Z = 84, A = 218
(C) Z = 84, A = 220 (D) Z = 82, A = 223

(89) Calculate the release energy in reaction, 88 Ra


226
o 86 Ra
222
 2He 4 . Kinetic energy of
D  particle is 4.78 MeV . Parent element is stable.
(A) 8 MeV (B) 4.78 MeV (C) 4.87 MeV (D) none of these

(90) Energy emitted from a star is 2.7 u 1036 Js 1 . Find the decrement in its mass.

(A) 3 u 1018 kg s 1 (B) 3 u 1019 kg s 1 (C) 3 u 10 20 kg s 1 (D) 3 u 10 21 kg s 1

(91) Series of emitted particles from the nucleus (Z 92) is D, D, E , E , D, D, D, D,

E , E , D, E , E , D . Then Z of final nucleus = .

(A) 78 (B) 82 (C) 74 (D) 76


(92) 1H
2
 1H3 o 2 He4  n , potential energy due to repulsive force between two nucleus in

nuclear fusion is 7.7 u 10  14 J . How much temperature should be given to the gas so this
reaction be possible ?
(A) 105 K (B) 103 K (C) 109 K (D) 107 K
(93) What this reaction suggest ?

4 1H1 o 2 He4  2 1e


0
 26 MeV.

(A) E  decay (B) J  decay (C) Fusion (D) Fission

490
(94) D  particles are emitted from stable radio active element having atomic mass mumber 208.
Energy of emitted D  particles is E. Find the energy of disintegration.

(A) (B) (C) 52 E (D) E


52 51
E E
51 52

Ans. : 78 (B), 79 (A), 80 (C), 81 (A), 82 (B), 83 (B), 84 (C), 85 (C), 86 (B), 87 (A),
88 (A), 89 (C), 90 (B), 91 (A), 92 (C), 93 (C), 94 (A)

Radio active constant and Activity


l In a specimen of radio active material, if the number of undisintegrated nuclei of an element
at time t is N and there after if 'N nuclei disintegrate in time interval 't , then

lim 'N dN
is called the rate of disintegration or the decay rate or activity (I) of
' t oo ' t dt
that element at time t. Activity means the number of nuclei decaying per unit time.

vN
dN
l The decay rate is proportional to the number of undisintegrated nuclei at that time.
dt

?  ON (negative sign indicates that as time passes N decreases.)


dN
I
dt

O Radioactive constant = decay constant, unit = s–1) its value depends on the type of
disintegrating element. For different unstable isotops of the same element, the values of O are
different.
l O o large o I o more o shortlived elements.

O o small o I o less o longlived elements.

l O is independent of temperature pressure, electric field and magnetic field.

§ dN ·
For the nucleus of a given element O ¨ O ¸ shows the probability of disintegration per
© Ndt ¹
l

unit time.
Units of Activity
l 1 disintegration oceur in one second, then activity of body is called 1 Becquerel.
1 Bq = 1 disintegration / sec.
l If 3.7 × 1010 disintegration per second take place, the activity of a substance called 1 curie (Ci).
1 Ci = 3.7 × 1010 disintegration / sec.
1 Ci = 3.7 × 1010 Bq.
Exponential law of Radio active Disintegration

 ON Ÿ  O dt
dN dN
dt N

491
ª t 0 o N N0 º
³ O ³
N t
« t t o N N »¼
dN
¬
dt
N
N0 0

? > ln N @ NN0  O > t @0


t

? ln N  ln N0  Ot

§ N·
? ln ¨  Ot
© N 0 ¸¹

? eO t Ÿ N N 0 e O t
N
N0

Similarly, I I 0 e O t and M M 0 e  O t M 0 = Initial mass, M = mass after disintegration.

l As time passes, the number of nuclei and activity decreases exponentially. This curve is called
decay curve. For I o t and M o t similar graph is obtained.

N N0

t
Half life :
l The time interval in which the number of nuclei of radioactive element becomes half of its
§W ·
value at the beginning, is called half life of that element ¨ 1 ¸ .
© 2¹

W1
N0 e O t , t and N
N0
N 2
2

 O W1 O W 1 OW 1
1
Ÿ 2 Ÿ 2
N0 2 2 2
N0 e e e
2

? O W1 Ÿ
ln2
O O
ln 2 W
0.693
1
2 2

l Half life of different radio active elements are from 10–7 s to 1010 Yr.

492
0
§1·
O t 0 time N N0 Ÿ
N
¨ ¸
N0 ©2¹
1
N0 N §1·
t W1 time N Ÿ ¨ ¸
2 2 N0 ©2¹
2
N0 N §1·
t 2 W 1 time N Ÿ ¨ ¸
2 4 N0 ©2¹
3
N0 N §1·
t 3 W 1 time N Ÿ ¨ ¸
2 8 N0 ©2¹

§1·
n W1 
o
n
¨ ¸ .
N
©2¹
t time
2 N0

Where n
t given time
W1 Half life
2

Time undecayed part Decayed part


t=0 100 % 0%

t = W 12 50 % 50 %

t = 2W 12 25 % 75 %

t = 3W 12 12.5 % 87.5 %

t = 4W 12 6.25 % 93.75 %

Mean life W
The time interval during which the number of nuclei of a radioactive element becomes eth part of its
original, is called Mean or average life W of that element. (e = 2.718)

N0 e O t , t W and N , N 0 e O W Ÿ e1 e O W
N0 N0
l N
e e

Ÿ OW 1 Ÿ W
1
O

other informations : – W1 0.693 W


2

– W W 1 u 1.44
2

493
l For the emission of D and E , total O t O D  OE


1 1 1
?  W 1 (t ) W 1 (D ) W1
1 1 1
W( t ) WD WE Similarly, (E)
2 2 2

Half life of At 215 is 100 Ps , The time in which activity of its specimen becomes
1 th
(95) 16
of its

original is .

(A) 400 Ps (B) 6.3 P s (C) 40 P s (D) 300 P s

(96) Activity of Radio active element at time t1 is R1 and at time t2 is R2 . t2 ! t1 . If the mean

life W then .

R1  R2
(A) R1 t1 (B) constant
t2  t1
R2 t2

§t t · § t ·
(C) R2 (D) R2 R1 exp ¨ 1 ¸
© W t2 ¹
R1 exp ¨ 1 2 ¸
© W ¹

(97) Quantity of Radon is 16 g and its half life is 3.8 days. Find the disintegration of Radon in 19 days ?

(A) 5 g (B) 0.5 g (C) 15.5 g (D) none of these

3 th
(98) Half-life of radio active element is 30 days. Find the time to disintetrate its mass.
4

(A) 15 days (B) 45 days (C) 30 days (D) 60 days

1 th
(99) Find the half life of radio active element whose activity becomes
16
of original in 30 years.

(A) 90 years (B) 120 years (C) 15 years (D) 7.5 years
(100) Find the decay constant of an element in Que : 99.

(A) 0.0924 Yr 1 (B) 9.24 Yr 1 (C) 924 Yr 1 (D) 0.0688 Yr 1

(101) Initially number of atoms in radio active element A and B are equal. Their half lives are 1 hr and
2 hr respectively. Find the ratio of rate of disintegration after 2 hr.

(A) 4 : 2 (B) 1 : 1 (C) 1 : 2 (D) 8 : 1

(102) Find the number of disintegration per second in 2.3 g, 90 Th


230 whose half life = 2.4 u 1011 s.

(A) 6 u 1021 (B) 0.73 u 1010 (C) 1.73 u 1010 (D) 109
494
(103) Half life of radio active isotop is 10 min. At a given moment number of radio active nucleus are
108 . Number of nucleus after 5 min = .
8 108
(A) (B) 104 (C) 2 u 107 (D)
10
2 2
(104) Half life of Pa218 is 3 min and mass of specimen is 16 g. How much mass is remaining after 15 min.

(A) 3.2 g (B) 2.0 g (C) 1.6 g (D) 0.5 g


(105) Half life of radio active element is 5 min. % of element remains undecayed after
25 min.

(A) 25 % (B) 75 % (C) 6.25 % (D) 3.125 %


(106) Activity of recenty bought bottle of radio active Tritium is 3 %, “seven years old” labelled on the
bottle, then is was made before how many years ? (Half life = 12.5 Yr)

(A) before 220 years (B) before 420 years (C) before 63 years (D) before 70 years
(107) Half life of radio active element is 10 years, then its average life is .

(A) 14.4 yr (B) 20 yr (C) 15 yr (D) 28.8 yr


(108) Activity of radio active sample at time t1 is I1 and at time t2 is I2. If the half life of sample is
W1
, the number of undecayed nucleus in t2  t1 time is directly proportional to .
2 I1  I 2
(A) I1 t2  I 2 t1 (B) I1  I 2 (C) W1 (D) ( I 2  I1 ) W 1
2 2
(109) Half life of radio active element is 20 min. Time to decay from 20% to 80 % is .

(A) 20 min (B) 40 min (C) 25 min (D) 30 min


(110) Half lives of radio active element for D and E decay are 8 year and 24 year respectively. After
12 year, its activity is how much percentage of its original activity ?

(A) 50 (B) 12.5 (C) 25 (D) 6.25


(111) There are 4 × 1016 nucleus in a radio active sample. Half life of element is 10 days. Find the
number of decayed nucleus in 30 days.

(A) 0.5 × 1016 (B) 2 × 1016 (C) 3.5 × 1016 (D) 1 × 1016
(112) Mass of radio active sample at t = 0 is 10 g. After two mean life mass of sample is
(A) 1.36 g (B) 2.50 g (C) 3.70 g (D) 6.30 g
(113) Half life of Radium is 1600 year. Mass of sample is 100 g. The time for which its mass becomes
25 g is years.

(A) 6400 (B) 2400 (C) 3200 (D) 4800


495
(114) After decayed two radio active nucleus P and Q becomes stable element R. At t = 0, the number
of nucleus in P and Q are 4 N0 and N0 respectively. Half lives of P and Q are 1 min and 2 min
respectively. When stable element R formed, number of nucleus in P and Q are equal. Then
number of nucleus in stable element R is .

(A) 2 N0 (B) 3 N0 (C) 3 N0


2
(D) 9 N0
2

(115) Half life of radio active isotop X is 50 year, after decayed it converts into stable element Y. If the
proportion of X and Y is 1 : 15, approximate life of rock is .

(A) 100 year (B) 150 year (C) 200 year (D) 250 year

(116) Sample of Cu decayed upto in 15 min and converts in Zn, its Half life is .
7
8

(A) 10 min (B) 15 min (C) 5 min (D) 7.5 min


(117) A radio active element X decayed into new element Y. If creation rate of element Y is R then the
graph of R o t would be.
R R
(A) (B)

t t

R R
(C) (D)

t t

(118) Undecayed part of radio active sample in half time of half life is .

2 1
(A) (B) (C) (D)
1 1 3
2 2 4 2

Ans. : 95 (A), 96 (C), 97 (C), 98 (D), 99 (D), 100 (A), 101 (B), 102 (C), 103 (D), 104 (D),
105 (D), 106 (D), 107 (A), 108 (D), 109 (B), 110 (C), 111 (C), 112 (A),
113 (C), 114 (D), 115 (C), 116 (C), 117 (C), 118 (A)

496
Assertion - Reason type Question :
Instruction : Read assertion and reason carefully, select proper option from given below.
(a) Both assertion and reason are true and reason explains the assertion.
(b) Both assertion and reason are true but reason does not explain the assertion.
(c) Assertion is true but reason is false.
(d) Assertion is false and reason is true.

(119) Assertion : ZX
A undergoes two D  decay, two E  decays and two J  decays and the

daughter product is Z  2 X A8 .

Reason : In D  decay the mass number decreases by 4 and atomic number decreases by 2.
In E – decay the mass number remain unchanged, but atomic number increases by
1 only.
(A) (a) (B) (b) (C) (c) (D) (d)
(120) Assertion : All nuclei are not of same size.
Reason : Size depends on atomic mass.
(A) (a) (B) (b) (C) (c) (D) (d)
(121) Assertion : X - rays are used for studying the structure of crystals.
Reason : The difference between the atoms of crystal is of the order of wavelength of X - ray.
(A) (a) (B) (b) (C) (c) (D) (d)
§ ·
W1 ¨ W 1  W .¸
(122) Assertion : If half life and mean life of radio active element are and t ¨ 2 ¸
2 © ¹

1
Reason : Mean life =
decay constant

(A) (a) (B) (b) (C) (c) (D) (d)


(123) Assertion : Isobars are the nuclei having same mass number A but different atomic number Z.
Reason : Neutrons and Protons are present inside nucleus.
(A) (a) (B) (b) (C) (c) (D) (d)
(124) Assertion : The ionisation potential of Hydrogen is 13.6 eV, the ionised potential of double
ionised lithium is 122.4 eV.

13.6
Reason : Energy in nth state of Hydrogen atom is E n eV.
n2
(A) (a) (B) (b) (C) (c) (D) (d)
(125) Assertion : Radio active nuclei emit E particle.
Reason : Electrons exist inside the nucleus.
(A) (a) (B) (b) (C) (c) (D) (d)

497
(126) Assertion : If the half life of radio active substance is 40 days, then 25 % substance decays in
20 days.
t
W1 .
n
§1·
Where
n
Reason : N N0 ¨ ¸
© 2¹ 2

(A) (a) (B) (b) (C) (c) (D) (d)


(127) Assertion : Balmar series lies in the visible region of electro magnetic spectrum.
1 § 1 1 ·
Reason : R ¨ 2  2 ¸ Where, n 3, 4, 5.
O ©2 n ¹
(A) (a) (B) (b) (C) (c) (D) (d)
(128) Assertion : A certain radio active substance has a half life of 30 days. Its disintegretion
constant is 0.0231 day–1.
Reason : Decay constant varies inversly as half-life.
(A) (a) (B) (b) (C) (c) (D) (d)
(129) Assertion : Energy is released in nuclear fission.
Reason : Total binding energy of the fission fragments is larger than the total binding energy
of the parent nucleus.
(A) (a) (B) (b) (C) (c) (D) (d)
(130) Assertion : Heavy water is preferred over ordinary water as a moderator in reactors.
Reason : Heavy water, used for slowing down the neutrons, has lesser absorption probability
of neutrons than ordinary water.
(A) (a) (B) (b) (C) (c) (D) (d)
Ans. : 119 (B), 120 (B), 121 (A), 122 (B), 123 (B), 124 (B), 125 (C), 126 (D), 127 (A),
128 (B), 129 (A), 130 (A)
Comprehension Type Questions :
Passage :
A single electron orbits arounds a stationary nucleus of charge + Ze, Where Z is a constant and
e is the magnitude of the charge. It requires 47.2 eV to excite the electron from the second orbit
to the third orbit. (The ionization energy of Hydrogen atom = 13.6 eV. radius = 5.3 × 10–11 m,
c = 3 × 108 ms–1, h = 6.6 × 10–34 Js) Based on the above facts, answer the following questions :
(131) The value of Z is = .
(A) 1 (B) 2 (C) 3 (D) 5
(132) The energy required to excite the electron from the second excited state to the third excited state
is = eV.
(A) 47.2 eV (B) 14.53 eV (C) 16.53 eV (D) 18.53 eV
(133) The energy required to excite the electron from the first excited state to the second excited state
is = eV.
(A) 47.2 eV (B) 16.53 eV (C) 255 eV (D) none of these
(134) The minimum wavelength of the electromagnetic radiation required to deport the electron first
orbit to an upper orbit is .

(A) 48.5 A (B) 36.4 A (C) 45.8 A (D) 34.6 A


D D D D

498
(135) The kinetic energy, potential energy, total energy and angular momentum of the electron in the
first orbit have respective values given by.

(A) 340 eV,  340 eV,  680 eV, 1.05 u 1034 Js

(B) 340 eV,  680 eV,  340 eV, 1.05 u 1034 Js

(C) 680 eV,  340 eV,  680 eV, 2.05 u 1034 Js

(D) 680 eV,  1360 eV,  680 eV, 2.05 u 1034 Js


(136) The radius of the first Bohr’s orbit is = .

(A) 0.53 A (B) 0.106 A (C) 5.3 A (D) 1.06 A


D D D D

Passage : For identical Hydrogen gas has some atoms in the lowest energy level A and some in a upper
energy level B. The atoms of the gas make transition to a higher energy level by absorbing
photon of energy 2.7 eV. Subsequently, the atoms emit radiation of only six different photon
energies. Some of the emitted photons have an energy of 2.7 eV, some have more energy and
some less than 2.7 eV. Based on the above facts, answer the following questions :
(137) The principle quantum number of the initially excited level B is .
(A) 2 (B) 4 (C) 6 (D) 8
(138) The ionization energy for the gas atom is eV.
(A) 14.4 eV (B) 13.6 eV (C) 3.4 eV (D) 1.51 eV
(139) The emitted photons will have energy value E (in eV). The photons emitted have maximum and
minimum energy values E and e respectively.
(A) e = 1.35 eV and E = 13.5 eV (B) e = 0.7 eV and E = 1.35 eV
(C) e = 0.7 eV and E = 13.5 eV (D) none of these
Passage :
Nuclei of a radio active element A are being produced at a constant rate D . The element has a
decay constant O . At time t 0 , there are N 0 , nuclei of the element. At time t, number of nuclei

of A is N. Also for D 2 N0 O , the number of nuclei of A after one half life is and the
N1
2

limiting value of N as t o f is N f . Based on the above facts, answer the following questions :

(140) N as a function of t is given by,

(A) N N0 
D
O

1  e O t (B) N N0 e O t

(C) N N0 
D
O

1  e O t (D) N
1 ª
O ¬
D  D  O N 0 e O t º
¼

499
(141) The value of in terms of N 0 is = .
N1
2

(A) (B) (C) (D)


N0 3 N0 3 N0 3 N0
N1 N1 N1 N1
2 2 2 2 2 4 2 8

(142) The value of N f in terms of N 0 is = .

(A) Nf 3 N0 (B) N f o 0 (C) Nf N0 (D) Nf 2 N0

Passage : In the Bohr model of the Hydrogen atom, the electron revolves in a circular orbit of radius
ro = 0.53 A around the nucleus. Based on the above facts, answer the following question.
D

(143) The velocity of electron is nearly ms–1.

(A) 2.2 u 106 (B) 2.6 u 106 (C) 2.8 u 106 (D) 2.9 u 106
(144) The velocity of electron is n times the velocity of light. The value of n is = .

(A) (B) (C) (D)


1 1 1 1
81 101 137 337

(145) The electric potential energy in eV.


(A) –13.6 (B) –27.2 (C) –36 (D) –54
(146) The kinetic energy in eV is .
(A) 13.6 (B) –27.2 (C) –36 (B) –54
(147) The total energy in eV is .
(A) 13.6 (B) –13.6 (C) 27.2 (D) –27.2
Ans. : 131 (D), 132 (C), 133 (A), 134 (B), 135 (B), 136 (B), 137 (B), 138 (A),
139 (C), 140 (D), 141 (B), 142 (D), 143 (A), 144 (C), 145 (B), 146 (A), 147 (B)

Match the columns :

Match the column-1 with column-2 Ñ

column-1 column-2

(a) In this reaction mass of product is less (p) D  decay

than the mass of reactants.

(b) Energy per nucleon Increases. (q) E  decay

(c) Conservation of atomic mass number. (r) Nuclear Fission

(d) Conservation of charge. (s) Nuclear Fusion

500
(A) (a ) o ( p, q, r , s), (b) o ( p, q ), (c) o (r , s ), (d ) o ( p, q, r , s )
(B) (a ) o (q ), (b ) o (r ), (c ) o ( p ), ( d ) o ( s )

(C) (a ) o ( s ), (b ) o (r ), (c ) o ( q ), (d ) o ( p )

(D) (a ) o ( p ), (b) o ( s ), (c) o (q ), (d ) o ( r )


(149)
column-1 column-2

(a) Nuclear - Fusion (p) Some matter converts into energy

(b) Nuclear - Fission (q) Generally, possible for the nucleus


with less atomic number

(c) E  decay (r) Generally, possible for the nucleus


with high atomic number

(d) Endo thermic reaction (s) Possible due to weak nuclear force

(A) (a ) o ( p, r ), (b) o (q, r ), (c) o ( p, q, s ), ( d ) o ( r , s )


(B) (a ) o (q , s ), (b ) o ( p , s ), (c ) o (q , r , s ), (d ) o ( p, q , r , s )

(C) (a ) o ( s ), (b ) o ( s, p ), (c ) o ( p, q , r , s ), (d ) o (r )

(D) (a ) o ( p, q ), (b) o ( p, r ), (c ) o ( p, s ), (d ) o ( p, q, r )
(150)
column-1 column-2
(a) Transition between two energy levels of an atom. (p) Characteristics of X-ray
(b) emission of electron from the matter. (q) Photo-electric effect
(c) Moseley’s law. (r) Hydrogen spectrum
(d) Conversion of energy of photon in to energy of an (s) E  decay
electron

(A) (a ) o ( p, r ), (b) o ( p, q, s ), (c) o ( p ), (d ) o ( q )


(B) (a ) o ( p ), (b ) o ( s ), (c ) o ( r ), (d ) o ( q, s )

(C) (a ) o ( p, q, r , s ), (b) o ( s ), (c ) o ( p ), ( d ) o ( s )

(D) (a ) o ( s, r ), (b) o (r ), (c) o ( s, p ), (d ) o ( q, r )


Ans. : 148 (A), 149 (D), 150 (A)

501
14 & 15
Electronic Devices and Communication Systems
Semiconductor Electronics :
l The electrons in inner orbit of atom are bounded with nucleus so there is no change in their energy
levels but the electrons in outer orbit are shared with other atoms so there is the change in their
energy levels.
l For the electron in crystal different energy levels are available then the energy of electron in
independent atom. These energy levels are called Energy Band.
l Electron can transit in any level of upper band as per energy gained by it and it behave as free
electron and take part in conduction process. So this upper band is called conduction band.
l The energy difference between minimum energy of conduction band and maximum energy of
valence band is called band gap. Band gap is represented by E g .
l In the band gap region of energy any energy lavel does not exist. It implies electron cannot possess
energy in this rigion. This region is called forbidden gap.
l In semiconductors the valence band is completly filled and conduction band is completely empty at 0K.
Explanation of conductor, insulator and semiconductor based on band theory :
Conductor : In conductor the conduction-band and valence-band superpose to each other. There
is no farbidden gap in conductors.
Insulators : Forbidden gap is large in insulators ( E g > 7eV). The insulator like diamond the
forbidden gap is of 6 eV.
Semiconductor : Forbidden gap is small in semiconductor ( E g < 3eV). In silicon it is of 1.1 eV.
Extrinsic Semiconductor : The semiconductor in which proper impurities are added in proper proportion
are called extrinsic semiconductor.
N-Type Semiconductors :
l The conductivity of pure semiconductors can be drastically changed by adding impurities in the right
proportion. This process of adding impurities in the semiconductor is known as doping.
l In Germenium or Silicon mainly the conduction process is due to the electrons obtained by addition
of pentavalent donor impurities. In these crystals majority charge carriers are electrons. These
pentavalent inpurities are called donor impurities. For N type semiconductor ne ! nh
P - Type Semiconductors :
l In these type of semiconductors mainly the conduction process takes place by holes.
l For P-type semiconductors holes are mojority charge carriers and nh ! ne .
l P-type semiconductor is formed by doping of trivalent impurities in pure semiconductor. These
impurities are called accepter impurities.
l In stable condition the rate of production of electrons and holes as well as the recombination rate
are same.
l Recombination rate = R ne nh

502
l For intrinsic semiconductor ne nh ni , So recombination rate R ne nh R ni 2

ni 2 ne nh

R = Recombination co-efficient, nh = number density of hols, ne = number density of electrons


ni = number density of electron in pure semiconductor.

P-N Junction Diode :


l When P-type and N-type semiconductors are joined permenently the P-N functiondiode is formed.
In this device, anode and cathod two electrodes are there so it is called P-N junction diode.
l At small region at P-N junction in N type, electron and in P type, holes are not there. These micro
regions are depleted from their respective majority charge carriers. So is called depletion region.
In N part of this region positive ions and P part of this rigion nagetive ions are there, so electric field
is formed in the direction from N to P.
l Width of depletion region is of the order of 0.5 P m .
l The distribution of electric potential in deplection region is called depletion barrier.
l For Si depletion barier is of 0.7 V and that for Ge is 0.3 V.
Forward bias
l When the positive terminal of battery is connected to P side of P-N junction and negative terminal is
connected to N side of junction then such a connection is known as forward bias. In which external
electric field and electric field of depletion region are in opposite direction.
Reverse bias :
l When the positive terminal of battery is connected to N side of P-N junction and negative terminal is
connected to P side of junction then such a connection is known as reverse bias. In which external
electric field and electric field of depletion region are in same direction (helping mode.)
l In the forward bias connection of P-N junction the minimum voltage of external battery for which
the current increases rapidly is called ‘thresold voltage’ or ‘cut in voltage’
l In the reverse bias connection of P-N junction the voltage for which currrent increase rapidly is
called ‘Break down’ voltage. It is denoted by VR . If increase in current is, due to Zener effect then
it is called Zener break down voltage VZ and due to avalanche effect then it is called avalanche
break down voltage V.
Rectification and Rectifier :
l The process of converting AC energy in to DC energy is called rectification.
l The circuit which convert AC energy in to DC energy is called rectifier
l Half wave Rectifier : The rectifier which gives DC current/voltage during half cycle out of full
cycle of AC input voltage is called half wave retifier.
503
A
D Vin
t

AC V
Supply Vout
t

B
l Full wave rectifier : During both the half cycle of AC, the AC energy is converted in to DC
energy is called full wave rectifier.
A M
Vin
D1
RL t

AC C.T
Supply N Vout
t

B
D2
Types of diodes and their symbols :
N K
(1) P-N Junction diode : A P

A K
(2) Zener diode :

K
(3) Light emitting diode (LED) Ñ A

A K
(4) Photo diode :

A K
(5) Solar cell :

(1) A pure silicon block is connected with battery of 2V emf at temperature 300 K. The length of
cross section is 10 cm and cross sectional area is 1.0 u 10-4 m2.. How much electric current will
flow through this block ? Mobility of electron is 0.14 m2 V–1 s–1 and number density is 1.5 u 1016 m–3.
(A) 6.72 u 10–4 A (B) 6.72 u 10–5 A (C) 6.72 u 10–6 A (D) 6.72 u 10–7 A
504
(2) Find the number density of impurity atom added to convert pure Silicon semiconductor having
conductivity 6400 1 m-1, in to n type semiconductor. Mobility of electron is 0.133 m2 V 1 s 1 .
Neglect proporation of holes in conductor.
(A) 3 u 1022 m–3 (B) 3 u 1023 m–3 (C) 3 u 1024 m–3 (D) 3 u 1021 m–3
(3) When in the given semiconductor Indium impurity is added the number density of holes is obtained
4.5 u 1023 m–3, then find number density of electrons. For given semiconductor ni = 1.5 u 1016 m–3.
(A) 3 u 109 m–3 (B) 4 u 109 m–3 (C) 5 u 108 m–3 (D) 6 u 10–9 m–3
(4) Find current flowing through 5: resistance, (Consider both the diodes as an ideal diodes)

A 5:
(A) 2.0 A (B) 1.0 A
10: B (C) 0.5 A (D) 0

5:

5v

(5) Current flowing through the given circuit, I = A. (consider all the diodes as an
ideal diodes)
D2 5 :

D1 : 5 :
(A) 0.5 A (B) 1 A
10
20
: (C) 1.5 A (D) 2 A
5 D3

10V

(6) Current drown from the battery in given circuit is, I = (Diode D1 and D2 are ideal diode)
20 : D1

(A) 40 A (B) 50 A
5 5
D2 30 :
(C) 10 A (D) 20 A
5 5

I 20 :

5v

(7) In the given circuit with 220 V (rms) AC voltage source a diode and a capacitor are connected in
series, then what will be the voltage across two plate of capacitor ?
D

(A) 720 V (B) 110 V


220 V C
(C) 311.1 V (D) 110 2 V

505
(8) Current flowing in the given circuit, I = (D1 and D2 are ideal diode)
6:

D1
(A) (B)
5 5
A A
6 7
D2 3:
6V 6: (C)
1
A (D)
5
A
2 4

(9) What will be the potential difference between two terminal of 2 : resistance in the given circuit ?
consider diode as an ideal diode.

: D
(A) 10 V (B) 0 V
3

20V (C) 20 V (D) 12 V

2:
(10) If in the given circuit when the voltage across two ends A and B is 15 V and Zener breakdown
voltage is 6 V then the potential difference across two ends of resistor R will be
.
A

R (A) 6 V (B) 11 V
(C) 9 V (D) 17 V

RL

B
(11) The LED is working with 6 V battery and resistance R. When 10 mA current is passes through
it the voltage drop obtained is 2 V, then value of R will be .
(A) 40 k: (B) 4 k: (C) 200 : (D) 400 :

(12) The wavelength of light incident on photo diode is 1700 nm, then it’s energy gap E g will be
how much ?
(A) 0.073 eV (B) 1.20 eV (C) 0.73 eV (D) 1.16 eV
(13) Find the current flowing in the following circuit. Consider diode as an ideal diode.
D1
I 2k:
(A) 0 (B) 9.65 mA
(C) 10.0 mA (D) 10.36 mA
E = 20V
D2

(14) In the following circuit P–N junction diodes D1, D2 and D3 are connected between A and B.
Consider diodes as an ideal diodes. Arrange resistance obtained in the circuit when supply
voltage is connected across A and B of following values (i) –10 V, –5 V (ii) –5 V, –10 V
(iii) –4 V, –12 V

506
D1
R
D2
R (A) (i) < (ii) < (iii) (B) (iii) < (ii) < (i)
D3 R (C) (ii) = (iii) < (i) (D) (i) { (iii) < (ii)
R
R 4
4
A B

(15) In the following question choose proper pairs from Column-1 and Column 2.
Column - 1 Column - 2
(a) Zener Diode (p) In photography
(b) Photo Diode (q) In control of traffic light
(c) LED (r) In cd player
(d) Solar cell (s) In Voltage Regulated power supply

(A) a o p b o q c o r d o s
(B) a o q b o r c o s d o p
(C) a o r b o q c o p d o s
(D) a o s b o r c o q d o p
(16) The pieces of Aluminium and Silicon are placed in closed room at 280 K temperature, then which
of the following statement is corrent ? (Initial temperature is 300 K)
(A) Resistance of both decreases.
(B) Resistance of both increases.
(C) Resistance of Aluminium decreases, but resistance of silicon increases
(D) Resistance of Aluminium increases, but resistance of silicon decreases
(17) The rms value of AC signal in half wave rectifier is .
(A) Equal to value of DC (B) Less then value of DC
(C) Greater then value of DC (D) Zero
(18) Which diode is in forward bias from the following ?

(A) (B)
R +10 V R –5 V

R R
(C) (D)
–5 V –10 V +12 V +5 V

Ans. : 1 (D), 2 (B), 3 (C), 4 (C), 5 (C), 6 (B), 7 (C), 8 (C), 9 (B), 10 (C), 11 (D),
12 (C), 13 (C), 14 (C), 15 (D), 16 (C), 17 (B), 18 (B)

507
Junction Transistor, Working of Transistor, Characteristic of Transistor Amplifier
(common-emitter configuration) and Oscillator.
l Transister was invented by John Bardeen, walter Barten and Willium Schotky.
Transistor : Transistor is a device made up of two PN Junction diodes. Transistor is prepared
by placing a thin chip of semiconductor of opposite type between two same type of semiconductors. It
have three regions : Base Region (B), Emitter region (E) and collector region (C). Base (B) and
collector (C) are in reversed bias mode and emitter (E) and Base (B) are in forward biased condition.
Transistor are of two types :
(1) P-N-P Transistor :
Schemetic diagram Symbolic diagram

C
E C
P N P B
(Emitter) (Collector)

E
(Base)
(2) N-P-N Transistor B
:
Schemetic diagram Symbolic diagram

C
E C
N P N B

E
B
l Three different types of circuit in case of a transistor :
(1) Common Base (CB) (2) Common Collector (CC) (3) Common Emitter (CE)
(1) Common Base Circuit :
(i) Input charecteristic
Equation of Current :
IE I B  IC ( IE Emitter current, I B Base current, IC Collector curret)
I E o VBE ( VCB = constant)

VCB1
VCB2
IE
A
AB = ,E
BC = 9BE

C B

(cut in voltage) VBE

508
Characteristic input resistance, ri =
BC
AB

9BE
ri
,E ( VCB = Constant)

(ii) Output Characteristic :

IC o VCB ( IE = Constant)

(I E )3
IC (I E )2 Characteristic Output Resistance
(mA)
(I E )1 § 9CB ·
r0 ¨ ¸ ( I E = Constant)
© IC ¹
(V)
0 –2 –4 –8 VCB
(2) Common Emitter Circuit (CE) :
(i) Input Characteristic :

(VCE )1
(VCE ) 2
IB § 9BE ·
(mA) Input resistance ri ¨ ¸ ( VCE = Constant)
© IB ¹

0 (V)
VBE

(ii) Output Characteristic :

§ 9CE ·
I B3
(i) Output resistance r0 ¨ ¸ ( I = Constant)
IC I B2 © IC ¹ B
(mA)
I B3 (ii) Current gain ( E )

§ I ·
E ¨ C¸ ( VCE = Constant)
© IB ¹
(V)
(iii) Transconductance ( g m )
VCE

IC
gm = =
9BE ri

l Unit of g m is mho ( )

509
Equations for Common Base Amplifire :

(1) Current gain ( dc


IC
(2) Voltage gain ( AV )
IE

D   90
IC
AV = ( D u resistance gain)
9i
ac
IE

(3) Power gain (A P )

'P0
= D 2 u resistance gain
'Pi
AP

Equations for Common Emitter Amplifire :

(1) Current Gain (Edc )

IC
dc
IB

IC
ac
IB
( VCE = Constant)

Reletion between D and

1 1
1
D

(2) Voltage gain (A V )

= – E ˜ r = –g m R L
V0 RL IE RL
AV = = – r I
VS i B i

(3) Power gain

= AV ˜ Ai =  ˜
'VCE ˜ 'I C RL
|A P | 
'VBE ˜ 'I B ri

RL
? Ap E2 gmER L
ri

Applications of Transistor
(i) As a switch (ii) As an amplifier (iii) As an oscillator
l Equations of oscillating Frequency

, L = Inductance C = Capacitance
1
2S LC
f

510
(19) In an N – P – N common emitter amplifier when load resistance is 18 k the voltage gain
obtained is 270. If input resistance of circuit is 3 k then what will be transconductance and
current gain for that ?
(A) 0.015 , 45 (B) 0.03 , 25 (C) 0.02 , 20 (D) 0.04 , 20
(20) In an N – P – N common base transistor circuit 4 % electrons from emitter combines with
holes in base. If collector current is 24 mA then find emitter current and current gain.
(A) 40 mA, 0.85 (B) 20 mA, 0.93 (C) 25 mA, 0.96 (D) 30 mA, 0.96
(21) The base current changes by 250 PA when 175 mV input signal is applied to CE amplifire. If
output voltage is 5V then find output resistance (R L ) and voltage gain for that.
(A) 1 k:, 10 (B) 3 k:, 12.5 (C) 70 k:, 28.8 (D) 0.7 k:, 28.8
(22) In N-P-N common emitter amplifier input voltage is changed by 200 mV, the collector current
changes by 5 mA. Current gain in this circuit is 100. What should be the value of load resistance
to obtain power gain equal to 5000 ?
(A) 3000 : (B) 2 k: (C) 1000 : (D) 4000 :
(23) In the circuit, given in figure keeping voltage at base resistance RB , equal to +15V, voltage VBE
and VCE both becomes zero, then find I c , I E and E for that circuit.
+15V
3 k: (A) 7 mA, 250 mA, 38.7
IE
RB IB (B) 5 mA, 50 $ , 100
+15 V (C) 5 mA, 75 $ , 66.6
(D) 10 mA, 200 $ , 50
200 k:

(24) AC Current gain for N-P-N common emitter circuit is 150. Input resistance of transisstar is
500 : . To obtain power gain equal to 1000 for this circuit, find the value of load resistance.
(A) 93.75 W (B) 22.22 W (C) 200 W (D) 300 W
(25) For common emitter transistor the current gain is 0.98. If input and output load resistance are
70 : and 5 k respectively then find voltage gain and power gain.
(A) 2200, 15700 (B) 400, 217150 (C) 2500, 121500 (D) 3500, 171500
(26) The current gain for a transistor is 50. When it is used as common emitter it’s input resistance is
1 k: and maximum value of input voltage is 0.01V, then find value of collector current at that
time.
(A) 500 $ (B) 50 mA (C) 0.5 $ (D) 0.5 mA
(27) In common base amplifire input resistance and load resistance are 3W and 24W respectively. If
current gain is 0.6, then find voltage gain.
(A) 0.48 (B) 48 (C) 4.8 (D) 480
(28) In common base transistor amplifier current gain is 0.5. Emitter current is 7mA then find base
current.
(A) 5.5 mA (B) 4.5 mA (C) 2.5 mA (D) 3.5 mA
511
20
(29) In transistor oscillator circuit, L mH and C 0.02 ) , then find resonance frequency of oscillator
S2
(A) 25 mHz (B) 25 kHz (C) 2.5 kHz (D) 250 kHz
Ans. : 19 (A), 20 (C), 21 (D), 22 (B), 23 (C), 24 (B), 25 (D), 26 (A), 27 (C), 28 (D), 29 (B)

Logic Gates
Number of probability in logic gates is 2n , Where n number of Inputs
OR Gate : Boolean equation : Y = A + B

A B Y=A+B
0 0 0 Y=A+B A
0 1 1
1 0 1
B
1 1 1
Circuit symbol
Truth Table
Circuit explaining working
When all input are zero then output is zero
AND Gate : Boolean equation : Y = A < B

A B Y
0 0 0 A Y=A<B
0 1 0
1 0 0 B
1 1 1 A B
Truth Table Circuit symbol Circuit explaining working
When all input are 1 output is 1.
NOT Gate : Boolean equation : Y=A (Gate with only one input)
Bulb
A Y
0 1
A
1 0 Y A
Truth Table
Circuit symbol
output is opposite to input Circuit explaining working

NAND Gate : Boolean equation : Y=A< B A + B (Output of AND gate is given to input of NOT gate to
prepare NAND gate.)

A B Y1 =A < B Y=A <B A Y1 Bulb


0 0 0 1 B
Y=A <B
0 1 0 1
1 0 0 1
1 1 1 0 A B
A
B
Y=A <B
Truth Table
Circuit symbol Circuit explaining working

512
When main input A and B are (0,1) and (1,0) output is always 1.
NOR Gate : Boolean equation : Y = A+B = A < B (Output of OR gate is given to input of NOT gate to
prepare NOR gate)
A B A+B A+B
A+B A Y1
Y
0 0 0 1
B
0 1 1 0
B
1 0 1 0
1 1 1 0 A
Y A+B
Trooth Table

Circuit symbol Circuit explaining working

When either of input or both is present out put is always zero


Note : OR, AND and NOT gates are called fundamantal logic gates NAND and NOR gates are called
universal gate.
XOR Gate : Boolean equation : Y = A< B + A < B
Main input A and B are same (0, 0) and
Truth Table
(1, 1) the final output is zero.
A B A B A<B A<B
Y = A<B  A<B A A<B
0 0 1 1 0 0 0
0 1 1 0 1 0 1
1 0 0 1 0 1 1
1 1 0 0 0 0 0 B
A<B

Y = A< B  A< B
XNOR Gate : Boolean equation : Y = A<B + A<B = (A+B) <(A+B)

XNOR = NOT + XOR


A B Y
0 0 1 A Y
0 1 0
B
1 0 0
1 1 1
Circuit symbol
Truth Table

Main input A and B are same (0, 0) and (1, 1) the final output is 1.
(30) Find output Y for following circuit :
A
2 Y
B

(A) 0 (B) 1 (C) 0, 1 (D) 1, 0


513
(31) As shown in the figure the combination of OR gate and short circuited NAND will behave as
which gate ?
A
OR NAND Y
B

(A) NOT gate (B) OR gate (C) NAND gate (D) NOR gate
(32) Find output Y for following circuit.

0
G1

A
Y
B
G2

1 G3

(A) 0 (B) 1 (C) Between 0 and 1 (D) can’t say any thing
(33) Following combination will behave as which gate ?

Y’
Y

(A) AND gate (B) NAND gate (C) OR gate (D) XOR gate
(34) In the following circuit combinations output at A, B and C are respectively........

1
(A) 1

0
(B)
1

(C) 1
0
1

(A) 0, 1, 1 (B) 0, 1, 0 (C) 1, 1, 0 (D) 1, 0, 1

514
(35) Which gate will be obtained by combination of following gates ?

A
Y
B

(A) XOR (B) AND (C) NAND (D) OR


(36) Which gate will be obtained by combination of following gates ?

(A) NOT (B) NAND (C) XOR (D) OR


(37) Which of following is correct option for given combination of gates ?

A
B
Y

(A) Y A<B  B (B) Y A<B  B<A (C) Y A<B  B (D) Y (A  B).B


(38) For the given combination of gate which option from the following is correct ?

(A) A<B X (B) A+B = X (C) A < B X (D) A  B X


(39) Following combination of gates will behave as which gate ?

A
G1
G3 Y
B
G2

(A) AND gate (B) XOR gate (C) NOR gate (D) NAND gate
Ans. : 30 (A), 31 (D), 32 (A), 33 (A), 34 (C), 35 (B), 36 (D), 37 (C), 38 (A), 39 (D)

Experimental Techniques

(1) Draw forward bias charecteristic of P-N Junction diode and find static resistance and dynamic resistance.
(2) Draw reverse characteristic curve of Zener diode and find breakdown voltage.
(3) Obtaine input and output characteristic curves of common emitter N-P-N transistor and final current gain.
(4) Differentiate diode, LED, Transistor, IC, resistance and capacitor from the given componants of circuit.

515
(1) P-N Junction Diode :
I f o Vf (Forward Bias)

If 1
(i) Static resistance at point P
OA
(mA) P AP tan D
1
(ii) Dynamic Resistance
AB
tan E
E AP
(V)
D
0 B A
Vf

(2) Zener Diode :

0
Vr (V)

Breakdown Voltage

mA

(3) Common Emitter :


(i) Input characteristic (ii) Output charateristic
I B o VBE ( VCE Constant) IC o VCE ( I B Constant)

IC 3 I B3
VCE = Constant
IC
IB mA
mA
IC 2 I B2

I C1 I B1

'I C
VBE (V) VCE ( V2 )

'I B
E

(40) The resistance of P-N junction in forward and reverse bias are respectively , and
(A) 100 :, 106 : (B) 106 :, 100 : (C) 102 :, 106 : (D) 106 :, 102 :
(41) The value of thresold voltage for Ge and Si are respectively and
(A) 0.7 V, 0.3 V (B) 0.4 V, 0.5 V (C) 0.3 V, 0.8 V (D) 0.3 V, 0.7 V

516
(42) P-Side is earthed for a P–N junction diode and –3V electric potential is givnen to N-side then
diode .
(A) Will not conduct electric current (B) Will pass electric current partially
(C) Will conduct electric current (D) will breakdown
(43) In P–N junction diode the depletion region is formed due to
(A) Driffing of holes (B) Diffution of constituent ions
(C) Driffing of electrons (D) Displacement of impurity ions
(44) For a transistar D 0.95 . If change in emitter current is 10 mA then change in base current will
be .

(A) 10.5 mA (B) 0.5 mA (C) 9.5 mA (D) 20 mA


19

(45) In common base amplifire the input resistance is 4 : and load resistance is 32 : . D 0 .5 ,
then what will be voltage gain ?
(A) 2 (B) 8 (C) 4 (D) 8
(46) For a transistor D = 0.95 . If change in emitter current is 100 mA then calculate change in
collector current.
(A) 90 mA (B) 100 mA (C) 80 mA (D) 95 mA
(47) For common base amplifier and common-emitter amplifire the phase difference between input
and output voltage are respectively and .
(A) 0, 180q (B) 180q, 0 (C) 0q, 0q (D) 180q, 180q
(48) What is the relation between current gain D and E for transistor ?

(A) E 1 D
D
(B) E D
1 D
(C) E 1 D
D
(D) E D
1 D

Ans. : 40 (A), 41 (D), 42 (C), 43 (B), 44 (B), 45 (C), 46 (D), 47 (A), 48 (B)

517
Communication Syatem

Basic Communication System :


Basic communication system consist of source of information, transmitter, link and receiver.

Source of Transmission
Transmitter Receiver
Information Channel

Noise

(i) Information : Any massage or thought can be represented by information. Information can be
individual or in group. Information can be in the form of symbol, code, pair of words, picture.

(ii) Transmitter : In radio transmission the transmitter consist of transducer, modulator,


amplifire and transmitting antena.

Transducer Ñ Used to cenvert sound waves in electrical signals.

Modulator : It admixed audio electric signals with radio waves of high frequency.

Amplifire : Increases the efficiency of modulated waves.

Antenna : Waves are transmitted through antenna placed in space.

(iii) Communication channel :


l Communication channel is a medium connecting transmitter and receiver. The information is
transmitted through this channel and reaches to receiver. Transmission link is other name of
transmission channel.

l In radio and T.V. transmission the free space is used as transmission channel.

Wireless communication : The communication system in which there is no connection of


conducting wire between transmitter and receiver is called ‘wireless communication’.

l In telephone system, two wire line is used as a transmission channel.

(iv) Receiver :
l Receiver section amplify the received signals which are transmitted through transmission
channal. The signals passes through the demodulation process in the receiver and are
converted into the original information using proper electronic device.

l Electrical signal given to loudspeaker converts it in to sound signal.

l Picture tube converts electrical signals into a picture.

518
Communication is of two type :
(i) Point to Point Communication mode
In Point to Point communication mode, communication takes over a link between single
transmitter and single receiver.
For example, communication through telephone system.
(ii) Broadcast mode : In the broadcast mode there are large number of receivers corresponding
to single transmiter for example, the transmission through radio and TV system.
Analog Signal and Digital Signal
l For transmission the information waves are converted in to electrical waves, which are
called signal.
Signal are of two types :
(i) Analog signal :
l An analog signal is a continuously varifing signal with respect to time.
l It’s value can be any one between maximum and minimum value.
l It is represented by sine wave
e.g. output signal of microphone or video camera.
(ii) Digital signal :
l The signal which have only two values which does not change with time is called Digital
signal.
l It’s minimum value is 0 and maximum value is 1.
l 0 and 1 is called bit. The group of bit is called byte.
e.g. Digital signal are used in the digital communication.
Modulation :
l The process of superposing low frequency audio signals on waves with high frequency is
called modulation.
l The low frequency signal is called modulating signal or modulating wave.
l The high frequency wave carry the information so is called carrier wave and the resultant
mixed wave is called modulated wave.
l Carrier wave is of sine wave form, which is mathematically represented as follows :

ec E c sin(Zc t  I)

Where, E c = amplitude of carrier wave

Zc angular frequency

I Initial phase of wave

519
Types of Modulation :
There are three types of modulation :
(1) Amplitude modulation (AM)
(2) Frequency modulation (FM)
(3) Phase modulation (PM)
(1) Amplitude Modulation :
l A modulation in which the amplitude of the carrier wave Ec is varied in accordance with the
instantaneous value of the modulating wave is called amplitude modulation.

l Carrier wave : ec E c sin(Zt  I )

l Modulating wave : em E m sin Zm t

?e (E c  em ) sin Zc t

(The amplitude of carrier wave is varying according to instantaneous value of modulating signal)

? e (E c  E m sin Zm t )sin Zc t

E c (1  sin Zmt )sin Zc t


Em
Ec

e E c (1  ma sin Zm t )sin Zc t is called mathematical form of amplitude modulated wave.

Em
Ec is called modulation index
l ma

l Generally it’s value is between 0 and 1. If ma is greater then 1 then AM wave gets distorted.
Modulation index in percentage :

Em E max – E min
ma
Ec E max + E min

E max  E min
u 100%
E max  E min
ma (%)

(2) Frequency Modulation :


l The process of changing the frequency of Audio signal in carrier wave is called
frequency modulation.

l Frequency co-efficient G ( f max  f c )

f c  f min

520
G
l mf is called modulation index (For Frequency)
m

Where, f m maximum frequency

G f max  f c f c  f min
?mf
fm fm fm

Demodulation :
l The process of separating sound-waves from modulated wave is called ‘demodulation’ or
‘detection’
l This process is reverse process of modulation.
l A circuit separating these waves is called ‘demodulation circuit’ or ‘detector circuit’
Propagation of Electromagnetic Waves :
l Propagation of waves is done in three ways :
(1) Ground wave propagation or surface-wave propagation (2) Sky wave propagation (3) Space wave
or Tropospheric propagation
(1) Ground Wave Propagation or Surface propagation :
l In the ground wave propagation electromagnetic waves propagate along the surface of
Earth, so it is called ground wave propagation.
l The waves with frequency greater than 2 MHz can not propagate to long distance via
surface wave propagation.
l The waves in MW band ( Frequency 550 kHz – 1600 kHz) of AM radio propagate through
ground wave.
(2) Sky Wave Propagation :
l The electromagnetic waves emitted by transmitter reflected from ionosphere situated at
height 60 km to 300 km and received at large distance by a receiver, this propagation is
called sky wave propagation.
l The propagation of radio waves with frequence 2 MHz to 30 MHz is done by sky waves.
l The gas molecules in Earth’s atmosphere are ionized and seperated in electrons and positive
ions. This atmosphere is called ionosphere.
l The waves with frequencies corresponding to SW (Short Wave) band of radio broad casting
can be propagated to long distance by ionosphere.
(3) Space Wave or Tropospheric Wave Propagation :
l The waves propagating to receiver directly from the transmitting antenna in a straight-line
path or by reflecting from Earth are called space waves.
l Radio waves having frequency 30 MHz to 300 MHz are propagates by space wave
propagation.

521
Line of Sight Communication : A

dT dR
Receiving
Transmitting dm
Antenna
Antenna
hR
hT

Earth
O

l If the distance between transmitting antenna and receiver antenna is large, then the curvature of
Earth comes in the ways of space wave propagation, because radio-waves can be received only
up to the distance determined by line of sight.
l If receiving antenna is placed at height hR and height of transmitting antenna is hT, then
maximum communication range is given by,

dM 2 hT R  2hR R

l Area covered, A Sd 2 2SRh


l Population covered = Population density u Area covered
Important Equation :

(1) Wave length, O


c
f

Where c = Velocity of light, f = frequency of wave, O = wave length


(2) Carrier wave, ec E c sin(Zc t  I)
E c = Amplitude of carrier wave

Zc = angular frequency, I = initial phase

(3) Permittivity of medium in the presence of free electrons : H H0  Ne


2

mZ2

Kc H 1  Ne
2
(4) Dielectric constant of medium with respect to vaccum : H0 mZ H0
2

(5) Refractive index of ionized medium : n kc 1 Ne 2


mZ 2 H 0

Where, N = electron density of medium, e = charge of electron = 1.6 u 1019 C , m = mass of

electron = 9.1 u 1031 kg , H0 = Permittivity of free space (Vaccum) = 8.854 u 1012 F m 1 , Z


= Angular frequency of Radio-waves = 2Sf

n 1  81N
f2

522
(6) Critical frequency, f c 9 N max
(7) Line of sight (Communication Range)
d 2hr , Where h = height of antenna, r = radius of Earth
(8) Covered Area, A Sd 2 2Srh
(9) Covered Population = Population density u covered Area (A)
(10) Inductive reactance ZL

Capacitive reactance
1
ZC
(49) The height of TV transmitter tower is 150 m at one place. To double the coverage area what
should be the height of tower ?
(A) 150 m (B) 300 m (C) 75 m (D) 450 m
(50) Height of TV antenna is 200 m. If average population density is 4000 km 2 , then how many
people can view TV programme ? Radius of Earth, Re 6400 km .
(A) 3.2 u 108 (B) 3.2 u 107 (C) 3.2 u 106 (D) 3.2 u 105
(51) Find the height hT of transmitting antenna if receiving antenna is at height 32 m. For
satisfactorily line of sight communication between two antenna the maximum distance is 45.5 km.
Radius of Earth Re 6400 km .
(A) 50 m (B) 75 m (C) 25 m (D) 100 m
(52) For an amplitude modulated wave, find the maximum and minimum amplitude. Modulation Index
is 25% and amplitude of carrier wave 8 V and frequency of carrier wave is 1 MHz.
(A) 12 V, 5 V (B) 10 V, 6 V (C) 10 V, 2 V (D) 15 V, 3 V
(53) In morning the maximum electron density of ionosphere is 1010 m3 . At Noon maximum
electron density increases to 3 u1010 m3 , then find the ratio of critical frequency at morning
and at noon.
(A) 1.732 (B) 1.414 (C) 2.000 (D) 2.236
(54) The electron density of ionosphere layers E , F1 , F2 are 2 u1011 m 3 , 5 u 1011 m 3 , 8 u 1011 m 3
respectively, then find the ratio of critical frequency for reflected radio waves.
(A) 2 : 3 : 4 (B) 2 : 4 : 3 (C) 3 : 2 : 4 (D) 4 : 3 : 2
(55) For a co-axial cable inductance is 0.80 + and capacitance is 20 pF, then find impedence of this
cable.
(A) 100 : (B) 4 u 103 : (C) 4 u 102 : (D) 200 :
(56) In a radio wave receiver to tune short wave and medium wave station the L–C circuit is used.
In it capacitance are same but inductunce of coil Ls and Lm are different then.
(A) Ls ! Lm (B) Ls  Lm (C) Ls Lm (D) Not any one

Ans. : 49 (B), 50 (B), 51 (A), 52 (B), 53 (A), 54 (A), 55 (D), 56 (B)

523
Passage based Question :

Passage : In a common base transistor amplifier the input resistance is 200 : and output resistance is
2000 : . If D 0.95 then,

(57) (i) Voltage gain is .


(A) 50 (B) 75 (C) 95 (D) 9.5
(58) (ii) Power gain is .
(A) 50 (B) 75 (C) 9.025 (D) 90.25
Passage : In a common emitter amplifier input resistance is 1 k: and output resistance is 5 k . Input
signal is of 10 mV and E 100 .
(59) (i) So output voltage =
(A) 1.25 V (B) 2.25 V (C) 2.5 V (D) 5 V
(60) (ii) Power gain
(A) 50,000 (B) 5500 V (C) 6 KV (D) 7500 V
Passage : An N–P–N transistor is connected in common emitter configuration as follow. Collector
supply is 8V. Voltage drop is 0.8V at the load resistance of 800 : connected in collector

circuit. If D then,
25
26

`
IC
C
R0 V0
IB
E
Vi
VBE IE

(61) (i) Collector emitter voltage VCE = .

(A) 8.2 V (B) 6.2 V (C) 7.2 V (D) 5.2 V


(62) (ii) If input resistance of transistor is 100 : then power gain will be .
(A) 1000 (B) 5000 (C) 2500 (D) 3000
Passage : A carrier wave of frequency 70 MHz and amplitude 40 V is modulated by an audio signal of
frequency 2 kHz and amplitude 20 V.
(63) (i) Modulation index will be .
(A) 20 % (B) 30 % (C) 40 % (D) 50 %
(64) (ii) Frequency spectrum of AM waves is .
(A) 70,000 kHz – 69998 kHz (B) 65250 – 62050 kHz
(C) 52100 kHz – 45020 kHz (D) 72150 – 18750 kHz
Ans. : 57 (D), 58 (C), 59 (D), 60 (A), 61 (C), 62 (B), 63 (D), 64 (A)

524
Assertion - Reason type Question :

Instruction : Read assertion and reason carefully, select proper option from given below.

(a) Both assertion and reason are true and reason explains the assertion.

(b) Both assertion and reason are true but reason does not explain the assertion.

(c) Assertion is true but reason is false.

(d) Assertion is false and reason is true.

(65) Assertion : Charge carriers in pure semicanductor are thermionic.


Reason Ñ Control on number of charge carrier is easy.
(A) a (B) b (C) c (D) d
(66) Assertion : When two P–N junction diodes are placed back to back it behave as N–P–N
transistor
Reason Ñ Transistor is a device working by electric current, a triode valve is a device working
by voltage.
(A) a (B) b (C) c (D) d
(67) Assertion : Resistivity of semiconductor increases with temperature.
Reason Ñ At high temprature more co-valent bonds breaks.
(A) a (B) b (C) c (D) d
(68) Assertion : Common base amplifire gives voltage gain with phase difference.
Reason Ñ The width of depletion region in P-N junction diode increases with increase in
reverse bias.
(A) a (B) b (C) c (D) d
(69) Assertion : Common emitter amplifire gives voltage gain with phase difference.
Reason Ñ The width of depletion region increases in P-N junction diode with increase in
reverse bias.
(A) a (B) b (C) c (D) d
(70) Assertion : Electrons in conduction band possess more energy than electrons in valence bond.
Reason Ñ Mobility of electrons and holes are equal.
(A) a (B) b (C) c (D) d
(71) Assertion : The signal which possess the level either 0 or 1 is called digital signal.
Reason Ñ The signal which changes continuously with time is called analog signal.
(A) a (B) b (C) c (D) d
(72) Assertion : Photo diode is used in reverse bias mode.
Reason Ñ In reverse bias decrease in minority charge is considereble.
(A) a (B) b (C) c (D) d
(73) Assertion : Mainly transistor is used in common emitter configuration.
Reason Ñ In common emitter circuit large current gain and voltage gain is obtained.
(A) a (B) b (C) c (D) d

525
(74) Assertion : At definite temperature with less doped situation the conductivity of silicon is more.

Reason Ñ Conductivity of pure semiconductor is less than the doped P-type semiconductor.
(A) a (B) b (C) c (D) d
(75) Assertion : Normally AM broadcasting is used, because it keep away the receiver complexity.

Reason Ñ Required band width in FM is less than required band width of AM.
(A) a (B) b (C) c (D) d
(76) Assertion : Modulated waves are not received directly by headphone.

Reason Ñ High frequency waves.


(A) a (B) b (C) c (D) d
(77) Assertion : Sky waves are not used for transmission of TV signals.

Reason Ñ Frequency range of TV signal is of 60 MHz to 100 MHz


(A) a (B) b (C) c (D) d

Ans. : 65 (A), 66 (B), 67 (B), 68 (A), 69 (A), 70 (D), 71 (A), 72 (A), 73 (B), 74 (A),
75 (D), 76 (A), 77 (A)

Match the columns :

(78) Gate Truth Table


(a) AND (p) (q)
A Y A B Y
0 1 0 0 1
1 0 1 0 1
(b) OR
0 1 1
1 1 0
(c) NOT (r) (s)
A B Y A B Y
0 0 0 0 0 0
1 0 0 1 0 1
(d) NAND 0 1 0 0 1 1
1 1 1 1 1 1

(A) a o s b o p c o q d o r
(B) a o s b o r c o p d o q
(C) a o r b o s c o p d o q
(D) a o s b o q c o r d o p

526
(79)
Column-1 Column-2
(a) Convenient frequency for satellite (p) 15.625 kHz
communication
(b) Line frequency of TV system in India (q) 15790 Hz
(c) Lower frequency used in satellite (r) 3000 MHz
communication
(d) At which frequency radio Silon 19.0 m (s) 0.84 Hz
broadcast
(A) a o r b o p c o s d o q
(B) a o q b o r c o s d o p
(C) a o p b o r c o q d o s
(D) a o s b o p c o r d o q

(80)
Symbol Gate

(a) (p) NOR

A
(b) B
(q) XOR (Special OR gate)

(c) (r) Equivalent to XOR gate


Y

or
A
B

(d) (s) NOT

(A) a o r b o s c o p d o q
(B) a o s b o r c o q d o p
(C) a o s b o p c o q d o r
(D) a o p b o q c o r d o s

Ans. : 78 (C), 79 (A), 80 (B)

527

You might also like